Test 1 (1/3)

Réussis tes devoirs et examens dès maintenant avec Quizwiz!

A 35 year old male patient with no health problems states he had the flu shot last year and asks if it is necessary to have it again this year. What is the nurses best response

Yes because the vaccine guards against a specific virus and reduced your chances of acquiring thr flu

Simple Facemask nursing interventions:

-be sure mask fits securely over nose and mouth -assess skin and provide skin care to the area covered by the mask -monitor the patient closely for risk for aspiration -provide emotional support to the patient who feels claustrophobic -suggest to the health care provider to switch the patient from a mask to the nasal cannula during eating

T-piece rationales:

-condensation interferes with flow rate delivery of FiO2 and may drain into the tracheostomy if not emptied -If the port is occluded, the patient can suffocate. -the weiight of the T-piece pulls on the tracheostomy and causes pain or erosion of skin at the insertion site -An adequate flow rate is needed to meet the inspiration effort of the patient. If not, the patient will be "air-hungry."

Nasal Cannula nursing interventions:

-ensure that prongs are in th enares properly -apply water-soluble jelly to nares PRN -assess the patency of the nostrils -assess the patient for charges in respiratory rate and depth

dehydration (hypovolemia)

fluid intake/retention does not meet body's fluid needs; results in fluid volume deficit signs & symptoms include: -thready, increased pulse rate -decreased BP and orthostatic hypotension -increased respirations, dyspnea -flat neck and hand veins -diminished peripheral pulses -decreased urine output -poor skin turgor -hypoactive bowel sounds/constipation

tactile (vocal) fremitus

vibration of the chest wall produced when the patient speaks

PICC

what type of VAD would you want to use for someone with chronic osteomyelitis who needs to be on an IV antibiotic for 3 months or longer?

A patient with active tuberculosis is ordered to take isoniazid (INH), pyrazinamide (PZA), and rifampin (RIF) and asks the nurse why it is necessary to take three antibiotics. What is the nurse's best answer?

"Three antibiotics help prevent bacterial drug resistance. Multidrug therapy provides quicker destruction of organisms and combats drug resistance. It does not allow for lower dosing or decrease side effects. Taking these three drugs does not produce a synergistic effect.

Which instruction should the nurse include in an exercise teaching plan for a patient with chronic obstructive pulmonary disease (COPD)?

"Use the bronchodilator before you start to exercise."

A young adult client refuses an influenza vaccine, saying, "I'm healthy and won't get that sick if I get the flu." Which is the best response by the nurse?

"You may spread the disease to people who are more at risk for severe symptoms.

A patient tells the nurse that after 3 weeks of multidrug therapy to treat tuberculosis (TB), the symptoms seem to have resolved. What does the nurse tell this patient?

"You will need to continue therapy for at least 6 months." Even though patients feel better and are no longer contagious, TB drug therapy must continue for 6 months or longer to avoid relapse and drug resistance. Directly observed therapy is used for patients who may have difficulty complying with treatment. It is important to continue taking all drugs in the regimen to avoid drug resistance.

A patient is being treated with ciprofloxacin 500 mg PO twice daily due to possible exposure to inhalation anthrax. What is the nurse's best answer when the patient asks how long this medication must be taken?

"You will need to take the medication for about 2 months." When medication is given for prophylaxis related to inhalation anthrax exposure, the patient will need to take it for 60 days and may take it longer if exposure was heavy. Ten days is not enough time for adequate prevention, and 6 and 12 months are too long.

An emergency department nurse is making a general appearance assessment on a preschool child just admitted to the emergency department. Which general assessment findings indicate the child "looks bad"?

- cold extremities - color pale - lethargic

A 55-yr-old patient with increasing dyspnea is being evaluated for a possible diagnosis of chronic obstructive pulmonary disease (COPD). When teaching a patient about pulmonary spirometry for this condition, what is the most important question the nurse should ask?

"Have you taken any bronchodilators today?"

A patient diagnosed with mild obstructive sleep apnea asks the nurse if surgery will be necessary. What does the nurse tell this patient?

"Mild symptoms can be treated by changing sleeping positions or by losing weight." Mild symptoms can be treated with conservative measures such as repositioning or weight loss. More severe apnea will require noninvasive positive pressure and then surgery if that is not effective. Medications treat daytime sleepiness that results from chronic sleep apnea but does not treat the cause.

A patient is scheduled for surgical removal of a neck tumor followed by radiation treatments. The patient asks the nurse why radiation therapy is not performed prior to the surgery. How does the nurse respond?

"Radiation therapy slows tissue healing." Radiation treatment slows tissue healing and is generally done after surgery. It is not withheld until after surgery because of tissue edema or changes in respiratory function. Radiation does not increase the risk of metastasis.

Indications for a Tracheostomy (5)

-Acute airway obstruction -Airway protection, ex. Airway reconstruction after laryngeal trauma or cancer surgery -Prolonged intubation or need for mechanical ventilation -Control of Pulmonary secretions -Obstructive sleep apnea

Nasal Cannula

-Flow rates of 1-6 L/min -O2 concentration of 24%-44% (1-6 L/min) -Flow rate >6 L/min does not increase O2 because anatomical dead space is full -Assess patency of nostrils -Assess for changes in respiratory rate and depth

Hypoxia can be caused by these factors for a patient with tracheostomy:

-Ineffective oxygenation before, during, and after suctioning -Use of a catheter that is too large for the artificial airway -Prolonged suctioning time -Excessive suction pressure -Too frequent suctioning

Key Nursing Assessments and Safe Care Practices (cont)

-Measure and record cuff pressure every 8 hours (indicated for coffed tracheostomy tubes) -Cuff pressure must be maintained at 14 to 20 mmHg -Excessive cuff pressure may cause complications including tracheomalacia, tracheal stenosis, or tracheoesophageal fistula

Bronchial and Oral Hygiene

-Turn/reposition every 1 to 2 hours, support out-of-bed activities, encourage early ambulation -Coughing and deep breathing, chest percussion, vibration, and postural drainage promote pulmonary cure -Avoid glycerin swabs or mouthwash containing alcohol for oral care; assess for ulcers, bacterial/fungal growth, infection

Parts of Tracheostomy Tube: Obturator

-Used to facilitate insertion of the outer cannula when inserting or changing a tracheostomy tube or in the even of accidental decannulation -Removed immediately after tube placement and kept at patients bedside.

Simple Facemask rationales:

-a poorly fitting mask reduces the FiO2 delievered -pressure and moisture under the mask may cause skin breakdown -the mask limits the patient's ability to clear the mouth, especially if vomitting occurs -emotional support decreases anxiety, which contributes to a claustrophobic feeling -use of the cannula prevents hypoxemia during eating

Venturi Mask rationales:

-an accurate flow rate ensures FiO2 delivery -if the venturi orifice is covered, the adaptor does not function and oxygen delivery varies. FiO2 is altered if kinking occurs or if the mask fits poorly.

Aerosol Mask, Face Tent, Tracheostomy Collar nursing interventions:

-assess that aerosol must escapes from the vents of the delivery system during inspiration and expiration -empty condensation water container as needed -change the aerosol water container as needed

Gold Classifications of COPD Severity

0 (At risk) 1 (Mild) 2 (Moderate) 3 (Severe) 4 (Very Severe)

To maintain patient safety, the nurse would question the health care provider about the prescription for prednisone if the patient also had which condition? 1 Systemic fungal infection 2 Diabetes mellitus 3 Congestive heart failure 4 Renal insufficiency

1 RATIONALE: Systemic fungal infection would be a contraindication to the use of prednisone because the drug can interfere with the body's ability to fight infection. While blood sugars may increase and fluid retention may occur, diabetes, congestive heart failure, and renal insufficiency are not absolute contraindications to the use of prednisone, although it should be used with caution.

A patient reporting a soreness in thr throat is diagnosed with strep throat, to prevent complications such as rheumatic heart disease, this patient should receive antibiotic treatment within what time frame

1 week

When admitting a patient with a diagnosis of asthma exacerbation, the nurse will assess for what potential triggers? Select all that apply. 1, Exercise 2, Allergies 3. Emotional Stress 4. Decreased humidity 5. Upper respiratory infections

1, 2, 3, 5 RATIONALE: Although the exact mechanism of asthma is unknown, there are several triggers that may precipitate an attack. These include allergens, exercise, air pollutants, upper respiratory infections, drug and food additives, psychologic factors, and gastroesophageal reflux disease (GERD). Decreased humidity is not a trigger.

During admission of a patient diagnosed with non-small-cell lung carcinoma, the nurse questions the patient related to a history of which risk factors for this type of cancer? Select all that apply 1. Asbestos exposure 2. Exposure to uranium 3. Chronic interstitial fibrosis 4. History of cigarette smoking 5. Geographic area in which the patient was born 6. New Choice

1, 2, 4 RATIONALE: Non-small-cell carcinoma is associated with cigarette smoking and exposure to environmental carcinogens, including asbestos and uranium. Chronic interstitial fibrosis is associated with the development of adenocarcinoma of the lung. Exposure to cancer-causing substances in the geographic area where the patient has lived for some time may be a risk, but not necessarily where the patient was born.

The nurse is caring for a patient who is receiving continuous oxygen (O2) therapy. The nurse knows that the method of O2 administration for the patient depends upon which factors? Select all that apply. 1. Financial resources 2. Patient's cooperation 3. Comfort of the device 4. Patient's cultural status 5. Humidification required 6. Fraction of inspired oxygen (FiO2) required

1,2,3,5,6 RATIONALE: The method of oxygen administration depends upon the patient's fraction of inspired oxygen (FiO2) needed, the mobility status, the level of humidification needed, resources available, level of cooperation, and comfort of the device. The patient's cultural status does not influence the type of O2 device used.

The health care provider requests a computed tomography (CT) scan for a patient suspected of having a mediastinal mass. What should the nurse assess prior to the test? Select all that apply. 1 Blood urea nitrogen (BUN) 2 Serum creatinine 3 Blood sugar levels 4 Hypersensitivity to shellfish 5 Blood hemoglobin levels

1,2,4 RATIONALE The CT scan involves administering a contrast agent; therefore, the nurse should assess the renal function to determine the safety of contrast administration. The renal function can be assessed by checking the laboratory values of BUN and serum creatinine. The contrast agent is usually an iodine-based compound; therefore, the nurse should also check if the patient is hypersensitive to shellfish, as it contains iodine. Checking blood sugar levels and hemoglobin levels are important, but not necessary prior to a CT.

When admitting a 45-year-old female with a diagnosis of pulmonary embolism, the nurse will assess the patient for which risk factors? Select all that apply. 1. Obesity 2. Pneumonia 3. Malignancy 4. Cigarette smoking 5. Prolonged air travel

1,3,4,5 RATIONALE: An increased risk of pulmonary embolism is associated with obesity, malignancy, heavy cigarette smoking, and prolonged air travel with reduced mobility. Other risk factors include deep vein thrombosis, immobilization, surgery within the last three months, oral contraceptives and hormone therapy, heart failure, pregnancy, and clotting disorders. Pneumonia is not a risk factor.

A patient with advanced stage chronic obstructive pulmonary disease (COPD) experiences significant weight loss. The nurse expects that what will be included on the patient's plan of care? Select all that apply. 1. Encourage rest before meals. 2. Encourage high fluid intake with food. 3. Discontinue oxygen therapy while eating. 4. Encourage high-protein supplements between meals. 5. Use steroids and theophylline to relieve bloating

1,4 RATIONALE: Decreased appetite and weight loss occur because of systemic inflammatory processes in COPD. Resting before taking meals helps the patient reduce dyspnea and conserve energy. High-protein supplements can be taken between meals to compensate for the catabolic effects of COPD. Fluid intake should be encouraged between meals rather than with food to prevent distension of the stomach. Supplemental oxygen therapy while eating is beneficial to the patient. Steroids and theophylline cause bloating and do not relieve it.

The nurse is teaching a patient how to use an inhaler. What steps should the nurse teach the patient? Please place selected options in sequential order. 1. Shake the inhaler. 2. Keep breathing in slowly and as deeply as possible. 3. Place the inhaler near or in your mouth as instructed by your health care provider (HCP) 4. Breathe out all the way. 5. As you start breathing in slowly through the mouth, press down on the inhaler one time 6. Hold your breath as you count to 10 slowly.

1,4,3,5,2,6 RATIONALE: Shake the inhaler. Next breathe out all the way and then, while holding the inhaler as prescribed by your HCP, place the inhaler in or near your mouth. Breathe in slowly through the mouth and press down on the inhaler one time. Keep breathing in slowly and as deeply as possible. Then hold your breath while counting slowly to 10.

2 ways oxygen is carried in blood

1. Dissolved in plasma (arterial) 2. Bound to hemoglobin (peripheral)

When performing endotracheal suctioning, the nurse applies suctioning while withdrawing and gently rotating the catheter 360 degrees for which of the following time periods? a) 30 to 35 seconds b) 20 to 25 seconds c) 0 to 5 seconds d) 10 to 15 seconds

10 to 15 seconds In general, the nurse should apply suction no longer than 10 to 15 seconds because hypoxia and dysrhythmias may develop, leading to cardiac arrest. Applying suction for 30 to 35 seconds is hazardous and may result in the patient's developing hypoxia, which can lead to dysrhythmias and, ultimately, cardiac arrest. Applying suction for 20 to 25 seconds is hazardous and may result in the patient's developing hypoxia, which can lead to dysrhythmias and, ultimately, cardiac arrest. Applying suction for 0 to 5 seconds would provide too little time for effective suctioning of secretions.

To reduce the risk for tracheal damage, how much should you keep the cuff pressure between

14 and 20mm hg or 20 and 30 cm H2O

Chest Tube Chambers

1: collects fluid draining from patient 2: Water seal prevents air from reentering patient's pleural space 3: Suction control of system

A patient with an acute exacerbation of chronic obstructive pulmonary disease (COPD) needs to receive precise amounts of oxygen. Which equipment should the nurse prepare to use? 1 Oxygen tent 2 Venturi mask 3 Nasal cannula 4 Oxygen-conserving cannula

2 The Venturi mask delivers precise concentrations of oxygen and should be selected whenever this is a priority concern. An oxygen tent is not required for a patient with COPD. A nasal cannula delivers a less-precise amount of oxygen. An oxygen-conserving cannula is not appropriate for this patient. Text Reference - p. 590

Which test result identifies that a patient with asthma is responding to treatment? 1.An increase in CO2 levels 2.A decreased exhaled nitric oxide 3.A decrease in white blood cell count 4.An increase in serum bicarbonate levels

2 RATIONALE: Nitric oxide levels are increased in the breath of people with asthma. A decrease in the exhaled nitric oxide concentration suggests that the treatment may be decreasing the lung inflammation associated with asthma and adherence to treatment. An increase in CO2 levels, decreased white blood cell count, and increased serum bicarbonate levels do not indicate a positive response to treatment in the asthma patient.

A patient needs to receive oxygen at a 28% concentration. The nurse will set up which oxygen delivery device? 1 Nasal cannula 2 Venturi mask 3 Simple face mask 4 Nonrebreather mask

2 RATIONALE: The Venturi mask is a high-flow device that delivers fixed concentrations of oxygen (e.g., 24%, 28%), independent of the patient's respiratory pattern). Nasal cannulas, simple face masks, and nonrebreather masks are examples of low-flow devices that deliver less precise concentrations of oxygen

A patient presents to the outpatient clinic with nasal trauma. On examination, the nurse finds localized pain, edema, epistaxis, and crepitus on palpation. The patient does not have nasal congestion or difficulty in breathing. Which conditions should the nurse suspect? 1.Acute sinusitis 2. Nasal fracture 3. Allergic rhinitis 4. Deviated nasal septum

2 RATIONALE: Trauma to the nose is the major cause of nasal fracture. It is manifested as localized pain, edema, epistaxis, and crepitus on palpation. Acute sinusitis presents with pain over the affected sinuses, purulent nasal discharge, and fever. Allergic rhinitis presents with sneezing, itchy eyes and nose, and watery nasal discharge. Deviated nasal septum presents with obstructed nasal breathing and nasal congestion. TEST-TAKING TIP Note the number of questions and the total time allotted for the test to calculate the times at which you should be halfway and three-quarters finished with the test. Look at the clock only every 10 minutes or so.

A patient diagnosed with acute respiratory distress syndrome is being mechanically ventilated with 12 cm of positive-end expiratory pressure (PEEP). Upon assessment, the nurse notes deterioration of vital signs and absent breath sounds in the right lung field. What is the most likely cause for this finding? 1 Deterioration of the disease 2 Pneumothorax 3 Decreased cardiac output 4 Obstructed endotracheal tube

2 A complication of PEEP may be a pneumothorax as a result of overdistention of the alveoli. If deterioration of the disease were the cause, both lung sounds would be decreased equally. Decreased cardiac output would affect vital signs, but not breath sounds. An obstructed endotracheal tube would affect both lung fields. Text Reference - p. 1663

A patient had an intradermal tuberculin skin test (Mantoux) administered 48 hours ago. The nurse assesses the injection site and identifies a 12-mm area of palpable induration. How should the nurse interpret this result? 1.Definitive evidence that the patient does not have tuberculosis 2. A significant indication that the patient has been exposed to tuberculosis 3. Delayed hypersensitivity with a high likelihood of infection with tuberculosis 4. A negative test that cannot be interpreted as ruling out the presence of tuberculosis

2 RATIONALE: An area of 12 mm of induration at the injection site 48 hours after a Mantoux test is considered significant for a past or current tuberculin infection. An induration of less than 5 mm is considered a negative result. The other answer options are incorrect conclusions related to the findings.

A patient has an initial positive PPD (purified protein derivative) skin test result. A repeat PPD's result is also positive. No other signs or symptoms of tuberculosis or allergies are evident. Which medication(s) does the nurse anticipate will be prescribed? 1 Penicillin 2 Isoniazid (INH) 3 INH and an antibiotic 4 Theophylline (Theo-Dur)

2 RATIONALE: Isoniazid (INH) is the most commonly initially prescribed medication to treat patients exposed to tuberculosis (TB). Penicillin and theophylline would not be prescribed for treatment of TB exposure. INH plus an antibiotic would not likely be prescribed for this scenario.

The nurse caring for a patient with diabetes mellitus and chronic obstructive pulmonary disease (COPD) on oral prednisone (Deltasone) will monitor which parameter regularly? 1. Blood pressure 2. Blood sugar 3. Hemoglobin A1c 4. Bowel Sounds

2 RATIONALE: Corticosteroids such as prednisone can lead to elevated blood sugar, especially for patients with diabetes. For this reason, it is useful to monitor the patient's blood sugar. The patient's blood pressure and bowel sounds will not be affected. The hemoglobin A1c will demonstrate average blood sugars over the past three months, which would not evaluate blood sugar since beginning prednisone.

The nurse is caring for a patient with respiratory failure. What measures can the nurse take to prevent the development of stress ulcers in the patient? Select all that apply. 1. Use of parenteral feedings 2. Initiate early enteral nutrition 3. Use smaller tidal volumes 4. Use antiulcer agents such as proton pump inhibitors. 5. Use measures to prevent development of hypotension and shock.

2, 4,5 RATIONALE: Early initiation of enteral nutrition helps to prevent mucosal damage. Prophylactic management includes antiulcer agents such as proton pump inhibitors. Stress ulcer management strategies include correction of predisposing conditions such as hypotension, shock, and acidosis. Use of smaller tidal volumes does decrease the risk of volutrauma but does not affect stress ulcers.

patient with cystic fibrosis is being treated by the nurse with chest physiotherapy for the left upper lobe (LUL) of the lung. In which order should the nurse perform the following actions? Put a comma and space between each answer choice (1, 2, 3, 4). 1.Effective coughing with expectoration 2.Albuterol (Proventil) nebulizer therapy 3.Application of percussion and vibration 4.Position patient on right side with head up

2,1,3,4 RATIONALE: Chest physiotherapy for the LUL begins with a short-acting bronchodilator to facilitate ventilation during therapy. After bronchodilator use, the patient is positioned on the right side with the head up to expose the LUL for treatment and drainage. Percussion and vibration are performed to loosen secretions. Then the nurse should assist the patient with coughing effectively and expectorating the thick secretions.

The health care provider requests arterial blood gases (ABGs) for a patient. While performing the test, what should the nurse tell the patient about the test? Select all that apply. 1 The test is done by using pulse oximetry. 2 The test is done to assess changes in acid-base balance. 3 The test is done to assess changes in arterial oxygen (PaO2). 4 The test is done to assess changes in arterial carbon dioxide (PaCO2). 5 The test is done to assess changes in blood urea nitrogen and serum creatinine.

2,3,4 RATIONALE: ABGs are used to assess changes in pH (acid status), PaO2, PaCO2, bicarbonate (base status), and saturation (SaO2). These parameters indicate the oxygen saturation and pH values in arterial blood. Pulse oximetry is used intermittently or continuously to assess arterial oxygen saturation (SpO2) only and does not provide information about other arterial blood gases. ABGs do not provide information regarding blood urea nitrogen and serum creatinine.

A patient with bacterial pharyngitis is prescribed penicillin. The nurse knows which of the following about this drug? Select all that apply. 1. Is given intravenously 2. Has to be taken for 10 days 3. Is taken several times a day 4. Is taken to prevent rheumatic fever 5. Helps to control accompanying fever 6. Prevents transmitting the disease within 12 hours of taking the first dose

2,3,4 RATIONALE: Oral penicillin needs to be taken several times a day for 10 days to prevent rheumatic fever, which is a sequela to the infection. Patients must take the drug orally for 24-48 hours before they are considered noncontagious. It has no effect on fever.

A nurse is caring for a patient with bronchitis. When reviewing the medical records, the nurse finds that the patient has decreased lung compliance. What are the conditions in which lung compliance decreases? Select all that apply. 1 Chronic obstructive pulmonary disease (COPD) 2 Pneumonia 3 Pulmonary edema 4 Pleural effusion 5 Sarcoidosis

2,3,4,5 RATIONALE: Lung compliance is a measurement of the ease with which the lungs expand. It decreases in cases of pneumonia, pulmonary edema, and pleural effusion because fluid builds up in the lungs. Lung compliance also decreases when lung tissue becomes less elastic, as in sarcoidosis. In COPD, lung compliance increases, since there is destruction of the alveolar walls and loss of tissue elasticity.

The patient has a prescription for albuterol 5 mg via nebulizer. Available is a solution containing 2 mg/mL. Calculate how many milliliters the nurse should use to prepare the patient's dose. Fill in the blank. __mL

2.5 RATIONALE: 5 mg ÷ 2 mg/mL = 2.5 mL

Room air % of oxygen

21%

How much oxygen does one need to survive?

21% on room air

HCO3

22-26

Nasal Cannula FiO2 delievered:

24% at 1 L/min 28% at 2 L/min 32% at 3 L/min 36% at 4 L/min 40% at 5 L/min 44% at 6 L/min

Aerosol Mask, Face Tent, Tracheostomy Collar FiO2 delivered:

24-100% FiO2 with flow rates of at least 10 L/min; provides high humidity.

Which instructions should the nurse include when teaching self-care to a patient with acute pharyngitis? Select all that apply. 1 Drink citrus juices. 2 Restrict fluid intake. 3 Gargle with warm salt water. 4 Suck on popsicles or hard candies. 5 Use a cool-mist vaporizer or humidifier

3,4,5 RATIONALE: Symptom relief is a major goal of nursing management in a patient with acute pharyngitis. The nurse should instruct the patient to gargle with warm salt water, suck on popsicles or hard candies, and use a cool-mist vaporizer or humidifier. Citrus juices can irritate the throat and should not be recommended. The patient should increase fluid intake to keep the secretions thin so they can be easily expectorated.

How much does 1 liter equal?

3-4%

Which patient is at highest risk of developing pneumonia

32 year old trauma patient on a mechanical ventilator

paCO2

35-45

PO2

35-46

Of these patients waiting for an influenza immunization, which one would be eligible to receive the live attenuated vaccine (Flumist) instead of the trivalent inactivated vaccine (Fluzone)?

35-year-old with allergies

The patient with asthma asks the nurse, "How will I know when my inhaler is empty?" What is the best response by the nurse? 1. "There will be no sound when shaking the canister." 2. "The canister will float in water." 3. "Your wheezing will not improve despite inhaler use." 4. "You need to keep track of how many puffs you have used and how many puffs are available."

4 RATIONALE: Show the patient how to determine how many puffs are available in the canister. Then teach the patient to document each time a puff is used. It is important to teach the patient this simple way to check the inhaler. The canister may or may not produce a sound when shaking. Floating the canister in water used to be recommended, but is no longer recommended because water can enter the chamber. It is not reasonable to wait for the patient's wheezing to worsen before getting a new inhaler.

The nurse is caring for a patient with a non-massive pulmonary embolism (PE). What is the best standard for treatment? 1. Tissue plasminogen activator (tPA) 2. Alteplase (Activase) 3. Warfarin (Coumadin) 4. Enoxaparin (Lovenox)

4 RATIONALE: Subcutaneous administration of low-molecular-weight heparin has been found to be safer and more effective than use of unfractionated heparin. It is also the recommended choice of treatment for patients with non-massive PE. Criteria for fibrinolytic therapy in PE include hemodynamic instability and right ventricular dysfunction. Fibrinolytic therapy dissolves pulmonary embolisms and the source of the thrombus. Low molecular weight heparin (Lovenox) is becoming more common for non-massive pulmonary embolism. Warfarin should be initiated within the first 24 hours s of heparinization and is typically administered for three to six months. Warfarin therapy is the standard treatment for non-massive PE.

Which statement about a deviated septum is correct? 1. All septal deviations are symptomatic 2. A septoplasty will not correct a deviated septum 3. Epistaxis is a rare manifestation of a deviated septum 4. The aim of medical management is nasal allergy contro

4 RATIONALE: The aim of medical management is symptom control because medical management of a deviated septum is focused on symptom control of nasal inflammation and congestion. All septal deviations are symptomatic, a septoplasty will not correct a deviated septum, and an epistaxis is a rare manifestation of a deviated septum are incorrect because minor septal deviations are typically asymptomatic, septoplasty will properly align a deviated septum, and epistaxis is one of many common manifestations of septal deviation.

The patient has decided to use the voice rehabilitation that offers the best speech quality even though it must be cleaned regularly. The nurse knows that this is what kind of voice rehabilitation? 1. Electromyograph 2. Intraoral electrolarynx 3. Neck type electrolarynx 4. Transesophageal puncture

4 RATIONALE: The transesophageal puncture provides a fistula between the esophagus and trachea with a one-way valved prosthesis to prevent aspiration from the esophagus to the trachea. Air moves from the lungs, vibrates against the esophagus, and words are formed with the tongue and lips as the air moves out the mouth. The electromyography and both electrolarynx methods produce low-pitched mechanical sounds.

The nurse is caring for a patient with a pulmonary artery (PA) catheter. The nurse knows that this type of catheter is used to obtain: 1. Mixed pulmonic vein blood gas sampling 2. Mixed atrial blood gas sampling 3. Mixed arterial blood gas sampling 4. Mixed venous blood gas sampling

4 RATIONALE: PA catheters are used to sample mixed venous blood gases. It consists of venous blood that has returned to the heart and mixes in the right ventricle. This sampling helps to assess the presence of inadequate tissue oxygen delivery or abnormal oxygen consumption in a patient with impaired cardiac output or hemodynamic instability. It does not assess blood from the pulmonic vein. It is not arterial blood and not blood from the atrial chamber.

A patient is learning to use a metered-dose inhaler (MDI). Which statement by the patient indicates the need for additional teaching? 1 "I will shake the canister before use." 2 "I will hold my breath for as long as I can after inhalation." 3 "I should wait at least one minute between puffs of medication." 4 "I should avoid using a spacer with this inhaler because it is ineffective."

4 RATIONALE: A spacer should be avoided with dry powder inhalers; they are helpful to use with MDIs. Metered dose inhalers require shaking of the canister, the breath should be held to increase absorption of the medication into the lungs, and the patient should wait one to two minutes between puffs.

Simple Facemask FiO2 delievered:

40% at 5 L/min 45%-50% at 6 L/min 55-60% at 8 L/min

A client is scheduled to receive 1000 mL of normal saline in 24 hours. The nurse should set the infusion pump to deliver how many milliliters per hour? _____________ mL/hr

42 1000 mL divided by 24 hours = 41.6 mL/hr

A nurse knows that under normal physiologic conditions of tissue perfusion, a patient will have what percent of oxygen dissociate from the hemoglobin molecule?

50%

The nurse knows that under normal physiologic conditions of tissue perfusion, a patient will have what percent of oxygen dissociate from the Hemoglobin molecule?

50%

Right lung produces

60-65% of oxygen Cancer tumors are mostly here

Partial Rebreather Mask

60-75% with flow between 6-11 L/min

pH

7.35-7.45

Normal PaO2 (arterial blood gas)

80-100mmHg

What is O2 sat goal for COPD?

88-92%

The nurse is providing instructions on safety measures to a patient who has a laryngectomy stoma. Which action of the patient prevents crusting of the laryngectomy stoma?

A laryngectomy stoma is a hole made in the neck during surgery that the patient can breathe through. Applying a nasal wash spray around the stoma such as Alkalol every one to two hours will keep the stoma moist and prevent it from crusting. Covering the stoma while coughing or during activities like shaving or applying makeup prevents the entry of foreign particles. Washing the area around the stoma with a moist cloth will clean any secretions. Wearing a plastic cloth while taking a shower will prevent the entry of water into the stoma.

intraosseous cath

A man with severe burns over 90% of his body has been brought to the ED. Rescue personnel were unable to establish IV access during transport to hopsital. What type of IV device would be most appropriate?

When mechanical ventilation is not required

A noncuffed tube is used

What are the changes in a patient after undergoing a total laryngectomy? Select all that apply.

A total laryngectomy is the removal of the larynx. It is associated with loss of speech, taste, and smell. A total laryngectomy does not disturb the vision or physical mobility in the patient.

Which factors will the nurse consider when calculating the CURB-65 score for a patient with pneumonia (select all that apply)? A. Age Correct B. Blood pressure Correct C. Respiratory rate Correct D. Oxygen saturation E. Presence of confusion Correct F. Blood urea nitrogen (BUN) level Correct

A. Age B. Blood pressure C. Respiratory rate E. Presence of confusion F. Blood urea nitrogen (BUN) level

A patient with acute dyspnea is scheduled for a spiral computed tomography (CT) scan. Which information obtained by the nurse is a priority to communicate to the health care provider before the CT? A. Allergy to shellfish B. Apical pulse of 104 C. Respiratory rate of 30 D. Oxygen saturation of 90%

A. Allergy to shellfish

A patient who is experiencing an acute asthma attack is admitted to the emergency department. Which assessment should the nurse complete first? A. Listen to the patient's breath sounds. B. Ask about inhaled corticosteroid use. C. Determine when the dyspnea started. D. Obtain the forced expiratory volume (FEV) flow rate.

A. Listen to the patient's breath sounds.

A nurse working in long-term care is assessing residents at risk for the development of a decubitus ulcer. Which one would be most at risk? Select one: A. an 86-year-old who is bedfast B. a 92-year-old who uses a walker C. an 83-year-old who is mobile D. a 75-year-old who uses a cane

A. an 86-year-old who is bedfast Most pressure ulcers occur in older adults as a result of a combination of factors, including aging skin, chronic illness, immobility, malnutrition, fecal and urinary incontinence, and altered level of consciousness. The bedfast resident would be most at risk in this situation.

A 58-year-old woman who has been diagnosed with throat cancer 1 week ago comes to the clinic today to discuss surgical options with her health care provider. She is very tearful and appears sad when the nurse calls her back to the examination room. Based on her diagnosis, which clinical manifestation will the nurse likely observe in the patient? A. Hoarseness B. Severe chest pain C. Low hemoglobin level (anemia) D. Numbness and tingling of the face

A. b/c the cancer will obstruct the airway

A physician has ordered that a medication be given "stat" for a patient who is having an anaphylactic drug reaction. At what time would the nurse administer the medication? Select one: A. immediately after the order is noted B. whenever the patient asks for it C. not until verifying it with the patient D. at the next scheduled medication time

A. immediately after the order is noted A stat order is a single order, and it is carried out immediately. This is a legal order. The nurse would not wait until the next scheduled medication time or verify the order with the patient. With a p.r.n. order, the patient receives medication when it is requested or required.

Labs for COPD

ABG Sputum samples CBC hgb and hct serum electrolytes serum aat cxr pulmonary function test

Assessment of Oxygenation

ABG analysis is best way to determine need for oxygen therapy.

Venturi Mask

Adaptor located between bottom of mask and O2 source Great for COPD patients

Venturi Mask

Adaptor located between bottom of mask and O2 sources Delivers precise O2 concentration—best device for chronic lung disease Switch to nasal cannula during mealtimes

The nurse is caring for a patient with complaints of allergic nasal rhinitis. The nurse knows that the treatment includes what?

Adequate fluid intake is essential when the patient is taking antihistamines. Antibiotics are not appropriate for allergic rhinitis. Second-generation antihistamines are preferred over first-generation ones because of their nonsedating effects. Initially monotherapy is prescribed, and if that is not effective, then a two-drug combination may help.

A patient hospitalized for pneumonia has the priority patient problem of ineffective airway clearance related to fatigue, chest pain, excessive secretions and muscle weakness. What nursing intervention helps to correct these problems

Administering respiratory therapy in a timely manner to decrease bronchospasms

The nurse is conducting an in service for the hosptial staff about practices that help prevent pneumonia among at risk patients, which nursing intervention is encouraged as standard practice

Administering vaccines to patients at risk

Subcutaneous Emphysema

Air present in the subcutaneous tissue

The nurse is caring for an older adult with Alzheimer's disease. The nurse provides meticulous oral care to prevent which condition?

Airway obstruction Patients who have an altered mental state are at risk for thickening and hardening of oral secretions, or inspissated secretions, that can block the airway and lead to death. Oral hygiene can prevent this. Inspissated secretions do not cause laryngeal edema, pharyngeal abscess, or pharyngitis.

Alveolitis

Alveolar inflammation Occurs from the presence of immune cells in the alveoli

Alveoli change due to age-physiologic change

Alveolar surface area decreases Diffusion capacity decreases Elastic recoil decreases Bronchioles and alveolar ducts dilate Ability to cough decreases Airways close early

NURSING SAFITY PRIORITY: When cappig the tube

Always deflate the cuff before capping the tube with the decannulation cap; otherwise, the patient has no airway.

The nurse is caring for a pregnant patient who has a coccidioidomycosis fungal infection, which is also known as "valley fever." What medication would most likely be ordered for this patient?

Amphotericin B IV Patients with valley fever who are pregnant or have a severe infection will receive amphotericin B IV. Non-pregnant patients with mild infection may receive fluconazole, ketoconazole, or voriconazole PO. Amoxicillin is an antibiotic and is not used to treat fungal infections.

Which people are at greatest risk for developing TB in the US

An alcoholic homeless man who occasionally stays in a shelter, a person with immune dysfunction or HIV, foreign immigrants

Tracheostomy

An opening in the trachea created by a surgical incision for the purpose of establishing an airway

Chest wall change to age-physiologic change

Anteroposteior diameter increases. Thoraz becomes shorter. Progessive kyphoscoliosis occurs. Chest wall compliance (elasticity) decreases. Mobility of chest wall may decrease. Osteoporosis is possible, leading to chest wall abnormalities.

The nurse is reviewing home care instructions for a patient diagnosed with acute viral rhinitis. Which medication order does the nurse question?

Antibiotic

An 80 year old man has a diagnosis of chronic rhinitis. Which statement is relevant to the care of the patient

Antihistamines and decongestants should be used with caution

How to apply suctioning

Apply it intermittently for only 10 to 15 seconds. After 15 seconds, end the suctioning procedure.

Stroke patients are at high risk for pneumonia related to what condition?

Aspiration

The nurse understands which is the primary risk factor for lung cancer? Air pollution Cigarette smoking Chronic exposure to asbestos Occupational radiation exposure

B

The nurse develops a plan of care to prevent aspiration in a high-risk patient. Which nursing action will be most effective? A. Turn and reposition immobile patients at least every 2 hours. B. Place patients with altered consciousness in side-lying positions. C. Monitor for respiratory symptoms in patients who are immunosuppressed. D. Insert nasogastric tube for feedings for patients with swallowing problems.

B. Place patients with altered consciousness in side-lying positions.

An experienced nurse instructs a new nurse about how to care for a patient with dyspnea caused by a pulmonary fungal infection. Which action by the new nurse indicates a need for further teaching? A. Listening to the patient's lung sounds several times during the shift B. Placing the patient on droplet precautions and in a private hospital room. C. Increasing the oxygen flow rate to keep the oxygen saturation above 90% D. Monitoring patient serology results to identify the specific infecting organism

B. Placing the patient on droplet precautions and in a private hospital room.

A nurse is administering a liquid medication to an infant. Where will the nurse place the medication to prevent aspiration? Select one: A. under the tongue B. between the gum and the cheek C. on the front of the tongue D. in front of the teeth and gums

B. between the gum and the cheek A dropper is used to give infants or very young children liquid medications while holding them in a sitting or semisitting position. The medication is placed between the gum and the cheek to prevent aspiration.

An elderly woman in a long-term care facility has fallen and sustained several injuries. Which of her injuries would be the most serious fall-related injury? Select one: A. lacerated lip B. fractured hip C. fractured ulna D. thigh contusion

B. fractured hip Falls can occur at any age, but a large percentage of elderly adults in long-term settings suffer a fall. Hip fractures are among the most serious fall-related injuries.

The patient has been diagnosed with head and neck cancer. Along with the treatment for the cancer, what other treatment should the nurse expect?

Because 50% of patients with head and neck cancer are malnourished before treatment begins, many patients need enteral feeding via a gastrostomy tube because the effects of treatment make it difficult to take in enough nutrients orally, whether surgery, chemotherapy, or radiation are used. Nasal packing could be used with epistaxis or with nasal or sinus problems. Peripheral skin care would not be expected because it is not related to head and neck cancer.

Laboratory Tests

Blood Sputum Standard chest x-rays, digital chest radiography, CT Ventilation and perfusion scan Pulse oximetry (noninvasive)

Lab tests

Blood Sputum CXR Ventilation and perfusion scan Pulse oximetry

How to preven thypoxia

By hyperoxygenating the patient with 100% oxygen with a manual resuscitation bag attached to an oxygen source.

While suctioning a patient, vagal stimulation occurs. What is the appropriate nursing action? Instruct the patient to cough and deep breathe. Place the patient in a high Fowler's position. Stop suctioning and oxygenate the patient with 100% oxygen. Instruct the patient to breathe slowly and deeply.

C.

Which of following statements most accurately captures the role of chronic illness in the lives of older adults? Select one: A. While chronic diseases used to be the leading cause of death, this is no longer the case. B. More older adults die from acute illnesses than from chronic diseases. C. Chronic illnesses constitute the leading cause of death for older adults. D. While cancer rates have fallen, other chronic diseases remain a common cause of death.

C. Chronic illnesses constitute the leading cause of death for older adults. Chronic illnesses constitute the leading cause of death for older adults, exceeding those attributed to acute illnesses. The presence of heart disease as a cause of death has decreased in recent years, while at the same time cancer has become more prevalent.

The nurse is caring for a patient with idiopathic pulmonary arterial hypertension (IPAH) who is receiving epoprostenol (Flolan). Which assessment information requires the most immediate action by the nurse? A. The oxygen saturation is 94%. B. The blood pressure is 98/56 mm Hg. C. The patient's central IV line is disconnected. D. The international normalized ratio (INR) is prolonged.

C. The patient's central IV line is disconnected.

Which anatomic site is recommended for intramuscular injections for adults? Select one: A. subcutaneous fat B. vastus lateralis C. ventrogluteal muscles D. epidermis of inner forearm

C. ventrogluteal muscles The ventrogluteal site involves the gluteus medius and gluteus minimus muscles in the hip area. This site is recommended for adults because there are no large nerves or blood vessels, it is removed from bone tissue, it is clean, and the patient may lie on the back, abdomen, or side for the injection.

How do you tell the difference between normal nasal secretions versus CSF?

CSF dries with a yellow glucose halo

Obstructive Sleep Apnea nonsurgical management

Change of sleep position, weight loss, positive-pressure ventilation

An older adult resident in a long-term care facility becomes confused and agitated, telling the nurse "Get out of here! You're going to kill me!" Which action will the nurse take first?

Check the resident's oxygen saturation

The nurse is caring for a patient following a thoracotomy. Which of the following findings requires immediate intervention by the nurse? a) Pain of 5 on a 1 to 10 pain scale b) Heart rate: 112 bpm c) Moderate amounts of colorless sputum d) Chest tube drainage of 190 mL/hr

Chest tube drainage of 190 mL/hr The nurse should monitor and document the amount and character of drainage every 2 hours. The nurse will notify the primary provider if drainage is 150 mL/hr or greater. The other findings are normal following a thoracotomy; no intervention is required.

A patient with severe chronic obstructive pulmonary disease (COPD) tells the nurse, "I wish I were dead! I'm just a burden on everybody." Based on this information, which nursing diagnosis is most appropriate?

Chronic low self-esteem related to physical dependence

A patient with COPD needs instruction on the measures to prevent pneumonia. What information does the nurse include

Clean all respiratory equipment you have at home

Goals for Patients with Pneumonia

Clear Breath sounds normal breathing patterns no signs of hypoxia normal cxr no complications related to pneumonia

The nurse is counseling a young woman about drug therapy with isoniazid and rifampin to treat tuberculosis. Before developing the teaching plan, what must the nurse assess for first?

Contraceptive methods used Rifampin can interfere with oral contraceptives, and women using these should be taught to use a backup method of contraception during treatment and up to 1 month after treatment ends. Ethambutol can have effects on vision, including color vision, but isoniazid and rifampin do not. Other drugs can increase the risk of gout. Pyrazinamide can cause increased sensitivity to sunlight.

Anti-inflammatory Agents

Corticosteroids NSAIDs Leukotriene Antagonists Immunomodulators

The nurse understands which symptom to be a hallmark objective sign of lung disease?

Cough

The nurse understands which symptom to be hallmark subjective sign of lung disease?

Cough

Non-invasive ways of assisting patients with breathing

Cough, Early Ambulation, Positioning

A patient is admitted to the hospital for treatment of pneumonia. Which nursing assessment finding best indicates that the patient is responding to antibiotics

Cough, clear sputum, temperature of 99F, pulse ox at 96% on room air

Adventitious sounds

Crackles Wheezes Rhonchi Pleural friction rub

20

DO NOT GIVE POTASSIUM AT A RATE GREATER THAN ____ mEq/hr

A patient with acute viral rhinitis has asked the nurse about medications that can be taken to relieve symptoms. Which statement by the patient reflects a need for further instruction?

Decongestant sprays should not be used for more than three days to prevent rebound nasal congestion. Antibiotics may not be effective if the patient still has the cold. Saline nasal spray can be used to relieve congestion. The patient should not drive if taking an antihistamine.

The nurse understands that the expected assessment for the older adult related to the natural aging process of the respiratory system includes which finding?

Decrease in respiratory muscle strength

Partial Rebreather

Delivers O2 60-75% with flow rate of 6-11 L/min One-third exhaled tidal volume with each breath Adjust flow rate to keep reservoir bag inflated

Simple Facemask

Delivers O2 up to 40-60% Minimum of 5L/min Mask fits securely over nose and mouth Monitor closely for risk of aspiration

Simple Facemask

Delivers up to 40-60% At minimum of 5 L/min

Which statement about pharyngitis is correct?

Development of stridor or indications of airway obstruction should be considered a medical emergency!

Yes for IV site for emergency but not for maintenance (not everyone is on fluids

Does every patient in the hospital need and IV?

Low-Flow Oxygen Delivery Systems

Does not provide enough flow to meet total oxygen and air volume Nasal cannula (1-6 L) Facemask Simple Partial rebreather Non-rebreather

Asthma Therapy

Drugs Exercise Supplemental Oxygen

Control Therapy Drugs

Drugs used to reduce airway sensitivity to prevent asthma attacks from occurring

Xerostomia

Dry mouth

Posterior Nasal Bleeding

Emergency! Can cause rapid blood loss and it is harder to see

COPD

Emphysema and Chronic bronchitis. Characterized by bronchospasm and dyspnea. Tissue damage is not reversible; increases in severity, eventually leads to respiratory failure

Steatorrhea

Excessive fat in stools

The nurse is assessing a patient who underwent nasoseptoplasty 24 hours ago. Which finding requires immediate intervention by the nurse?

Excessive swallowing Excessive swallowing in a patient who has undergone a nasoseptoplasty may indicate posterior nasal bleeding and requires immediate attention. Because of the very vascular nature of the face, ecchymosis is a normal finding in the patient who has undergone a nasoseptoplasty. Edema is a normal reaction to any kind of trauma, including that caused by surgery, so it is not an unexpected finding for this patient. A sore throat is a common side effect of endotracheal intubation.

Other noninvasive testing

Exercise testing and skin testing

The abdominal thrust maneuver (formerly known as the Heimlich maneuver) for acute airway obstruction attempts to achieve which outcome?

Expel the air remaining in the victim's lungs The quick inward motion of the abdominal thrust maneuver expels the air remaining in the victim's lungs, and with it, the foreign body causing the obstruction.

A client who is taking rimfampin tells the nurse that his urine looks orange, what action should the nurse take

Explain this is expected

Which condition is suspected in a patient who is exposed to sun and asbestos?

Exposure to sun and asbestos are risk factors for head and neck cancer. Rhinitis is suspected when the patient is exposed to animal dander and house molds. Sinusitis is suspected when the patient has inflammation of the mucosa that blocks the opening to the sinuses. Influenza is suspected when the patient is exposed to an influenza virus.

True or False: Flammable solutions containing high concentrations of alcohol or oil should not be used in rooms with oxygen. Therefore, hand hygiene using alcohol-based foams or gels should be avoided when caring for patients on oxygen therapy. True False

False

Relevant Patient History

Family and personal data Smoking (pack-years) Drug use Allergies Travel, geographic area of residence Nutritional status Cough, sputum production, chest pain, dyspnea, PND, orthopnea

Which assessment finding in a patient who has received omalizumab (Xolair) is most important to report immediately to the health care provider?

Flushing and dizziness

A nurse caring for a patient post-laryngectomy begins to suction the patient. The nurse notes that the mucous is blood-tinged. What is the most appropriate nursing action?

Following a laryngectomy, it is important to maintain a patent airway, which can be done by regularly suctioning through a tracheostomy. The nurse should continue to monitor the patient. Initially the mucus will be blood tinged, so there is no need to notify the surgeon. A saline bolus should not be given through tracheostomy because this can block the airway and cause asphyxia. A change of position is not necessary.

A patient has undergone nasal surgery. Which finding indicates a safety priority to the nurse?

Frequent swallowing Frequent swallowing may indicate a posterior nasal bleed that will require intervention by the surgeon. Restlessness may be present, especially if the patient is in pain. Excessive sedation may be related to residual anesthesia effects or pain medications. Nausea and vomiting, while important to treat, are not the safety risk the posterior bleed would be. Good care involves all of these aspects; however, the initial focus should be on the potential presence of bleeding.

A newly admitted patient with pneumonia has an oral temperature of 102° F, an oxygen saturation of 93%, diminished breath sounds bilaterally, and the patient is unable to cough effectively. The nurse has received orders for oxygen therapy, intravenous antibiotics, antipyretic medication, and sputum specimen collection. What is the nurse's first action?

Give IV antibodics

A patient who had sinus surgery has a surgical incision under the lip. The nurse intervenes when a well intentioned family member performs which action in attempt to make the patient feel better

Hands the patient a tissue to blow the nose

Labs/Vital signs used to measure a person's perfusion

HbG, Hct, CBC and SpO2

Non-Rebreather Mask

Highest O2 level Can deliver FIO2 greater than 90% Used for unstable patients requiring intubation Ensure valves are patent and functional

Community-Based Care

Home care management Teaching for self-management: Stoma care Communication Smoking cessation Psychosocial preparation Health care resources

The nurse is caring for a patient who is receiving an inspired oxygen concentration of more than 60%. What value of arterial O2 (PaO2) would indicate hypoxemic respiratory failure? Record your answer using a whole number.

Hypoxemic respiratory failure is commonly defined as a PaO2 less than 60 mm Hg when the patient is receiving an inspired oxygen concentration of 60% or more. This definition incorporates two important concepts. First, the PaO2 level indicates inadequate oxygen saturation of hemoglobin; second, this PaO2 level exists despite administration of supplemental oxygen at a percentage (60%) that is about three times that in room air (21%).

Complications of Suctioning

Hypoxia Tissue (mucosal) trauma Infection Vagal stimulation, bronchospasm Cardiac dysrhythmias from induced hypoxia

INH is prescribed as a prophylactic measure for a client whose spouse has active TB, what statement by the client indicate there is a need for further teaching

I sometimes allow our children to sleep in our bed at night, I know I also have TB because the skin test was positive, I will be skipping wine but enjoying the cheese at my neighbors party

The nurse is teaching a patient about the combination drug therapy that is used in the treatment of TB, which patient statement indicates the nureses instructions were effective

I will take 3 drugs, isoniazid, rimfampin and pyrazinamide then ethambutol may be added later

Le Fort Fracture

I — Nasoethmoid complex fracture II — Maxillary and nasoethmoid complex fracture III — Combination of I & II plus orbital-zygoma fracture; often called craniofacial disjunction

Goals for COPD

Improve oxygenation and reduce carbon dioxide retention prevent weight loss minimize anxiety improve activity tolerance prevent respiratory infection

Lungs change due to age-nursing interventions

Include inspectuin, palpatation, percussion, and auscultation in lung assessments. Help patient actively maintain healht and fitness. Assess patient's respirations for abnormal breathing patterns. Encourage frequent oral hygiene.

Chronic Obstructive Pulmonary Disease (COPD)

Includes: emphysema, chronic bronchitis Characterized by bronchospasms and dyspnea Tissue damage not reversible, increases in severity, eventually leads to respiratory failure

An elderly patient was diagnosed with influenza one week ago. What should the nurse include in the teaching plan for the patient? Select all that apply.

Increase fluid intake Use appropriate hand-washing techniques. Avoid the use of diphenhydramine

Upper Airway Obstruction

Interruption in airflow through nose, mouth, pharynx, or larynx Life-threatening emergency Early recognition essential in preventing further complications, including respiratory arrest

Lung Biopsy

Invasive Obtain tissue for histologic analysis, culture, cytologic examination May be performed in patient's room

A nurse is caring for a patient who is orally intubated and mechanically ventilated. The nurse understands that this patient is at an increased risk for developing ventilator associated pneumonia. When planning care for this patient, what pathophysiological concepts regarding an artificial airway does the nurse recognize as a contributing factor to the development of this condition? Select all that apply.

It bypasses the protective airway mechanisms It allows aspiration of secretions from the oropharynx and stomach

The radiology report of a client who has had a chest x-ray shows consolidation in a segment of the client's left lung. This is typical of which type of pneumonia?

Lobar

Emphysema

Loss of lung elasticity and hyperinflation Dyspnea Air gets trapped due to loss of recoil in alveolar walls Collapse of small airways (bronchioles)

Hypoxemia

Low levels of oxygen in the blood

A patient taking antibiotics to treat rhinosinusitis reports facial pain over the affected sinuses. Which comfort measure does the nurse suggest in addition to the antibiotic therapy?

Moist heat packs over the affected sinuses Moist heat packs over the sinuses can alleviate some discomfort. Decongestant medications are also indicated. Frequent nose-blowing is not recommended. Patients should be taught to avoid placing the sinuses in a dependent position.

Granulomas

Noncancerous inflammatory growths

Complications: Tube Obstruction Nursing Interventions!:

Obstruction can occur from secretions or cuff displacement. Nursing interventions: -Assist patient to cough and deep breathe -Provide inner cannula care -Humidify the oxygen source to provide moisture and prevent hardened secretions -Suction as needed If tube obstruction occurs as a result of cuff prolapse, contact physician or advanced practice nurse

The nurse is discussing activity management with a patient who is postoperative following thoracotomy. What instructions should the nurse give to the patient regarding activity immediately following discharge? a) Walk on a treadmill 30 minutes daily. b) Perform shoulder exercises five times daily. c) Walk 1 mile 3 to 4 times a week. d) Use weights daily to increase arm strength.

Perform shoulder exercises five times daily. The nurse emphasizes the importance of progressively increased activity. The nurse also instructs the patient on the importance of performing shoulder exercises five times daily. The patient should ambulate with limits and realize that the return of strength will likely be gradual and likely will not include weight lifting or lengthy walks.

Class V of Dyspnea

Performance not indicated or recommended; too difficult.

Anatomic Dead Space

Places where air flows but the structures are too thick for gas exchange

Fine crackles, fine rales, high-pitched rales characterisitics :

Popping, discontinuous sounds causes by air moving into previously deflated airways; sounds like hair being rolled between fingers near the ear. "Velcro" sounds late in inspirtation usually associated with restrictive disorders.

Which findings indicate that a patient is suffering from bacterial pharyngitis? Select all that apply.

Positive throat culture White blood cell (WBC) count of 14,000/mm3

Aveoli change to age-rationales

Potential for mechanical or infectious respiratory complications is increased in these situations. Theupright position minimzes ventilation-perfusion mismatching.

Nursing Priority

Promote oxygenation by ensuring a patent airway

Which finding by the nurse for a patient with a nursing diagnosis of impaired gas exchange will be most useful in evaluating the effectiveness of treatment?

Pulse oximetry reading of 92%

Head and Neck Cancer Interventions

Radiation therapy Chemotherapy Biotherapy Cordectomy Laryngectomy

When the patient arrives to the unit, she is assessed and is in acute respiratory distress. Her respirations are labored and her respiratory rate is 34. She states that she is severely short of breath. Her oxygen saturation is 82% on O2 at 2 L via nasal cannula. Based on these findings, what should the nurse do next?

Rapid response team Tracheotomy

An older patient has a persistent cough with hemoptysis and has a known exposure to tuberculosis. A tuberculin skin test reveals a reaction less than 5 mm. The nurse documents that this test result indicates which condition?

Reduced immune function

Palliation

Relieving symptoms rather than the cure

When caring for a patient who is three hours postoperative laryngectomy, what is the nurse's highest priority assessment?

Remember the airway, breathing, and circulation (ABCs) with prioritization. Airway patency is always the highest priority and is essential for a patient undergoing surgery surrounding the upper respiratory system. Comfort, drainage, and vital signs follow the ABCs in priority.

Lobectomy

Removal of a lung lobe

Pneumonectomy

Removal of the entire lung

Cordectomy

Removal of vocal cord

A nurse is educating a patient in anticipation of a procedure that will require a water-sealed chest drainage system. What should the nurse tell the patient and the family that this drainage system is used for? a) Removing excess air and fluid b) Monitoring pleural fluid osmolarity c) Maintaining positive chest-wall pressure d) Providing positive intrathoracic pressure

Removing excess air and fluid Chest tubes and closed drainage systems are used to re-expand the lung involved and to remove excess air, fluid, and blood. They are not used to maintain positive chest-wall pressure, monitor pleural fluid, or provide positive intrathoracic pressure

The nurse is instructing a patient with tuberculosis about combination drug therapy. What are the most common instructions that the patient should follow for all the antitubercular drugs? Select all that apply.

Report yellowing of the skin and any darkened urine immediately. "Refrain from drinking alcoholic beverages

The nurse is caring for an 80-year-old patient with chronic obstructive pulmonary disease (COPD) who was admitted for treatment of malnutrition. The patient develops a cough and the nurse notes a temperature of 39° C, an oxygen saturation of 94%, and crackles in both lungs. What is the nurse's priority action?

Request chest x-ray

What ABGs are at risk in COPD?

Respiratory Acidosis CO2 increased

Muscle Strength change to age-physiologic change

Respiratory muscle strength, especially the diaphragm and the intercostals, decreases.

What systems work together to provide sufficient tissue perfusion to the body?

Respiratory, cardiovascular, hematologic systems

Respiratory failure

Reversible Retains CO2

Which upper respiratory infection is often triggered by a hypersensitivity reaction to airborne allergens?

Rhinitis

Cor Pulmonale

Right-sided heart failure caused by pulmonary disease

Class II of Dyspnea

Satisfactory, mild breathlessness. Complete performance is possible without pause or assistance but not entirely normal

Fibrosis

Scar tissue formation

Which symptom indicates that a patient's pharyngitis is most likely bacterial and not viral?

Scarlatiniform rash

A patient newly diagnosed with asthma is being discharged. The nurse anticipates including which topic in the discharge teaching?

Self-administration of inhaled corticosteroids

hyponatremia

Serum sodium level below 136 mEq/L -cerebral changes are the most obvious problems -general muscle weakness -increased intestinal motility, causing nausea and diarrhea -drug therapy: IV Na and salt tabs -diet: processed food & snack food -causes: NPO, diuretics, vomiting, diarrhea, kidney disease, low-salt diet

Bronchodilators

Short and long acting Relaxes smooth muscle Cholinergic antagonists Methylxanthines

A patient reports difficulty breathing, facial pain, sneezing or couging, green or bloody nasal drainage, productive cough, and low grade fever. Which disorder does the nurse suspect.

Sinusitis

The nurse is caring for a patient who was admitted with pneumonia. Which position assumed by the patient leads the nurse to suspect that the patient is developing hypoxia?

Sitting in tripod position A patient with hypoxia will assume the tripod position (seated and positioned leaning on the hands, often leaning on an over-the-bed table). The patient who is hypoxic will not assume a side-lying or prone position because these positions will only increase a patient's feelings of inability to obtain enough air. Elevating the head of the bed 45° will not be adequate to relieve the smothering feelings associated with hypoxia

A patient presents to the emergency department with facial trauma after a motor vehicle accident. The nurse notes extensive bruising behind the ears and suspects which of the following conditions?

Skull fracture and brain trauma Extensive bruising in the mastoid area is associated with skull fracture and brain trauma. Lower jaw fractures and spinal fractures are not associated with extensive bruising behind the ear. Soft tissue injury may cause bruising, but this finding is related to more severe injury.

The role of the respiratory system

Specific human need: oxygenation and tissue perfusion Organs/tissues important in meeting the need: The respiratory system- lungs: oxygen intake heart: oxygen delivery 1 blood vessels RBCs: oxygen delivery 2

Educational Needs of the Patient: Speech communication

Speech and Communication -Patient can speak with a cuffless tube, a fenestrated tube, or a speaking valve -Provide writing tablet, communication board -Be sure call light is within reach

A patient is admitted to the hospital to rule out pneumonia. Which infection control technique does the nurse maintain

Standard precautions and no respiratory isolation

A patient is admitted to the hosptial with pneumonia, which approach to the administration of antibiotics does the nurse expect the physiciam to order

Start broad spectrum IV antibiotic therapy without a delay

A patient reporting a sore throat also has a temperature of 101.4, scarlatinform rash, and a positive rapid test throat culture, this patient will most likely be treated for which type of bacterial infection

Streptococcus

Psychosocial assessment

Stress may worsen some respiratory problems Chronic respiratory disease may cause changes in family roles, social isolation, financial problems due to unemployment or disability Discuss coping mechanisms, offer access to support systems

A patient with suspected TB is admitted to the hospital, along with a private room, which nursing intervention is appropriate related to isolation procedures

Strict respiratory isolation and use of specially designed facemasks

The nurse is teaching the patient and family about care of a peritonsillar abscess at home. For what symptoms does the nurse indicate the need for the patient to go to the emergency room

Strider and drooling

Which interventions should the nurse include in the plan of care for a patient undergoing radiation therapy who develops a sore throat and difficulty swallowing? Select all that apply.

Sucking on ice chips Gargling with saline water Using mouthwashes and throat sprays Sucking on ice chips will be beneficial to lessen sore throat pain. Gargling with saline water will help to soothe the throat and reduce difficulty in swallowing. Water and fluids should be increased, not decreased, to prevent a sore throat. Analgesic drugs are painkillers and should not be reduced. Mouthwashes and throat sprays have a local anesthetic agent that provides temporary relief from pain.

Tracheotomy

Surgical incision into the trachea to create an airway

Rhinoplasty

Surgical reconstruction of the nose

A patient reports experiencing chest pain, headache, and cough with sputum production, fever, and dyspnea. What does the nurse anticipate upon assessment? Select all that apply.

Tachycardia Diminished chest expansion Crackles upon auscultation This patient has symptoms of pneumonia. Symptoms include tachycardia due to hypoxemia, chest pain with decreased or unequal chest expansion, and crackles upon auscultation due to fluid in the interstitial and alveolar areas. A sore throat and nasal drainage are symptoms of an upper respiratory disorder.

An adult has been diagnosed as having pulmonary tuberculosis. What education should the nurse provide before the patient is started on isoniazid (INH) therapy? Select all that apply.

Take a daily multivitamin. Avoid alcoholic beverages

What nursing interventions may help prevent the complication of pneumonia for a surgical patient

Teaching, coughing, deep breathing exercises and use of incentive spirometry

A patient who is about to undergo a supraglottic partial laryngectomy asks the nurse what the surgeon will remove during this procedure. The nurse explains to the patient that the surgeon will remove which structures?

The hyoid bone, false cords, and the epiglottis A supraglottic partial laryngectomy involves removal of the hyoid bone, false cords, and the epiglottis. A hemilaryngectomy involves removal of one true cord, one false cord, and one-half of the thyroid. A total laryngectomy involves removal of the entire larynx. Laser surgery involves removal of the tumor only.

The nurse has taught a patient about influenza infection control. Which patient statement indicates the need for further teaching?

The intranasal vaccine can be given to everybody in the family. The intranasal flu vaccine is approved for healthy patients ages 2-49 who are not pregnant. Washing hands frequently is the best way to prevent the spread of illnesses such as the flu. Avoiding kissing and shaking hands are two ways to prevent transmission of the flu. A new recommendation from the Centers for Disease Control and Prevention (CDC) for controlling the spread of the flu is to sneeze or cough into the upper sleeve rather than into the hand.

b. Have patient perform coughing and deep-breathing exercises, then reassess

The nurse has just received a patient from the recovery room who is somewhat drowsy, but is capable of following instructions. Pulse oximetry has dropped from 95% to 90%. What is the priority nursing intervention? a. Administer oxygen at 2L/min by nasal cannula, then reassess b. Have the patient perform coughing and deep-breathing exercises, then reassess c. Administer Narcan to reverse narcotic sedation effect d. Withhold narcotic pain medication to reduce sedation effect

c. In older adult patients

The nurse is inspecting the patient's cest and observes an increase in anteroposterior diameter of the chest. When is this an expected finding? a. With pulmonary mass b. Upon deep inhalation c. In older adult patients d. ith chest trauma

c. Observe the patient for other findings assoiated with a pneumothorax

The nurse is palpating the patient's chest and identifies an increased tactile fremitus or vibration of the chest wall produced when the patient speaks. What does the nurse do next? a. Observe for other findings associated with subcutaneous emphysema b. Document the observation as an expected normal finding c. Observe the patient for other findings associated with a pneumothorax. d. Document the observation as a pleural friction rub

d. "When was the last time you were hospitalized?"

The nurse is performing a respiratory assessment on the older adult patient. Which question is not appropriate to ask when using the Gordon's Functional Health Pattern Assessment approach? a. "How has your general health been?" b. "Have you had any colds this past year?" c. "Do you have sufficient energy to do what you like to do?" d. "When was the last time you were hospitalized?"

b. Normal pH, decreased PaO2, normal PaCO2

The nurse is reviewing ABG results from an 86-year-old patient. Which results would be considered normal findings for a patient of this age? a. Normal pH, normal PaO2, normal PaCO2 b. Normal pH, decreased PaO2, normal PaCO2 c. Decreased pH, decreased PaO2, normal PaCO2 d. Decreased pH, decreased PaO2, decreased PaCO2

d. 50-year-old man with a PaO2 of 84

The nurse is reviewing partial pressure of arterial oxygen (PaO2) levels for several adult patients. Which patient has a PaO2 that is lower than expected for his age? a. 40-year-old man with a PaO2 of 96 b. 85-year-old man with a PaO2 of 83.5 c. 65-year-old man with a PaO2 of 92 d. 50-year-old man with a PaO2 of 84

a. Orthopnea

The nurse is taking a history on a patient who reports sleeping in a recliner chair at night because lying on the bed causes shortness of breath. How is this documented? a. Orthopnea b. Paroxysmal nocturnal dyspnea c. Orthostatic nocturnal dyspnea d. Tachypnea

b. Limited to bed or chair and experiences shortness of breath at rest

The nurse makes observations about several respiratory patients' abilities to perform activities of daily living in order to quantigy the level of dyspnea. Which patient is considered to have Class V dyspnea? a. Experiences subjective shortness of breath when walking up a flight of stairs b. Limited to bed or chair and experiences shortness of breath at rest c. Can independently shower and dress, but cannot keep pace with similarly aged people d. Experience shortness of breath during aerobic exercise such as jogging

a. Increased red blood cell

The nurse reviews the complete blood count results for the patient who has chronic obstructive pulmonary disease (COPD) and lives in a high mountain area. What lab results does the nurse expect to see for this patient? a. Increased red blood cells b. Decreased neutrophils c. Decreased eosinophils d. Increased lymphocytes

A patient with a nasal fracture is scheduled for a fracture reduction surgery in two weeks. What preoperative instructions should a nurse give to the patient? Select all that apply.

The nurse should instruct the patient to avoid intake of NSAIDs at least two weeks before the surgery to reduce the risk of bleeding. The patient should avoid smoking because smoking cessation promotes better wound healing. Avoiding exercise is a postoperative instruction and helps to prevent bleeding. Applying ice is a postoperative instruction to decrease swelling. Alcohol tends to increase swelling and should be avoided after the operation.

The nurse is suctioning the tracheostomy tube of a patient. Which action by the nurse would be a priority ?

The nurse should supply oxygen for 30 seconds after suctioning and before starting the next suction. Suctioning should be performed for a short period, such as for 10 seconds, but not for five minutes. Suctioning should be discontinued if the heart rate falls from the baseline by 20 beats/minute. The suction pressure of 90 to 100 mm Hg is not sufficient for effective suctioning; therefore, the nurse should adjust the suction pressure to 120 to 150 mm Hg.

b. Coughing and deep breathing

The older patient is confined to bed and is therefore prone to decreased alveolar surface and elastic recoil. Which intervention is best to address these physiologic changes? a. Adequate nutritional intake b. Coughing and deep breathing c. Fluids to thin secretions d. Periods of rest and sleep

CLEAN WITH ALCOHOL, SCRUB HUB, VIGOROUSLY BEFORE SCRUBBING

The patient is receiving an intermittent IV ABX therapy. What does the nurse anticipate doing in order to maintain an asceptic port?

Usually need to inject contrast because sometimes contrast needs to be injected quickly

The radiology tech calls the nurse and informs the nurse that the patient is going to be picked up for a CT angiography. The radiologist asks the nurse regarding the patient's IV access. Why would the radiology tech be concerned with the type of IV access?

What happens if someone doesn't have a gag reflex?

They can't protect their airway

Rhonchus (rhonchi) association:

Thick, tenacious secretions Sputum production Obstruction by foreign body Tumors

Rhoncus is associated with what respiratory diseases?

Thick, tenacious secretions, sputum production, obstruction by foreign body, tumors

What is the main nursing priority for respiration problems?

To promote oxygenation by ensuring a patent airway

A patient with pulmonary tuberculosis is being started on combination therapy. What does the nurse explain to the patient as the purpose of combination therapy?

To shorten therapy by 6 months

A patient with pulmonary tuberculosis is being started on combination therapy. What does the nurse explain to the patient as the purpose of combination therapy?

To shorten therapy by 6 months Combination medication shortens therapy by 6 to 12 months. Tuberculosis medications should be taken as ordered without missing a dose. Combination therapy is not related to disease resistance. Medications may be changed based upon the patient's ability to tolerate drugs.

The nurse is providing education about head and neck cancer at a community fair. What does the nurse list as the main cause of head and neck cancers?

Tobacco and smokeless tobacco cause 85% of head and neck cancer. Alcohol can be a cause of oral, pharyngeal, laryngeal, and esophageal cancers, but it is not the main cause. Infections and trauma are unrelated to head and neck cancer.

A patient reports throat soreness and dryness, throat pain, pain on swallowing, and difficulty swallowing. Which disorder does the nurse suspect?

Tonsillitis

Which surgical procedure involves the removal of the entire larynx in patients with laryngeal cancer?

Total laryngectomy Total laryngectomy involves complete removal of the larynx. Laser surgery destroys or reduces a tumor by a laser beam. In laryngofissure, no cord is removed since it treats an early lesion. In transoralcordectomy, a tumor is removed through laryngoscope.

Which technique of voice restoration in a laryngectomy involves creation of a fistula between the esophagus and trachea in a patient?

Transesophageal puncture is a method of voice restoration in patients who have undergone laryngectomy. This procedure involves the creation of a fistula between the esophagus and trachea. Esophageal speech is a technique in which the patient inhales air, traps it in the esophagus, and releases it to create sound. Intraoral electrolarynx and neck type electrolarynx are two types of electrolarynx devices that create speech by using sound waves.

Accessory respiratory muscles

Trapezius, sternacostimastoid, and intercostals

Which information will the nurse include in the asthma teaching plan for a patient being discharged?

Tremors are an expected side effect of rapidly acting bronchodilators.

Physical appearance of COPD patients?

Tripod breathing position Barrel- chest Nails: clubbing

hypocalcemia

Trousseau's sign and Chvostek's sign may appear in someone with ______________

Fenestrated Tracheostomy Tube

Tube with opening in outer cannula. Used to wean client from tracheostomy tube by ensuring client can tolerate breathing though natural airway. Allows client to speak and cough. If the patient has trouble with any of placement, patency, size, and fenestration. Do not cap the tube until the problem is identified and corrected.

Cancer of the Nose & Sinuses

Tumors rare, benign or malignant Seen with exposure to dust from wood, textiles, leather, flour, nickel, chromium mustard gas, radium Slow onset, resembles sinusitis Lymph enlargement often occurs on side with tumor mass Surgical removal is treatment; may be combined with radiation (IMRT)

Bronchial Hygiene

Turn and reposition the patient every 1 to 2 hours, support out of bed activites, and encourage ambulation.Coughing and deep breathing, combined with the chest percussion, vibration, and postural drainage, promote pulmonary hygiene.

Anatomy & Physiology

Upper respiratory tract Lower respiratory tract Lungs Accessory muscles of respiration Oxygen delivery and the oxygen-hemoglobin dissociation curve

Infection

Use sterile technique to preven infection during suctioning and tracheostomy care. Assess stoma site q.shift. Use 4x4 pads around tracheostomy DO NOT cut the dressing because small bits of gauze could then be aspirated through the tube.

Reliever Drugs

Used to stop asthma attacks once it has started

hypertension

Used to treat _______________, the "ACE inhibitors" are drugs that disrupt the renin-angiotensin II pathway by reducing the amount of ACE made so that less angiotensin II is present.

Noninvasive Positive-Pressure Ventilation (NPPV)

Uses positive pressure to keep alveoli open, improve gas exchange without airway intubation BiPAP CPAP

Noninvasive Positive Pressure Ventilation (NPPV)

Uses positive pressure to keep alveoli open, improve gas exchange without airway intubation BiPAP- good for COPD CPAP

Pulmonary Vasculature change to age-physiologic change

Vascular resistance to blood flow through pulmonary vascular system increases. Pulmonary capillaries blood volume decreases. Risk for hypoxia increases.

NURSING SAFETY PRIORITY: If the tube is dislodged on an immature tracheostomy?

Ventilate the patient using a manual resuscitation bag and facmask while another nurse calls the Rapid Response Team.

Oxygen/Carbon Dioxide exchange 3 step process

Ventilation (airway/breathing), Diffusion (oxygenation/gas exchange, Transport (circulation/perfusion)

Drug therapy with antihistamines and decongestants to treat sinusitis are used with caution in the older adult because of which possible side effects

Vertigo, hypertension, insomnia, blurred vision

Upon assessment of a patient with chest pain, the nurse finds unequal chest expansion, crackles with diminished breath sounds, tachypnea, and a fever. Which laboratory data finding would lead the nurse to believe the patient has pneumonia?

WBC 12,000/mm3 An elevated white blood count is associated with bacterial infections such as pneumonia. A WBC of 12,000/mm3 is elevated. Any WBC below 10,000/mm3 is considered normal.

The nurse is reviewing the lab results for an older adult patient with pneumonia, which lab value frequently seen in patients with pneumonia may not be seen in this patient

WBC 12-18

HEAD DOWN, FEET UP TRANDELBURG ALTERED NEED BLOOD TO GO TO THE HEART

What position does the nurse anticipate the patient being placed in?

SO STOP, CATH FLOW HELPS LOOSEN ANYTHING IN THE IV, YOU INSTILL IT IN IT THEN REMOVE IT AFTER LIKE 10 MIN TO TRY AND SAVE THE CATH

When attempting to flush a catheter the nurse meets resistance. The nurse stops and recognizes that this could be a clot formation. What does the nurse anticipate doing?

Orthopnea

When patients sleep semi- sitting up because breathlessness is worse than lying down

c. Provide a diet that is high in calories and protein Rationale: Clients with COPD have difficulty obtaining enough calories and protein due to fatigue and early satiety. Therefore, the food that is consumed should be high in calories and protein.

When planning the care for a client with COPD, the nurse should include which of the following interventions? a. Schedule respiratory treatments following meals. b. Have the client sit in a chair for 2-hr periods three times a day c. Provide a diet that is high in calories and protein. d. Combine activities to allow for longer rest periods between activities

b. Decline in forced expiraory volume in 1 second

Which aspect of PFTs would be considered a normal result in the older adult? a. Increased forced vital capacity b. Decline in forced expiratory volume in 1 second c. Decrease in diffusion capacity of carbon monoxide d. Increased functional residual capacity

B. diuretics

Which medication classification must be assessed during the preoperative period because it can cause an electrolyte imbalance during surgery? A.Corticosteroids B.Diuretics C.Phenothiazines D.Insulin

Thoracentesis

Which procedure has a risk for the complication of pnemothorax? a. Thoracentesis b. Bronchoscopy c. PFT d. Ventilation-perfusion scan

Leukoplakia

White, patchy lesions

Tissue trauma

Xan result form frequent suctioning, prolonged suctioning time, excessivfe suction pressure, and nonrotation of the catheter. Therefore suction only prn.

2.2 ; 500

______ lb of weight gained is equivalent to 1 liter of fluid retained -- weight change of 1 lb is equal to a fluid volume change of about ______ mL

bucks

________ boot traction helps with muscle spasms from a hip fracture

cardiac

_________ monitoring is important for someone who has hyperkalemia

medical-surgical nursing

a specialty practice area in which nurses promote, restore, or maintain optimal health for patients from 18 to older than 100 years of age

A client has a peripherally inserted central catheter (PICC) line and the primary nurse is updating the care plan. For which common complications does the nurse assess? (Select all that apply.) a. Phlebitis b. Pneumothorax c. Thrombophlebitis d. Excessive bleeding e. Extravasation

a. Phlebitis c. Thrombophlebitis Rational: Although the complication rate with PICC lines is fairly low, the most common complications are phlebitis, thrombophlebitis, and catheter-related bloodstream infection. Pneumothorax, excessive bleeding, and extravasation are not common complications.

paranasal sinuses

air filled cavities within the bones that surround the nasal passages

subcutaneous emphysema

air present in the subcutaneous tissue

crepitus

air trapped in and under the slin

The nurse is teaching a client about isoniazid (INH) and rifampin (RIF) drug therapy for tuberculosis (TB). The nurse instructs that while on these medications, the client should avoid consuming which food?

alcohol

atelectasis

alveolar collapse

Changes in the Respiratory system related to aging

alveoli lungs pharynx and larynx pulmonary vasculature excerise tolerance muscle strength susceptibility to infection chest wall

fluid overload (hypervolemia)

an excess of body fluid signs & symptoms include: -weight gain -increased BP -bounding, increased pulse rate -increased respirations, dyspnea -crackles in the lungs -distended neck and hand veins -headache, confusion -orthopnea -pitting edema -ascites -hyperactive bowel sounds/diarrhea

Therapies for Pneumonia

antibiotic therapy oxygen therapy incentive spirometry (to prevent atelectasis) pharmacologic tx non-pharm tx

turbinates

are three bones that protrude intot he nasal cavities from the internal portion of the nose

laryngopharynx

area located behind the larynx from the base of the tongue to the esophagus

A client has just had a central venous access line inserted. What is the nurse's next action? a. Beginning the prescribed infusion as soon as possible b. Confirming placement of the catheter by x-ray c. Having the infusion team start the IV therapy d. Confirming that solutions are appropriate for the central line

b. Confirming placement of the catheter by x-ray Rational: A central venous access device, once placed, needs an x-ray confirmation of proper placement before it is used. The bedside nurse would be responsible for beginning the infusion once placement has been verified. Any IV solution can be given through a central line.

posteroanterior

back to front

hemoptysis

bleeding from the airway

thrombosis

blood clot in the vein

hemoptysis

blood in the sputum; most often seen in patients with chronic bronchitis or lung cancer

year

bones can take up to a _______ to completely heal

genu varum

bowing of the legs, or _______ _________, is a key feature of Paget's disease

A new nurse is preparing to start an IV on a client who is dehydrated and needs significant fluid volume. The new nurse selects a butterfly needle for the infusion. What action by the supervising nurse is best? a. Help the new nurse with the procedure as needed. b. Make sure the new nurse has the correct dressing. c. Stop the new nurse and review the procedure in private. d. Get the ultrasonic vein finder to help illuminate veins.

c. Stop the new nurse and review the procedure in private. Rational: Winged (butterfly) needles generally are used for single doses of medications or for blood sampling. They would not be used for large volumes of fluid or kept in for any length of time. The other options do not acknowledge that the new nurse's actions are incorrect and should be stopped.

phosphorus

calcium and ___________ have a reciprocal relationship

rapid response team (RRT)

called to intervene when patient is deteriorating (before a respiratory or cardiac arrest occurs)

AA nurse is administering oxygen to a client with chest pain who is restless, what method of oxygen administration will most likely prevent further increase in the clients anxiety

cannula

lipid

change ______ tubing every 24 hr

blood

change ________ tubing within 4 hr

propofol (Diprovan)

change __________ tubing every 6 to 12 hr

implanted ports

chosen for patients who are expected to require IV therapy for more than a year -- always check for blood return before giving a drug through this -- flushed after each use and at least once a month

paget's disease (osteitis deformans)

chronic metabolic disorder where bone is excessively broken down (osteoclastic activity) -- skull is typically soft, thick, and enlarged -- fractures never completely heal

The nurse is providing discharge instructions about pneumonia to a patient and family, which discharge information is the nurse sure to include

complete antibiotics as prescribed, rest, drink fluids, and minimize contact with crowds

Asthma (Definition)

condition in which reversible airflow obstruction in the airways occurs intermittently due to inflammation, bronchospasm or mucous production

eustachian tubes

connect the nasopharynx with the middle ears and open during swallowing to equalize pressure within the middle ear

Which assessment finding for a client with a peripherally inserted central catheter (PICC) line requires immediate attention? a. Initial dressing over site is 3 days old. b. Line has been in for 4 weeks. c. A securement device is absent. d. Upper extremity swelling is noted.

d. Upper extremity swelling is noted. Rational: Upper extremity swelling could indicate infiltration, and the PICC line will need to be removed. The initial dressing over the PICC site should be changed within 24 hours. This does not require immediate attention, but the swelling does.

osteogenic sarcoma

dreaded complication of Paget's disease -- can affect the femur and humerus & may affect old fracture sites

paget's

elevated alkaline phosphatase is related to _________ disease (180 range)

hypercarbia

elevated blood levels of carbon dioxide

alkaline phosphatase (ALP)

elevated serum ___________ ______________ would be seen in someone diagnosed with osteogenic sarcoma

The nurse has decided to immediately place a patient in respiratory isolation because the patient is exhibiting which sign/symptoms of the prodromal (early) stage of inhalation anthrax? Select all that apply.

fEVER, mild chest pain, and Mediastinal "widening" (chest x-ray)

fat embolism syndrome (FES)

fat globules are released into the blood stream (12-48 hours after injury) -- results from fractures or fracture repair (fractured hip and pelvis present greatest risk)

Which symptom of pneumonia may present differently in the older adult than in the younger adult?

fever Older adults may not have a fever and may have a lower-than-normal temperature with pneumonia. Crackles on auscultation may be present in all age groups as the result of fluid in the lungs. All age ranges may have a headache with pneumonia. Wheezing is an indication of narrowed airways and can be found in all age groups.

Weaning

gradual decrease in tube size; ultimate removal of tube

internal fixation

hardware holds bone fragments in proper alignment -- immediately restores bone strength

Stridor

harsh, grading, vibrating sound while breathing

Best way to reestablish an open airway

head tilt, chin lift

Non-Rebreather Mask

highest O2 level can deliver FIO2 greater than 90% Used for unstable patients requiring intubation ensure valves are patent and funcitonal

aphonia

inability to produce sound

sun

increase of Vitamin D through dietary intake and daily _____ exposure is important for someone who has osteomalacia

osteomyelitis (acute)

infection of the bony tissue -- bone pain is constant -- fever, redness, heat, elevated WBC

thrombophlebitis

inflammation due to a blood clot

phlebitis and post-infusion phlebitis

inflammation of the vein wall

nasotracheal

insert a tube into the trachea via the nose

orotracheal

insert a tube into the trechea via the mouth

bronchoscopy

is the insertion of a tube in the airways, usually as far as the secondary bronchi, to view airway structures and obtain tissue samples for biopsy or culture

hemodialysis catheter

large lumens accommodate hemodialysis or pheresis procedure (harvests specific blood cells)

osteomalacia

loss of bone related to Vitamin D deficiency (malabsorption of Vitamin D from the small bowl)

CO2 narcosis

loss of sensitivity to high levels of CO2

hypoxemia

low levels of oxygen in the blood

invasive interventions for COPD

lung reduction surgery preoperative care and testing operative procedure by median sternotomy or VATS postoperative care and close monitoring for complications

In a client w/ pneumonia, what is the most important nursing intervention?

managing hypoxemia

submucous resection (SMR)

may be needed to straighten a deviated septum when chronic "stuffy" nose, snoring, sinusitus, or discomfort occurs

400 to 600 mL/day

minimum urine amount needed to excrete toxic waste products

hip fractures

most common injury in older adults -- osteoporosis is a risk factor

xerostomia

mouth becomes dry (if the salivary glands are in the irradiation path)

Diffusion

movement of gases in the alveoli and across the alveolar-capillary membrane

hypotonic infusion

moves water into cells and expands them Ex: ½ NS (0.45% NaCl) ¼ NS (0.225% NaCl)

closed reduction

moving the bones by palpation to realign them

Absorption atelectasis

new onset of crackles/decreased breath sounds

Rhinoplasty

nose job; a surgical reconstruction of the nose

10

only use a ____ mL syringe when flushing a central line

glottis

opeing between the true vocal chords

oxygen concentrator

oxygen extractor is a machine that removes nitrogen from room air, increasing oxygen leves to more than 90%

Oxygen therapy improves?

oxygenation and tissue perfusion

pneumothorax

partial or complete collapse of the lung

Communication between a patient with a tracheostomy and the nurse requires

patience on the part of both the patient and the nurse.

Positive airway pressure

patient interfaces. A, Subject wearing a nasal mask and headgear with positive airway pressure being delivered. Positive pressure is delivered only to the nasal airway; opening of the mouth may cause air to leak and decrease the efficacy of the positive-pressure regimen. B, Subject wearing nasal pillows. Positive pressure is delivered directly into the nares rather than covering the nose or mouth; only minimal headgear is necessary to anchor the interface in place. C, Subject wearing a full face mask. Positive pressure is delivered to both the nose and the mouth based on the subject's own breath-to-breath partitioning of nasal and oral airflow.

external fixation device

pins are inserted through the skin in the affected bone -- important to pay attention to pin site and pin care -- allows early ambulation

anatomic dead space

places where air flows but the structures are too thick for gas exchange

Ventilation

process of moving air in and out of lungs (airflow)

erythroplakia

red, velvety patches

Oxygen therapy purpose

relieves hypoxemia

cordectomy

removal of the vocal cord

An early sign of hypoxia

restlessness

hypercarbia

retention of CO2; increased partial pressure of arterial carbon dioxide PaCO2 levels)

Which upper respiratory infection is often triggered by a hypersensitivity reaction to airborne allergens?

rhinitis Allergic rhinitis (hay fever or allergies) is triggered by a hypersensitive reaction to airborne allergens, especially plant pollens or molds. These infections can occur in the sinuses (sinusitis) or throat (pharyngitis); however, the initial trigger is the hypersensitive allergic reaction. Tonsillitis is a contagious airborne infection that has settled in the tonsils on either side (or both sides) of the throat.

Complications of Pneumonia

sepsis pericarditis (spread of infection to heart) meningitis endocarditis (infection of endocardium and heart valves)

mediastinal shift

shift of central thoracic structures toward one side

dyspnea

shortness of breath

orthopnea

shortness of breath occuring when lying down and is relieved by sitting up

A patient is admitted with a diagnosis of avian influenza (H5N1). For what symptoms specific to avian influenza does the nurse assess the patient?

shortness of breath, diarrhea, and bleeding

hypomagnesemia

since hypocalcemia often occurs with ____________________, interventions aim to restore normal serum calcium levels

morton's neuroma

small tumor in the digital foot -- causes pain and a burning sensation -- surgically removed

epiglottis

small, elastic structure attached along one edge to the top of the larynx.

hypoactive ; muscle

someone with hypokalemia may have ____________ bowel sounds and shallow respirations due to ____________ weakness

metastasizes

spreads

Tracheostomy

stoma (opening) that results from tracheotomy May be temporary or permanent

Acinys

structural unit consisting of a respiratory bronchiole, an alveolar duct, and an alveolar sac.

anterior nares

the external openings into the nasal cavities

pack years

the number of packs per day multiplied by the number of years the patient has smoked

dissociates

the oxygen molecule

2

to ensure that the right blood products are given to the right patient, the patient is identified using two identifiers by _____ qualified health care professionals to ensure patient safety

patient is having her tonsils removed. patient asks the nurse what function the tonsils normally serve. Which of the following would be the most accurate response?

tonsils help to guard the body from invasion of organisms.

oxygen

treatments for FES include bed rest and __________ therapy

osteomyelitis (chronic)

ulceration resulting from sinus tract formation (foot ulcers are most common)

Oxygen therapy goal

use lowest fraction of inspired oxygen for acceptable blood oxygen level without causing harmful side effects

midline catheter

used for therapies lasting 1 to 4 wk -- inserted through vein in upper arm

central IV therapy

vascular access device (VAD) placed in central circulation, specifically within superior vena cava (SVC) near junction with right atrium

High-Flow Oxygen Delivery Systems - can deliver 24%-100% at 8-15L/min

venturi mask face tent aerosol mask tracheostomy collar t-piece

fremitus

vibration

isotonic infusion

water does not move into or out of body's cells -- risk for fluid overload Ex: LR, NS, D5W

insensible water loss

water loss through skin, lungs, stool

abductor pillow

what device is used to ensure proper joint alignment and to prevent crossing of the legs in a patient post-op care of a hip fracture?

protein, calcium, vitamin D, vitamin K, magnesium

what kinds of food would a nurse suggest to promote good wound healing?

lactated ringers

what type of IV fluid would someone be on who is NPO?

pain

when someone has an amputation it's important to ask about their _______ level

infiltration

when the infusion of the IV solution goes into the extravascular space (pt. may complain of pain)

dexa scan

which non-invasive procedure is used to diagnose osteoporosis?

leukoplakia

white, patchy lesions

blood

whole blood, packed red blood cells, platelets, fresh-frozen plasma, albumin, and clotting factors can be transfused via the __________

fixed occlusion

wiring the jaws together with the mouth in a closed position

color, sensation, movement, and temperature

with a fracture, you want to assess these four things (CSMT) & compare side to side

culture

you'll want to get an order for a __________ to see what type of organism is causing the infection (bacteria, virus, fungi)

A nurse educator is reviewing the indications for chest drainage systems with a group of medical nurses. What indications should the nurses identify? Select all that apply. a) Chest trauma resulting in pneumothorax b) Need for postural drainage c) Post thoracotomy d) Pleurisy e) Spontaneous pneumothorax

• Post thoracotomy • Spontaneous pneumothorax • Chest trauma resulting in pneumothorax Chest drainage systems are used in treatment of spontaneous pneumothorax and trauma resulting in pneumothorax. Postural drainage and pleurisy are not criteria for use of a chest drainage system.

A client who must begin oxygen therapy asks the nurse why this treatment is necessary? What would the nurse identify as the goals of oxygen therapy? Select all that apply. a) To provide adequate transport of oxygen in the blood b) To clear respiratory secretions c) To reduce stress on the myocardium d) To decrease the work of breathing e) To provide visual feedback to encourage the client to inhale slowly and deeply

• To provide adequate transport of oxygen in the blood • To reduce stress on the myocardium • To decrease the work of breathing Oxygen therapy is designed to provide adequate transport of oxygen in the blood while decreasing the work of breathing and reducing stress on the myocardium. Incentive spirometry is a respiratory modality that provides visual feedback to encourage the client to inhale slowly and deeply to maximize lung inflation and prevent or reduce atelectasis. A mini-nebulizer is used to help clear secretions

The medical-surgical unit has one negative airflow room. Which of these four clients who have just arrived on the unit should the charge nurse admit to this room?

Client with possible pulmonary tuberculosis who currently has hemoptysis

Fracture of the Nose interventions

Closed reduction Rhinoplasty Nasoseptoplasty

Alopecia

Hair loss

Class I of Dyspnea

No breathlessness; normal

Epistaxis

Nosebleed

Hypoxemia

Poor blood oxygen levels

What is preventive therapy for asthma?

Use every day regardless of symptoms

pleura

continuous smooth muscle with two surfaces that totally enclose the lungs

Pneumonia

an infection of the lungs; excess fluid in the lungs resulting from inflammatory process

noninvasive interventions for COPD

breathing techniques, positioning, effective coughing, hydration, vibratory device oxygen therapy drug therapy exercise conditioning suctioning

A coworker tells the nurse that she will not get the flu shot because she believes it is better to develop her own immunity to the flu. What does the nurse tell this coworker?

"You are putting your clients at increased risk for serious respiratory illness."

Parts of Tracheostomy Tube: Speaking valve

-Cleint must be able to control secretions before deflation of cuff and speaking valve attempted -Placed on the hub or end of a tracheostomy tube to allow air to enter on inspiration -Air is directed around the tube and out the upper airway, through the vocal cords upon expiration, allowing speech -May assist with swallowing

gradual decrease in tube size; ultimate removal of tube

FALSE Rationale: Flammable solutions containing high concentrations of alcohol or oil are not used in rooms in which oxygen is in use. However this does not include alcohol-based hand rubs.

A patient who was admitted the previous day with pneumonia complains of a sharp pain of 7 (based on 0 to 10 scale) "whenever I take a deep breath." Which action will the nurse take next? A. Auscultate breath sounds. B. Administer the PRN morphine. C. Have the patient cough forcefully. D. Notify the patient's health care provider.

A. Auscultate breath sounds.

Nasotracheal

Inserting a tube into the trachea via the nose

Pulmonary Vasculature change due to age-nursing interventions

Assess patient's level of consciousness and cognition.

Pharynx and Larynx change to age-physiologic change

Muscle atrophy Vocal chords become slack Laryngeal muscles lose elasticity and airways lose cartilage

The nurse is caring for a patient with a cuffed tracheostomy and is aware the patient is at risk for developing which complication? Pneumothorax Tracheomalacia Subcutaneous emphysema Trachea-innominate artery fistula

B.

Positive Airway Pressure

BiPAP APAP CPAP

Which conditions may cause patients to be at risk for aspiration pneumonia

Continuous tube feeding, bronchoscopy procedure, decreased level of consciousness, stroke

As the assessment is completed, the nurse observes that the patient has a large amount of thick secretions visible in the trach. What is the priority nursing action? Add pulmonary toileting to daily interventions. Instruct the UAP to sit with the patient until she is calmer. Call the respiratory therapist for a stat bronchodilator treatment. Suction the artificial airway and remove the secretions.

D.

Hypoxia

Decreased tissue oxygenation

Lower respiratory tract

Main stem bronchus

A patient with emphysema is experiencing shortness of breath. To relieve this patient's symptoms, the nurse should assist her into what position?

Sitting upright, leaning forward slightly

The nurse is providing teaching to a client who has been diagnosed with bacterial rhinosinusitis. Which instruction does the nurse include when teaching this client about his diagnosis?

"Be sure to complete the full course of antibiotics."

The nurse is assessing a patient's sleep patterns to determine the presence of sleep apnea. What is a specific question that the nurse asks to determine this?

"Do you snore loudly or experience daytime sleepiness?" Snoring and daytime sleepiness are signs of sleep apnea and are appropriate questions to ask. Falling asleep easily is an effect of sleep apnea but can also indicate other sleep disorders. Dreaming and loss of muscle control are not indicators of sleep apnea.

The nurse provides dietary teaching for a patient with chronic obstructive pulmonary disease (COPD) who has a low body mass index (BMI). Which patient statement indicates that the teaching has been effective?

"I can have ice cream as a snack every day."

A patient has a stoma after undergoing a laryngectomy, and the nurse is preparing the patient for discharge. Which statement by the patient indicates that more teaching is needed?

"I should not cover the stoma while coughing or sneezing." Patients who have undergone a laryngectomy should be taught to cover the stoma when coughing or sneezing. They should also cover the stoma while shaving to prevent hair from getting inside. The stoma should be cleaned with soap and water. Patients should take appropriate precautions when around water and should not swim.

The nurse is instructing a nursing student on how to prevent pneumonia in an older adult who is receiving mechanical ventilation. Which statement by the student indicates a need for further teaching?

"I will provide meticulous oral care every 24 hours and as needed Should be preformed q 12 hrs.

The nurse is providing health education to a patient regarding ways to prevent influenza. Which statement made by the patient shows effective learning?

"I will refrain from attending public meetings if I feel I am getting sick. As the infected person may spread the disease, he or she must avoid public meetings to reduce the risk of spreading the disease. Drugs prescribed should be taken to reduce the risk of influenza. Paying attention to public health announcement will reduce the risk of spreading the disease. Similarly, giving nonperishable food to family members will avoid going out, thus reducing the risk of spreading the disease.

Which statements by the patient with rhinitis indicate ineffective learning about reducing the risk of spreading colds? Select all that apply.

"I will stop my cough reflex when I am in a crowded place or with the family." "I will have minimal contact with people who have chronic respiratory problems."

The nurse is counseling a patient whose parent has just been diagnosed with tuberculosis (TB). The patient tells the nurse that the parent was exposed several years ago, but developed symptoms only recently. What does the nurse tell this patient about his or her risk of contracting the disease?

"People are infectious to others only when symptoms are present." It is important to remind patients that people with TB are infectious only when manifestations of the disease occur. Patients being treated for TB are not considered contagious after 2-3 weeks of drug therapy. The only way to diagnose TB is with testing and by evaluation of symptoms. Treatment is initiated when the disease is confirmed.

A patient has a sore throat; fever; enlarged, red tonsils; and tender, swollen lymph nodes. A rapid antigen test (RAT) performed in the clinic is negative for group A beta-hemolytic streptococcus. What does the nurse tell this patient?

"The provider will have final results of a culture in 2 days."

The nurse has been instructed to administer tuberculosis (TB) medication to a patient who has been noncompliant by directly observed therapy. Which statement by the nurse will assist the patient in understanding this therapy?

"You must swallow your pills in front of me."

The nurse is providing preoperative teaching to a patient who is about to undergo a supraglottic partial laryngectomy. The patient asks the nurse whether his voice will be normal after surgery. How does the nurse respond?

"You will be hoarse after surgery, but your voice may become normal. Patients who have a supraglottic partial laryngectomy may have a normal or a hoarse voice after recovery from surgery since the false cords are removed. Patients who have a total laryngectomy will not have a natural voice. Patients who have a laryngofissure will have a normal voice. Patients who have a hemilaryngectomy will have a hoarse voice.

A patient with acute shortness of breath is admitted to the hospital. Which action should the nurse take during the initial assessment of the patient? A. Ask the patient to lie down to complete a full physical assessment B. Briefly ask specific questions about this episode of respiratory distress C. Complete the admission database to check for allergies before treatment D. Delay the physical assessment to first complete pulmonary functions test

*B* When a patient has severe respiratory distress only information pertinent to the current episode is obtained, and a more thorough assessment is deferred until later.

Hazards & Complications of Oxygen Therapy

-Combustion -Oxygen-induced hypoventilation Hypercarbia—retention of CO2 CO2 narcosis—loss of sensitivity to high levels of CO2 -Oxygen toxicity -Absorption atelectasis—new onset of crackles/decreased breath sounds -Drying of mucous membranes -Infection

Parts of Tracheostomy Tube: Cuff

-Cuff seals airway and protects client from aspiration of oral or gastric secretions -Used for patients who cannot protect airway (those on ventilator) -Pilot balloon on outside of tube indicates presence or absence of air in balloon

Key Nursing assessments and safe care practices cont

-Ensure patent airway -Assess need for suctioning every 2 hrs; suction prn -Clean/change inner cannula every 8 hours and prn (per facility protocol)

Assessment of the Lungs & Thorax

-Inspect thorax with patient sitting up -Observe chest, compare one side with the other -Work from the apex, move downward toward base (from side to side) -Rate, rhythm, depth of inspiration as well as symmetry of chest movement -Examine AP diameter with lateral diameter -Distance between ribs (intercostal space) -Palpate to assess respiratory movement, symmetry -Crepitus -Diaphragmatic excursion -Lung sounds -Adventitious sounds

Key Nursing Assessments and safe care practices

-Monitor Vital Signs frequently -Continous pulse oximetry -Assess for bilateral breath sounds

Respiratory Assessment

-Nose and sinuses -Pharynx, trachea, larynx -Lungs and thorax Movement/symmetry/fremitus Resonance Breath sounds -General appearance (muscle development) -Skin and mucous membranes

Educational needs of the Patient: Emotional Care

-Observe and acknowledge frustration -Use normal tone of voice when speaking -Tracheostomy tube does not alter hearing or comprehension

Types of Airways: 4

-Oropharyngeal airway -Nasal trumpet -Endotracheal tube -tracheostomy tube

T-piece nursing interventions:

-empty condensation from the tubing -keep the exhalation port open and uncovere. Position the T-piece so that it does not pull on the tracheostomy or endotrachal tube. -make sure the humidifier creates enough mist -a mist should be seen during inspiration and expiration

Aerosol Mask, Face Tent, Tracheostomy Collar rationales:

-humidification should be delivered to the patient -emptying prevents the patient from being lavaged with water, promotes an adequate flow rate, and ensures a continued prescribed FiO2 -adequate humidification is ensured only when there is sufficient water in the canister

Partial rebreather mask nursing interventions:

-make sure that the reservior does nto twist or kink, which results in a deflated bag -adjust the flow rate to keep the reservoir bag inflated

Venturi Mask nursing interventions:

-perform constant survelliance to ensure an accurate flow rate for the specific FiO2 -keep the orifice for the venturi adaptor open and uncovered -provide a mask that fits snugly and tubing that is free of kinks -assess the patient for dry mucous membranes. change to a nasal cannula during mealtime

Non-rebreather mask nursing interventions:

-requires close monitoring -make sure that valves and rubber flaps are patenet, functional, and not stuck. Remove mucus or salvia -closely assess the patient on increases FiO2 via non-rebreather mask. Intubation is the only eay to provide more precise FiO2.

Important aspects to assess in a respiratory system history

-smoking history -childhood illnesses: asthma, pneumonia, communicable diseases, hay fever, allergies, eczema, frequent colds, croup, cystic fibrosis -adult illnesses: pneumonia, sinustis, tuberculosis, HIV and AIDS, lung disease such as emphysema and sarcoidosis, diabetes, hypertension, heart disease, influeneza, pneumoncoccal (Pneumovax), BCG vaccinations, surgeries of the upper or lower respiratory system, injuries to the upper or lwoer respiratory system, hospitalizations, date of last chest x-ray, pulmonary function test, or other diagnostic tests and results, recent weight loss, night sweats, sleep disturbances, lung disease and condition of family members, geographic areas of recent travel, and occupation and leisure activities

The nurse is assessing a patient who smokes and notes gray-colored sputum with specks of brown. What action would be appropriate for this patient? 1. No action is required, as it is considered normal. 2. Administer oxygen therapy to the patient. 3. Assess oxygen saturation through pulse oximetry. 4. Inform the health care provider

1 RATIONALE Gray-colored sputum with specks of brown is a normal finding in an individual who smokes, so no action is required. Administering oxygen therapy, assessing oxygen saturation, and informing the health care provider are not necessary for this patient.

A nurse is monitoring the SvO2 of a patient. The SvO2 value is 50%. What does this value indicate? 1 Less oxygen is being delivered to the tissues. 2 Less oxygen is being consumed by the tissues. 3 The cardiac output is normal. 4 The patient is hypoventilating

1 RATIONALE: SvO2 is the measure of venous oxygen saturation. It can be measured through a pulmonary artery catheter. The normal values range between 60% to 80%. A decrease in SvO2 indicates that less oxygen is being delivered to the tissues, causing tissue hypoxia. A decrease in SvO2 also indicates that more oxygen is being consumed by the tissues, causing an imbalance of demand and consumption. It is an early indicator of a change in the cardiac output. It does not indicate hypoventilation; a measurement of CO2 is more appropriate for measuring hypoventilation.

What patient positioning strategies should the nurse use while caring for a patient with acute respiratory distress syndrome (ARDS)? Select all that apply. 1, Prone position 2, Supine position 3, Continuous lateral rotation therapy 4. Kinetic therapy 5. Lateral positioning

1, 3, 4 RATIONALE: Some patients with ARDS have a marked improvement in arterial oxygen (PaO2) when turned from the supine to the prone position with no change in fraction of inspired oxygen (FIO2). Continuous lateral rotation therapy and kinetic therapy are other strategies that can be used for patients with ARDS. Continuous lateral rotation therapy involves continuous, slow, side-to-side turning of the patient by rotating the actual bed frame less than 40 degrees. Kinetic therapy involves rotating the patient side to side 40 degrees or more. In a supine position, the heart and the mediastinal mass may put pressure on the lungs, predisposing the patient to atelectasis. Therefore, a supine position is not advisable. Lateral positioning is also not suitable, as it may cause pooling of secretions.

A nurse is providing care to a patient with hypoxemia secondary to an intrapulmonary shunt. Which nursing interventions provide better improvement in the patient's condition? Select all that apply 1. Positive pressure ventilation (PPV) via tight-fitting mask 2. Benzodiazepines to reduce intrapulmonary shunt 3.24% to 32% oxygen by face mask or Venturi mask 4. PPV via an endotracheal tube 5. Oxygen administration at 1 to 3 L/minute by nasal cannula

1, 4 RATIONALE: Hypoxemia secondary to an intrapulmonary shunt usually requires PPV. PPV provides oxygen therapy and humidification, decreases the work of breathing, and reduces respiratory muscle fatigue. In addition, PPV assists in opening collapsed airways and decreasing shunt. PPV is provided via an endotracheal tube (most frequently) or noninvasively by means of a tight-fitting mask. Benzodiazepines are an antianxiety drug and do not reduce intrapulmonary shunt. 24% to 32% oxygen by face mask or Venturi mask and oxygen administration at 1 to 3 L/minute by nasal cannula does not improve shunt. TEST-TAKING TIP: Come to your test prep with a positive attitude about yourself, your nursing knowledge, and your test-taking abilities. A positive attitude is achieved through self-confidence gained by effective study. This means (a) answering questions (assessment), (b) organizing study time (planning), (c) reading and further study (implementation), and (d) answering questions (evaluation).

A 61-year-old patient with asthma is admitted to the hospital. The nurse understands that symptoms of asthma include which of the following? Select all that apply. 1. Wheezing 2. Chest tightness 3. Crackles 4. Cough 5. Pink frothy sputum

1,2,4 RATIONALE: Symptoms of asthma include cough, chest tightness, and wheezing. Crackles are heard when fluid has accumulated in the lungs, which is not consistent with asthma. Pink frothy sputum is seen with pulmonary edema

Nasal Cannula

1-6 L/min 24%- 44% O2 concentration

A young adult patient who denies any history of smoking is seen in the clinic with a new diagnosis of chronic obstructive pulmonary disease (COPD). The nurse should plan to teach the patient about

1-antitrypsin testing.

The nurse is assigned to care for an 83-year-old patient with an acute asthma exacerbation. Which arterial blood gas (ABG) result would prompt the nurse to notify the provider immediately? 1 pH 7.32, PaO2 70 mm Hg, PaCO2 50 mm Hg 2 pH 7.30, PaO2 74 mm Hg, PaCO2 65 mm Hg 3 pH 7.48, PaO2 79 mm Hg, PaCO2 43 mm Hg 4 pH 7.40, PaO2 65 mm Hg, PaCO2 38 mm Hg

2 RATIONALE: A markedly elevated PaCO2 is indicative of impending respiratory failure, requiring immediate medical attention. While the other ABGs are abnormal, the ABG requiring immediate intervention is that with the highest PaCO2 and associated acidosis.

How much water needs to stay in chest tube? What does that do?

2 cm Prevents backflow of air

A patient presents with epistaxis. The primary health care provider places an anterior packing in the form of a compressed sponge to control the bleed. Which nursing interventions would be appropriate for this patient? Select all that apply. 1. Provide extra packing material. 2. Administer mild opioid medication. 3. Administer prophylactic antibiotics. 4. Provide saline nasal spray. 5. Administer nonsteroidal antiinflammatory drugs

2,3 RATIONALE: Nasal packing is painful because of the pressure applied to stop the bleeding. Mild opioid medication should be given to the patient for analgesia. Patients with nasal packing are susceptible to staphylococci infection, so prophylactic antibiotics should be administered. Extra packing material is no longer required as anterior packing is already done. Saline nasal spray is recommended after packing removal. Nonsteroidal antiinflammatory drugs are discouraged due to the risk of bleeding.

A patient presents to the emergency department with acute exacerbation of asthma. What actions should the nurse perform to monitor the patient's respiratory and cardiovascular systems? Select all that apply. 1. Take a chest radiograph. 2. Auscultate the lung sounds. 3. Measure blood pressure and respiratory rate. 4. Monitor arterial blood gases (ABGs) and pulse oximetry. 5 Check the patient's temperature

2,3,4 RATIONALE: It is essential to monitor respiratory and cardiovascular systems in case of acute exacerbation of asthma. Auscultating lung sounds, measuring blood pressure and respiratory rate, and monitoring ABGs and pulse oximetry are required to monitor these systems. Chest radiographs are seldom useful in the management of an acute asthma attack. Checking the temperature may not contribute to monitoring respiratory and cardiovascular systems.

A patient has bronchodilator therapy ordered. What information should the nurse include when educating the patient about bronchodilator therapy? Select all that apply. 1 Bronchodilators increase alveolar perfusion. 2 Bronchodilators increase alveolar ventilation. 3 Bronchodilators may worsen hypoxemia at times. 4 Rebound bronchoconstriction may occur with bronchodilators. 5 Side effects of bronchodilators may include tachycardia and hypertension

2,3,5 RATIONALE: Bronchodilators increase alveolar ventilation by relieving bronchospasm. These medications can sometimes worsen hypoxemia by redistributing the inspired gas to areas of decreased perfusion. Side effects of bronchodilators may include tachycardia and hypertension due to activation of adrenergic receptors. Bronchodilators do not increase alveolar perfusion. Bronchodilators do not produce rebound bronchoconstriciton.

The nurse is monitoring a patient who has pneumonia with thick secretions. The patient is having difficulty clearing the secretions. Which of these would be appropriate nursing interventions for this patient? Select all that apply. 1. Perform postural drainage every hour. 2. Encourage the patient to rest and limit activity. 3. Provide adequate hydration by encouraging fluid intake. 4. Provide analgesics as prescribed to promote patient comfort. 5. Teach the patient how to cough effectively to bring secretions to the mouth.

2,3,5 RATIONALE: nterventions for pneumonia include teaching the patient how to cough effectively to remove secretions, providing adequate hydration, and encouraging rest. Hydration is important in the supportive treatment of pneumonia to prevent dehydration and loosen secretions. Individualize and carefully monitor fluid intake if the patient has heart failure. It is not necessary to provide postural drainage every hour. Providing analgesics will not help the patient clear secretions.

An alcoholic patient was involved in a road traffic accident and sustained a minor head injury. What nursing interventions may increase the risk of respiratory failure in such patients? Select all that apply. 1.The administration of steroids to relieve pain and edema 2.The administration of an opioid analgesic to relieve the pain 3.The administration of prophylactic antibiotics to prevent infection 4.The administration of benzodiazepines to prevent development of seizures 5. The administration of nonsteroidal antiinflammatory drugs to relieve pain and edema

2,4, RATIONALE: Opioids and benzodiazepines have a respiratory depressant action in a dose-dependent manner. Since the patient is alcoholic and has sustained a head injury, the chance of going into respiratory failure is higher with the use of opioids and benzodiazepines. Steroids, antibiotics, and nonsteroidal antiinflammatory drugs have no respiratory depressive action. Steroids and nonsteroidal antiinflammatory drugs may improve the patient's condition if respiratory failure is caused by brain stem edema.

The nurse is caring for a patient on a ventilator. What measures can a nurse take to prevent development of barotraumas in such a patient? Select all that apply. 1 Initiate enteral nutrition early. 2 Use smaller tidal volumes. 3 Choose peak inspiratory pressure. 4 Choose varying amounts of positive end-expiratory pressure (PEEP). 5 Advise use of measures that increase intrathoracic pressure

2,4,5 RATIONALE: To avoid barotraumas, patients with acute respiratory distress syndrome (ARDS) are often ventilated with smaller tidal volumes and varying amounts of PEEP to minimize oxygen requirements and intrathoracic pressure. Large tidal volume and elevated plateau and peak inspiratory pressures are associated with the risk of barotrauma. Early initiation of enteral nutrition helps in protecting the gut mucosa and has no role in preventing barotraumas.

When teaching a patient about using a dry powder inhaler, what instructions should be included in this teaching? Select all that apply. 1. Shake the inhaler well before use. 2. Avoid shaking the inhaler before use. 3. Slow down the inspiration. 4.Increase the inspiration. 5. Keep the device moisture-free

2,4,5 RATIONALE: With dry powder inhalers, there is no need to shake before use. Inspiration should be rapid, and the device should be kept moisture-free to protect the dry powder. Shaking the inhaler well before use and slow inspiration are actions performed when using metered-dose inhalers.

The OR nurse is setting up a water-seal chest drainage system for a patient who has just had a thoracotomy. The nurse knows that the amount of suction in the system is determined by the water level. At what suction level should the nurse set the system? a) 20 cm H2O b) 5 cm H2O c) 15 cm H2O d) 10 cm H2O

20 cm H2O The amount of suction is determined by the water level. It is usually set at 20 cm H2O; adding more fluid results in more suction.

What is the oxygen saturation in room air?

21%. every liter of oxygen that we deliver brings that number up 10%

A nurse is teaching a patient how to manage fatigue induced by radiation therapy. What statement by the patient indicates a correct understanding of the lesson? 1. "I will walk three to four hours every day to increase my level of energy." 2. "It's most important for me to avoid asking for help so I can become more independent." 3. "Because I have the most energy in the morning, I will plan my errands during this time." 4. "I will keep myself busy continuously throughout the daylight hours."

3 RATIONALE: "Because I have the most energy in the morning, I will plan my errands during this time" indicates that the patient understands the importance of doing activities that are most important to them and to rest during periods of low energy. Fatigue is a common side effect of radiation therapy and usually begins a few weeks into therapy. "I will walk three to four hours every day to increase my level of energy" indicates that the patient does not understand that scheduling activities for a period of three to four hours is excessive and does not allow time for adequate rest periods. It is important that patients suffering from radiation-induced fatigue identify support systems as a means of assistance. Avoiding requests for help would be counter to this teaching. Continuous engagement in activity would not provide periods of much needed rest for a patient who is fatigued from radiation therapy.

A patient is admitted to an emergency department with injuries of the face and nose. A nurse notices a clear, pink-tinged discharge from the nostrils of the patient, even after controlling the nasal bleed. What could be the cause of the discharge? 1 Skull fracture 2 Septal deviation 3 Cerebrospinal fluid (CSF) leak 4 Epistaxis

3 RATIONALE: A clear and pink-tinged discharge from the nose even after control of nasal bleeding suggests a cerebrospinal fluid (CSF) leak. It is an emergency situation and can lead to life-threatening complications. Skull fracture is manifested as ecchymosis of the eyes. There is no clear discharge in the event of a septal deviation or epistaxis.

The nurse is caring for a patient with complaints of allergic nasal rhinitis. The nurse knows that the treatment includes: 1. Antibiotics 2. First-generation antihistamines 3.Adequate fluid intake when taking antihistamines 4. A two-drug combination with an antihistamine and corticosteroid spray

3 RATIONALE: Adequate fluid intake is essential when the patient is taking antihistamines. Antibiotics are not appropriate for allergic rhinitis. Second-generation antihistamines are preferred over first-generation because of their nonsedating effects. Initially monotherapy is prescribed, and if that is not effective then a two-drug combination may help.

The nurse evaluates that nursing interventions to promote airway clearance in a patient admitted with chronic obstructive pulmonary disease (COPD) are successful based on which finding? 1. Absence of dyspnea 2. Improved mental status 3. Effective and productive coughing 4. PaO2 within normal range for the patient

3 RATIONALE: Airway clearance is evaluated most directly as successful if the patient can engage in effective and productive coughing. Absence of dyspnea, improved mental status, and PaO2 within normal range for the patient show improved respiratory status but do not evaluate airway clearance.

The nurse is monitoring a patient through pulse oximetry. What is monitored through pulse oximetry? 1 Inspired oxygen concentration 2 Expired oxygen concentration 3 Arterial oxygen saturation (SpO2) 4 Venous oxygen saturation

3 RATIONALE: Pulse oximetry is a noninvasive method to determine oxygen saturation levels. It may be used intermittently or continuously to assess SpO2. Pulse oximetry cannot be used to assess inspired oxygen concentration, expired oxygen concentration, or venous oxygen saturation. These can be measured through arterial blood gases measurements or pulmonary artery pressure monitoring.

What is the priority nursing assessment in the care of a patient who has a tracheostomy? 1. Electrolyte levels and daily weights 2. Assessment of speech and swallowing 3. Respiratory rate and oxygen saturation 4. Pain assessment and assessment of mobility

3 RATIONALE: The priority assessment in the care of a patient with a tracheostomy focuses on airway and breathing. These assessments supersede the nurse's assessments that also may be necessary, such as nutritional status, speech, pain, and swallowing ability.

The nurse is caring for a patient admitted with a barbiturate overdose. The patient is comatose with blood pressure (BP) 90/60, apical pulse 110, and respiratory rate 8. Based upon the initial assessment findings, the nurse recognizes that the patient is at risk for which type of respiratory failure? 1 Hypoxemic respiratory failure related to shunting of blood 2 Hypoxemic respiratory failure related to diffusion limitation 3 Hypercapnic respiratory failure related to alveolar hypoventilation 4 Hypercapnic respiratory failure related to increased airway resistance

3 The patient's respiratory rate is decreased as a result of barbiturate overdose, which caused respiratory depression. The patient is at risk for hypercapnic respiratory failure resulting from the decreased respiratory rate, and thus decreased CO2 elimination. Barbiturate overdose does not lead to shunting of blood, diffusion limitations, or increased airway resistance.

A nurse is caring for a patient diagnosed with acute lung injury who has a PaO2 of 48 mm Hg. Which condition does the nurse expect? 1.Dysoxia 2.Hypoxia 3.Hypoxemia 4.Hyperventilation

3 RATIONALE: Hypoxemia, an inadequate amount of oxygen in the blood, frequently is quantified as a PaO2 of less than 50 mm Hg. If allowed to progress, hypoxemia can result in hypoxia, which is defined as an inadequate amount of oxygen available at the cellular level such that cells experience anaerobic metabolism. Dysoxia is a condition characterized by an inability of the cells to use oxygen properly despite adequate levels of oxygen delivery. There is not enough information to determine if the patient is hyperventilating.

A patient presents with a lung abscess. What treatment option would be the most appropriate? 1. Postural drainage 2. Chest physiotherapy 3. Reduction of fluid intake 4. Antibiotic treatment

4 RATIONALE: As there are mixed bacteria in a lung abscess, starting a broad spectrum antibiotic is the appropriate treatment option. Postural drainage and chest physiotherapy are not recommended, as they may cause spillage of infection to other bronchi and spread the infection. Reducing fluid intake is not advisable; instead, adequate fluid intake is recommended.

A client with a respiratory condition is receiving oxygen therapy. While assessing the client's PaO2, the nurse knows that the therapy has been effective based on which of the following readings? a) 84 mm Hg b) 120 mm Hg c) 45 mm Hg d) 58 mm Hg

84 mm Hg : In general, clients with respiratory conditions are given oxygen therapy only to increase the arterial oxygen pressure (PaO2) back to the client's normal baseline, which may vary from 60 to 95 mm Hg.

The nurse performs follow-up care for a group of patients who have previously had tuberculosis. Which patients are most at risk for developing secondary tuberculosis (TB)? Select all that apply.

A 34-year-old with HIV infection A 75-year-old who is recovering from a hip replacement A 7-year-old who is undergoing chemotherapy for leukemia

A

A 58-year-old woman who has been diagnosed with throat cancer 1 week ago comes to the clinic today to discuss surgical options with her health care provider. She is very tearful and appears sad when the nurse calls her back to the examination room. Based on her diagnosis, which clinical manifestation will the nurse likely observe in the patient? A. Hoarseness B. Severe chest pain C. Low hemoglobin level (anemia) D. Numbness and tingling of the face

Which patient is most at risk for the development of either community or hospital-acquired pneumonia?

A 76-year-old who has limited mobility because of osteoarthritis A 76-year-old patient with limited mobility is at high risk for both community- and hospital-acquired pneumonia. The 8-month-old is at a slightly increased risk but not as high as the 76-year-old who is limited in mobility. An individual who works in the textile industry is at an increased risk for community-acquired pneumonia, but not hospital-acquired pneumonia, as is the adolescent who has type 1 diabetes.

c. Take the talet with a full glass of water Rationale: Guaifenesin (Mucinex) is an expectorant and should be taken with a full glass of water to decrease the viscosity of secretions. Extra doses should not be taken. The client should contact the physician if the cough lasts longer than 1 week or is accompanied by fever, rash, sore throat, or persistent headache. Fluids are needed to decrease the viscosity of secretions. The medication does not have to be taken with meals.

A client has a prescription to take guaifenesin (Mucinex). The nurse determines that the client understands the proper administration of this medication if the lent states that he or she will: a. Take an extra dose if fever develops b. Take the medication with meals only c. Take the tablet with a full glass of water d. Decrease the amount of daily fluid intake

a. Right pneumothorax Rationale: Pneumothorax is characterized by restlessness, tachycardia, dyspnea, pain with respiration, asymmetrical chest expansion, and diminished or absent breath sounds on the affected side. Pneumothorax can cause increased airway pressure because of resistance to lung infaltion. Acute respiratory distress syndrome and pulmonary embolism are not characterized by absent breath sounds. An endotracheal tube that is inserted too far can cause absent breath sounds, but the lack of breath sounds most likely would be on theleft side because of the degree of curvature of the right and left main stem bronchi.

A client has been admitted with chest trauma after a motor vehicle accident and has undergone subsequent intubation. A nurse checks the client when the high-pressure alarm on the ventilator sounds, ad notes that the client has absence of breath sounds in the right upper lobe of the lung. The nurse immediately assess for other signs of: a. Right pneumothorax b. Pulmonary embolism c. Displaced endotracheal tube d. Acute repiratory distress syndrome

d. Bronchospasm Rationale: Cromolyn sodium (Intal) is an inhaled nonsteroidal antiallergyagent and a mast cell stabilizer. Undesirable side effects associated with inhalatio therapy of cromolyn sodium are bronchospasm, cough, nasal congestion, throat irritation, and wheezing. Clients receiving this medication orally may experience pruritus, nausea, diarrhea, and myalgia.

A cromolyn sodium (Intal) inhaler is prescribed for a client with allergic asthma. A nurse provides instructions regarding the side effects of this medication. The nurse tells that client that which undesirable effect is associated with this medication? a. Insomnia b. Constipation c. Hypotension d. Bronchospasm

For patients receiving mechanical ventilation

A cuffed tube is used in acute care settings.

Chest Tube

A drain placed in the pleural space Allows the lung re-expansion

Which tube has openings on the surface of the cannula to permit airflow?

A fenestrated tube has openings on the surface of the outer cannula that permit air to flow over the vocal cords. Speaking tracheostomy tubes, tracheostomy tubes with foam-filled cuffs, and tracheostomy tubes with cuffs and pilot balloons do not have openings on the surface of the cannula.

A patient had an open reduction repair of a bilateral nasal fracture. The nurse plans to implement an intervention that focuses on both nursing and medical goals for this patient. Which intervention should the nurse implement?

A goal that is common to nursing and medical management of a patient after rhinoplasty is to prevent the formation of a septal hematoma and potential infections resulting from a septal hematoma. Therefore the nurse helps to keep the nasal packing in the nose. The packing applies direct pressure to oozing blood vessels to stop postoperative bleeding. A medical goal includes realigning the fracture with an external or internal splint. The nurse helps maintain the airway by humidifying inspired air because the nose is unable to do so following surgery because it is swollen and packed with gauze.

Oxygen Concentrator

A machine that removes nitrogen fro room air increasing oxygen levels to more than 90%

b. Salmeterol first and then the beclomethasone Rationale: Salmeterol (Srevent Diskus) is an adrenergic type of bronchodilator and beclomethasone dipropionate is a glucocorticoid. Bronchodilators are always administered before glucocorticoids when both are to be given on the same schedule. This allows for widening of the air passages by the bronchodilator, which then makes the glucocorticoid me effective.

A nurse has an order to give a client salmeterol (Serevent Diskus), two puffs, and beclomethasone dipropionate (Qvar), two puffs, by metered-dose inhaler. The nurse administers the medication by giving the: a. Beclomethasone first and then the salmeterol b. Salmeterol first and then the beclomethasone c. Alternating a single puff of each, beginning with the salmeterol d. Alternating a single puff of each, beginning with the beclomethasone

d. "I should not be contagious after 2 to 3 weeks of medication therapy." Rationale: The client is continued on medication therapy for 6 to 12 months, depending on the situation. The client generally is considered not to be contagious after 2 to 3 weeks of medication therapy. The client is instructed to wear a mask if there will be exposure to crowds until the medication is effective in preventing transmission. The client is allowed to return to work when the results of three sputum cultures are negative.

A nurse has conducted discharge teaching with a client diagnosed with tuberculosis. The client has been taking medication for 1 1/2 weeks. The nurse evaluates that the client has understood the information if the client makes which of the following statements? a. "I need to continue drug therapy for 2 months." b. "I can't stop at the mall for the next 6 months." c. "I can return to work if a sputum culture comes back negative." d. "I should not be contagious after 2 to 3 weeks of medication therapy."

d. Difficulty in discriminating the color red from green Rationale: Ethambutol causes optic neuritis, which decreases visual acuity and the ability to discriminate between the colors red and green. This poses a potential safety hazard when a client is driving a motor vehicle. The client is taught to report this symptom immediately. The client also is taught to take the medication with food if gastrointestinal upset occurs. Impaired hearing results from antitubercular therapy with streptomycin. Orange-red discoloration of secretions occurs with rifampin (Rifadn).

A nurse has given a client taking ethambutol (Myambutol) information about the medication. The nurse determines that the client understands the instructions if the client states to report immediately: a. Impaired sense of hearing b. Gastrointestinal side effects c. Orange-red discoloration of body secretions d. Difficulty in discriminating the color red from green

d. Promote carbon dioxide elimination Rationale: Pursed-lip breathing facilitates maximal expiration for clients with obstructive lung disease. This type of breathing allows better expiration by increasing airway pressure that keeps air passages open during exhalation. Options a,b, & c are not the purpose for this type of breathing.

A nurse instructs a client to use the pursed-lip method of breathing and the client asks the nurse about the purpose of this type of breathing. The nurse responds, knowing that the primary purpose of pursed-lip breathing is to: a. Promote oxygen intake b. Strengthen the diaphragm c. Strengthen the intercostal muscles d. Promote carbon dioxide elimination

b. Continuous bubbling in the water seal chamber Rationale: Continuous bubbling in the water seal chamber suggests an air leak, indicating a need for further action.

A nurse is assessing a client who has a chest tube in place following thoracic surgery. Which of the following findings indicates a need for further action? a. Fluctuation of drainage in the tubing with inspiration b. Continuous bubbling in the water seal chamber c. Drainage of 75 mL in the first hr after surgery d. Several small, dark-red blood clots in the tubing

a. Emphysema Rationale: The client with emphysema has hyperinflation of the alveoli and flattening of the diaphragm. These lead to increased anteroposterior diameter, referred to as barrel chest. The client also has dyspnea with prolonged expiration and has hyperresonant lungs to percussion.

A nurse is assessing a client with chronic airflow limitation and notes that the client has a "barrel chest." The nurse interprests that this client has which of the following forms of chronic airflow limitation? a. Emphysema b. Bronchial asthma c. Chronic obstructive bronchitis d. Bronchial asthma and bronchitis

c. Resuscitation equipment Rationale: The nurse administering naloxone for suspected opioid overdose should have resuscitation equipment readily available to support naloxone therapy if it is needed. Other adjuncts that may be needed include oxygen, mechanical ventilator, and vasopressors.

A nurse is preparing to administer a dose of naloxone hydrochloride (Narcan) intravenously to a client with an intravenous opioid overdose. Which supportive medical equipment would the nurse paln tohave at the client's bedside if needed? a. Nasogastric tube b. Paracentesis tray c. Resuscitation equipment d. Central line insertion tray

b. Particulate respirator, gown, and gloves Rationale: The nurse who is in contact with a client with tuberculosis should wear an individually fitted particulate respirator. The nurse also wears a gown when the possibility exits that the clohing could become contaminated, such as when giving a bed bath.

A nurse is preparing to give a bed bath to an immobilized client with tuberculosis. The nurse should wear which of the following items when performing this care? a. Surgical mask and gloves b. Particulate respirator, gown, and gloves c. Particulate respirator and protective eyewear d. Surgical mask, gown, and protective eyewear

d. "I should apply suction while inserting the catheter into my tracheostomy." Rationale: Suction should only be applied on withdrawal of the catheter to prevent tracheal tissue trauma.

A nurse is providing discharge instructions to a client following a tracheostomy. Which of the following statements by the client indicates a need for further instruction? a. "I need to inspect the stoma for signs of infection of skin irritation." b. "I will clean the cannula with half-strength peroxide and rinse with saline." c. "I can remove the old twill tape once the new tape is in place." d. "I should apply suction while inserting the catheter into my tracheostomy."

The nurse receives a change-of-shift report on the following patients with chronic obstructive pulmonary disease (COPD). Which patient should the nurse assess first?

A patient with a respiratory rate of 38 breaths/min

Which complication does the nurse suspect in a patient with breathing difficulty who has ecchymosis?

A patient with breathing difficulty may have sinusitis, rhinitis, or septal deviation. However, the patient has ecchymosis under the eyes, and the presence of ecchymosis is suggestive of orbital or basilar skull fracture. Rhinitis is suspected when the patient has a history of exposure to house molds, animal dander, or dust mites. Sinusitis is suspected when the nurse observes edematous mucosa and discolored purulent nasal drainage. Septal deviation is a deviation in the nasal septum.

Which type of foods should be given to a patient who has oral mucositis?

A patient with oral mucositis will have irritation and ulceration in the mouth. Bland and soft foods, such as fruit and vegetable juices, should be given to the patient to avoid irritation. Potato chips are hard to chew and can further irritate the oral mucosa. Pico de gallo and chicken fried steak may be spicy and can irritate the oral mucosa.

A patient has an infection caused by β-hemolytic streptococci. What condition does the nurse correlate with this bacterial infection?

A peritonsillar abscess is a complication of bacterial acute pharyngitis that is most often caused by β-hemolytic streptococci. Sinusitis is caused by rhinovirus and coronavirus and is characterized by inflammation of the sinuses. Fungal pharyngitis is a fungal infection caused by Candida albicans. Acute viral rhinitis is caused by a virus, but not by β-hemolytic streptococci.

The registered nurse has completed the training of a licensed practical nurse about his or her responsibilities. Which action of the licensed practical nurse needs correction?

A registered nurse, but not a licensed practical nurse, can assess the patient's risk of aspiration. A licensed practical nurse can perform suctioning of the tracheostomy tube, provide tracheostomy care or change the tracheostomy dressing, and evaluate the patient's health after suctioning.

Which statement best describes pneumonia

A serious inflammation of the bronchioles from various causes

A patient has been on a mechanical ventilator with an endotracheal tube for three weeks, and the primary health care provider wants to perform a tracheostomy. Which benefits are associated with tracheostomy in a patient? Select all that apply.

A tracheostomy is a surgically created opening to provide ventilation. It facilitates the removal of secretions, bypasses an airway obstruction, and provides long-term mechanical ventilation. Subcutaneous crepitus and narrowing of the trachea are complications associated with tracheostomy.

After morning care, the student nurse is to perform tracheostomy care under the RN's supervision. Which instructions does the RN give the student nurse? (Select all that apply.) Create a sterile field. Change trach ties if soiled. Remove old dressings and excess secretions. Suction the tracheostomy tube after the trach care. Clean the inner cannula with full-strength hydrogen peroxide.

A, B, C

The nurse recognizes that a patient with sleep apnea may benefit from which intervention? (select all that apply.) A. Weight loss B. Nasal mask to deliver BiPAP c. A change in sleeping position D. Medication to increase daytime sleepiness E. Position- fixing device that prevents tongue subluxation

A, B, C, E

An older patient is receiving standard multidrug therapy for tuberculosis (TB). The nurse should notify the health care provider if the patient exhibits which finding? A. Yellow-tinged skin B. Orange-colored sputum C. Thickening of the fingernails D. Difficulty hearing high-pitched voices

A. Yellow-tinged skin

While listening to the posterior chest of a patient who is experiencing acute shortness of breath, the nurse hears these sounds. How should the nurse document the lung sounds? Click here to listen to the audio clip a. Pleural friction rub b. Low-pitched crackles c. High-pitched wheezes d. Bronchial breath sounds

ANS: C Wheezes are continuous high-pitched or musical sounds heard initially with expiration. The other responses are typical of other adventitious breath sounds. DIF: Cognitive Level: Understand (comprehension) REF: 483 TOP: Nursing Process: Assessment MSC: NCLEX: Physiological Integrity

13. The nurse admits a patient who has a diagnosis of an acute asthma attack. Which statement indicates that the patient may need teaching regarding medication use? a. "I have not had any acute asthma attacks during the last year." b. "I became short of breath an hour before coming to the hospital." c. "I've been taking Tylenol 650 mg every 6 hours for chest-wall pain." d. "I've been using my albuterol inhaler more frequently over the last 4 days."

ANS: D The increased need for a rapid-acting bronchodilator should alert the patient that an acute attack may be imminent and that a change in therapy may be needed. The patient should be taught to contact a health care provider if this occurs. The other data do not indicate any need for additional teaching. DIF: Cognitive Level: Apply (application) REF: 482 TOP: Nursing Process: Assessment MSC: NCLEX: Physiological Integrity

Which is a serious complication of pharyngitis caused by group A streptococcal bacteria?

Acute glomerulonephritis Acute glomerulonephritis is a serious complication of a streptococcal group A infection, which may occur 7-10 days after the infection. Pulmonary empyema is a collection of pus in the pleural space caused by pneumonia or an infected effusion. Meningitis is an infection of the meninges of the brain and can be caused by bacteria, but is not a result of having had a group A streptococcal infection. Laryngitis can be a common result of an upper respiratory infection, but is not considered a serious complication of group A streptococcus.

A patient who has pneumonia reports having chest pain associated with inspiration. The nurse notifies the provider and anticipates implementing which order?

Administering analgesic medications to alleviate discomfort Pleuritic chest pain occurs with inspiration and is a common clinical manifestation in patients with pneumonia; analgesic medications are given to alleviate discomfort. This pain is caused by inflammation of the parietal pleura, not by an increase in infection, so another antibiotic is not indicated. Because the pain is associated with inspiration, it is not due to myocardial infarction, so cardiac enzyme testing is not indicated. Supplemental oxygen is used for hypoxemia.

A patient with laryngeal cancer is admitted to the medical-surgical unit the morning before a scheduled total laryngectomy. Which preoperative intervention can be accomplished by an LPN/LVN working on the unit?

Administering preoperative antibiotics and anxiolytics Administering medication is a skill within the LPN/LVN scope of practice. As a reminder, anxiolytics must be administered AFTER the operative consent has been signed, or the consent will be invalid. The patient's nutritional status and need for nutritional supplements should be assessed by the RN or a registered dietitian as part of the multidisciplinary care team. The surgeon is responsible for discussing the laryngectomy procedure, answering any questions, and having the patient sign the operative consent form. Patient teaching is the responsibility of the RN because it requires complex critical thinking skills.

a. Assess vital signs and respiratory staus and notify the physician of the findings.

After a bronchoscopy procedure, the patient coughs up sputum which contains blood. What is the best nursing intervention for this patient? a. Assess vital signs and respiratory status and notify the physician of the findings. b. Monitor the patient for 24 hours to see if blood continues in the sputum c. Send the sputum to the lab for cytology for possible lung cancer d. Reassure the patient this is a normal response after a bronchoscopy

Cover when outside, use mild soap, gargle with salt water, suck on ice chips, throat sprays (lidocaine), and soft clothing.

After the radiation therapy begins, the patient visits the clinic stating that her throat is sore, she is having difficulty swallowing, and the skin on her throat is red, tender, and peeling. What patient teaching should the nurse provide?

An older patient is diagnosed with pneumonia. To assist with comfort during the admission interview, what does the nurse do?

Allow the patient to rest at frequent intervals Patients with pneumonia often have pain, fatigue, and dyspnea, which can cause anxiety. The nurse should allow frequent rest periods and should pace the interview and assessment according to the patient's fatigue level. The patient should be allowed to choose whether to get into bed or remain up in a chair.

The nurse is scheduled to administer seasonal influenza vaccinations to the residents of a long-term care facility. What would be a contraindication to the administration of the vaccine to a resident?

Although current vaccines are highly purified and hypersensitivity reactions are extremely uncommon, a hypersensitivity to eggs precludes vaccination because the vaccine is produced in eggs. Advanced age and a history of respiratory illness are not contraindications for influenza vaccination.

Which statements about anthrax infection are correct? Select all that apply.

Although rarely occurring naturally, inhalation anthrax is nearly 100% fatal without treatment. As macrophages in the lungs engulf the anthrax spores, the organism leaves its capsule and replicates. Toxins produced by the organisms in the lungs create massive edema, suppressing neutrophil action Dyspnea, diaphoresis, and sudden onset of breathlessness are common in late stages of the disease

Gas exchange takes place where

Alveoli

b. Venturi mask Rationale: The venturi mask delivers the most accurate oxygen concentration. It is the best oxygen delivery system for the client with chronic airflow limitation because it delivers a precise oxygen concentration. The face tent, aerosol mask, and tracheostomy collar are also high-flow oxygen delivery systems, but most often are used to administer high humidity.

An oxygen delivery system is prescribed for a client with chronic obstructive pulmonary disease to deliver a precise oxygen concentration. Which oxygen delivery system would the nurse anticipate to be prescribed? a. Face tent b. Venturi mask c. Aerosol Mask d. Tracheostomy collar

The nurse knows that under normal physiologic conditions of tissue perfusion, a patient will have what percent of oxygen dissociate from the hemoglobin molecule? A) 25% B) 50% C) 75% D) 100%

Answer: B Rationale: Oxygen dissociates with the hemoglobin molecule based on the need for oxygen to perfuse tissues. Under normal conditions, 50% of hemoglobin molecules completely dissociate their oxygen molecules when blood perfuses tissues that have an oxygen tension (concentration) of 26 mm Hg. This is considered a "normal" point at which 50% of hemoglobin molecules are no longer saturated with oxygen. (See Figure 27-8.)

The nurse is caring for a patient with a cuffed tracheostomy and is aware the patient is at risk for developing which complication? A) Pneumothorax B) Tracheomalacia C) Subcutaneous emphysema D) Trachea-innominate artery fistula

Answer: B Rationale: Tracheomalacia can develop because of the constant pressure exerted by the cuff, causing tracheal dilation and erosion of cartilage. Pneumothorax can develop during any tracheostomy procedure if the thoracic cavity is accidentally entered. Subcutaneous emphysema can develop during any tracheostomy procedure if air escapes into fresh tissue planes of the neck. Trachea-innominate artery fistula can occur any time a malpositioned tube causes its distal tip to push against the lateral wall of the tracheostomy.

While suctioning a patient, vagal stimulation occurs. What is the appropriate nursing action? A) Instruct the patient to cough. B) Place the patient in a high Fowler's position. C) Oxygenate the patient with 100% oxygen. D) Instruct the patient to breathe slowly and deeply.

Answer: C Rationale: Vagal stimulation may occur during suctioning and result in severe bradycardia, hypotension, heart block, ventricular tachycardia, asystole, or other dysrhythmias. If vagal stimulation occurs, stop suctioning immediately and oxygenate the patient manually with 100% oxygen. Repositioning the patient, slow deep breathing, and coughing will not address the cardiovascular effects of vagal stimulation.

The nurse understands that the expected assessment for the older adult related to the natural aging process of the respiratory system includes which finding? A) Tightening of the vocal cords B) Decrease in residual volume C) Decrease in the anteroposterior diameter D) Decrease in respiratory muscle strength

Answer: D Rationale: As a person ages, vocal cords become slack, changing the quality and strength of the voice; the anteroposterior diameter increases; respiratory muscle strength decreases; and the residual volume increases.

The nurse is caring for a patient admitted to the ED after experiencing a fall while rock climbing. The patient has several facial fractures. Which objective assessment finding is most serious? A) Malaligned nasal bridge B) Blood draining from one of the nares C) Crackling of the skin (crepitus) upon palpation D) Clear glucose positive fluid draining from nares

Answer: D Rationale: Blood or clear fluid (cerebrospinal fluid, or CSF) may drain from one or both nares. However, the presence of glucose in the clear drainage indicates that CSF is draining, which could be caused by a skull fracture, a serious complication. A malaligned nasal bridge and crepitus may be observed when evaluating general facial fractures.

A patient whose tracheostomy was inserted 30 minutes ago is recovering in the postanesthesia recovery unit when the patient coughs and expels the tracheostomy tube. How should the nurse respond?

As long as the patient is not in acute respiratory distress after dislodging the tracheostomy tube, the nurse should use a sterile hemostat to maintain an open airway until a sterile tracheostomy tube can be reinserted into the tracheal opening. The nurse would not suction the tracheostomy opening at this point. If the patient is in respiratory distress, the nurse will use an Ambu bag and mask to ventilate the patient temporarily. The tracheostomy is an open surgical wound that has not had time to mature into a stoma.

Fine crackles, fine rales, high-pitched rales association:

Asbestosis Atelectasis Interstitial fibrosis Bronchitis Pneumonia Chronic pulmonary disease

In a long term care facility for older adults and immunocompromised patients, one employee and several patients have been diagnosed with influenza. What does the supervising nurse do to decrease risk of infection to other patients

Ask employees with flu symptoms to stay at home for up to 5 days after onset of symptoms

A patient with sleep apnea who has a new order for continuous positive airway pressure (CPAP) with a facemask returns to the outpatient clinic after 2 weeks with a report of ongoing daytime sleepiness. Which action should the nurse take first?

Ask the patient whether CPAP has been used consistently at night The nurse should assess whether the patient has actually consistently been using CPAP at night because patients may have difficulty with the initial adjustment to this therapy. With APAP, the pressures are adjusted continuously depending on the patient's needs; this may be more comfortable for the patient. Modafinil treats narcolepsy or daytime sleepiness; it does not treat the cause of sleep apnea, but it may be used to help some of the side effects of obstructive sleep apnea. A nasal mask may be an option for the patient if he or she is finding the facemask used with CPAP uncomfortable.

Thoracentesis

Aspiration of pleural fluid or air from pleural space. -Stinging sensation and feeling of pressure -Correct position -Motionless patient -Follow-up assessment for complications

The nurse received a client from the post-anesthesia care unit (PACU) who has a chest tube to a closed drainage system. Report from the PACU nurse included drainage in the chest tube at 80 mL of bloody fluid. Fifteen minutes after transfer from the PACU, the chest tube indicates drainage as pictured. The client is reporting pain at "8" on a scale of 0 to 10. The first action of the nurse is to: a) Administer prescribed pain medication. b) Notify the physician. c) Assess pulse and blood pressure. d) Lay the client's head to a flat position.

Assess pulse and blood pressure. The client has bled 120 mL of bloody drainage in the chest drainage system within 15 minutes. It is most important for the nurse to assess for signs and symptoms of hemorrhage, which may be indicated by a rapid pulse and decreasing blood pressure. The nurse may then lay the client in a flat position and notify the physician.

An older patient with pneumonia has become more confused during the initial assessment. What action will the nurse take initially?

Assess the patient's oxygen saturation Patients who have altered level of consciousness are often hypoxic. The nurse should assess oxygen saturation to evaluate the possible cause if this occurs. The nurse may evaluate orientation, but the oxygen saturation is more important and should be performed initially. It is not necessary to notify the Rapid Response Team. A bronchodilator medication is not indicated.

Lung Biopsy follow-up care:

Assess vital signs, breath sounds at least every 4 hours for 24 hours Assess for respiratory distress Report reduced/absent breath sounds immediately Monitor for hemoptysis

Which assessment procedure is used to clinically assess for a swallowing dysfunction? Select all that apply.

Assessment of a patient's ability to swallow is performed using videofluoroscopy, fiberoptic endoscopy, and by a speech physiotherapist. A charge nurse and chest physiotherapist do not assess for swallowing dysfunction.

The nurse is caring for a group of patients on the pulmonary unit. Which patient is at greatest risk for having pulmonary hypertension (PH)? 29-year old male who is overweight 32-year-old female with a family history of PH 43-year-old male with history of right-sided heart failure 50-year-old female with history of blood clots in the pulmonary artery

B

While the Rapid Response Team is at the bedside, the patient's healthcare provider arrives. The provider writes several orders. Which order is most important for the nurse to implement immediately? Transfer to ICU Increase O2 to 3 L per nasal cannula ABGs 30 min after oxygen is increased Methylprednisolone sodium succinate (Solu-Medrol) 40 mg IVP

B

The nurse is caring for an older adult client who has been admitted for dehydration and needs IV fluids. Which location does the nurse choose to place a peripheral IV on this client?

B [basilic vein] The most appropriate veins for peripheral IV therapy include the dorsal venous network and the basilic, cephalic, and median veins. However, an older client has poor skin turgor on the back of the hand, making this a poor selection. The palmar side of the wrist should be avoided because the median nerve is located there, causing increased pain and difficulty with stabilization. The cephalic vein, although large and prominent in most people, is not the best choice because the sensory branch of the median nerve intersects with it up to three times. The best choice is the basilic vein.

A patient has just been admitted with probable bacterial pneumonia and sepsis. Which order should the nurse implement first? A. Chest x-ray via stretcher B. Blood cultures from two sites C. Ciprofloxacin (Cipro) 400 mg IV D. Acetaminophen (Tylenol) rectal suppository

B. Blood cultures from two sites

Radiation therapy is planned for a patient with head and neck cancer. Which medication will the nurse administer to the patient before initiating the therapy?

Because radiation therapy can cause dry mouth, the nurse should administer pilocarpine hydrochloride medication to increase the production of saliva. Zanamivir is used in the treatment of influenza. Olopatadine is an antihistamine used in the treatment of allergic rhinitis. Pseudoephedrine is an oral decongestant used in medications for sinusitis.

The nurse teaches a patient who is experiencing oral mucositis due to radiation therapy about care measures. During a follow-up visit, the patient reports severe mouth irritation. Which action of the patient indicates the need for further teaching?

Because the oral mucosa is affected during radiation therapy, the patient will experience dry mouth and oral mucositis. Commercial mouthwashes can irritate the mouth severely, and the nurse should instruct the patient to avoid them. A patient who experiences oral mucositis will have pain; sucking ice chips can cause numbness and relieve the pain. Rinsing the mouth with salt and baking soda four to six times a day will promote healing. Equal proportions of antacid, diphenhydramine, and topical lidocaine reduces severe irritation.

b. The patient does not respond to supplemental oxygen

Before a bronchoscopy procedure, the patient received benzocaine spray as a topical anesthetic to numb the oropharynx. The nurse is assessing the patient after the procedure. Which finding suggests that the patient is developing methemoglobinemia? a. The patient has a decreased hematocrit level b. The patient does not respond to supplemental oxygen c. The blood sample is a bright cherry red color d. The patient experiences sedation and amnesia

The nurse is providing tracheostomy care to a patient. Which nursing actions should be a priority while providing this care? Select all that apply.

Before applying suction, the nurse should auscultate for lung sounds to check for the presence of rhonchi or coarse crackles; if they are present, the nurse can encourage the patient to cough up secretions. The nurse should remove the soiled dressing while providing tracheostomy care. Before starting the procedure, the nurse should explain the procedure to the patient to reduce anxiety. Tracheostomy care is provided as many times as needed per day. The patient should be in a semi-Fowler's position during tracheostomy care.

The nursing instructor is evaluating a student nurse's teaching plan about preventive measures for head and neck cancer in patients. Which statement of the student nurse does the nursing instructor correct?

Both chewing and smoking tobacco cause head or neck cancer; chewing tobacco should not be recommended. Cigarettes should generally be avoided. Alcoholics and people with poor oral hygiene are at greater risk of developing head and neck cancer.

Invasive Diagnostic Tests

Bronchoscopy Thoracentesis

A nurse is assigned to care for a client with a tracheostomy tube. How can the nurse communicate with this client? a) By suctioning the client frequently b) By supplying a magic slate or similar device c) By placing the call button under the client's pillow d) By providing a tracheostomy plug to use for verbal communication

By supplying a magic slate or similar device The nurse should use a nonverbal communication method, such as a magic slate, note pad and pencil, and picture boards (if the client can't write or speak English). The physician orders a tracheostomy plug when a client is being weaned off a tracheostomy; it doesn't enable the client to communicate. The call button, which should be within reach at all times for all clients, can summon attention but doesn't communicate additional information. Suctioning clears the airway but doesn't enable the client to communicate.

8. A student nurse is developing a teaching plan for an adult patient with asthma. Which teaching point should have the highest priority in the plan of care that the student is developing? A) Gradually increase levels of physical exertion. B) Change filters on heaters and air conditioners frequently. C) Take prescribed medications as scheduled. D) Avoid goose-down pillows.

C Feedback: Although all of the measures are appropriate for a client with asthma, taking prescribed medications on time is the most important measure in preventing asthma attacks.

A new nurse is observed caring for a patient who has a chest tube in place after a partial lobectomy. Which action by the nurse is inappropriate and requires a more experienced nurse to intervene? A. Positioning the patient in a semi-Fowler's position B. Encourage the patient to increase his fluid intake C. Clamping the chest tube while the patient is ambulating to the bathroom D. Administering the patient's prescribed narcotic analgesic before activity

C RATIONALE: Clamping of chest tubes is generally contraindicated. Clamping would allow a buildup of secretions or air in the pleural space and inhibit lung expansion. Patients with chest tubes are usually placed in the semi-Fowler's position for the sake of comfort and ease of breathing. Fluid intake is not a major concern in light of the information provided. Administering the patient's prescribed narcotic analgesic before activity is an expected standard of care for this patient.

An older patient with a history of arthritis has fallen after an episode of dizziness. Laboratory data reveal anemia and stool positive for occult blood. Which assessment question is the most appropriate for the patient's health situation? Select one: A. "What herbal remedies or supplements do you use regularly?" B. "Do you take any medication for high blood pressure?" C. "Do you take aspirin for the treatment of pain or inflammation?" D. "Does your family doctor ask you to get regularly scheduled blood work?"

C. "Do you take aspirin for the treatment of pain or inflammation?" Anemia and stool positive for occult blood could indicate the presence of gastrointestinal (GI) bleeding. Since aspirin is commonly implicated in episodes of GI bleeding this question would be the best to ask the patient at this time. The other questions may or may not relate to the patient's condition and are less likely to be related to anemia and blood in the stool.

The nurse discusses management of upper respiratory infections (URI) with a patient who has acute sinusitis. Which statement by the patient indicates that additional teaching is needed? A. "I can take acetaminophen (Tylenol) to treat my discomfort." B. "I will drink lots of juices and other fluids to stay well hydrated." C. "I can use my nasal decongestant spray until the congestion is all gone." D. "I will watch for changes in nasal secretions or the sputum that I cough up."

C. "I can use my nasal decongestant spray until the congestion is all gone."

A patient has acute bronchitis with a nonproductive cough and wheezes. Which topic should the nurse plan to include in the teaching plan? A. Purpose of antibiotic therapy B. Ways to limit oral fluid intake C. Appropriate use of cough suppressants. D. Safety concerns with home oxygen therapy

C. Appropriate use of cough suppressants.

Which of the following social changes is increasing the number and complexity of ethical dilemmas that nurses face? Select one: A. Diminished fiscal constraints B. Expansion of the hospice movement C. Greater numbers of older adults D. Outdated medical technologies

C. Greater numbers of older adults Nurses face more ethical dilemmas as a result of new medical technologies and increased fiscal constraints. The complexity is increased by larger numbers of older adults to serve. The expansion of hospice care helps nurses deal with ethical dilemmas.

You are providing care for an 82-year-old man whose signs and symptoms of Parkinson disease have become more severe over the past several months. The man tells you that he can no longer do as many things for himself as he used to be able to do. What factor should you recognize as impacting your patient's life most significantly? Select one: A. Age-related changes B. Neurologic deficits C. Loss of independence D. Tremors and decreased mobility

C. Loss of independence This patient's statement places a priority on his loss of independence. This is undoubtedly a result of the neurologic changes associated with his disease, but this is not the focus of his statement. This is a disease process, not an age-related physiological change.

When flushing a client's central line with normal saline, the nurse feels resistance. Which action does the nurse take first? A. Decrease the pressure being used to flush the line. B. Obtain a 10-mL syringe and reattempt flushing the line. C. Stop flushing and try to aspirate blood from the line. D. Use "push-pull" pressure applied to the syringe while flushing the line.

C. Stop flushing and try to aspirate blood from the line. If resistance is felt when flushing any IV line, the nurse should stop and further assess the line. Aspiration of blood would indicate that the central line is intact and is not obstructed by thrombus. Continuing or reattempting to flush the line, or using a push-pull action on the syringe, might result in thrombus or injection of particulate matter into the client's circulation.

The nurse teaches a patient with chronic bronchitis about a new prescription for Advair Diskus (combined fluticasone and salmeterol). Which action by the patient would indicate to the nurse that teaching about medication administration has been successful? A. The patient shakes the device before use. B. The patient attaches a spacer to the Diskus. C. The patient rapidly inhales the medication. D. The patient performs huff coughing after inhalation.

C. The patient rapidly inhales the medication.

The nurse is caring for a patient with chronic obstructive pulmonary disease (COPD). Which information obtained from the patient would prompt the nurse to consult with the health care provider before administering the prescribed theophylline? A. The patient reports a recent 15-pound weight gain. B. The patient denies any shortness of breath at present. C. The patient takes cimetidine (Tagamet) 150 mg daily. D. The patient complains about coughing up green mucus.

C. The patient takes cimetidine (Tagamet) 150 mg daily.

The nurse observes a student who is listening to a patient's lungs who is having no problems with breathing. Which action by the student indicates a need to review respiratory assessment skills? A. The student starts at the apices of the lungs and moves to the bases. B. The student compares breath sounds from side to side avoiding bony areas. C. The student places the stethoscope over the posterior chest and listens during inspiration. D. The student instructs the patient to breathe slowly and a little more deeply than normal through the mouth.

C. The student places the stethoscope over the posterior chest and listens during inspiration.

While caring for an elderly man, the nurse observes that his skin is dry and wrinkled, his hair is gray, and he needs glasses to read. Based on these observations, what would the nurse conclude? Select one: A. The observations are not typically found in older adults. B. Extra teaching will be necessary to prevent complications. C. These are normal physiologic changes of aging. D. These are abnormal observations and must be reported.

C. These are normal physiologic changes of aging. Dry wrinkled skin, gray hair, and needing glasses to read are all commonly occurring and normal physiologic changes of aging. They are not abnormal and do not lead to complications.

Employee health test results reveal a tuberculosis (TB) skin test of 16-mm induration and a negative chest x-ray for a staff nurse working on the pulmonary unit. The nurse has no symptoms of TB. Which information should the occupational health nurse plan to teach the staff nurse? A. Standard four-drug therapy for TB B. Need for annual repeat TB skin testing C. Use and side effects of isoniazid (INH) D. Bacille Calmette-Guérin (BCG) vaccine

C. Use and side effects of isoniazid (INH)

The nurse assesses a patient with a history of asthma. Which assessment finding indicates that the nurse should take immediate action? A. Pulse oximetry reading of 91% B. Respiratory rate of 26 breaths/minute C. Use of accessory muscles in breathing D. Peak expiratory flow rate of 240 L/minute

C. Use of accessory muscles in breathing

An elderly patient has presented to the clinic with a new diagnosis of osteoarthritis. The patient's daughter is accompanying him and you have explained why the incidence of chronic diseases tends to increase with age. What rationale for this phenomenon should you describe? Select one: A. Chronic illnesses are diagnosed more often in older adults because they have more contact with the health care system. B. Older adults often have less support and care from their family, resulting in illness. C. With age, biologic changes reduce the efficiency of body systems. D. There is an increased morbidity of peers in this age group, and this leads to the older adult's desire to also assume the "sick role."

C. With age, biologic changes reduce the efficiency of body systems. Causes of the increasing number of people with chronic conditions include the following: longer lifespans because of advances in technology and pharmacology, improved nutrition, safer working conditions, and greater access (for some people) to health care. Also, biologic conditions change in the aged population. These changes reduce the efficiency of the body's systems. Older adults usually have more support and care from their family members. Assuming the "sick role" can be a desire in any age group, not just the elderly.

A nurse teaches adults preventive measures to avoid problems of middle adult years. Which of the following are the major health problems during the middle adult years? Select one: A. upper respiratory infections, fractures B. communicable diseases, dementia C. cardiovascular disease, cancer D. sexually transmitted diseases, drug abuse

C. cardiovascular disease, cancer The major health problems of the middle adult years are cardiovascular and pulmonary diseases, cancer, rheumatoid arthritis, diabetes mellitus, obesity, alcoholism, and depression. The risk for these health problems often depends on a combination of lifestyle factors and aging.

Mrs. Ash, an 88-year-old woman who lives alone, has deficits in vision and hearing. Her blood pressure medicine is making her dizzy. What response to these health problems would the home health nurse identify? Select one: A. altered consciousness B. risk for impaired judgment C. risk for accidental injury D. decreased social interaction

C. risk for accidental injury The older adult is at increased risk for accidental injury because of changes in vision and hearing, loss of muscle mass and strength, slower reflexes and reaction time, and decreased sensory ability. The effects of chronic illness and medications may also make the older adult more prone to accidents.

Pulmonary Arterial Hypertension (PAH)

Cause unknown Blood vessels constrict with increasing vascular resistance in the lung Heart fails w/o treatment, death w/in 2 years

A client who is to be admitted for minor surgery has a chest radiograph as part of the presurgical physical. The nurse s notified that the radiograph reveals that the client has pulmonary TB. What evidence of TB is provided by the radiograph

Cavities caused by caseation

Which drug is used as a targeted therapy for head and neck cancer?

Cetuximab is an anticancer agent used as a targeted therapy in head and neck cancer after standard chemotherapy. Olopatadine is effective in the treatment of rhinitis. Mometasone is a corticosteroid used in the treatment of sinusitis and allergic rhinitis. Ipratropium bromide is an anticholinergic used to treat rhinitis.

Obstructive Sleep Apnea surgical management

Change of sleep position, weight loss, positive-pressure ventilation

A patient has been started on ethambutol for tuberculosis. What adverse effect requires the patient to notify the provider?

Changes in vision When taking ethambutol for tuberculosis, the patient should report any vision changes to the provider as the medication can cause optic neuritis. Darkening of the urine, yellowing appearance of the skin, and increased bleeding or bruising is associated with liver toxicity or failure and may be seen with isoniazid, rifampin, and pyrazinamide.

Patients with which risk factor warrant close monitoring for and education about cancers of the nose and sinuses?

Chronic exposure to dust from wood, leather, or flour Chronic exposure to dust from wood, leather, or flour is known to be a risk factor for cancers of the nose and sinuses; individuals routinely exposed to these materials in a dust form should be encouraged to wear an appropriate mask. A history of allergies and sinus infections does not increase the risk of nasal/sinus cancers. A history of needing to sleep with the head of the bed elevated should be explored further for cause but is not likely to be related. Although an individual with nasal/sinus cancer may experience respiratory symptoms, the presence of persistent nasal drainage is not a risk factor in and of itself.

A patient who has chronic exposure to textile dust is fearful about the risk for sinus cancer after a coworker developed the disease. To help assess risk, the nurse asks the patient about which other risk factor?

Cigarette Smoking Patients who have exposure to common workplace substances have an increased risk of sinus cancer from these substances if they smoke. Alcohol intake, dietary fat, and exercise habits are all part of the usual health history, but they do not have a special concern for determining risk for sinus cancer.

Following a bioterrorism attack with anthrax, the emergency department nurse checks the medication room for ample supply of which medications? Select all that apply.

Ciprofloxacin Doxycycline Amoxicillin Rifampin Vancomycin Ciprofloxacin, in combination therapy with one or more of the drugs doxycycline, amoxicillin, rifampin, and vancomycin, is used for exposure and actual infection. Therefore all of these medications should be on hand. Gentamicin is not a drug used to treat anthrax infection.

The nurse assesses a patient with a heart rate of 42 and a blood pressure of 70/46. What type of hypoxia does the nurse determine this patient is displaying? a) Hypoxic hypoxia b) Histotoxic hypoxia c) Circulatory hypoxia d) Anemic hypoxia

Circulatory hypoxia Given this patient's vital signs, he appears to be in shock. Circulatory hypoxia is hypoxia resulting from inadequate capillary circulation. It may be caused by decreased cardiac output, local vascular obstruction, low-flow states such as shock, or cardiac arrest. Although tissue partial pressure of oxygen (PO2) is reduced, arterial oxygen (PaO2) remains normal. Circulatory hypoxia is corrected by identifying and treating the underlying cause.

The nurse notices clear nasal drainage in a patient newly admitted with facial trauma, including a nasal fracture. What should the nurse do first?

Clear nasal drainage suggests leakage of cerebrospinal fluid (CSF). The drainage should be tested for the presence of glucose, which would indicate the presence of CSF. Suctioning should not be done. Documenting the findings and monitoring are important after notifying the health care provider. A drip pad may be applied, but the patient should not be reassured that this is normal.

Other indicators of respiratory adequacy

Clubbing of fingers Weight loss Unevenly developed muscles Skin and mucous membrane changes General appearance Activity tolerance

Chest Tube Chamber 1

Collects fluid draining from patient

III

Combination of I & II plus orbital-zygoma fracture; often called craniofacial disjunction

craniofacial disjunction

Combination of nasoethmoid complex fracture, maxillary fracture, and orbital- zygoma fracture

Which diagnostic test is not used to diagnose TB

Complete blood count

Which statements about acute viral rhinitis are correct? Select all that apply.

Complications are more common in immunosuppressed individuals It is most often caused by many different viruses.

Which patients are at risk for developing hospital acquired pneumonia

Confused patient, patient with gram negative colonization of the mouth, malnourished patient

Air leak Correct Explanation: The nurse needs to observe for air leaks in the drainage system; they are indicated by constant bubbling in the water seal chamber, or by the air leak indicator in dry systems with a one-way valve. Tidaling is fluctuation of the water level in the water seal that shows effective connection between the pleural cavity and the drainage chamber and indicates that the drainage system remains patent.

Constant bubbling in the water seal of a chest drainage system indicates which of the following problems? a) Tension pneumothorax b) Air leak c) Increased drainage d) Tidaling

After being dischareged from the hospital a patient is diagnosed with TB, at the outpatient clinic, what is the correct procedure regarding public health policy in this case

Contact the public health nurse so that all individuals who may have come in contact with the patient can be screened

A patient with pneumococcal pneumonia is being treated with intravenous antibiotics. On the fifth day of treatment, the nurse notes a productive cough with white mucus. Which nursing action is correct?

Continue the current plan of care and reassess the patient periodically The cough with pneumococcal pneumonia is typically productive of purulent rusty brown or yellow mucus; white mucus production indicates resolution of the infection. It is not necessary to administer a bronchodilator or a different antibiotic. The provider does not need to be notified.

The nurse is counseling a young woman about drug therapy with isoniazid (INH) and rifampin (RIF) to treat tuberculosis. Before developing the teaching plan, what must the nurse assess for first?

Contraceptive methods used

A patient with head and neck cancer has undergone a cordectomy. Which complication is associated with this procedure?

Cordectomy is surgical removal of the vocal cords, which causes impaired verbal communication. The patient may have anxiety due to lack of knowledge regarding a surgical procedure; anxiety is not specifically associated with cordectomy. Acute pain is related to tissue injury during surgery. The presence of an artificial airway and the accumulation of mucus in the airways increases risk of aspiration; these complications are not associated with cordectomy.

Cricothyroidotomy

Creation of temporary airway by making a small opening in the throat between the thyroid cartilage and the cricoid cartilage

Home Oxygen Therapy

Criteria for equipment Patient education: Compressed gas in tank or cylinder Liquid oxygen in reservoir Oxygen concentrator -Needs to have 88% O2 sat before you can send them home with O2

33. A nurse is providing health education to the family of a patient with bronchiectasis. What should the nurse teach the patients family members? A) The correct technique for chest palpation and auscultation B) Techniques for assessing the patients fluid balance C) The technique for providing deep nasotracheal suctioning D) The correct technique for providing postural drainage

D Feedback: A focus of the care of bronchiectasis is helping patients clear pulmonary secretions; consequently, patients and families are taught to perform postural drainage. Chest palpation and auscultation and assessment of fluid balance are not prioritized over postural drainage. Nasotracheal suctioning is not normally necessary.

The nurse teaches a patient about discharge instructions after a rhinoplasty. Which statement, if made by the patient, indicates that the teaching was successful? A. "I can take 800 mg ibuprofen for pain control." B. "I will safely remove and reapply nasal packing daily." C. "My nose will look normal after 24 hours when the swelling goes away." D. "I will keep my head elevated for 48 hours to minimize swelling and pain."

D. "I will keep my head elevated for 48 hours to minimize swelling and pain."

The nurse provides discharge teaching for a patient who has two fractured ribs from an automobile accident. Which statement, if made by the patient, would indicate that teaching has been effective? A. "I am going to buy a rib binder to wear during the day." B. "I can take shallow breaths to prevent my chest from hurting." C. "I should plan on taking the pain pills only at bedtime so I can sleep." D. "I will use the incentive spirometer every hour or two during the day."

D. "I will use the incentive spirometer every hour or two during the day."

In response to a patient's complaint of pain, the nurse administered a PRN dose of hydromorphone (Dilaudid). In what phase of the nursing process will the nurse determine whether this medication has had the desired effect? Select one: A. Data collection B. Analysis C. Assessment D. Evaluation

D. Evaluation Evaluation, the final step of the nursing process, allows the nurse to determine the patient's response to nursing interventions and the extent to which the objectives have been achieved.

A patient with pneumonia has a fever of 101.4° F (38.6° C), a nonproductive cough, and an oxygen saturation of 88%. The patient complains of weakness, fatigue, and needs assistance to get out of bed. Which nursing diagnosis should the nurse assign as the highest priority? A. Hyperthermia related to infectious illness B. Impaired transfer ability related to weakness C. Ineffective airway clearance related to thick secretions D. Impaired gas exchange related to respiratory congestion

D. Impaired gas exchange related to respiratory congestion

Nurse H is providing care in the hospital for a 71-year-old male patient who is in the late stages of cancer and who has painful bone metastases. The client is non-responsive but groans and grimaces intermittently. Nurse H is drawing up a breakthrough dose of morphine for the patient, but Nurse R cautions that, "sure, that will address his pain, but it could depress his respiratory drive and actually kill him at this stage." Which of the following ethical principles is Nurse R prioritizing? Select one: A. Justice B. Fidelity C. Beneficence D. Nonmaleficence

D. Nonmaleficence Nurse R's emphasis on preventing harm to the patient, even during an act that may be motivated by altruism, is characteristic of the principle of nonmaleficence.

A patient who has just been admitted with community-acquired pneumococcal pneumonia has a temperature of 101.6° F with a frequent cough and is complaining of severe pleuritic chest pain. Which prescribed medication should the nurse give first? A. Codeine B. Guaifenesin (Robitussin) C. Acetaminophen (Tylenol) D. Piperacillin/tazobactam (Zosyn)

D. Piperacillin/tazobactam (Zosyn)

The nurse assists in the surgical removal of eschar from a trochanter pressure ulcer revealing the bone and tendons. The nurse correctly stages this ulcer as what stage? Select one: A. Stage 2 B. Stage 3 C. Stage 1 D. Stage 4

D. Stage 4 With stage 4 the subcutaneous tissue is lost, exposing muscle, bone, or both. Eschar must be removed to fully stage and treat a pressure ulcer.

The nurse teaches a patient who has asthma about peak flow meter use. Which action by the patient indicates that teaching was successful? A. The patient inhales rapidly through the peak flow meter mouthpiece. B. The patient takes montelukast (Singulair) for peak flows in the red zone. C. The patient calls the health care provider when the peak flow is in the green zone. D. The patient uses albuterol (Proventil) metered dose inhaler (MDI) for peak flows in the yellow zone.

D. The patient uses albuterol (Proventil) metered dose inhaler (MDI) for peak flows in the yellow zone.

A nurse is administering a medication that is formulated as enteric-coated tablets. What is the rationale for not crushing or chewing enteric-coated tablets? Select one: A. to facilitate absorption in the stomach B. to prevent absorption in the esophagus C. to prevent absorption in the mouth D. to prevent gastric irritation

D. to prevent gastric irritation Enteric-coated tablets are covered with a hard surface to impede absorption until the tablet has left the stomach. Enteric-coated tablets should not be chewed or crushed because the active ingredient of the drug is irritating to the gastric mucosa.

Hypoxia

Decrease in PaO2 levels causing signs of inadequate oxygenation

The nurse understands that the expected assessment for the older adult related to the natural aging process of the respiratory system includes what findings? (4)

Decrease in respiratory muscle strength, loosening of the vocal chords, increase in residual volume, and increase in anteroposterior diameter

CPAP

Delivers set positive airway pressure throughout each cycle of inhalation and exhalation Opens collapsed alveoli Used for atelectasis after surgery or cardiac-induced pulmonary edema; sleep apnea

Dyspnea

Difficulty breathing, shortness of breath

An adult patient diagnosed with rhinitis medicamentosa reports chronic nasal congestion. What does the nurse instruct the patient to do

Discontinue the use of the current nose drops or spray

Perfusion

Distribution of oxygen to the tissue and cells

Patients who are at high risk for TB would be asked which questions upon assessment

Do you have an immune dysfunction or HIV, do you abuse alcohol or inject recreational drugs, do you work in a crowded area such as a prison or mental health facility

Which action by the student nurse when providing tracheostomy care to a patient indicates a need for further teaching?

Dried secretions from the stoma are removed using gauze that is soaked in sterile water or normal saline, but not in alcohol. Hand hygiene reduces the risk of infection to the patient. Tracheostomy care is performed by positioning the patient in a semi-Fowler's position. The nurse can suction as needed when the patient is unable to cough up the thick and hard secretions.

The laryngoscopy report of a patient indicates the presence of a cancerous mass over the vocal cords. The patient is to be treated with radiation therapy. How can a nurse help the patient cope with the side effects of radiation therapy? Select all that apply.

Dry mouth is the most common side effect of radiation therapy. The nurse should instruct the patient to increase fluid intake because hydration will help the patient to relieve symptoms. Due to radiation therapy, the patient's saliva decreases in volume and becomes thick. Pilocarpine hydrochloride is helpful in increasing saliva production. Soft and bland foods cause less irritation to the oral mucosa and should be encouraged if the patient has oral mucositis. Alcohol cessation should be encouraged following the diagnosis of cancer to avoid complications. Artificial saliva also helps in keeping the mouth hydrated in xerostomia.

Routine tracheostomy care is on the treatment plan for a patient that has a fenestrated tracheostomy tube. What should be the order of actions performed by the nurse for proper tracheostomy care?

During routine tracheostomy care, the required sterile equipment should be assembled near the patient's bed. Hands should be washed before the procedure. Gloves and goggles are to be worn. At first, the inner cannula is unlocked and cleaned in sterile water or saline solution. The inner cannula should be replaced after cleaning. The stoma is then cleaned with sterile gauze to remove dried secretions. Retention sutures, if present, should be properly positioned and secured. Subsequently, tracheostomy ties are changed.

Clinical Manifestations of Respiratory Distress

Dyspnea Nasal flaring Use of accessory muscles to breathe Pursed-lip or diaphragmatic breathing Decreased endurance Skin, mucous membrane changes (pallor, cyanosis)

In which scenario is a patient likely to require emergency endotracheal intubation?

Dyspnea and stridor have occurred with bilateral vocal cord paralysis Bilateral vocal cord paralysis will not allow airflow into the trachea; this leads to a respiratory emergency and may require intubation. When the patient is unable to close a vocal cord due to paralysis, the risk is for aspiration during eating and drinking. Immediate coughing upon swallowing liquids may indicate aspiration and require treatment, but not endotracheal intubation. Polyps on the vocal cord can become edematous and will then interfere with closure of the vocal cords; this would also contribute to the risk for aspiration.

The nurse is caring for a patient who underwent tracheostomy. Which nursing intervention is likely to cause aspiration?

Eating watermelon Watermelon should be avoided because thin liquids cause difficulty in swallowing and aspiration. Thickened liquids should be given to the patient because they do not have a risk of aspiration. The patient should be given smaller and more frequent meals, which will reduce the risk of aspiration. The patient should be placed in an upright position to avoid aspiration.

After suctioning a tracheostomy tube, the nurse assesses the client to determine the effectiveness of the suctioning. Which findings indicate that the airway is now patent? a) Effective breathing at a rate of 16 breaths/minute through the established airway b) Increased pulse rate, rapid respirations, and cyanosis of the skin and nail beds c) Restlessness, pallor, increased pulse and respiratory rates, and bubbling breath sounds d) A respiratory rate of 28 breaths/minute with accessory muscle use

Effective breathing at a rate of 16 breaths/minute through the established airway Proper suctioning should produce a patent airway, as demonstrated by effective breathing through the airway at a normal respiratory rate of 12 to 20 breaths/minute. The other options suggest ineffective suctioning. A respiratory rate of 28 breaths/minute and accessory muscle use may indicate mild respiratory distress. Increased pulse rate, rapid respirations, and cyanosis are signs of hypoxia. Restlessness, pallor, increased pulse and respiratory rates, and bubbling breath sounds indicate respiratory secretion accumulation.

A patient with pneumonia has difficulty clearing secretions from the airway. Which nursing intervention does the nurse include in this patient's plan of care?

Encourage an intake of 2 liters of fluid per day Hydration is essential to help liquefy secretions so they can be mobilized more easily. The alert patient should be encouraged to drink at least 2 L per day. The other interventions may help indirectly and are part of the overall nursing management of pneumonia.

Muscle Strength change due to age-nursing interventions

Encourage pulmonary hygiene, and help patient actively maintain health and fitness.

Susceptibility to Infection change due to age-nursing interventions

Encourage pulmonary hygiene, and help patient actively maintain health and fitness.

The nurse is caring for a patient who has had abdominal surgery. Which action does the nurse take to help prevent pulmonary infection in this patient?

Encourage regular use of an incentive spirometer Postoperative patients, especially those who have had abdominal surgery, are less likely to take deep breaths and cough, so they do not clear their lungs of mucus, increasing their risk of pulmonary infection. Encouraging use of an incentive spirometer can help with this. Low-molecular-weight heparin is given to prevent blood clots and pulmonary emboli, but not infection. Intravenous antibiotics are usually not given prophylactically unless there is increased risk. Adequate analgesia may be a necessary adjunct to incentive spirometry to assist with comfort while taking deep breaths.

A patient is taking isoniazid, rifampin, pyrazinamide, and ethambutol for tuberculosis. The patient calls to report visual changes, including blurred vision and reduced visual fields. Which medication may be causing these changes?

Ethambutol

Pulmonary Function Tests

Evaluate lung volumes and capacities, flow rates, diffusion capacity, gas exchange, airway resistance, distribution of ventilation

The nurse assesses that an unconscious patient is struggling to breathe. What is the nurse's first action?

Extend the patient's head and neck. Patients who are unconscious often have airway obstruction caused by the tongue falling back or by excessive secretions. The initial step is to extend the head and neck. Inserting an oral or nasal airway device and suctioning may be done next. Once the airway is established, oxygen may be necessary.

If decannulation occurs after 72 hours? What should you do.

Extend the patients neck and open the tissues of the stoma with a curved Kelly clamp to secure the airway. With the obturator inserted into the tracheostomy tube, quickly and gently replace the tube and remove the obturator. Check for airflow thorugh the tube and for bilateral breath sounds. Notify more experienced nurse if unable.

Assessment of the Nose & Sinuses

External nose - Deformities or tumors Nares - Symmetry of size and shape Nasal cavity - Color, swelling, drainage, bleeding Mucous membranes - Abnormalities Septal deviation Turbinates

T/F: After the proper dose of epinephrine is administered to a patient having an anaphylactic reaction, the nurse must wait 60 min before repeating the dose

FALSE

T/F: Latex allergies are manifested only in the dermatitis, so latex free gloves are the only neccessary precautions for those with latex allergy

FALSE

What test is used to diagnose asthma?

FEV1 Forced expiratory volume in first second

Class III of Dyspnea

Fair, moderate breathlessness. Must stop during activity. Complete performance is possible without assistance, but performance may be too debilitating or time consuming.

What type of respirations are found in patients with COPD?

Fast and shallow 40-50 breaths per minute

What are side effects associated with radiation therapy? Select all that apply

Fatigue, dry mouth, and oral mucositis are side effects of radiation therapy. Constipation is a side effect associated with antihistamines. Nasal irritation is a side effect associated with the use of anticholinergics and antihistamines.

After several weeks of not feeling well a patient is seen in the physicians office for possible TB, if TB is present which assessment finding does the nurse expect to observe

Fatigue, night sweats, low grade fever

Characterisitcs of adventitious breath sounds

Fine crackles, fine rales, high-pitched rales Coarse crackles, low-pitched crackles Wheeze Rhonchus (rhonchi) Pleural friction rub

Laryngectomy Postoperative Care

First priorities are airway maintenance and ventilation Wound, flap, reconstructive tissue care Hemorrhage Wound breakdown Pain management Nutrition Speech and language rehabilitation

Laryngectomy Postoperative Care

First priorities are airway maintenance and ventilation**** Wound, flap, reconstructive tissue care Hemorrhage Wound breakdown Pain management Nutrition Speech and language rehabilitation

The nurse is caring for a child and observes widening of the nostrils, restlessness, and auscultates expiratory wheezes. What is the priority action by the nurse?

Flaring or widening nostrils, wheezing or whistling sounds during expiration, and restlessness indicate airway obstruction in the patient. The priority intervention is to maintain a patent airway until further treatment can be initiated. An IV line can be inserted but is not the priority action at this time. ABC's (airway, breathing, and circulation) are the priority in order to prevent a poor outcome. Food and fluid should be withheld until the airway is unobstructed to prevent aspiration. The child should not lie flat at this time, and the head of the bed should be elevated.

After the radiation therapy begins, the patient visits the clinic stating that her throat is sore, she is having difficulty swallowing, and the skin on her throat is red, tender, and peeling. What patient teaching should the nurse provide?

Gargle with saline Suck on ice chips Throat numbing spray

Which nursing intervention should the nurse include in the teaching plan for a patient with acute pharyngitis whose laboratory reports indicate the presence of a candidiasis infection?

Gargling with warm salt water helps in relieving swelling and discomfort in the throat. The patient may have aspirin or ibuprofen if he or she reports pain. Lemon is a citrus fruit and would result in more throat irritation. Warm and cool fluids are appropriate for patients with pharyngitis related to candida.

The nurse is working in the hospital during the flu season and knows that what is true?

Generalized myalgia or body aches are common flu symptoms. The onset of flu is abrupt and not insidious. Anorexia occurs, but not vomiting and diarrhea. Nuchal rigidity is impaired neck flexion resulting from muscle spasms of the neck and is related to meningeal irritation.

Cystic Fibrosis

Genetic disease affecting many organs Lethally impairing pulmonary function Error of chloride transport and produce thick mucus with low water content Mucus plugs up glands causing atrophy and organ dysfunction No cure average age 37

What differentiates cerebrospinal fluid leakage from normal nasal secretions?

Glucose Content Cerebrospinal fluid contains glucose and this can be tested using a dipstick test. In other nasal secretions, glucose is absent. Viscosity refers to the thickness of a fluid and both cerebrospinal fluid and nasal secretions are thin. Cerebrospinal fluid and nasal secretions will not contain lipid.

Which virus is a strain of the bird flu?

H5N1 H5N1 is the viral strain that causes bird flu. H1N7, H1N1, and H1N5 are the virus types that cause swine flu

Which statement is true about community-acquired pneumonia (CAP) as compared to health care-associated pneumonia (HAP)?

HAP's are more likely to be resistant to some antibiotics

Which actions known as the "ventilator bundle" have been shown to reduce the incidence of ventilator-associated pneumonia (VAP)? Select all that apply.

HOB Oral Care Hand Hygiene

Pharynx and Larynx change due to age-nursing interventions

Have face-to-face conversions with patient when possible.

An older adult patient often coughs and chokes while eating or trying to take medications, the patient insists that he is okay; but the nurse identifies the priority patient problem of risk for aspiration, which nursing interventions are used to prevent aspiration pneumonia

Head of bed should always be elevated during feeding, monitor the patients ability to swallow small bites, consult a nutritionist and obtain swallowing studies, monitor the patients ability to swallow saliva

Nursing Assessment of COPD

Health history Physical examination Clinical manifestations Labs Pulmonary function tests (PFTs)

The nurse should refer a client to the pulmonary clinic for suspected TB based on which clinical indicators reported during the initial interview

Hemoptysis, night sweats

High-Flow Oxygen Delivery Systems

High-flow—can deliver 24%-100% at 8-15 L/min Venturi mask Face tent Aerosol mask Tracheostomy collar T-piece

Which condition causes a patient to have the greatest risk for community acquired pneumonia

History of tobacco use

Which is a clinical manifestation of an early sign of laryngeal cancer?

Hoarseness for more than two weeks is an early sign of laryngeal cancer. Discolored purulent nasal drainage and tenderness at the ethmoidal sinuses are signs of acute sinusitis. Rhinorrhea, a condition where the nasal cavity is filled with mucous fluid, is a sign for influenza.

Types of Pneumonia

Hospital Acquired Pneumonia Healthcare Associated Pneumonia Ventilator Associated Community-Acquired

The nurse is caring for a patient who is scheduled to have a thoracotomy. When planning preoperative teaching, what information should the nurse communicate to the patient? a) How to splint the incision when coughing b) How to take prophylactic antibiotics correctly c) How to milk the chest tubing d) How to manage the need for fluid restriction

How to splint the incision when coughing Prior to thoracotomy, the nurse educates the patient about how to splint the incision with the hands, a pillow, or a folded towel. The patient is not taught how to milk the chest tubing because this is performed by the nurse. Prophylactic antibiotics are not normally used and fluid restriction is not indicated following thoracotomy.

The nurse is giving discharge instructions to a patient diagnosed with a viral pharyngitis. Which statement by the patient indicated need for further teaching

I will wait for my test results, then I can get a prescription for antibiotics

Le Fort Fractures

I — Nasoethmoid complex fracture II — Maxillary and nasoethmoid complex fracture III — Combination of I & II plus orbital-zygoma fracture; often called craniofacial disjunction

Which task can the registered nurse (RN) delegate to an a properly trained unlicensed assistive personnel (UAP) in the care of a stable patient who has a tracheostomy?

If the UAP have been trained in correct technique, UAP may suction the patient's oropharynx. Assessing the need for suctioning should be performed by an RN or licensed practical nurse, whereas swallowing assessment and the maintenance of cuff inflation pressure should be performed solely by the RN.

a. Encourage the patient to ambulate and change positions

In the older adult, there is a decreased number of functional alveoli. To assist the patient to compensate for this change related to aging, what does the nurse do? a. Encourage the patient to ambulate and change positions b. Allow the patient to rest and sleep frequently c. Have face-to-face conversations when possible d. Obtain an order for supplemental oxygen

c. There is an increase in anteroposterior ratio

In the older adult, there is a loss of elastic recoiling of the lung and decreased chest wall compliance. What is the result of this occurrence? a. The thoracic area becomes shorter b. The patient has an increased activity tolerance c. There is an increase in anteroposterior ratio d. The patient has severe shortness of breath

Aphonia

Inability to produce sound

Which statement about obstructive sleep apnea is correct?

Increased carbon dioxide levels stimulate neural centers to awaken the sleeper. When apnea occurs, changes in blood gases awaken the sleeper who repositions to correct the obstruction and then returns to sleep. Patients are able to sleep but do not get restful or deep sleep. Recurrent sleep apnea does not cause irreversible brain changes. Untreated obstructive sleep apnea does not cause neurologic apnea.

Which outcome indicates effectiveness modafinil administered to a patient with sleep apnea?

Increased daytime wakefulness The drug modafinil promotes daytime wakefulness in patients with sleep apnea. Hypertension is a long-term effect of untreated sleep apnea. Untreated sleep apnea results in loss of deep sleep at night. Having increased feelings of exhaustion after waking is a symptom of untreated sleep apnea.

Incentive spirometry for the treatment of pneumonia has which outcome objective?

Increased inspiratory muscle action and decreased atelectasis Incentive spirometry helps improve inspiratory muscle action and prevents or reverses atelectasis. It does not increase respiratory effort, reduce crackles and wheezes, or reduce sputum production.

Hypercarbia

Increased partial pressure of arterial carbon dioxide PACO2 levels

Fremitus

Increased vibrations felt on the chest wall

What are 2 ways that asthma occurs?

Inflammation Airway hyperresponsiveness leading to bronchoconstriction

Wheeze association:

Inflammation Bronchospasm Edema Secretions Pulmonary vessel engorgement (as in cardiac "asthma")

Chronic Bronchitis

Inflammation of bronchi and bronchioles cause by constant exposure to irritants Affects only airways, not alveoli Produces large amounts of thick mucus

Orotracheal

Inserting a tube into the trachea via the mouth

A nurse is caring for a patient who appears cachectic and pale but appears in no acute distress. The patient tells the nurse that he has had a chronic cough for months and produces a large amount of foul-smelling sputum. He also states that he occasionally suffers from a stabbing pain when taking a deep breath. When reviewing the patient's history, the nurse notes that the patient has a recent history of influenza. Breath sounds reveal decreased sound with rhonchi to the right lower lobe and percussion to the right lower lobe is dull. What procedure does the nurse anticipate preparing for?

Insertion of a thoracentesis needle and drainage

Lungs change to age-rationales

Inspection, palptation, percussion, and auscultation are needed to detect normal age-related changes. Health and fitness help keep losses in respiratory functioning to a minimum. Periodic breathing patterns (Cheynes Stokes) can occur. Oral hygiene aids in the removal of secretions.

The clinic nurse makes a follow-up telephone call to a patient with asthma. The patient reports having a baseline peak flow reading of 600 L/min, and the current peak flow is 420 L/min. Which action should the nurse take first?

Instruct the patient to use the prescribed albuterol (Proventil).

Which factor helps prevent wound breakdown?

Intake of a protein-rich diet Intake of a protein-rich diet promotes wound healing. Wound breakdown is caused by poor nutrition, alcohol use, radiation therapy, and smoking.

A patient with sinusitis is prescribed an anticholinergic nasal spray. Regarding which nasal spray should the nurse educate the patient?

Ipratropium bromide is an anticholinergic nasal spray used to treat sinusitis. Phenylephrine is a decongestant nasal spray used to treat rhinitis and sinusitis. Triamcinolone is a corticosteroid nasal spray used to treat sinusitis. Cromolyn spray is a mast cell stabilizer nasal spray used to treat rhinitis and sinusitis.

A patient has an HIV infection but the TB skin test shows an induration of less than 10 mm and no clinical symptoms of TB are present, which medication does the patient receive for a period of 12 months to prevent TB

Isoniazid (INH)

A client who has been homeless and has spent the past 6 months living in shelters has been diagnosed with TB. Which medications does the nurse expect to be ordered for this client?

Isoniazid (INH), rifampin (Rifadin), pyrazinamide (PZA), ethambutol (Myambutol)

The nurse is caring for a patient who needs education on his medication therapy for allergic rhinitis. The patient is to take Cromolyn Nasalcrom daily. In providing education for this patient, how should the nurse describe the action of the medication?

It inhibits the release of histamine and other chemicals

NURSING SAFETY PRIORITY: Temp of air entering a tracheostomy

Keep the temperature of the air entering a tracheostomy between 98.6 and 100.6 F, and never exceed 104F

A patient who is experiencing an acute asthma attack is admitted to the emergency department. Which assessment should the nurse complete first?

Listen to the patient's breath sounds.

A patient recovering from pneumonia tells the nurse that his sputum smells bad. The nurse suspects the patient may have developed what condition?

Lung abscess A lung abscess may occur after pneumonia, aspiration, or obstruction. It is characterized by pleuritic chest pain, fever, and foul-smelling sputum. Tuberculosis is an airborne disease that produces sputum, cough, weight loss, and hemoptysis. Pulmonary empyema is a collection of pus in the pleural space commonly caused by an infection or a pulmonary abscess. Severe acute respiratory syndrome is an inflammatory respiratory disorder that is easily spread airborne.

Hemoglobin

Male 14 to 18 g/dL Female 12-16 g/dL

II

Maxillary and nasoethmoid complex fracture

Caponometry and capnography

Measure amount of carbon dioxide present in exhaled air Normal pressure or PETCO2 is between 20 and 40 mm Hg

For a client who has a chest tube connected to a closed water-seal drainage system, the nurse should include which action in the care plan? a) Measuring and documenting the drainage in the collection chamber b) Maintaining continuous bubbling in the water-seal chamber c) Keeping the collection chamber at chest level d) Stripping the chest tube every hour

Measuring and documenting the drainage in the collection chamber The nurse should regularly measure and document the amount of chest tube drainage to detect abnormal drainage patterns, such as may occur with a hemorrhage (if excessive) or a blockage (if decreased). Continuous bubbling in the water-seal chamber indicates a leak in the closed chest drainage system, which must be corrected. The nurse should keep the collection chamber below chest level to allow fluids to drain into it. The nurse shouldn't strip chest tubes because doing so may traumatize the tissue or dislodge the tube.

The nurse is caring for a pediatric patient with pertussis who is currently in the catarrhal phase of the illness. What manifestations will the nurse most likely find on assessment of this patient?

Mild cough Pertussis occurs in three distinct phases. During the first (catarrhal) phase, the patient may present with signs and symptoms of the common cold, including a mild cough. After 1 or 2 weeks, the paroxysmal phase occurs, characterized by a severe cough and bloody sputum, and potentially complicated by pneumonia. The third phase is the convalescent phase, which can last for several months.

The nurse is admitting a patient diagnosed with an acute exacerbation of chronic obstructive pulmonary disease (COPD). How should the nurse determine the appropriate O2 flow rate?

Minimize oxygen use to avoid oxygen dependency.

A nurse caring for a client with sever dyspnea who is receiving oxygen via a venturi mask, what should the nurse do when caring for the client

Monitor oxygen saturation levels when eating

Stridor

Monotone whistling noise that is high pitched It can vary from being musical in tone to rough and raspy and in most cases, it is more pronounced upon inspiration (breathing in)

Assessment of the Pharynx, Trachea, & Larynx

Mouth Posterior pharynx Neck - Symmetry, alignment, masses, swelling, bruises, use of accessory neck muscles for breathing Trachea - Palpate for position, mobility, tenderness, masses

Closed Reduction

Moving the bones by palpation to realign them

Which statements about pulmonary tuberculosis (TB) are correct? Select all that apply.

Mycobacterium tuberculosis is transmitted from person to person via the airborne route. Infected people are not infectious to others until manifestations of the disease occur. An asymptomatic period of up to years or decades can follow the time of primary infection Foreign immigrants, especially from Mexico, the Philippines, and Vietnam, are at greatest risk

Oxygen Therapies

Nasal Cannula Simple Facemask Venturi Mask Partial Rebreather Non-rebreather mask High-Flow Oxygen delivery systems

The nurse is caring for a patient who is experiencing mild shortness of breath during the immediate postoperative period, with oxygen saturation readings between 89% and 91%. What method of oxygen delivery is most appropriate for the patient's needs? a) Simple mask b) Nasal cannula c) Partial-rebreathing mask d) Non-rebreathing mask

Nasal cannula A nasal cannula is used when the patient requires a low to medium concentration of oxygen for which precise accuracy is not essential. The Venturi mask is used primarily for patients with COPD because it can accurately provide an appropriate level of supplemental oxygen, thus avoiding the risk of suppressing the hypoxic drive. The patient's respiratory status does not require a partial- or non-rebreathing mask.

What clinical manifestation related to respiratory difficulty does the nurse expect to observe in a patient who has experienced laryngeal trauma?

Nasal flaring Laryngeal trauma is likely to cause obstruction of the trachea. Nasal flaring is a manifestation of the body's attempt to compensate and bring in more oxygen. If the patient cannot compensate and bring in adequate oxygen, the saturation level will gradually decrease. The CO2level will most likely increase. Humidification may be indicated for long-term use, especially for high-flow oxygen, but poor response to humidification is not a manifestation of respiratory difficulty. Increased nasopharyngeal secretions would not necessarily be present as a result of laryngeal trauma.

A child diagnosed with a group B strep throat infection, in teaching the parents about treatment of the infection, what does the nurse instruct the parents

Need to complete penicillin or penicillin like antibiotics

Of the following oxygen administration devices, which has the advantage of providing high oxygen concentration? a) Venturi mask b) Catheter c) Non-rebreather mask d) Face tent

Non-rebreather mask The non-rebreather mask provides high oxygen concentration but it is usually poor fitting. The Venturi mask provides low levels of supplemental oxygen. The catheter is an inexpensive device that provides a variable fraction of inspired oxygen and may cause gastric distention. A face tent provides a fairly accurate fraction of inspired oxygen, but is bulky and uncomfortable. It would not be the device of choice to provide high oxygen concentration.

Pulmonary function testing

Noninvasive Evaluate lung volumes and capacities, flow rates, diffusion capacity, gas exchange, airway resistance, distribution of ventilation

Capnometry & Capnography

Noninvasive Measure amount of carbon dioxide present in exhaled air Normal pressure of PETCO2 is between 20 and 40 mm Hg

Epistaxis

Nosebleed is a common problem Cauterization of affected capillaries may be needed; nose is packed Posterior nasal bleeding is an EMERGENCY! Assess for respiratory distress, tolerance of packing or tubes Humidification, oxygen, bedrest, antibiotics, pain medications

A patient who has been experiencing an asthma attack develops bradycardia and a decrease in wheezing. Which action should the nurse take first?

Notify the health care provider

Educational Needs of the Patient: Nutrition, Speech communication

Nutrition-Swallowing may be more difficult -Encourage smaller, more frequent meals -Thicken liquids -Avoid water and other "thin" lquids -Sit upright while eating

Describe the Oxygen/Carbon Dioxide Exchange process

O2 is transferred from the air to the tissues and CO2 is excreted in the expired air

Subcutaneous emphysema

Occurs when there is an opening or tear in the trachea and air escapes into fresh tissue planes of the neck. Inspect and palpate for air under the skin around the new tracheostomy

The nurse is caring for the patient with acute pharyngitis. Which action by the nurse is most appropriate?

Offering a drink of water is correct because cool, bland liquids, such as water, will not irritate the pharynx. Citrus juices often irritate the pharynx, making it painful. Drinking warm or cold liquid is recommended, but consuming hot tea will irritate the pharynx and cause pain. Gargling with warm salt water can alleviate the symptoms of acute pharyngitis, but there are no recommendations to use hydrogen peroxide to alleviate the symptoms.

How many times to check cuff pressure?

Once per each shift, keeping the pressure at 14 to 20 mm Hg or 20 to 30 cm H2O

Mucositis

Open sores on mucous membranes

What are the clinical manifestations of oral mucositis? Select all that apply.

Oral mucositis is a common complication from radiation therapy. Pain, irritation, and ulceration are clinical manifestations of oral mucositis. Oral mucositis is not characterized by dental caries. Nasal decongestion is the clinical manifestation of sinusitis

A patient with bacterial pharyngitis is prescribed penicillin. The nurse knows what about this drug? Select all that apply.

Oral penicillin needs to be taken several times a day for 10 days to prevent rheumatic fever, which is a sequela to the infection. Patients must take the drug orally for 24 to 48 hours before they are considered noncontagious. It has no effect on fever.

Parts of Tracheostomy Tube: Outer Cannula

Outer Cannula-Fits into the stoma and keeps airway open. -Neckplate (or face plate) indicates size and type of tube on each side secures tube with tracheotomy ties

Non- Rebreather Mask

Over 10 L Highest O2 level Can deliver over 90% Used if patient's really distressed Bag should be fully inflated

Oxygen Delivery Systems: type used depends on-

Oxygen concentration required/achieved Importance of accuracy and control of oxygen concentration Patient comfort Importance of humidity Patient mobility

Which nursing assessment has the highest priority when caring for a patient with facial trauma?

Oxygenation Facial trauma has the potential to interfere with breathing by occluding the upper airways. The nurse should monitor the patient's oxygenation closely. Assessing for infection, pain level, and self-image are important but are not critical.

A critical concern for a patient returning to the unit after a surgical procedure is related to impaired oxygenation caused by inadequate ventilation, which arterial blood gas value and assessment finding indicates to the nurse that oxygen and incentive spirometry must be administered

PaO2 is 90 mm Hg with crackels

A patient is admitted to the hospital with pneumonia, what does the nurse expect the chest xray results to reveal

Patchy areas of consolidation

Pulmonary Vasculature change to age-rationales

Patient can become confused during acute respiratory conditions.

What statement made by the student nurse indicates that education regarding tracheostomy tubes has been effective?

Patients who use a tracheostomy tube may be able to learn to speak with the tube in place. Patients who have endotracheal tubes are not able to speak because no air bypasses the vocal chords completely. Patients with tracheostomy tubes can eat, will experience less discomfort, and will have less risk of damage to the vocal cords than patients with endotracheal tubes

The nurse is caring for a patient with cor pulmonale. The nurse should monitor the patient for which expected finding?

Peripheral edema

Which may be the most common feature of pneumonia and lung abscesses?

Pleuritic chest pain Pleuritic chest pain is the most common feature of pneumonia and lung abscesses. Rust-colored sputum is seen in pneumonia. Foul-smelling sputum is seen in lung abscesses. Mucopurulent sputum is seen in tuberculosis.

A patient with a recent diagnosis of bacterial pharyngitis caused by group A streptococcal infection calls the health care provider stating his has developed a cough, fever, chills, shortness of breath, and severe chest pain. Which complication does the nurse suspect?

Pneumonia

The nurse understands that which of the following is the most common manifestation of pneumonia in the older adult patient?

Pneumonia

Complications: What other comllications can occur

Pneumothorax -Subcutaneous Emphysema -Bleeding after first few days -Infection -Tracheomalacia -Tracheal stenosis -Tracheoesophageal fistula -Trachea-innominate artery fistula - medical emergency

Class IV of Dyspnea

Poor, marked breathlessness. Incomplete performance; assistance is necessary.

Which patient is the least likely to be at risk for developing pneumonia

Postop patient with a bedside commode

The nurse is preparing a community information packet about the bird flu, what info does the nurse include for public dissemention

Prepare a minimum of 2 weeks supply of food, water and routine prescription drugs, Listen to public health announcements and early warning signs for disease outbreaks, avoid traveling to areas where there has been a suspected outbreak of disease, in the event of an outbreak, avoid going to public areas such as churches or schools

The patient with a tracheostomy is receiving humidified air. Which rationale(s) for the patient's use of humidification are correct? Select all that apply.

Preventing formation of mucous plugs, warming secretions, and moisturizing secretions are correct because humidification is essential to prevent retention of tenacious secretions and formation of mucous plugs. Humidified air compensates for the loss of the upper airway to warm and moisturize secretions. Preventing lower airway heat loss is incorrect because humidified air compensates for the loss of the upper airway, not the lower airway, to warm and moisturize secretions. Additionally, it actively warms and therefore does not prevent heat loss.

Facial Trauma

Priority action is airway assessment Manifestations Stridor Shortness of breath/dyspnea Anxiety/restlessness Hypoxia and hypercarbia Decreased oxygen saturation Cyanosis, loss of consciousness CSF??

Facial Trauma

Priority action is airway assessment***** Manifestations Stridor Shortness of breath/dyspnea Anxiety/restlessness Hypoxia and hypercarbia (elevated blood levels of carbon dioxide) Decreased oxygen saturation Cyanosis, loss of consciousness

Neck trauma

Priority is to assess for and maintain patent airway Obstruction can occur from initial injury or resultant swelling

Erythropoiten

Produced by kidney to make up for the lack of oxygen

Nursing priority

Promote oxygenation by ensuring a patent airway

Partial Rebreather Mask

Provides 60%-75% with flow rate of 6-11 L/min One third exhaled tidal volume with each breath Adjust flow rate to keep reservoir bag inflated

Xerostomia is a condition associated with which of the following types of treatment?

Radiation therapy Xerostomia is dryness of the mouth which occurs with radiation therapy. It occurs when the salivary glands are in the irradiation path. The side effect is long-term and may be permanent.

Head and Neck Cancer Interventions

Radiation therapy Chemotherapy Cordectomy Laryngectomy

The nurse is teaching a patient with allergic rhinitis about the safe and effective use of his medications. What would be the most essential information to give this patient about preventing possible drug interactions?

Read drug labels carefully before taking OTC medications.

A client who is taking isoniazid (INH) and rifampin (RIF) to treat tuberculosis reports reddish-orange urine. Which action by the nurse is correct?

Reassure the client that this is an expected drug side effect.

The nurse in the long-term care facility is concerned about the health status of an 80-year-old resident. What early symptom would alert the nurse that this patient is developing pneumonia?

Recent onset of confusion The most common manifestation of pneumonia in the older adult is acute confusion caused by hypoxia. Other symptoms may include poor appetite (not vomiting), lethargy, fatigue, and weakness. Fever and cough may be absent.

Muscle Strength change to age-rationales

Regular pulmonary hygiene and overall fitness help maintain maximal functioning of the respiratory system and prevent illness.

A patient with the common cold used an over-the-counter (OTC) nasal decongestant for 12 days but still reports headache, sneezing, and nasal congestion. What could be the possible diagnosis?

Rhinitis medicamentosa The common cold usually subsides within 7 to 10 days, so when a drug is used for more than 7 days, it may be considered overuse. Overuse of nasal decongestants and chronic nasal inhalation of cocaine may cause rhinitis medicamentosa. Viruses cause viral rhinitis, also known as the common cold. Allergic rhinitis and perennial rhinitis (a type of allergic rhinitis) is caused by contact with allergens.

A client with nasal congestion, fever, and cough has been using over-the-counter medications for a week without improvement. The client exhibits tenderness to percussion over the sinuses and referred pain to the back of the head. These findings may indicate which condition?

Rhinosinusitis

A patient presenting with fever and muscle aches is diagnosed with influenza B. What antiviral medication does the nurse expect the health care provider to prescribe? Select all that apply.

Ribavirin Zanamivir Oseltamivir Ribavirin has been used in the treatment of influenza B. Zanamivir and oseltamivir have been used to shorten the duration of influenza A and influenza B. Amantadine and rimantadine are used in the prevention and treatment of influenza A.

The nurse had identified the priority patient problem of ineffective airway clearance with bronchospasms for a patient with pneumonia. The patient has no previous history of chronic respiratory disorders. The nurse obtains an order for which nursing intervention

Scheduled and PRN aerosol nebulizer bronchodilator treatments

The nurse is preparing a patient for discharge who has been treated for the prodromal stage of inhalation anthrax. What information is most important for the nurse to communicate to this patient?

Seek medical attention immediately if you begin to feel breathless The fulminant stage of inhalation anthrax usually begins after the patient has been feeling better for a day or so. This stage often begins with the patient experiencing breathlessness so it is important for the patient to seek help immediately if this symptom develops. A nurse should never encourage a patient to refill any medications unless it has been ordered by the health care practitioner. The patient should be given information about all of the symptoms of the fulminating stage, not given the impression that it is normal to feel worse before feeling better, so that any symptom will cause the patient to seek appropriate medical attention. Mild chest pain is expected with the prodromal stage of inhalation anthrax.

hyperkalemia

Serum potassium level greater than 5.0 mEq/L -cardiovascular changes are the most common cause of death -skeletal muscles twitch, hyperactive bowels -drug therapy: Kayexalate, insulin, D20 -education is key to prevention

hypernatremia

Serum sodium level over 145 mEq/L -altered cerebral function is the most common manifestation -interventions used when sodium levels become life threatening include hemodialysis and blood ultrafiltration -drug therapy: IV infusions or diuretics -ensure adequate water intake -causes: decreased water intake, increased sodium intake, kidney disease

The nurse is teaching a postoperative client who had a coronary artery bypass graft about using the incentive spirometer. The nurse instructs the client to perform the exercise in the following order: 1 Sit in an upright position. 2 Breathe air in through the mouth. 3 Exhale air slowly through the mouth. 4 Hold breath for about 3 seconds. 5 Place the mouthpiece of the spirometer in the mouth.

Sit in an upright position. Place the mouthpiece of the spirometer in the mouth. Breathe air in through the mouth. Hold breath for about 3 seconds. Exhale air slowly through the mouth. The nurse instructs the client, when using the incentive spirometer, the proper use of it. First, the client is to sit in an upright position. The client is then to place the mouthpiece of the spirometer in the client's mouth. Next, the client breathes air in through the mouth. This causes the incentive spirometer to be activated. The client holds his breath for about 3 seconds. Then, the client exhales slowly through the mouth.

Which diagnostic test are most likely to be done for a patient suspected of having community acquired pneumonia

Sputum gram stain, chest x ray

What must the nurse determine before discontinuing airborn precautions for a client with pulmonary TB

Sputum is free of acid fast bacteria

Head and neck cancers

Squamous cell carcinoma Slow growing Usually curable with early detection

Head and Neck Cancer

Squamous cell carcinoma and slow growing Begins with mucus that is chronically irritated, becoming tougher and thicker Leukoplakia and erythroplakia lesions

Wheeze characteristics:

Squeaky, musical, continuous sounds associated with air rushing through narrowed airways; may be heard without a stethoscope. Arise from the small airways. Usually do not clear with coughing.

Which clinical manifestation in the patient with facial trauma is the nurse's first priority?

Stridor Sounds like whale crying..is an indication of a partial airway obstruction and requires immediate attention. Although bleeding is important in all trauma patients, it is not the first priority in assessing the ABCs. The question does not specify where the bleeding is occurring. The type (venous or arterial) and quantity of the bleeding need to be noted. Visual acuity will be assessed in the secondary survey because it is not considered life-threatening. Pain must be addressed to fully evaluate a patient and complete a reliable examination; however, it is not the nurse's first priority.

Which of the patient's actions during self-management after a laryngectomy can lead to complications?

Swimming Swimming should be avoided after laryngectomy to prevent an infection pathway related to the stoma. Crusting around the stoma should be prevented with a saline solution rinse of the area. The stoma should be cleaned with mild soap and water to prevent infection. The stoma should be covered while coughing because the pressure can cause mucus secretions.

A nursing student is discussing a patient with viral pharyngitis with the preceptor at the walk-in clinic. What should the preceptor tell the student about nursing care for patients with viral pharyngitis?

Symptom management is the main focus of medical and nursing care.

A patient with HIV is admitted to the hosptial with a temp of 99.6 and reports bloody sputum, night sweats, feeling tired and short of breath, what are these assessment findings consistant with

TB

A previously infected patient with a dormant tuberculosis (TB) infection has experienced a reactivation of the disease. Which was likely a factor in this occurrence?

Taking prednisone for the past 3 weeks

The home health nurse is visiting a patient with chronic obstructive pulmonary disease (COPD). Which nursing action is appropriate to implement for a nursing diagnosis of impaired breathing pattern related to anxiety?

Teach the patient how to use pursed-lip breathing.

c. Diabetes mellitus Rationale: Terbutaline (Brethine) is a bronchodilator and is contraindicated in clients with hypersensitivity to sympathomimetics. It should be used with caution in clients with impaired cardiac function, diabetes mellitus, hypertension, hyperthyroidism, or a history of seizures. The medication may increase blood glucose levels.

Terbutaline (Brethine) is prescribed fora client with bronchitis. A nurse understands that this medication should be used with caution if which of the following medical conditions is present in the client? a. Osteoarthritis b. Hypothyroidism c. Diabetes mellitus d. Polycystic disease

The nurse is assessing a patient with facial trauma and observes clear drainage from the patient's left nostril. What is the nurse's next action?

Test the drainage for glucose Drainage from the nares that is positive for glucose would indicate that it is cerebrospinal fluid leaking from a skull fracture. This should be determined before calling the provider. Suctioning the nares in the presence of a possible skull fracture is contraindicated, as it may introduce bacteria through the opening. Packing may or may not be ordered by the provider, but it is not the nurse's next action.

The patient has a suspected lesion in the throat. Which tool is the nurse most likely to prepare at the bedside for the physician to use to further examine the patient?

The laryngoscope is the best tool to visualize the airway at the bedside. A stethoscope is used for listening to chest sounds. An ultrasound may be used to examine the throat, but first the health care provider will use the laryngoscope. A PET scan is a radiologic exam that is not completed at the bedside and does not visualize the airway.

c. Ventilates the client manually Rationale: If at any time an alarm is sounding and the nurse cannot quickly ascertain the problem, the client is disconnected from the ventilator and manual resuscitation is used to support respirations until the problem can be corrected. No reason is given to begin cardiopulmonary resuscitation. Checking vital signs is not the initial action. Although oxygen is helpful, it will not provide ventilation to the client.

The low pressure alarm sounds on a ventilator. A nurse assesses the client and then attempts to determine the cause of the alarm. The nurse is unsuccessful in determining the cause of the alarm and takes what initial action? a. Administers oxygen b. Checks the client's vital signs c. Ventilates the client manually d. Starts cardiopulmonary resuscitation

b. Encourage coughing and deep breathing

The nurse hears fine crackles during a lung assessment of the patient who is in the initial postoperative period. Which nursing intervention helps relieve this respiratory problem? a. Monitor the patient with a pulse oximeter b. Encourage coughing and deep breathing c. Obtain an order for a chest x-ray d. Obtain an order for high-flow oxygen

a. "I must take the medication exactly as prescribed." Rationale: Antiviral medications for influenza must be taken exactly as prescribed. These medications do not prevent the spread of influenza and clients are usually contagious for up to 2 days after the initiation of antiviral medications. Secondary bacterial infections may occur despite antiviral treatment. Side effects occur with these medications and may necessitate change in activities, especially when driving or operating machinery if dizziness occurs.

The nurse is caring for a client with a diagnosis of influenza who first began to experience symptoms yesterday. Antiviral therapy is prescribed and the nurse provides instructions to the client about the therapy. Which statement by the client indicates an understanding of the instructions? a. "I must take the medication exactly as prescribed." b. "Once I start the medication, I will no longer be contagious." c. "I will not get any colds or infections while taking this medication." d. "This medication has minimal side effects and I can return to normal activities."

A.

The nurse is caring for a patient admitted to the ED after experiencing a fall while rock climbing. The patient has several facial fractures. Which objective assessment finding is most serious? A. Malaligned nasal bridge B. Blood draining from one of the nares C. Crackling of the skin (crepitus) upon palpation D. Clear glucose positive fluid draining from nares

a. Decreased muscle strength and cough

The nurse is caring for an older patient and identifies a nursing diagnosis of Ineffective Airway Clearance. Which etiology for this diagnosis is related to the normal aging process? a. Decreased muscle strength and cough b. Increased carbon dioxide exchange b. Decreased residual volume d. Increased elastic recoil of the lungs

a. "Smoking increases my susceptibility to respiratory infections."

The nurse teaches the patient about the impact of cigarette smoking on the lower respiratory tract. Which statement by the patient indicates an understanding of the information? a. "Smoking increases my susceptibility to respiratory infections." b. "If I stop smoking the damage to my lungs will be reversed." c. "Cigarette smoke affects my ability to cough out secretions from the lungs." d. "Smoking makes the large and small airways get bigger."

The patient has an acute nasal fracture. What action by the nurse is best to maintain the patient's airway?

The nursing management goal for a patient with a nasal fracture is to maintain the airway by keeping the patient sitting upright. Applying ice to face and nose will reduce edema and bleeding and therefore indirectly assist with maintaining the airway, but sitting upright is paramount. Acetaminophen will provide analgesia but will not maintain the airway in a patient with a nasal fracture. Taking a hot shower will lead to increased swelling and should be avoided in the first 48 hours after a nasal fracture.

a. "When did the cough start?"

The patient comes to the physician's office for an annual physical. The patient reports having a persistent nagging cough. Which question does the nurse ask first about this symptom? a. "When did the cough start?" b. "Do you have a family history of lung cancer?" c. "Have you been running a fever?" d. "Do you have sneezing and congestion?"

b. Ineffective tissue perfusion

The patient with chronic respiratory disease presents with a decreased level of consciousness, dusky skin, pale mucous membranes, decreased capillary refill, and an increased respiratory rate. What is the priority nursing diagnosis? a. Ineffective airway clearance b. Ineffective tissue perfusion c. Decreased cardiac output d. Acute confusion

Please place options in order of importance. 1.Explain procedure to patient 2.Place tracheostomy care kit or appropriate sterile equipment at bedside 3.Place patient in semi-Fowler position 4.Put on goggles and clean gloves 5.Auscultate chest sounds 6.Change tracheostomy tubes

The procedure must first be explained to the patient. The proper equipment can be placed at the bedside. If the equipment is placed first, the patient might be frightened. Position the patient in semi-Fowler position. Next put on goggles and clean gloves and auscultate chest sounds before beginning the procedure to determine if suctioning is needed. Next change the tracheostomy cannula. After the tracheostomy has been changed and dressing applied, tracheostomy ties can be changed.

Wedge Resection

The removal of the peripheral portion of small, localized areas of disease

A patient who has acute viral rhinitis cares for an older family member who is susceptible to respiratory infections. Which action does the nurse suggest to this patient to help prevent the spread of infection?

Thoroughly wash hands after touching the face

Which surgery of neck cancer leads to absence of a natural voice?

Total laryngectomy Total laryngectomy results in the patient having no natural voice. In transoralcordectomy, the patient may have a natural or hoarse voice quality. In vertical laryngectomy, the resulting voice quality is hoarse. Also, after a supraglottic partial laryngectomy, the patient may have a natural or hoarse voice quality.

Upper respiratory tract

Trachea, mouth, nose, larynx, pharynx, and sinuses

Tracheostomy

Tracheal stoma (opening) that results from the tracheotomy

A patient presents to the outpatient clinic with nasal trauma. On examination, the nurse finds localized pain, edema, epistaxis, and crepitus on palpation. The patient does not have nasal congestion. Which conditions should the nurse suspect?

Trauma to the nose is the major cause of nasal fracture. It is manifested as localized pain, edema, epistaxis, and crepitus on palpation. Acute sinusitis presents with pain over the affected sinuses, purulent nasal discharge, and fever. Allergic rhinitis presents with sneezing, itchy eyes and nose, and watery nasal discharge. Deviated nasal septum presents with nasal congestion.

A patient with a history of pain and difficulty swallowing ignored the symptoms and later developed neck swelling, muffled voice, and bad breath. What could the original symptoms have been?

Untreated tonsillitis Neck swelling, muffled voice, and bad breath are symptoms of peritonsillar abscess. Untreated or partially treated acute tonsillitis may lead to the complication of peritonsillar abscess. Untreated rhinitis, untreated sinusitis, and untreated pharyngitis will not cause peritonsillar abscess

Which factor is least likely to be considered while assessing a patient's nutritional status?

Unusual bleeding Unusual bleeding is not a factor related to nutrition. Hair quality is decreased due to poor nutritional status. Generalized edema may occur due to inadequate nutritional intake. Decreased muscle mass decrease can be due to poor nutrition.

c. Fine crackles

Upon assessing the lungs, the nurse hears short, discrete popping sounds "like hair being rolled between fingers near the ear" in te bilateral lower lobes. How is this assessment documented? a. Rhonchi b. Wheezes c. Fine crackles d. Coarse crackles

The nurse assesses a patient with a history of asthma. Which assessment finding indicates that the nurse should take immediate action?

Use of accessory muscles in breathing

It is cold season and the school nurse been asked to provide an educational event for the parent teacher organization of the local elementary school. What should the nurse include in teaching about the treatment of pharyngitis?

Use of warm saline gargles or throat irrigations can relieve symptoms.

A patient comes to the walk in clinic reporting seasonal nasal congestion, sneezing, rhinorhea, and itchy watery eyes. The nurse identifies that the patient most likely has rhinitis and should also be assessed for sinusitis. How does the nurse assess for sinusitis

Use reflection of light throughout the tissues and observe for a red glow on the cheek

What should you teach a patient that has a peak flow measurement in the red zone?

Use rescue inhaler Get to ER

The nurse develops a teaching plan to help increase activity tolerance at home for an older adult with severe chronic obstructive pulmonary disease (COPD). Which instructions would be appropriate for the nurse to include in the plan of care?

Walk 15 to 20 minutes daily at least 3 times/week.

The nurse is caring for a patient with severe acute respiratory syndrome (SARS). What is the most important precaution the nurse should take when preparing to suction this patient?

Wearing a disposable particulate mask respirator and protective eyewear

a. Airway patency Rationale: Using the airway, breathing, and circulation priority-setting framework, the greatest risk to the client is airway obstruction. The priority assessment is to determine airway patency.

When caring for a client immediately following a total laryngectomy for laryngeal cancer, the nurse should give priority to which of the following assessments? a. Airway patency b. Oxygen saturation c. Breath sounds d. Gag reflex

The nurse is completing tracheostomy care. Which of these is the best method for ensuring the fit of tracheostomy ties?

When securing tracheostomy ties, place two fingers underneath the ties to ensure that they are not too tight around the patient's neck. The respiratory therapist may not be trained in changing the ties, or may not check them accurately. The patient may not be able to identify if the ties are too tight. One finger beneath the tie is too tight.

D.

When the nurse begins taking the patient's history, the patient asks, "Did you know that I have throat cancer and may not survive?" What is the appropriate nursing response? A. "Are you having difficulty swallowing?" B. "My mother had cancer, so I know how you must be feeling right now." C. "I am sure that your cancer can be cured if you follow your doctor's advice." D. "I know you have been diagnosed with cancer. Are you concerned about what the future may hold?"

A student nurse is performing first aid measures in a patient who has had epistaxis for 15 minutes after being admitted to the hospital. Which step taken by the student nurse needs correction?

While providing first aid measurement to a patient with epistaxis, the nurse should place the patient in a sitting position because it will reduce the blood pressure in the veins of the nose and reduce bleeding, thereby preventing the patient from swallowing blood. Anterior packing is used to prevent the flow of blood when the bleeding does not stop after 15 minutes. The nurse should elevate the head of the patient to prevent the flow of blood, and for the clear visualization of the nostrils. The nurse should pinch the lower soft part of the nose because this intervention helps to send the pressure back to the bleeding point in the nasal septum and stops the flow of blood.

Fixed Occlusion

Wiring the jaws together with the mouth in a position

A patient is scheduled for spirometry. Which action should the nurse take to prepare the patient for this procedure?

Withhold bronchodilators for 6 to 12 hours before the examination.

Is asthma more common in men or women? African-Americans or Caucasian?

Women African- Americans

A patient has a positive skin test results for TB, what explaination does the nurse give to the patient

You have been infected but this does not mean active disease is present

A patient has been treated for pneumonia and the nurse is preparing discharge instructions, the patient is capable of performing self care and is anxious to return to his job at the construction site. Which instructions does the nurse give this patient

You will continue to fee ltired and will fatigue easily for the next several weeks

osteoporosis

a chronic and progressive disease characterized by low bone mass and structural deterioration (spine, hip, and wrist are most often at risk) -- increase calcium and vitamin D intake

surfactant

a fatty protein that reduces surface tension in the alveoli

When assessing the client's peripheral IV site, the nurse observes a streak of red along the vein path and palpates a 4-cm venous cord. What is the most accurate documentation of this finding? a. Grade 3 phlebitis at IV site b. Infection at IV site c. Thrombosed area at IV site d. Infiltration at IV site

a. Grade 3 phlebitis at IV site Rational: The presence of a red streak and palpable cord indicates grade 3 phlebitis. No information in the description indicates that infection, infiltration, or thrombosis is present.

A client is to receive a blood transfusion. Before the transfusion, what action by the nurse takes priority? a. Verifying the client's identity b. Ensuring that the blood bank has enough blood c. Establishing a peripheral IV site d. Feeding the client before starting the blood

a. Verifying the client's identity Rational: Blood transfusion reactions can be devastating and can be prevented in large measure by positive client identification. This is accomplished by two professionals using two different client identifiers. Ensuring that the blood bank has enough blood would not be a normal nursing action, and transfusions can be given without regard to food and drink.

A client who is having a tunneled central venous catheter inserted begins to report chest pain and difficulty breathing. What action does the nurse take first? a. Administer the PRN pain medication. b. Prepare to assist with chest tube insertion. c. Place a sterile dressing over the IV site. d. Place the client in the Trendelenburg position.

b. Prepare to assist with chest tube insertion. Rational: An insertion-related complication of central venous catheters is a pneumothorax. Signs and symptoms of a pneumothorax include chest pain and dyspnea. Treatment includes removing the catheter, administering oxygen, and placing a chest tube. Pain is caused by the pneumothorax, which must be taken care of with a chest tube insertion. Use of a sterile dressing and placement of the client in a Trendelenburg position are not indicated for the primary problem of a pneumothorax.

A new nurse is securing the connections on a new IV administration set connected to a peripherally inserted central catheter (PICC) line with tape. Which action by the precepting nurse is most appropriate? a. Make sure the tape being used is from a sterile IV start kit. b. Stop the nurse and confirm that the Luer-Lok connections are tight. c. Help the new nurse document the set change appropriately. d. Show the new nurse how to turn back the corner of the tape for easy removal.

b. Stop the nurse and confirm that the Luer-Lok connections are tight. Rational: PICC line administration sets must be secured using the Luer-Lok to help prevent air emboli. When starting peripheral IVs, nurses must use the tape from the sterile IV start kit when possible, instead of using tape that might be dirty. Documentation is a critical function, but it does not take priority over doing a procedure correctly, nor does showing the new nurse time- and work-saving tips.

A client is admitted to the hospital for excessive nausea and vomiting, and a blood pressure of 90/50 mm Hg. A catheter of which gauge is most appropriate for the nurse to choose for this client's peripheral IV? a. 24 b. 22 c. 20 d. 18

c. 20 Rational: The nurse selects the access device most appropriate for the designated purpose. In this case, because a large amount of fluid will be needed as a result of excessive fluid loss, the appropriate needle is the 20-gauge catheter IV, because this is the most commonly used size in adults and it can be used for all fluids. The 22- and 24-gauge catheters will have a slower rate of flow, which may not be desirable with excessive fluid losses and low blood pressure. The 18-gauge catheter allows rapid flow of IV fluids. However, it requires a large vein and is more prone to irritation to the vein wall.

Which IV order does the nurse question? a. Flush Groshong catheter with 10 mL normal saline every 8 hours. b. Infuse 20 mEq potassium chloride in 1000 mL D5W at 50 mL/hr. c. Infuse 500 mL normal saline over 1 hour. d. Infuse 0.9% normal saline at keep vein open (KVO) rate.

d. Infuse 0.9% normal saline at keep vein open (KVO) rate. Rational: To be complete, IV orders for infusion fluids should specify the rate of infusion. This order does not specify the rate of infusion and is not considered complete.

In examining a peripheral IV site, the nurse observes a red streak along the length of the vein, and the vein feels hard and cordlike. What action by the nurse takes priority? a. Applying continuous heat b. Continuing to monitor site c. Elevating the extremity d. Removing the catheter

d. Removing the catheter Rational: The clinical manifestations described are those associated with phlebitis. Phlebitis is an inflammation of the vein. Its presence in a vein being used for IV therapy may be caused by mechanical forces associated with the IV device, or by chemical factors related to the composition and osmolarity of the drug solution. The key manifestation is that symptoms are directly associated with the vein, and the catheter must be removed. Warm compresses can be applied for 20 minutes four times daily after the catheter is removed. The site needs to be monitored after the catheter is removed. The arm is not swollen. Therefore, elevation of the extremity is not a correct option.

The nurse is making home visits to an older adult recovering from hip fracture and identifies the priority patient problem of risk for respiratory infection, which condition represents a factor of normal aging that would contribute to this increased risk

decreased strength of respiratory muscles

hypoxia

decreased tissue oxygenation

Venturi Mask

delivers 24%-50% O2 w/ flow rate of 4-10L/min delivers precise O2 concentration-best source for chronic lung disease switch to nasal cannula during meal times

infusion therapy

delivery of medications in solutions and fluids by parenteral route

A nurse is caring for a patient with coccidioidomycosis who has recently migrated from Mexico. When planning care for this patient, what manifestation noted on assessment does the nurse recognize as a sign of more severe coccidioidomycosis infection?

joint pain Coccidioidomycosis, also known as valley fever, is a fungal infection that is caused by an organism commonly found in the desert southwest regions of the United States, Mexico, and Central and South America. Joint pain is a symptom of more serious coccidioidomycosis infection. Cough, chest pain, and night sweats are symptoms of coccidioidomycosis; however, these are common in all infections and are not symptoms of more serious infection.

A patient in anaphylaxis who is going into respiratory failure will demonstrate which symptoms

laryngeal edema, hypoxemia, crackles, wheezing

In general, chest drainage tubes are not used for the patient undergoing a) lobectomy. b) pneumonectomy. c) wedge resection. d) segmentectomy.

pneumonectomy. Usually, no drains are used for the pneumonectomy patient because the accumulation of fluid in the empty hemothorax prevents mediastinal shift. With lobectomy, two chest tubes are usually inserted for drainage, the upper tube for air and the lower tube for fluid. With wedge resection, the pleural cavity usually is drained because of the possibility of an air or blood leak. With segmentectomy, drains are usually used because of the possibility of an air or blood leak.

A nurse is caring for a client with a venturi mask who is receiving 40% oxygen, what nursing actions are indicated

prevent the clients blanket from covering the adaptors orfices, check that the appropriate adaptor to deliver the prescribed FIO2 is attached to the mask

A patient is diagnosed with TB has been receiving treatment for 3 weeks and has clinically shown improvement, the family asks the nurse if the patient is still infectious, what is the nurses reply

the patient is not infectious but needs to continue treatment for at least 6 months

compartment syndrome

when increased tissue perfusion in a confined space causes decreased flow to the area

protruding

would you expect to see flat or protruding neck veins in a patient suffering from fluid volume excess?

A healthy patient expresses worries about developing tuberculosis (TB) after spending time at a family reunion and learning later that a family member is being treated for the disease. What does the nurse tell this patient?

"Among people exposed to the disease, only a small percentage develop active TB.

Which teaching by the student nurse to a patient who underwent a rhinoplasty may result in complications during the postoperative period?

"Apply a hot water bottle to the nose." Following rhinoplasty, a hot water bottle may increase the risk of bleeding due to tissue friability. Therefore, the student nurse should instruct the patient to use cool compresses, which helps to reduce swelling and bruising. The patient should rest in a semi-Fowler's position (30°-45°), which reduces pressure on the surgical area. Soft foods should be eaten to reduce pressure on the nearby tissues. Drinking lots of water will keep the patient well-hydrated.

A clinic nurse is providing teaching for a patient who has been diagnosed with a peritonsillar abscess. What does the nurse include in this patient's teaching?

"Go to the emergency department if drooling or stridor occur.

A 70-year-old patient has a complicated medical history including chronic obstructive pulmonary disease (COPD). Which patient statement indicates the need for further teaching about the disease?

"I am here to receive the yearly pneumonia shot again." Patients 65 years and older , as well as those who have chronic health problems, should be encouraged to receive the pneumonia vaccine, which is not given annually but only once. Older patients are encouraged to receive a flu shot annually because the vaccine changes, depending on anticipated strains for the upcoming year. It is a good idea to avoid large gatherings during cold and flu season. New recommendations from the Centers for Disease Control and Prevention (CDC) for controlling the spread of flu include coughing or sneezing into the upper sleeve rather than into the hand.

Which statement by a patient with a laryngectomy indicates a need for further discharge teaching?

"I can't put anything over my stoma to cover it. Loose clothing or a covering such as a scarf can be used to cover the stoma if the patient desires. To avoid aspiration, the patient with a laryngectomy should not swim. Mild soap and water is the proper way to clean the stoma; however, a shield should be used in the shower so a large amount of water does not enter it. The patient may project mucus when he laughs or coughs; reinforce with the patient and the family that this is normal and is to be expected.

The nurse is teaching a patient who underwent surgery for mandibular fracture with inner maxillary fixation about self-care upon discharge. Which statement regarding self-management made by the patient indicates effective learning?

"I will cut the wires if I vomit." With inner maxillary fixation, bones are realigned and then wired in place with the closed bite. If the patient vomits after an inner maxillary fixation, the patient should immediately cut the wires to prevent aspiration. Irrigating devices are important for maintaining oral health because a patient cannot brush after surgery until completely healed. The patient should consume only a liquid diet, because chewing is not possible after surgery. Rewiring should be done only by the licensed practitioner.

Which statement made by the patient indicates a need for further teaching about how to manage a breathing stoma?

"I will limit the amount of water I drink to keep the stoma dry." The patient with a stoma should stay hydrated to prevent secretions from thickening. The patient should increase the humidity in his or her home to keep the stoma moist. Using a shower shield on the stoma while bathing prevents entry of water into the airway. The stoma should be covered while shaving with an electric razor to prevent hair from entering the airway.

A patient who works in a warehouse has developed acute viral pharyngitis after inhaling noxious fumes. After counseling the patient about treatment for this condition, which statement by the patient indicates a need for further teaching?

"I will need to take antibiotics." Nursing management of acute viral pharyngitis focuses on supportive interventions. The patient should be taught to increase fluid intake, humidify the air, and use analgesics for pain. Gargling several times per day with saline can increase comfort. Viral pharyngitis is not infectious and does not need antibiotic therapy.

The nurse is providing health education to an elderly patient to prevent pneumonia. Which statements made by the patient demonstrate ineffective learning? Select all that apply.

"I will refrain from drinking nonalcoholic fluids. "I will refrain from obtaining the pneumococcal vaccination." Since drinking at least 3 liters of nonalcoholic fluids per day will reduce the risk for pneumonia, the patient needs to follow this guideline. In addition, not getting the pneumococcal vaccination will increase the risk of developing pneumonia; therefore the patient should obtain the vaccination. Smoking is a precipitating factor for pneumonia; therefore, to prevent pneumonia the patient should stay away from smoking. Indoor pollutants like dust and aerosols should also be avoided to prevent pneumonia. Staying away from public areas during flu season will decrease the spread of the disease.

A nurse is diagnosed with seasonal influenza, and on the second day of treatment with oseltamivir, she asks the supervising nurse when she may return to work on a hospital unit. What does the supervising nurse tell her?

"If you are feeling well and afebrile in 5 days, you may return to work." Individuals with influenza are contagious from 24 hours before the onset of symptoms and up to 5 days after symptoms begin. Antiviral medication only shortens the duration of symptoms but does not affect contagiousness. Those who continue to have symptoms, especially fever, should remain off work until those symptoms clear.

A nurse is providing discharge instructions for a patient with active tuberculosis (TB) who has been prescribed isoniazid. What information about medication administration does the nurse include when providing discharge instructions?

"Take the drug on an empty stomach.

A 75-year-old patient tells the nurse he is not planning to receive a "flu shot" this year because the shot makes him sick. What is the nurse's best response?

"The injectable flu vaccine is not a live virus and cannot cause influenza. The influenza vaccine is not a live virus and cannot cause disease. The intranasal vaccine is a live, attenuated vaccine and is not given to people over age 49. Immunity to influenza is not conferred in subsequent years because the strains of influenza virus change each year

A patient with active tuberculosis is ordered to take isoniazid (INH), pyrazinamide (PZA), and rifampin (RIF) and asks the nurse why it is necessary to take three antibiotics. What is the nurse's best answer?

"Three antibiotics help prevent bacterial drug resistance.

T-Piece

Delivers desire FIO2 for tracheostomy, laryngectomy, ET tubes Ensures humidifier creates enough mist Mist should be seen during inspiration and expiration Assess need for suctioning

T-piece

Delivers desired FIO2 for tracheostomy, laryngectomy, ET tubes Ensures humidification through creation of mist Mist should be seen during inspiration and expiration

Parts of Tracheostomy Tube: Inner Cannula

-Internal portion of tracheostomy tube -Fits snugly within the outer cannula -Promotes cleaning of secretions form inside the tube -Provides universal adapter for use with ventilator and other respiratory equipment -May be disposable or nondisposable

Indications for Suctioning (6)

-Rapid, shallow breathing -Moist, noisy, gurglin sounds -Decreased Spo2 -Visible secretions -Course crackles -Vibrations of loose secretions when placing hand over clients chest (tactile femitus)

Obstructive Sleep Apnea

-a breathing disrupiton during sleep that lasts at least 10 seconds and occursa minimum of 5 times in an hour Breathing disruption during sleep Excessive daytime sleepiness, inability to concentrate, irritability Nonsurgical management: Change of sleep position, weight loss, positive-pressure ventilation Surgical management: Change of sleep position, weight loss, positive-pressure ventilation

Nursing Cannula rationales:

-a poorly fitting nasal cannula leads to hypoxia and skin breakdown -this substance prevents mucosal irritation related to the drying effect of oxygen; promotes comfort -congestion or a deviated septum prevents effective delivery of oxygen through the nares -the respiratory pattern affects the amount of oxygen delivered. A different delivery system may be needed.

Partial rebreather mask rationales:

-deflation results in decreased oxygen delievered and increases the rebreathing of the exhaled air -the flow rate is adjusted to meet the pattern of the patient

stridor

-is a high-pitched breath sound resulting from turbulent air flow in the larynx or lower in the bronchial tree

Non-rebreather mask rationales:

-monitoring ensures proper functioning and prevents harm -valves should open during expiration and close during inhalation to prevent dramatic decrease in FiO2. Suffocation can occur if the reservior bag kinks or if the oxygen source disconnects -the patient may require intubation

A patient with chronic obstructive pulmonary disease (COPD) has a nursing diagnosis of imbalanced nutrition: less than body requirements. Which intervention would be most appropriate for the nurse to include in the plan of care?

Offer high-calorie snacks between meals and at bedtime.

A parent calls the ER about her child who reports a sever sore throat and refuses to drink fluids for take liquid pain meds. What is the most important question for the nurse to ask in order to determine the need to seek immediate medical attention?

Is the child drooling or do you hear stridor, a raspy rough sound when the child breathes?

An older adult patient asks the nurse how often on should receive the pneumococcal vaccine for pneumonia prevention, what is the nurses best response

It is usually given once but some older adults may need a second vaccination after 5 years

The nurse measures the pulse of a patient admitted with bronchitis and asthma and finds it to be 120 beats per minute. To which medication might the nurse attribute this finding? 1 Albuterol (Proventil HFA) 2 Montelukast (Singulair) 3 Acetaminophen (Tylenol) 4 Allopurinol (Zyloprim)

1 RATIONALE: Albuterol, used to treat asthma, increases the sympathetic nervous system response, and may increase the heart rate. Allopurinol, acetaminophen, and montelukast do not elevate the heart rate.

A patient experiences a chest wall contusion as a result of being struck in the chest with a baseball bat. The emergency department nurse would be most concerned if which finding is observed during the initial assessment? A. Paradoxic chest movement B. Complaint of chest wall pain C. Heart rate of 110 beats/minute D. Large bruised area on the chest

A. Paradoxic chest movement

The nurse is caring for a patient with cor pulmonale. The nurse should monitor the patient for which expected finding? A. Peripheral edema B. Elevated temperature C. Clubbing of the fingers D. Complaints of chest pain

A. Peripheral edema

Fracture of the Nose

Displacement of bone or cartilage can cause airway obstruction or cosmetic deformity; potential source of infection CSF may indicate skull fracture Interventions Closed reduction Nasoseptoplasty Rhinoplasty

A patient with pneumonia caused by aspiration after alcohol intoxication has just been admitted. The patient is febrile and agitated. Which health care provider order should the nurse implement first?

Draw aerobic and anaerobic blood cultures

d. PaO2 58 mm Hg Rationale: The PaO2 is decreased with acute respiratory failure.

A client is diagnosed with acute respiratory failure. The nurse should expect which of the following laboratory findings? a. Arterial pH 7.50 b. PaCO2 25 mm Hg c. SaO2 92% d. PaO2 58 mm Hg

The nurse is caring for a patient who is suspected of having chronic obstructive pulmonary disease (COPD). The nurse knows that what type of diagnostic test would confirm this diagnosis? 1. Spirometry 2. Chest x-ray 3. Arterial blood gas (ABG) 4. Computed tomography (CT) scan of the ches

1 RATIONALE: Spirometry is needed to confirm the presence of airflow obstruction and the severity of COPD. The patient is given a short-acting bronchodilator, and postbronchodilator values are compared with a normal reference value. Chest x-rays are not diagnostic but can show a flat diaphragm caused by hyperinflated lungs. ABGs are used after the patient has been diagnosed and usually in the more severe stages of the disease. CT scans are not used routinely to diagnose COPD.

Interventions for Upper Airway Obstruction

-Assess cause of obstruction -Maintenance of patent airway and ventilation -Cricothyroidotomy -Endotracheal intubation (nasotracheal or orotracheal) -Tracheotomy

Interventions for Upper Airway Obstruction

-Assess cause of obstruction -Maintenance of patent airway and ventilation Cricothyroidotomy Endotracheal intubation (nasotracheal or orotracheal) Tracheotomy

Educational needs of the Patient: Body Image

-Assist patient in setting realistic goals related to self-care -Work with the family-educate -Provide encouragement and positive reinforcement

A patient is in acute respiratory distress syndrome (ARDS) as a result of sepsis. Which measure most likely would be implemented to maintain cardiac output? 1 Administer crystalloid fluids or colloid solutions. 2 Position the patient in the Trendelenburg position. 3 Place the patient on fluid restriction and administer diuretics. 4 Perform chest physiotherapy and assist with staged coughing.

1 Low cardiac output may necessitate crystalloid fluids or colloid solutions in addition to lowering positive end-expiratory pressure (PEEP) or administering inotropes. The Trendelenburg position (not recommended to treat hypotension) and chest physiotherapy are unlikely to relieve decreased cardiac output, and fluid restriction and diuresis would be inappropriate interventions. Text Reference - p. 1669

The nurse cares for an immunocompetent patient. Which clinical manifestation is most indicative of pulmonary tuberculosis? 1. Mucopurulent sputum 2. Diarrhea and fatigue 3. Lymph node enlargement 4. Hematuria and dehydration

1 RATIONALE: A cough that progresses in frequency and produces mucoid or mucopurulent sputum is the most common symptom of pulmonary tuberculosis (TB). Diarrhea, and hematuria, and dehydration are manifestations not directly associated with pulmonary TB. Fatigue and lymph node enlargement may be seen with TB but are not as indicative as is the production of mucopurulent sputum.

The nurse cares for a patient with a diagnosis of tuberculosis. Which assessment finding best indicates that the patient has been following the prescribed treatment plan? 1 Negative sputum cultures 2 Clear breath sounds bilaterally 3 Decrease in the number of coughing episodes 4 Conversion of the Mantoux test from positive to negative

1 RATIONALE: A patient's sputum is expected to convert to negative within 3 months of the beginning of treatment. If it does not, the patient is either not taking the medication or has drug-resistant organisms. Bilaterally clear breath sounds and a decrease in coughing are good indications that the patient is following the prescribed plan, but they are not as confirmatory as negative sputum cultures. Once a person has been exposed to the tuberculosis-causing organism, the Mantoux test will always elicit a positive result.

A nurse is caring for a patient with bronchitis. The health care provider has advised arterial blood gas (ABG) analysis. The blood sample can be obtained from which blood vessels? Select all that apply. 1 Radial artery 2 Femoral artery 3 Pulmonary artery 4 Superior vena cava 5 Femoral vein

1,2 RATIONALE: Blood samples for ABG analysis are collected from the radial and femoral arteries, as they carry oxygenated blood. The pulmonary artery carries deoxygenated blood and is used for the measurement of mixed venous blood gas. The superior vena cava is used to measure the central venous pressure (CVP). The femoral vein is not used for ABG analysis, since it carries deoxygenated blood

A nurse is assessing the pulmonary functions of a patient and finds that the vital capacity and total lung capacity are reduced. What respiratory disorders could cause the reduced capacities? Select all that apply. 1. Atelectasis 2. Emphysema 3. Chronic bronchitis 4. Pulmonary fibrosis 5. Asthma

1,4 RATIONALE: Pulmonary function tests are used to differentiate restrictive pulmonary disorders from obstructive disorders. Reduction in vital capacity and total lung capacity indicates that the patient has restrictive pulmonary disorder. Both atelectasis and pulmonary fibrosis are examples of restrictive disorders. Emphysema, chronic bronchitis, and asthma are obstructive pulmonary disorders and are highly unlikely to be seen in this patient.

Alveoli change due to age-nursing interventions

Encourage vigorous pulmonary hygiene (patient to turn, cough, deep breathe) and to use incentive spirometry, especially if he or she is confined to bed or has had surgery. Encourage upright position

The purpose of this exercise is to prolong exhalation and thereby prevent bronchiolar collapse and air trapping. 1 Diaphragmatic breathing 2 Pursed-lip breathing (PLB) 3 Huff coughing 4 Chest physiotherapy (CPT)

2 RATIONALE: The purpose of PLB is to prolong exhalation and thereby prevent bronchiolar collapse and air trapping. PLB is simple and easy to teach and learn, and it gives the patient more control over breathing, especially during exercise and periods of dyspnea. Another type of breathing retraining exercise is diaphragmatic breathing, which focuses on using the diaphragm to achieve maximum inhalation and slow respiratory rate, not prolong exhalation. Huff coughing is an effective forced expiratory technique, not a breathing exercise to prolong exhalation. Chest physiotherapy (CPT) consists of postural drainage, percussion, and vibration and is for patients who have difficulty clearing excessive bronchial secretions.

A patient with acute respiratory distress syndrome (ARDS) has been put on a ventilator. What nursing measures alter the risk of development of pneumonia secondary to ventilator use? Select all that apply. 1 Using invasive monitoring devices 2 Maintaining proper hygiene of the patient 3 Keeping the patient on prolonged ventilation 4 Using aseptic techniques when suctioning the patient 5 Using the enteral route for feeding the patient

2,4 RATIONALE: Maintaining proper hygiene of the patient and using aseptic techniques may decrease the risks of pneumonia in the patient caused by the ventilator. The use of invasive monitoring devices and keeping the patient on prolonged ventilation increase the risk of pneumonia by facilitating passage of microorganisms into the airway. When a patient is on ventilator and on enteral feedings, there is a high risk of aspiration of feeds, causing pneumonia.

What nursing intervention is most appropriate to enhance oxygenation in a patient with unilateral malignant lung disease? 1.Positioning patient on right side 2. Maintaining adequate fluid intake 3. Positioning patient with "good lung" down 4. Performing postural drainage every four hours

3 RATIONALE: Therapeutic positioning identifies the best position for the patient, thus assuring stable oxygenation status. Research indicates that positioning the patient with the unaffected lung (good lung) dependent best promotes oxygenation in patients with unilateral lung disease. For bilateral lung disease, the right lung down has best ventilation and perfusion. Increasing fluid intake and performing postural drainage will facilitate airway clearance, but positioning is most appropriate to enhance oxygenation.

The nurse is caring for an alert patient with a diagnosis of recent nasal fracture. The nurse knows that initially the best way to maintain the airway is to: 1. Insert a nasal airway 2. Apply oxygen via a face mask 3. Apply ice to face and nose per protocol 4. Place patient in a semi-Fowler position

3 RATIONALE: Within the first 48 hours of the nasal fracture, ice needs to be applied to the face and nose in 10 to 20 minute intervals to help reduce edema and bleeding. Nasal airways should not be inserted into patients with a nasal fracture. In addition, airways should not be inserted into alert patients. Patients need to be placed in an upright position and not a semi-Fowler position. Oxygen via a face mask is not needed to keep the airway open.

When teaching the patient with chronic obstructive pulmonary disease (COPD) about smoking cessation, what information should be included related to the effects of smoking on the lungs and the increased incidence of pulmonary infections? 1. Smoking causes a hoarse voice 2. Cough will become nonproductive 3. Decreased alveolar macrophage function 4. Sense of smell is decreased with smoking

3 RATIONALE: The damage to the lungs includes alveolar macrophage dysfunction that increases the incidence of infections and thus increases patient discomfort and cost to treat the infections. Other lung damage that contributes to infections includes cilia paralysis or destruction, increased mucus secretion, and bronchospasms that lead to sputum accumulation and increased cough. The patient may be aware already of respiratory mucosa damage with hoarseness and decreased sense of smell and taste, but these do not increase the incidence of pulmonary infection.

The nurse is caring for a patient on mechanical ventilation. What are the nursing interventions that prevent the development of volutrauma in a patient on a ventilator? Select all that apply. 1 Sterile techniques 2 Strict hand washing 3 Smaller tidal volumes 4 Pressure-control ventilation 5 Mouth care and oral hygiene

3,4 RATIONALE: Volutrauma occurs when large tidal volumes are given to a mechanically ventilated patient. Because of the high tidal volume, the alveoli may become damaged and tear, allowing proteins and fluid to move into the alveolar spaces. This can be prevented by giving smaller tidal volumes or pressure-control ventilation. Strict hand washing, sterile technique during endotracheal suctioning, and frequent mouth care and oral hygiene are helpful to prevent ventilator-associated pneumonia and not volutrauma.

A severely dehydrated client requires a rapid infusion of normal saline and needs a midline IV placed. Which staff member does the ER charge nurse assign to complete this task? A. RN who is certified in the administration of oral and infused chemotherapy medications 2. RN with 2 years of experience in the ER who is skilled at insertion of short peripheral catheters 3. RN with 10 years of experience on a medical-surgical unit who has cared for many clients requiring IV infusions 4. RN with certified registered nurse infusion (CRNI) certification who is assigned to the ER for the day

4. RN with certified registered nurse infusion (CRNI) certification who is assigned to the ER for the day The nurse with CRNI certification is most likely to be able to quickly insert a midline catheter for a client who is dehydrated. The chemotherapy nurse and the ED nurse have the appropriate scope of practice, but will not be as skilled in inserting a midline IV catheter. The medical-surgical nurse may be skilled at inserting short peripheral catheters, but will not be skilled in inserting midline IV catheters.

Routine tracheostomy care is on the treatment plan for a patient that has a fenestrated tracheostomy tube. What should be the order of actions performed by the nurse for proper tracheostomy care? 1.Clean the stoma using 4 x 4 gauze pad and the tracheostomy faceplate using cotton swabs. 2.Wash hands and put on gloves and goggles. 3.Remove fenestrated inner cannula and rinse in a sterile solution, and replace it. 4.Change tracheostomy ties using a two-person change technique. 5.Maintain the position of the retention sutures near the stoma. 6.Assemble sterile equipment

6,2,3,1,5,4 RATIONALE: During routine tracheostomy care, the required sterile equipment should be assembled near the patient's bed. Hands should be washed before the procedure. Clean gloves and goggles are to be worn. At first, the inner cannula is unlocked and cleaned in sterile water or saline solution. The inner cannula should be replaced after cleaning. The stoma is then cleaned with sterile gauze to remove dried secretions. Retention sutures, if present, should be properly positioned and secured. Subsequently, tracheostomy ties are changed.

A patient with a history of asthma is having shortness of breath. The nurse discovers that the peak flow meter indicates a peak expiratory flow (PEF) reading that is in the red zone. What is the priority nursing action? Administer the rescue drugs. Take the patient's vital signs. Notify the patient's prescriber. Repeat the PEF reading to verify the results.

A

20. A nurse is caring for a patient who has been admitted with an exacerbation of chronic bronchiectasis. The nurse should expect to assess the patient for which of the following clinical manifestations? A) Copious sputum production B) Pain on inspiration C) Pigeon chest D) Dry cough

A Feedback: Clinical manifestations of bronchiectasis include hemoptysis, chronic cough, copious purulent sputum, and clubbing of the fingers. Because of the copious production of sputum, the cough is rarely dry. A pigeon chest is not associated with the disease and patients do not normally experience pain on inspiration.

18. A pediatric nurse practitioner is caring for a child who has just been diagnosed with asthma. The nurse has provided the parents with information that includes potential causative agents for an asthmatic reaction. What potential causative agent should the nurse describe? A) Pets B) Lack of sleep C) Psychosocial stress D) Bacteria

A Feedback: Common causative agents that may trigger an asthma attack are as follows: dust, dust mites, pets, soap, certain foods, molds, and pollens. Lack of sleep, stress, and bacteria are not common triggers for asthma attacks.

17. A nurse has been asked to give a workshop on COPD for a local community group. The nurse emphasizes the importance of smoking cessation because smoking has what pathophysiologic effect? A) Increases the amount of mucus production B) Destabilizes hemoglobin C) Shrinks the alveoli in the lungs D) Collapses the alveoli in the lungs

A Feedback: Smoking irritates the goblet cells and mucous glands, causing an increased accumulation of mucus, which, in turn, produces more irritation, infection, and damage to the lung.

The patient has severe chronic obstructive pulmonary disease (COPD). To reduce the size of hyperinflated emphysematous lungs and decrease airway obstruction, the health care provider would most likely recommend what as a surgical option? A.Lung volume reduction surgery (LVRS) B.Lung transplant C.Airway bypass D. Bullectomy

A RATIONALE: Three different surgical procedures have been used in severe COPD. One type of surgery is LVRS. The goal of this surgery is to reduce the size of the lungs by removing the most diseased lung tissue so the remaining healthy lung tissue can perform better. The rationale for LVRS is that reducing the size of the hyperinflated emphysematous lungs results in decreased airway obstruction and increased room for the remaining normal alveoli to expand and function. The procedure reduces lung volume and improves lung and chest wall mechanics. Lung transplantation would not be the first choice for the HCP because it is for carefully selected patients with advanced COPD and this patient's lung volume can be reduced. Bullectomy is for patients with emphysematous COPD who have large bullae. Airway bypass is a procedure still under evaluation.

The nurse is performing a tracheostomy dressing change for a patient. Which action is most appropriate when changing the dressing?

A dressing designed to be used for a tracheostomy or with an unlined gauze should be used to reduce the risk of complications in the patient. The nurse should not cut the gauze before using it because the patient may inhale the threads. The dressing should be changed frequently, not once every two days. The nurse should wash his or her hands before and after applying the dressing, following effective infection control measures.

A nurse is suctioning the airway of a patient with a tracheostomy tube in place. While suctioning, the nurse notices that the heart rate of the patient drops from 80 to 60 beats/minute. What nursing intervention is most appropriate in this case?

A drop in the heart rate during suctioning indicates hypoxia. If the heart rate drops or increases by 20 beats per minute while suctioning the airway through a tracheostomy tube, suctioning should be stopped immediately. No intermittent or continuous suction should be applied, because it may lead to hypoxia. The suction catheter should be rotated while applying suction.

Laryngectomee

A person who has had a laryngectomy

The provider discusses radiation therapy with the patient because her lesion is small and the cure rate is 80% or higher. The patient asks if her voice will return to normal. What is the appropriate nursing response? (Select all that apply.) A. "At first the hoarseness may become worse." B. "The more you use your voice, the quicker it will improve." C. "Gargling with saline may help decrease the discomfort in your throat." D. "Your voice will improve within 4 to 6 weeks after completion of the therapy." E. "You should rest your voice and use alternative communication during the therapy."

A, C, D, E

34. A nurse is working with a child who is undergoing a diagnostic workup for suspected asthma. What are the signs and symptoms that are consistent with a diagnosis of asthma? Select all that apply. A) Chest tightness B) Crackles C) Bradypnea D) Wheezing E) Cough

A, D, E Feedback: Asthma is a chronic inflammatory disease of the airways that causes airway hyperresponsiveness, mucosal edema, and mucus production. This inflammation ultimately leads to recurrent episodes of asthma symptoms: cough, chest tightness, wheezing, and dyspnea. Crackles and bradypnea are not typical symptoms of asthma.

A patient tells the nurse that her doctor just told her that her new diagnosis of rheumatoid arthritis is considered to be a "chronic condition." She asks the nurse what "chronic condition" means. What would be the nurse's best response? Select one: A. "Chronic conditions are defined as health problems that require management of several months or longer." B. "Chronic conditions are medical conditions that culminate in disabilities that require hospitalization." C. "Chronic conditions are those that require short-term management in extended-care facilities." D. "Chronic conditions are diseases that come and go in a relatively predictable cycle."

A. "Chronic conditions are defined as health problems that require management of several months or longer." Chronic conditions are often defined as medical conditions or health problems with associated symptoms or disabilities that require long-term management (3 months or longer). Chronic diseases are usually managed in the home environment. They are not always cyclical or predictable.

A nurse obtains a health history from a patient who has a 35 pack-year smoking history. The patient complains of hoarseness and tightness in the throat and difficulty swallowing. Which question is most important for the nurse to ask? A. "How much alcohol do you drink in an average week?" B. "Do you have a family history of head or neck cancer?" C. "Have you had frequent streptococcal throat infections?" D. "Do you use antihistamines for upper airway congestion?"

A. "How much alcohol do you drink in an average week?"

The nurse is caring for a patient with a nursing diagnosis of impaired skin integrity related to a stage III decubitus ulcer. What would be the most important outcome for this patient? Select one: A. The patient exhibits no signs or symptoms of infection B. The patient changes position every 2 hours C. The patient keeps the area clean and dry D. The patient knows prevention measures for decubitus

A. The patient exhibits no signs or symptoms of infection All options are appropriate outcomes for this patient, but the most important outcome is that the patient exhibits no signs or symptoms of infection.

What are the two major processes involved in the inflammatory phase of wound healing? Select one: A. blood clotting is initiated, WBCs move into the wound B. collagen is remodeled, avascular scar forms C. bleeding is stimulated, epithelial cells are deposited D. granulation tissue is formed, collagen is deposited

A. blood clotting is initiated, WBCs move into the wound The inflammatory phase of wound healing begins at the time of injury and prepares the wound for healing. The two major physiologic activities are blood clotting (hemostasis) and the vascular and cellular phase of inflammation.

2. The nurse prepares a patient with a left-sided pleural effusion for a thoracentesis. How should the nurse position the patient? a. Supine with the head of the bed elevated 30 degrees b. In a high-Fowler's position with the left arm extended c. On the right side with the left arm extended above the head d. Sitting upright with the arms supported on an over bed table

ANS: D The upright position with the arms supported increases lung expansion, allows fluid to collect at the lung bases, and expands the intercostal space so that access to the pleural space is easier. The other positions would increase the work of breathing for the patient and make it more difficult for the health care provider performing the thoracentesis. DIF: Cognitive Level: Apply (application) REF: 492 TOP: Nursing Process: Implementation MSC: NCLEX: Physiological Integrity

The nurse is caring for a patient following a wedge resection. While the nurse is assessing the patient's chest tube drainage system, constant bubbling is noted in the water seal chamber. This finding indicates which of the following problems? a) Tension pneumothorax b) Tidaling c) Increased drainage d) Air leak

Air leak The nurse needs to observe for air leaks in the drainage system; they are indicated by constant bubbling in the water seal chamber, or by the air leak indicator in dry systems with a one-way valve. Tidaling is fluctuation of the water level in the water seal that shows effective connection between the pleural cavity and the drainage chamber and indicates that the drainage system remains patent.

What is the priority nursing action for patients who have experienced facial trauma?

Airway assessment It can be difficult to determine the extent of trauma with a facial injury, so these patients should be monitored closely for adequacy of their airway. Obstruction related to the trauma can lead to further respiratory complications. Assessment of pulse oximetry, checking for bruising, and monitoring for pulse changes would also be part of the patient's care; however, the priority is to ascertain that the airway is patent.

Facial Trauma Interventions

Airway assessment Anticipate need for emergency intubation Tracheotomy Cricothyroidotomy Fixed occlusion Débridement

Facial Trauma Interventions

Airway assessment** Anticipate need for emergency intubation Time, Treatment Team Tracheotomy Cricothyroidotomy Fixed occlusion

A patient presents with a persistent runny nose, sneezing, and watery eyes. The patient also reports a recent onset of headache and nasal congestion. On further questioning, a nurse finds that the patient recently brought a cat home. What condition is the patient likely to have?

Allergic rhinitis can be caused by sporadic exposure to allergens such as animal dander. Symptoms such as runny nose, sneezing, and watery eyes caused by exposure to a cat are suggestive of allergic rhinitis. The symptoms of the common cold are similar to those of allergic rhinitis, but the common cold is not caused by pets. Symptoms such as a runny nose, sneezing, and watery eyes are not suggestive of septal deviation. These symptoms may be suggestive of influenza; however, influenza is not associated with pets.

A patient reports sneezing, itchy eyes and nose, and watery nasal discharge to a nurse. The nurse finds pale, boggy, and swollen nasal turbinate. Which category of medication does the nurse expect to be included in the patient's prescription? Select all that apply.

Allergic rhinitis is the inflammation of nasal mucosa due to allergens such as pollens, animal dander, house molds, or dust mites. The patient with allergic rhinitis will have pale, boggy, and swollen nasal turbinates. Decongestants increase the vasoconstriction of the blood vessels and help to reduce nasal congestion. H1 antihistamines are used in the treatment of rhinitis because these medications bind with H1 receptors to block histamine binding and reduce inflammation. Anticoagulants, when administered to a patient with allergic rhinitis, increase bleeding time and aggravate the risk of epistaxis. Neuraminidase inhibitors are used in the treatment of influenza but do not help in relieving allergic rhinitis. Nonsteroidal antiinflammatory drugs increase bleeding time and pose a risk for epistaxis when used in a patient with rhinitis.

b. Diminished breath sounds Rationale: The patient has sustained a blunt or a closed chest injury. Basic symptoms of a closed pneumothorax are shortness of breath and chest pain. A larger pneumothorax may cause tachypnea, cyanosis, diminished breath sounds, and subcutaneous emphysema. Hyperresonance also may occur on the affected side. A sucking sound at the site of injury would be noted with an open chest injury. The word blunt eliminates "d". Respiratory injuries increase respirations so that eliminates "a". A barrel chest is a characteristic finding in a client with COPD so that eliminates "c"

An emergency department nurse is assessing a client who has sustained a blunt injury to the chest wall. Which of these signs would indicate the presence of a pneumothorax in this client? a. A low respiratory rate b. Diminished breath sounds c. The presence of a barrel chest d. A sucking sound at the site of injury

The nurse is caring for a patient who has just been diagnosed with pulmonary tuberculosis and will be discharged with a prescription for isoniazid 300 mg orally each day. At what time should the nurse teach this patient to take this medication?

An hour before breakfast Isoniazid must be taken on an empty stomach to ensure adequate medication absorption so the best time for the patient to take this medication is an hour before breakfast. The patient would need to fast for two hours before taking the medication prior to bedtime to ensure that the stomach is empty. Taking the medication immediately before or after breakfast would not allow the stomach to be empty while the medication is absorbed.

Which nonsurgical method is best to restore speech in a patient who cannot achieve sound conduction on the skin?

An intraoral electrolarynx is a device used to restore speech in patients who have impaired sound conduction on the skin. The use of esophageal speech helps patients develop a few speaking skills, but not fluent speech. A neck type electrolarynx is an intraoral device that is placed on the chin, neck, or cheek to articulate sound. This method involves the use of skin; thus, this device cannot be used in patients with impaired sound conduction on the skin. Transesophageal puncture is a surgical procedure in which a fistula is created between the trachea and esophagus.

For a patient who is having an anaphylactiv reaction, which common symptoms will manifest almost immediately after being exposed to an allergen

Angioedema, apprehension, urticaria

The nurse recognizes that a patient with sleep apnea may benefit from which intervention(s)? (Select all that apply.) Weight loss Nasal mask to deliver BiPAP A change in sleeping position Medication to increase daytime sleepiness Position-fixing device that prevents tongue subluxation

Answer: A, B, C, E Rationale: All interventions listed are viable interventions that can be of benefit to patients who have sleep apnea. Patients should work with their providers of care to determine the severity of their sleep apnea and which specific interventions would be of most importance to them. Encouraging daytime sleepiness is the opposite of the effect needed for this patient.

A nurse teaches home-care precautions to a patient who has anterior nasal packing at the time of hospital discharge. During follow-up visits, the nurse observes that the patient's condition is aggravated. Which action of the patient is responsible for this condition?

Anterior nasal packing is applied to prevent the flow of blood from the nose of a patient who has had nasal surgery or bleeding from the nose. After applying the nasal packing, the nurse will instruct the patient to avoid blowing through the nose vigorously, because this can cause pressure on the blood vessels and increase the risk of bleeding. Aspirin can increase bleeding and slows clotting time in a patient who has epistaxis; the nurse will instruct the patient to avoid the use of aspirin for pain. Sneezing with mouth open helps to reduce bruising and swelling of the nose. The nurse will instruct the patient to keep the nasal packing in place for few days because this helps in preventing bleeding.

A family member of a client who has been diagnosed with severe acute respiratory syndrome (SARS) asks the nurse why the client is not receiving an antibiotic. How does the nurse respond to this family member?

Antibiotics are not effective because SARS is caused by a virus.

The RN is observing a new nurse providing care for a patient admitted with anterior epistaxis (nosebleed). What actions taken by the new nurse are appropriate for this patient? Select all that apply.

Apply direct lateral pressure to the nose for 10 minutes. Maintain Universal Body Substances Precautions. Apply ice or cool compresses to the nose. Instruct the patient not to blow the nose for 24 hours The correct position for a patient with an anterior nosebleed is upright and leaning forward to prevent blood from entering the stomach and to avoid aspiration. The nurse should instruct the patient to apply pressure for 10 minutes by pinching the nares. Standard Precautions should be utilized because bodily fluid is present. Ice or cool compresses will help stop the bleeding. The patient should not attempt to blow the nose for 24 hours because the newly formed clot can become dislodged.

A patient comes to the emergency department (ED) with a bruised and swollen nose after being hit with a baseball 2 days ago. X-ray reveals a displaced fracture of the nose. Which order does the nurse anticipate implementing?

Applying a cool compress and administering analgesic medication A patient who has a displaced nasal fracture will require a closed reduction, which should be performed within 24 hours after the injury. After 24 hours, the fracture is difficult to reduce because of edema and scar formation, so the provider will wait up to several days until the swelling is gone to reduce the fracture. The nurse should expect to provide care that limits pain and swelling, which includes cold compresses and analgesics. Packing the nose is not necessary unless there is uncontrolled bleeding. A rhinoplasty is done for complex fractures or those that don't heal properly.

The community health nurse is planning tuberculosis treatment for a patient who is homeless and heroin-addicted. Which action will be most effective in ensuring that the patient completes treatment?

Arrange for a health care worker to watch the patient take the medication Because this patient is unlikely to adhere to long-term treatment unless directly supervised while taking medications, the best option is to arrange for directly observed therapy (DOT). Giving a patient who is homeless and addicted to heroin written instructions on how to take prescribed medications is placing too much responsibility on the patient to follow through. Also, the question does not indicate whether the patient can read. Simply because the patient can state the names and side effects of medications does not mean that the patient understands what the medications are and why he or she needs to take them. A patient who is homeless may be more concerned with obtaining shelter and food than with properly taking his or her medication.

A patient has been diagnosed with nasal fracture. What measure does the nurse plan to initiate immediately upon admission to prevent complications?

Assessment for an airway obstruction Prompt recognition of an airway obstruction provides the opportunity to remedy the problem early. Although the other answers all address potential problems in patients with upper respiratory disorders, the immediate need at time of admission is to confirm and maintain a patent airway.

The nurse is planning care for an 80-year-old long-term care patient who takes a histamine-2 blocker and who is confused most of the time. To help prevent pulmonary infection in this patient, which nursing action is included in the plan of care?

Assist the patient with all oral intake

A patient has lobar pneumonia. To help ensure that the expected outcome of maintaining an oxygen saturation of 95% or greater is met, which nursing intervention is most important?

Assist with coughing, deep-breathing, and incentive spirometry every 2 hours Assisting the patient to clear the airway of secretions is most important for increasing oxygen saturation because it allows improved oxygenation. Assessing breath sounds and respiratory effort; monitoring vital signs; and obtaining a complete blood count, sputum, and blood cultures are important interventions but are not the priority.

28. A nurse is developing a teaching plan for a patient with COPD. What should the nurse include as the most important area of teaching? A) Avoiding extremes of heat and cold B) Setting and accepting realistic short- and long-range goals C) Adopting a lifestyle of moderate activity D) Avoiding emotional disturbances and stressful situations

B Feedback: A major area of teaching involves setting and accepting realistic short-term and long-range goals. The other options should also be included in the teaching plan, but they are not areas that are as high a priority as setting and accepting realistic goals.

29. A nurse is assessing a patient who is suspected of having bronchiectasis. The nurse should consider which of the following potential causes? Select all that apply. A) Pulmonary hypertension B) Airway obstruction C) Pulmonary infections D) Genetic disorders E) Atelectasis

B, C, D Feedback: Bronchiectasis is a chronic, irreversible dilation of the bronchi and bronchioles. Under the new definition of COPD, it is considered a disease process separate from COPD. Bronchiectasis may be caused by a variety of conditions, including airway obstruction, diffuse airway injury, pulmonary infections and obstruction of the bronchus or complications of long-term pulmonary infections, or genetic disorders such as cystic fibrosis. Bronchiectasis is not caused by pulmonary hypertension or atelectasis.

A 55-year-old woman with a long history of COPD and 40 years of smoking cigarettes is being admitted to the pulmonary stepdown unit from the ED. The ED nurse reports that the patient is on oxygen at 2 L per nasal cannula after having bronchodilator respiratory treatment in the ED. She has bilateral expiratory wheezes and crackles both anteriorly and posteriorly. A saline lock was placed in her right forearm for intermittent medications. Based on the patient's diagnosis, which clinical manifestations would the nurse expect to see when assessing this patient? (Select all that apply.) Bradycardia Shortness of breath Use of accessory muscles Sitting in a forward posture Barrel chest appearance

B, C, D, E

A patient was admitted with a diagnosis of respiratory failure 3 weeks ago. She required an artificial airway (tracheostomy) to help clear her secretions. The previous shift nurse reports that the patient had a very restless night with a drop in her O2 saturation level several times despite her O2 being set at 40% via trach collar. The previous shift nurse also reports that the patient experienced tachycardia and tachypnea during the night. The nurse immediately checks on the patient and finds that she appears anxious and her vital signs are as follows: Blood pressure: 128/84 mm Hg Heart rate: 114 (sinus tachycardia) Respiratory rate: 24 and labored Temperature: 99.4º F (axillary) O2 saturation: 91% on 40% O2 via trach collar Which of these findings are cause for concern?

B, C, E

31. An interdisciplinary team is planning the care of a patient with bronchiectasis. What aspects of care should the nurse anticipate? Select all that apply. A) Occupational therapy B) Antimicrobial therapy C) Positive pressure isolation D) Chest physiotherapy E) Smoking cessation

B, D, E Feedback: Chest physiotherapy, antibiotics, and smoking cessation are cornerstones of the care of patients with bronchiectasis. Occupational therapy and isolation are not normally indicated.

A diabetic patient's arterial blood gas (ABG) results are pH 7.28; PaCO2 34 mm Hg; PaO2 85 mm Hg; HCO3- 18 mEq/L. The nurse would expect which finding? A. Intercostal retractions B. Kussmaul respirations Correct C. Low oxygen saturation (SpO2) D. Decreased venous O2 pressure

B. Kussmaul respirations Correct

Based on the patient's vital signs, what is the appropriate nursing action? Inform the provider of abnormal vital signs. Complete an assessment of airway and respiratory status. Provide patient teaching regarding relaxation techniques. Notify the Rapid Response Team for extra assistance.

B.

A patient with newly diagnosed lung cancer tells the nurse, "I don't think I'm going to live to see my next birthday." Which response by the nurse is best? A. "Would you like to talk to the hospital chaplain about your feelings?" B. "Can you tell me what it is that makes you think you will die so soon? " C. "Are you afraid that the treatment for your cancer will not be effective?" D. "Do you think that taking an antidepressant medication would be helpful?"

B. "Can you tell me what it is that makes you think you will die so soon? "

The nurse teaches a patient about pursed lip breathing. Which action by the patient would indicate to the nurse that further teaching is needed? A. The patient inhales slowly through the nose. B. The patient puffs up the cheeks while exhaling. C. The patient practices by blowing through a straw. D. The patient's ratio of inhalation to exhalation is 1:3.

B. The patient puffs up the cheeks while exhaling.

The nurse is caring for an older patient with a fractured hip. Which pain control goal would be the most realistic for the patient? Select one: A. The patient will experience relief from pain. B. The patient's self-report of pain will remain below 5 out of 10 while hospitalized. C. Scheduled and breakthrough analgesia will be administered as needed. D. The patient will state that being in a state of comfort.

B. The patient's self-report of pain will remain below 5 out of 10 while hospitalized. The goal that the patient's self-report of pain will remain below 5 out of 10 while hospitalized is realistic, specific, and achievable. The patient experiencing relief from pain is not specific. The goal that the patient will state being in a state of comfort is neither realistic nor measurable. Providing analgesia is a nursing goal.

The nurse assessing a client's peripheral IV site obtains and documents information about it. Which assessment data indicate the need for immediate nursing intervention? A. Client states, "It really hurt when the nurse put the IV in." B. The vein feels hard and cordlike above the insertion site. C. Transparent dressing was changed 5 days ago. D. Tubing for the IV was last changed 72 hours ago.

B. The vein feels hard and cordlike above the insertion site. A hard, cordlike vein suggests phlebitis at the IV site. The IV should be discontinued and restarted at another site. It is common for IVs to cause pain during insertion. An intact transparent dressing requires changing only every 7 days. Tubing for peripheral IVs should be changed every 72 to 96 hours.

When assessing a patient with a sore throat, the nurse notes anterior cervical lymph node swelling, a temperature of 101.6° F (38.7° C), and yellow patches on the tonsils. Which action will the nurse anticipate taking? A. Teach the patient about the use of expectorants. B. Use a swab to obtain a sample for a rapid strep antigen test. C. Discuss the need to rinse the mouth out after using any inhalers. D. Teach the patient to avoid use of nonsteroidal antiinflammatory drugs (NSAIDs).

B. Use a swab to obtain a sample for a rapid strep antigen test.

A patient diagnosed with TB agrees to take the medication as instructed and to complete the therapy, when does teh nures tell the patient is the best time to take the medication

Bedtime

Hemoptysis

Bleeding from the airway

Bleeding

Bleeding in small amounts from the tracheotomy incision can be expected for the first few days, but constant oozing is abnormal. Wrap gause around th tube andpack gause gently into the wound to apply pressure to the bleeding sites.

36. A nurse is teaching a patient with asthma about Azmacort, an inhaled corticosteroid. Which adverse effects should the nurse be sure to address in patient teaching? A) Dyspnea and increased respiratory secretions B) Nausea and vomiting C) Cough and oral thrush D) Fatigue and decreased level of consciousness

C Feedback: Azmacort has possible adverse effects of cough, dysphonia, oral thrush (candidiasis), and headache. In high doses, systemic effects may occur (e.g., adrenal suppression, osteoporosis, skin thinning, and easy bruising). The other listed adverse effects are not associated with this drug.

Excerise Tolerance change to age-physiologic change

Body's response to hypoxia and hypercarbia decreases.

Obstructive Sleep Apnea

Breathing disruption during sleep signs:::Excessive daytime sleepiness, inability to concentrate, irritability Assessment & Treatments: ESS, sleep study Nonsurgical management: Change of sleep position, weight loss, positive-pressure ventilation Surgical management: Adenoidectomy, uvulectomy, or uvulopalatopharyngoplasty

Lung sounds

Bronchial Bronchovesicular Vesicular

Lung sounds

Bronchial Bronchovesicular Vesicular

Coarse crackles and low-pitched crackles association:

Bronchitis Pneumonia Tumors Pulmonary edema

Crackles are associated with what respiratory diseases?

Bronchitis, Pneumonia, pulmonary edema and chronic pulmonary edema

A patient has an endotracheal tube in place for mechanical ventilation. Which nursing action is most important to prevent infection in this patient?

Brushing the patient's teeth every 12 hour

9. A student nurse is preparing to care for a patient with bronchiectasis. The student nurse should recognize that this patient is likely to experience respiratory difficulties related to what pathophysiologic process? A) Intermittent episodes of acute bronchospasm B) Alveolar distention and impaired diffusion C) Dilation of bronchi and bronchioles D) Excessive gas exchange in the bronchioles

C Feedback: Bronchiectasis is a chronic, irreversible dilation of the bronchi and bronchioles that results from destruction of muscles and elastic connective tissue. It is not characterized by acute bronchospasm, alveolar distention, or excessive gas exchange.

2. A nurse is creating a health promotion intervention focused on chronic obstructive pulmonary disease (COPD). What should the nurse identify as a complication of COPD? A) Lung cancer B) Cystic fibrosis C) Respiratory failure D) Hemothorax

C Feedback: Complications of COPD include respiratory failure, pneumothorax, atelectasis, pneumonia, and pulmonary hypertension (corpulmonale). Lung cancer, cystic fibrosis, and hemothorax are not common complications.

4. The nurse is assessing a patient whose respiratory disease in characterized by chronic hyperinflation of the lungs. What would the nurse most likely assess in this patient? A) Signs of oxygen toxicity B) Chronic chest pain C) A barrel chest D) Long, thin fingers

C Feedback: In COPD patients with a primary emphysematous component, chronic hyperinflation leads to the barrel chest thorax configuration. The nurse most likely would not assess chest pain or long, thin fingers; these are not characteristic of emphysema. The patient would not show signs of oxygen toxicity unless he or she received excess supplementary oxygen.

Restoration of adequate oxygenation and tissue perfusion by oxygen delivery adjustments and oxygen therapy when respiratory problems interfere with meeting tissue oxygen needs.

Hgb, Hemoglobin. RBCs, red blood cells.

The nurse is assessing an older adult who has been diagnosed with bacterial pharyngitis. Which assessment finding is typically associated with this medical diagnosis, but may not be present in the older adult patient

High fever and elevated white blood cell count

Which information will the nurse include in the asthma teaching plan for a patient being discharged? A. Use the inhaled corticosteroid when shortness of breath occurs. B. Inhale slowly and deeply when using the dry powder inhaler (DPI). C. Hold your breath for 5 seconds after using the bronchodilator inhaler. D. Tremors are an expected side effect of rapidly acting bronchodilators.

D. Tremors are an expected side effect of rapidly acting bronchodilators.

A 22-year-old client is seen in the ER with acute right lower quadrant abdominal pain, nausea, and rebound tenderness. It appears that surgery is imminent. What gauge catheter does the ED nurse choose when starting this client's intravenous solution? A. 24 B. 22 C. 18 D. 14

C. 18 An 18-gauge catheter is the size of choice for clients who will undergo surgery. If they need to receive fluids rapidly, or if they need to receive more viscous fluids (such as blood or blood products), a lumen of this size would accommodate those needs. Neither a 24-gauge nor a 22-gauge catheter is an appropriate size (too small) for clients who will undergo surgery. If it becomes necessary to administer fluids to the client rapidly, another IV would be needed with a larger needle—18, for example. Administering through the smallest gauge necessary is usually best practice, unless the client may be going into hypovolemic status (shock). A 14-gauge catheter is an extremely large-gauge needle that is very damaging to the vein.

A patient with right lower-lobe pneumonia has been treated with IV antibiotics for 3 days. Which assessment data obtained by the nurse indicates that the treatment has been effective? A. Bronchial breath sounds are heard at the right base. B. The patient coughs up small amounts of green mucus. C. The patient's white blood cell (WBC) count is 9000/µL. D. Increased tactile fremitus is palpable over the right chest.

C. The patient's white blood cell (WBC) count is 9000/µL.

A patient is scheduled for pulmonary function testing. Which action should the nurse take to prepare the patient for this procedure? A. Give the rescue medication immediately before testing. B. Administer oral corticosteroids 2 hours before the procedure. C. Withhold bronchodilators for 6 to 12 hours before the examination. D. Ensure that the patient has been NPO for several hours before the test.

C. Withhold bronchodilators for 6 to 12 hours before the examination.

A nurse assessing a patient's wound documents the finding of purulent drainage. What is the composition of this type of drainage? Select one: A. clear, watery blood B. large numbers of red blood cells C. white blood cells, debris, bacteria D. mixture of serum and red blood cells

C. white blood cells, debris, bacteria Purulent drainage is made up of white blood cells, liquefied dead tissue debris, and both dead and live bacteria. Purulent drainage is thick, often has a musty or foul odor, and varies in color (such as dark yellow or green), depending on the causative organism. Serous drainage is composed primarily of the clear, serous portion of the blood and from serous membranes. Serous drainage is clear and watery. Sanguineous drainage consists of large numbers of red blood cells and looks like blood. Bright-red sanguineous drainage is indicative of fresh bleeding, whereas darker drainage indicates older bleeding. Serosanguineous drainage is a mixture of serum and red blood cells. It is light pink to blood tinged.

b. Continuous cough

In performing a respiratory assessment, which finding is considered the principal or main sign of respiratory disease? a. Sputum production b. Continuous cough c. Fever with congestion d. Increased respiratory rate

What will be the drug of choice for a patient whose cancer overexpresses the epidermal growth factor receptor?

Cetuximab Cetuximab is prescribed to patients whose cancer overexpresses the epidermal growth factor receptor. It blocks epidermal growth factor receptors in the tumors and normal tissues. Salagen is used in xerostomia because it acts as a saliva stimulant. Salivart is also prescribed in xerostomia because it acts as artificial saliva. Cisplatin is chemotherapy used to destroy cancer cells.

A patient taking ethambutol for tuberculosis is receiving discharge teaching from the nurse. What is the most important sign or symptom of a serious adverse reaction to this medication that the nurse should teach this patient?

Change in vision Ethambutol can cause optic neuritis leading to blindness. The damage can be reversed if the problem is caught in time, so the patient should be instructed to immediately report any changes in vision to the health care provider. Severe nausea and vomiting can occur in the presence of alcohol but fatigue and anorexia are not worrisome signs on their own. This drug may precipitate gout, which causes aching of the feet, so the patient should be taught to increase fluid intake; however, this adverse reaction is not as serious as potential blindness.

An older adult resident in a long-term care facility becomes confused and agitated, telling the nurse "Get out of here! You're going to kill me!" Which action will the nurse take first?

Check the resident's oxygen saturation A common reason for sudden confusion in older patients is hypoxemia caused by undiagnosed pneumonia. The nurse's first action should be to assess oxygenation by checking the pulse oximetry. Determining the cause of the confusion is the primary goal of the RN. A complete neurologic examination may give the RN other indicators of the cause for the patient's confusion and agitation; this will take several minutes to complete. Administering lorazepam may make the patient more confused and agitated because antianxiety drugs may cause a paradoxical reaction, or opposite effect, in some older patients. Depending on the results of the patient's pulse oximetry and neurologic examination, this may be an appropriate next step. Notifying the resident's primary care provider is not a primary measure.

A patient reports a headache, nasal congestion, and fever for the past three days. A nurse examines the patient's nose and sinus areas thoroughly. What findings would suggest that the patient has sinusitis? Select all that apply.

Clinical findings that indicate sinusitis include hyperemic and edematous mucosa, tenderness over the involved sinuses, and enlarged turbinates. The inflammation results in increased blood supply to the affected area, which leads to hyperemic and edematous mucosa. Due to the inflammation, there may be tenderness over the involved frontal and/or maxillary sinuses. The turbinates may enlarge due to congestion. Clear nasal discharge is not a sign of sinusitis. Patients with sinusitis usually have a purulent nasal discharge. Nosebleed is not a manifestation of sinusitis.

What are symptoms of Cor Pulmonale?

Collapsed alveoli Hypoxia Pulmonary hypertension Cyanosis

A patient with tuberculosis (TB) who is homeless and has been living in shelters for the past 6 months asks the nurse why he must take so many medications. What information will the nurse provide in answering this question? Select all that apply.

Combination drug therapy is effective in preventing transmission. Combination drug therapy is the most effective method of treating TB. Multiple drug regimens destroy organisms as quickly as possible. The use of multiple drugs reduces the emergence of drug-resistant organisms.

A patient sustains an injury to the nose. Surgery is scheduled to repair the deviated nasal septum. The nurse expects to assess what patient symptoms? Select all that apply.

Common signs that are suggestive of a nasal septal deviation are nasal congestion, frequent nosebleeds, and difficulty in breathing through the nose. As a response to injury, blood circulation increases, causing congestion and frequent nosebleeds. A deviated septum may obstruct the air pathway, leading to difficulty in breathing through the nose. Nasal swelling and redness do not indicate deviated nasal septum but may indicate other injuries to the nose.

A patient with a history of tonsillitis complains of difficulty breathing. Which patient assessment data warrants emergency interventions by the nurse?

Contraction of neck muscles during inspiration indicates that the patient is using accessory muscles for breathing and is in serious respiratory distress. The reddened and enlarged tonsils indicate pharyngitis. The increased temperature, diaphoresis, and chills indicate an infection, which could be β-hemolytic streptococcus or fungal infection, but not an emergency situation for the patient.

Which surgery is associated with the partial removal of one vocal cord?

Cordectomy is the partial removal of one vocal cord. Hemilaryngectomy involves the removal of one side of the larynx. Supraglottic laryngectomy is the removal of the epiglottis and false vocal cords. Supracricoid laryngectomy is the removal of the entire supraglottis, false and true vocal cords, and thyroid cartilage, including the paraglottic and preepiglottic spaces.

The school nurse is providing information to high school students about influenza prevention. What should the nurse emphasize in teaching to prevent the transmission of the virus? Select all that apply.

Covering the nose and mouth when coughing is an effective way to prevent the spread of the virus. Obtaining an influenza vaccination helps prevent the flu. Staying at home helps prevent direct exposure of others to the virus. Drinking fluids helps liquefy secretions but does not prevent influenza. Antibiotic therapy is not used unless the patient develops a secondary bacterial infection.

A patient has a known allergy to dogs and is prescribed cromolyn spray. What should the nurse instruct the patient regarding the use of the spray?

Cromolyn is a mast cell stabilizer that inhibits the secretion of histamine and other inflammatory mediators. It is used to prevent symptoms of rhinitis and should be used 10 to 15 minutes before likely exposure to a known allergen. It is not recommended for daily use. Using the spray after allergen exposure would not be beneficial. Using the spray after the symptoms appear would also not help the patient.

Tracheotomy

surgical incision into trachea for purpose of establishing an airway

The nurse is transporting a patient with chest tubes to a treatment room. The chest tube becomes disconnected and falls between the bed rail. What is the priority action by the nurse? a) Clamp the chest tube close to the connection site. b) Call the physician. c) Cut the contaminated tip of the tube and insert a sterile connector and reattach. d) Immediately reconnect the chest tube to the drainage apparatus.

Cut the contaminated tip of the tube and insert a sterile connector and reattach. If the patient is lying on a stretcher and must be transported to another area, place the drainage system below the chest level. If the tubing disconnects, cut off the contaminated tips of the chest tube and tubing, insert a sterile connector in the cut ends, and reattach to the drainage system. Do not clamp the chest tube during transport.

14. A nursing is planning the care of a patient with emphysema who will soon be discharged. What teaching should the nurse prioritize in the plan of care? A) Taking prophylactic antibiotics as ordered B) Adhering to the treatment regimen in order to cure the disease C) Avoiding airplanes, buses, and other crowded public places D) Setting realistic short-term and long-range goals

D Feedback: A major area of teaching involves setting and accepting realistic short-term and long-range goals. Emphysema is not considered curable and antibiotics are not used on a preventative basis. The patient does not normally need to avoid public places.

15. A nurse is documenting the results of assessment of a patient with bronchiectasis. What would the nurse most likely include in documentation? A) Sudden onset of pleuritic chest pain B) Wheezes on auscultation C) Increased anterior-posterior (A-P) diameter D) Clubbing of the fingers

D Feedback: Characteristic symptoms of bronchiectasis include chronic cough and production of purulent sputum in copious amounts. Clubbing of the fingers also is common because of respiratory insufficiency. Sudden pleuritic chest pain is a common manifestation of a pulmonary embolism. Wheezes on auscultation are common in patients with asthma. An increased A-P diameter is noted in patients with COPD

The nurse is caring for a patient admitted to the ED after experiencing a fall while rock climbing. The patient has several facial fractures. Which objective assessment finding is most serious? A. Malaligned nasal bridge B. Blood draining from one of the nares C. Crackling of the skin (crepitus) upon palpation D. Clear glucose positive fluid draining from nares

D. Facial fracture

What is an autosomal recessive, multisystem disease characterized by altered function of the exocrine glands? A. Sickle-cell disease B. Tay-Sachs disease C. Spinal muscular atrophy D. Cystic fibrosis (CF)

D RATIONALE: CF is an autosomal recessive, multisystem disease characterized by altered function of the exocrine glands. This defect primarily affects the lungs, pancreas and biliary tract, and sweat glands. Sweat glands excrete increased amounts of sodium and chloride. While sickle-cell disease, Tay-Sachs disease, and spinal muscular atrophy are all autosomal recessive, multisystem diseases, they are not characterized by altered function of the exocrine glands.

The nurse is preparing to flush a PICC line. The protocol specifies using 50 units of heparin. Available is a multidose vial containing heparin, 10 units/mL. Which syringe does the nurse use to draw up and administer the heparin? A. 1mL B. 3mL C. 5mL D. 10 ml

D [the 10-mL syringe] Always use a 10-mL syringe when flushing PICC lines because a smaller syringe creates higher pressure, which could rupture the lumen of the PICC line.

25. A nurse is caring for a patient with COPD. The patients medication regimen has been recently changed and the nurse is assessing for therapeutic effect of a new bronchodilator. What assessment parameters suggest a consequent improvement in respiratory status? Select all that apply. A) Negative sputum culture B) Increased viscosity of lung secretions C) Increased respiratory rate D) Increased expiratory flow rate E) Relief of dyspnea

D, E Feedback: The relief of bronchospasm is confirmed by measuring improvement in expiratory flow rates and volumes (the force of expiration, how long it takes to exhale, and the amount of air exhaled) as well as by assessing the dyspnea and making sure that it has lessened. Increased respiratory rate and viscosity of secretions would suggest a worsening of the patients respiratory status. Bronchodilators would not have a direct result on the patients infectious process.

A home care nurse makes the following assessments of a wound: increased drainage and pain, increased body temperature, red and swollen wound, and purulent wound drainage. What wound complication do these assessments indicate? Select one: A. fistula B. dehiscence C. evisceration D. infection

D. infection Symptoms of infection usually become apparent within 2 to 7 days after an injury or surgery; often the patient is at home. Symptoms include purulent drainage; increased drainage; pain, redness, and swelling around the wound; increased body temperature; and increased WBCs.

A nurse is administering a medication to a patient for acute pain. Of the various routes for drug administration, which would be chosen because it is absorbed more rapidly? Select one: A. oral-coated medications B. topical skin medications C. liquid oral medications D. injected medications

D. injected medications Injected medications are usually absorbed more rapidly than oral or topical medications.

The laboratory has just called with the arterial blood gas (ABG) results on four patients. Which result is most important for the nurse to report immediately to the health care provider? A. pH 7.34, PaO2 82 mm Hg, PaCO2 40 mm Hg, and O2 sat 97% B. pH 7.35, PaO2 85 mm Hg, PaCO2 45 mm Hg, and O2 sat 95% C. pH 7.46, PaO2 90 mm Hg, PaCO2 32 mm Hg, and O2 sat 98% D. pH 7.31, PaO2 91 mm Hg, PaCO2 50 mm Hg, and O2 sat 96%

D. pH 7.31, PaO2 91 mm Hg, PaCO2 50 mm Hg, and O2 sat 96%

Thrombocytopenia

Decreased numbers of platelets

The nurse is planning to deflate the tracheostomy cuff of a patient. Which task is least helpful in preventing secretions from being aspirated during deflation?

Deflating the cuff during inspiration would be least helpful in preventing aspiration because the tracheostomy cuff should be deflated during the patient's expiration and exhaled gas helps propel secretions into the mouth. Suctioning the tracheostomy tube and then the mouth are important steps in preventing secretions from being aspirated during cuff deflation. Having the patient cough and therefore clear the lower airway before cuff deflation minimizes risk of aspiration. Suctioning the tracheostomy tube and then the mouth are important steps in preventing secretions from being aspirated during cuff deflation.

Simple Facemask

Delivers O2 up to 40%-60% Minimum of 5 L/min (5-8 L/min) Mask fits securely over nose and mouth Monitor closely for risk of aspiration

Which statement about pharyngitis is correct?

Development of stridor or indications of airway obstruction should be considered a medical emergency. Pharyngitis can lead to stridor and other indications of airway obstruction due to the swelling of the tissues; this can lead to a medical emergency requiring intubation if not identified early and treated urgently. Diphtheria is a bacterial infection, not viral. Organisms spread throughout the throat can actually be varied and thus a thorough throat culture is needed to facilitate accurate diagnosis. Viral and bacterial pharyngitis are difficult to differentiate on physical examination alone.

A patient reports recurrent rhinitis and is prescribed diphenhydramine. Understanding the side effects associated with the medication, the nurse should instruct the patient to avoid which activity?

Diphenhydramine causes drowsiness and sedation. It can increase the risk of injury if the patient engages in activities like driving or operating machinery. Therefore, the patient should be asked not to drive while on the drug. Watching television, exercising, and sexual activity do not pose any danger to the patient during the treatment.

The nurse is taking a history on a patient who presents with symptoms of pharyngitis. Sore throat with dry sensation, pain on swallowing, and low grade fever. The patient mentions plans to take an overseas trip. Which immunization does the nurse suggest the patient should have, if not already received, if not already recieved, before leaving

Diphtheria

What could cause vocal cord paralysis?

Direct trauma

A patient who has been using a nasal decongestant spray to treat symptoms of rhinitis is experiencing severe rhinitis medicamentosa. The nurse notes significant nasal swelling and notifies the provider. Which treatment does the nurse expect the provider to order for this patient?

Discontinue the drug Discontinuation of the drug is the treatment of rebound rhinitis caused by overuse of decongestant nasal drops or spray. Antihistamines, leukotriene inhibitors, and mast cell stabilizers are used to treat symptoms of allergic rhinitis.

Chest wall change due to age-nursing interventions

Discuss the normal changes of aging. Discuss the need for increased rest periods during excretes. Encourage adequate calcium intake (espically during a woman's premenopause phase).

Sarcoidosis

Disease of inflammation of unknown cause that can affect any organ, but the lung is involved most often Treatment: corticosteroids

Fracture of the Nose

Displacement of bone or cartilage can cause airway obstruction or cosmetic deformity; potential source of infection CSF may indicate skull fracture

A patient is about to begin drug therapy for the treatment of tuberculosis (TB). What information is most important for the nurse to give to this patient prior to the start of therapy?

Do not drink alcohol It is most important for patients who are beginning drug therapy for tuberculosis to refrain from the use of alcohol. This is because all medications that treat tuberculosis can cause damage to the liver. Medications for TB should be taken at bedtime to help prevent nausea. The diet should include vitamins C and B and be rich in iron and protein, not carbohydrates.

The nurse is assessing a patient who has a chest tube in place for the treatment of a pneumothorax. The nurse observes that the water level in the water seal rises and falls in rhythm with the patient's respirations. How should the nurse best respond to this assessment finding? a) Document that the chest drainage system is operating as it is intended. b) Gently reinsert the chest tube 1 to 2 cm and observe if the water level stabilizes. c) Inform the physician promptly that there is in imminent leak in the drainage system. d) Encourage the patient to do deep breathing and coughing exercises.

Document that the chest drainage system is operating as it is intended. Fluctuation of the water level in the water seal shows effective connection between the pleural cavity and the drainage chamber and indicates that the drainage system remains patent. No further action is needed.

Nerve puncture may have occurred.

During the insertion of an IV catheter, a patient with dehydration reports feeling "pins and needles" in his arm. The nurse is aware that this sensation may have been caused by what?

A. Nerve puncture may have occurred.

During the insertion of an IV catheter, a patient with dehydration reports feeling "pins and needles" in his arm. The nurse is aware that this sensation may have been caused by what? A. Nerve puncture may have occurred. B. The patient's dehydration caused this sensation. C. The vein has collapsed during the catheter insertion. D. The vein has been accessed properly for the infusion.

A patient is seen in the health care providers office and is diagnosed with community acquired pneumonia, what are the most common symptoms the patient will have

Dyspnea, hypoxia, chest discomfort

Excerise Tolerance change to age-rationales

Early assessment helps prevent complications.

Which method is the best way to prevent outbreaks of pandemic influenza?

Early recognition and quarantine The recommended approach to disease prevention consists of early recognition of new cases and implementing community and personal quarantine to reduce exposure to the virus. Public gatherings should be avoided only if a widespread outbreak has occurred in a community. No vaccine is available for pandemic influenza. The pneumonia vaccine is recommended for high-risk populations because pneumonia may be a complication of influenza. The current influenza vaccine is updated, reevaluated, and changed yearly to meet anticipated changes in the virus. When a cluster of cases is discovered in an area, the antiviral drugs oseltamivir and zanamivir should be widely distributed to help reduce the severity of the infection and to decrease mortality.

A patient who experiences recurrent respiratory tract infections has chosen natural therapy. What is the most appropriate preparation for the nurse to use in this therapy?

Echinacea can reduce the incidence and duration of upper respiratory tract infections when used on a short-term basis. It should be used cautiously in patients with conditions affecting the immune system and also in asthmatic patients because it increases the risk of allergic reaction. Aloe is recommended for constipation. Ginger is used to treat nausea and vomiting during pregnancy. Hawthorn is used for mild to moderate heart failure.

Susceptibility to Infection change to age-physiologic change

Effectiveness of the cilia decreases. Immunoglobulin A decreases. Alveolar macrophages are altered.

Lung Compliance

Elasticity

Hypercarbia

Elevated blood levels of carbon dioxide

Which measure aids in reducing anxiety in a patient with head and neck cancer who is scheduled for surgery?

Encouraging the patient and family to discuss their fears Patients with head and neck cancer and their families can be extremely anxious before surgery as they face multiple unknowns. Attentive listening to their concerns, whatever they may be, serves to reduce some of the anxiety they are experiencing. No one can provide definite answers to all of the uncertainties they face; however, addressing the ones that are overwhelming them at this time will aid their comprehension of teaching and improve their overall experience. Teaching will be an important component of care; however, they must be able to effectively receive the information provided. Anxiety interferes with receptiveness and learning of new information. Cure rates and treatment options are important to discuss if that is their major concern and should be addressed when indicated but not as an automatic response to apparent anxiety.

A nurse is caring for a client who has a tracheostomy and temperature of 103° F (39.4° C). Which intervention will most likely lower the client's arterial blood oxygen saturation? a) Use of a cooling blanket b) Encouragement of coughing c) Incentive spirometry d) Endotracheal suctioning

Endotracheal suctioning Endotracheal suctioning removes secretions as well as gases from the airway and lowers the arterial oxygen saturation (SaO2) level. Coughing and using an incentive spirometer improve oxygenation and should raise or maintain oxygen saturation. Because of superficial vasoconstriction, using a cooling blanket can lower peripheral oxygen saturation readings, but SaO2 levels wouldn't be affected.

What is the priority of care for the nurse when working with patients who are experiencing disorders of the upper respiratory tract?

Ensuring a patent airway to promote oxygenation

Why Do We Need Oxygen?

Essential for life and function of cells/tissues

The standard laryngectomy plan of care for a patient admitted with laryngeal cancer includes these interventions. Which intervention will be most important for the nurse to accomplish before the surgery?

Establish a means for communicating during the immediate postoperative period, such as a Magic Slate or an alphabet board. In the immediate postoperative period, relieving pain and anxiety is going to be a major priority. Because the patient will be unable to communicate verbally, establishing a way to communicate before the surgery will help by having a plan in place. Aspiration is not a risk after a total laryngectomy because no connection is present between the mouth and the respiratory system. It will be several weeks before the patient will need to address appropriate clothing; overloading the patient with too much information before surgery is unnecessary. Suctioning and wound care is discharge teaching that can be started after the surgery when the patient and significant others are more likely to retain the information owing to decreased preoperative anxiety. The significant others can observe the care and then can begin to take over more of the care while the patient is still in the hospital in a supervised environment.

A patient is being discharged from the emergency department after being treated for epistaxis. In teaching the family first aid measures in the event the epistaxis would recur, what measures should the nurse suggest? Select all that apply.

First aid measures to control epistaxis include tilting the patient's head backwards and placing the patient in a sitting position, leaning forward. Applying ice compresses, pinching the soft lower portion of the nose or inserting a small gauze pad into the bleeding nostril should stop the bleeding within 15 minutes. Tilting the head back or forward does not stop the bleeding, but rather allows the blood to enter the nasopharynx, which could result in aspiration or nausea/vomiting from swallowing blood. Lying down also will not decrease the bleeding.

Perfusion

Flow of blood through capillary bed

Nasal Cannula

Flow rates of 1-6 L/min O2 concentration of 24-44% (1-6 L/min Flow rate >6L/min does not increase O2 because anatomical dead space is full Assess patency of nostrils Assess for changes in respiratory rate and depth

A patient is diagnosed with sinusitis. What type of corticosteroid does the nurse anticipate discussing with the patient prior to discharge? Select all that apply.

Flunisolide, triamcinolone, and fluticasone furoate are the corticosteroids used to treat sinusitis. Zileuton is a leukotriene-receptor inhibitor used to treat sinusitis. Clemastine is a first-generation antihistamine used to treat sinusitis.

What information is important to share with a patient who is being discharged after treatment for pneumonia? Select all that apply.

Get an annual influenza immunization. Avoid contact with all persons with colds or influenza. Stop or reduce any intake of tobacco and tobacco products Individuals who have had pneumonia need to be instructed to avoid contact with ill persons, stop or reduce smoking, and get an annual influenza immunization and a pneumococcal immunization as recommended by the health care provider. The patient recovering from pneumonia is advised to avoid crowded places such as malls and churches, so the patient would not be able to resume all regular activities. Pneumococcal immunizations are usually given once after age 65 and may be given 5 years after that if the patient is at high risk.

The nurse is preparing to administer a trivalent influenza vaccine (TIV) to a 70-year-old patient with chronic obstructive pulmonary disease (COPD). While reviewing the patient's immunization record, the nurse notes that the patient received a pneumococcal polysaccharide vaccine (PPV23) 10 years prior. Which action does the nurse take?

Give the TIV and suggest that the patient receive a second PPV23 There is some evidence that a second PPV23 is helpful for preventing pneumonia in patients with chronic lung disease if more than 5 years have passed since the initial PPV23. The nurse should suggest this to the patient or the provider. Administering the TIV and reminding the patient to receive this annually is correct, but the nurse should recommend a second PPV23 since this patient meets criteria for this booster. All patients over 50 years and those with chronic lung disease should receive the flu vaccine annually. Patients over 50 years of age cannot receive the live virus vaccine.

Labs drawn for Pneumonia

Gram stain culture and sensitivity CBC ABGs Serum BUN Electrolytes Creatinine CXR Pulse Oximetry

Which statement is true about community-acquired pneumonia (CAP) as compared to health care-associated pneumonia (HAP)?

HAPs are more likely to be resistant to some antibiotics HAPs are more likely to be resistant to some antibiotics, most likely related to the widespread use of antibiotics in the health care environment. Pneumonias acquired in the community are less likely to be caused by organisms that have been exposed to antibiotics and developed resistance. The fibrin and edema that accompanies the inflammation with pneumonia can stiffen the lung in both CAP and HAP. As red blood cells and fibrin move into the alveoli with pneumonia, the infection spreads to other areas of the lung in both CAP and HAP.

A patient admitted to the hospital to rule out TB, what type of mask doe the nurse wear when caring for this patient

HEPA respirator mask

The nurse assesses a patient with intrinsic laryngeal cancer. Which finding is most characteristic of this type of cancer?

Hoarseness occurs early in the course of most intrinsic (vocal cord) laryngeal cancers. The tumor prevents accurate approximation of the vocal cords during phonation, resulting in mild hoarseness. Chronic foul breath, difficulty swallowing, and a nagging dry, nonproductive cough may also be seen with laryngeal cancer but are not as common or consistent as hoarseness.

Patient is diagnosed with "strep throat"; which interventions should this patient receive:

Humidification of the air Saline gargles Increased fluid intake Oral antibiotics

A 76-year-old patient who is recovering from influenza A reports severe dry mouth and constipation. After reviewing the patient's medication list, the nurse suspects the patient is experiencing the anticholinergic effect of which medication?

Hydroxyzine

A patient is diagnosed with pneumonia, during auscultation of the lower lung fields, the nurse hears coarse crackles and identifies the patient problem of impaired oxygenation. What is the underlying physiologic condition associated with the patients condition

Hypoxemia

A home health nurse is visiting a home care client with advanced lung cancer. Upon assessing the client, the nurse discovers wheezing, bradycardia, and a respiratory rate of 10 breaths/minute. These signs are associated with which condition? a) Hypoxia b) Semiconsciousness c) Delirium d) Hyperventilation

Hypoxia As the respiratory center in the brain becomes depressed, hypoxia occurs, producing wheezing, bradycardia, and a decreased respiratory rate. Delirium is a state of mental confusion characterized by disorientation to time and place. Hyperventilation (respiratory rate greater than that metabolically necessary for gas exchange) is marked by an increased respiratory rate or tidal volume, or both. Semiconsciousness is a state of impaired consciousness characterized by limited motor and verbal responses and decreased orientation.

A 70-year-old client has a complicated medical history including chronic obstructive pulmonary disease (COPD). Which client statement indicates the need for further teaching about the disease?

I am here to receive the yearly pneumonia shot again."

A mother calls the clinic asking for a prescription for Amoxicillin for her 2-year-old son who has what the nurse suspects to be viral rhinitis. What should the nurse explain to this mother?

I'll certainly inform the doctor, but if it is a cold, antibiotics won't be used because they do not affect the virus.

The patient with a tracheostomy needs suctioning. These interventions are included in the plan for suctioning. In what order should the nurse perform these actions?

If suctioning is indicated, the needed supplies are gathered and the suction source checked before washing hands and putting on eye protection. Sterile technique is used to open the package and prepare the equipment. Then one hand is designated as contaminated to facilitate sterile suctioning with the other hand. The patient may be preoxygenated before the catheter is inserted gently without suction to minimize the amount of oxygen removed from the lungs. Then suction is applied intermittently while withdrawing the catheter in a rotating manner for 10 seconds. If another suction pass is needed, the catheter is rinsed with sterile water between passes until airway is clear or three passes have occurred (the patient will rest after three passes before additional suctioning). After suctioning the oropharynx, the lung sounds will be assessed and the time, amount, character of secretions, and response to suctioning will be recorded.

NURSING PRIORITY: Feeling puffyness around tracheostomy.

If the skin around a new tracheostomy is puffy and you can feel a crackling sensation when pressing on the skin, notify the physician immediately.

A nurse is caring for a client after a thoracotomy for a lung mass. Which nursing diagnosis should be the first priority? a) Anxiety b) Deficient knowledge: Home care c) Impaired physical mobility d) Impaired gas exchange

Impaired gas exchange Impaired gas exchange should be the nurse's first priority. After ensuring that the client has adequate gas exchange, she can address the other diagnoses of Anxiety, Impaired physical mobility, and Deficient knowledge: Home Care.

When caring for a patient, the nurse assesses tachypnea, a cough, and restlessness. The lung sounds have fine, scattered crackles and the chest x-ray shows new bilateral interstitial and alveolar infiltrates. The nurse is aware that the patient may have an acute lung injury (ALI). In what order does the nurse expect the physiologic changes of ARDS to occur if it happens with this patient? Put a comma and space between each answer choice (1, 2, 3, 4, etc.) 1. Atelectasis 2. Interstitial edema 3. Refractory hypoxemia 4. Surfactant dysfunction 5. Increased inflammatory response 6. Decreased gas exchange surface area

In the injury or exudative phase of acute respiratory distress syndrome (ARDS; one to seven days after acute lung injury) there is interstitial edema and surfactant dysfunction that lead to atelectasis. Widespread atelectasis decreases lung compliance, hyaline membranes form, and refractory hypoxemia occurs. In the reparative or proliferative phase (one to two weeks after acute lung injury) there is an increased inflammatory response, which worsens hypoxia. In the fibrotic phase (two to three weeks after acute lung injury) the lung tissue is remodeled by collagen and fibrous tissue, which decreases the available surface area for gas exchange.

The nurse is reviewing lab results for a patient who has pneumonia, which lab value does the nurse expect to see for this patient.

Increased WBC

Which groups are at greatest risk for drug-resistant Streptococcus pneumoniae? Select all that apply.

Individuals older than age 65 years Older adults exposed to children from a daycare environmenT

Which test results indicate a patient has clinically active TB

Induration of 12 mm and positive sputum

After receiving the subq Mantoux skin test, a patient with no risk factors return to the clinic in the required 48-72 hours for the test results, which assessment finding indicates a positive result

Induration/hardened area measures 10 mm or greater

The nurse is caring for a patient who is hospitalized for pneumonia. Which nursing diagnosis has the highest priority?

Ineffective airway clearance r/t thick secretions in trachea and bronchi

New onset confusion may be a great early sign of what for the geriatric population?

Infection

What are the rare complications of pharyngitis? Select all that apply.

Infection of supraglottic structures Infection of epiglottic structures

What is bronchitis?

Inflammation of the lining of bronchial tubes, which carry air to and from the lungs. Characterized by persistent cough.

Wheezes are associated with what respiratory diseases?

Inflammation, bronchospasm (bronchoconstriction), edema, secretions, pulmonary vessel engorgement

A febrile patient presents to the emergency department with a headache, chills, fatigue, nausea, vomiting, and diarrhea. What illness does the nurse suspects that the patient has?

Influenza B Symptoms of influenza B may include nausea, vomiting, and diarrhea. The influenza viruses (A, B, and C) all include headache, muscle aches, fever, chills, fatigue, and weakness. Influenza is identified as A, B, or C; there is no combination or AB.

A student nurse is providing postoperative care to a patient who has undergone a laryngectomy and has a tracheostomy tube. Which action of the student nurse indicates the need for further training?

Infusing the normal saline solution into the tracheostomy tube can cause hypoxia and endothelial cell damage. Encouraging the patient to cough frequently removes the sputum from the lungs. Positioning the patient in a semi-Fowler's position decreases tension on the sutures and prevents edema and aspiration in the patient. Suctioning when coughing to remove secretions is a sign of good tracheostomy care.

Tracheostomy Tube: How are they inserted, how long are they used for, benefits.

Inserted though a surgical opening (stoma) in the neck. Used for long-term mechanical ventilation. More comfortable, increased mobility for the patient.

A patient who has been homeless and has spent the past 6 months living in shelters has been diagnosed with confirmed tuberculosis (TB). Which medications does the nurse expect to be ordered for the patient?

Isoniazid, rifampin, pyrazinamide (PZA), ethambutol The combination of isoniazid, rifampin, pyrazinamide, and ethambutol is used to treat tuberculosis. Metronidazole is used to treat anaerobic bacteria and some parasites, but is not effective against tuberculosis. Acyclovir is used to treat viral infection. Flunisolide is a corticosteroid that is useful in asthma or other airway disease to prevent wheezing. Prednisone is a steroid. Guaifenesin is a mucolytic. Ketorolac is an NSAID that is used for short-term pain relief. Salmeterol and cromolyn sodium would most likely be given to patients with respiratory difficulties such as poorly controlled asthma from allergic sources. Dexamethasone is a steroid.

NURSING SAFETY PRIORITY: Preventing accidental decannulation during tracheostomy care by

Keeping the old ties or holder on the tube while applying new ties or holder or by keeping a hand on the tube until it is securely stable.

Which is an orbital-zygoma fracture?

Le fort III A Le fort III is an orbital-zygoma fracture called a craniofacial disjunction. A Le fort II is a maxillary and nasoethmoid complex fracture. Nasal and maxillary fractures are simple fractures of the bones after which they are named.

Lung Cancer

Leading cause of cancer deaths worldwide Poor long-term survival w/ late-stage diagnosis

How would you care for a patient with an anterior nosebleed?

Lean forward Teach not to blow nose and limit activity

A patient with suspected initial infection of tuberculosis (TB) is admitted to the respiratory intensive care unit (ICU). The nurse caring for the patient reviews the patient's recent chest x-ray. Where on the patient's chest x-ray will the nurse most likely find evidence of the patient's infection? Select all that apply.

Left lower lobe Right lower lobe Right middle lobe Initial infection of tuberculosis (TB) is most often seen in the middle or lower lobes of the lung. The upper lobes of the lung are not the primary location of initial infection of TB.

What are the symptoms of laryngeal cancer? Select all that apply.

Leukoplakia (white patches on the lining of the cheeks of the mouth), hoarseness, and difficulty swallowing are symptoms of laryngeal cancer. A scratchy throat is a symptom of acute pharyngitis. Discolored nasal drainage is a symptom of sinusitis.

Status Asthmaticus

Life- threatening Acute episodes of airway obstruction Often does not respond to common therapy Treatment: IV therapy, potent systemic bronchodilator, steroids, epinephrine, O2 End up being intubated or tracheotomy

Upper Airway Obstruction

Life- threating Causes poor gas exchange Maintain suctioning of secretions

A patient who has had a recent laryngectomy continues to report pain. Which medication would be best used as an adjunct to a narcotic once the patient can take oral nutrition?

Liquid NSAIDs NSAIDs are an excellent adjunct when used with narcotics or opioid analgesia. Steroids will not help in pain relief and will delay healing. An opioid antagonist will reverse the effect of the narcotic. Diazepam has no pain-relieving properties.

A patient is diagnosed with a head and neck lesion that is highly suspicious of cancer. In what way does the nurse demonstrate psychosocial support for the patient and family?

Listens closely to their concerns regarding quality of life after treatment This situation requires attentive listening to the concerns of the family. Based on their history and knowledge of head and neck cancer, their specific needs will vary, but the important thing is to listen and address their concerns. While the other answers are all potentially part of the overall process, their initial anxiety will limit their ability to receive other meaningful information. The prognosis and treatment recommendations will be based on the extent and type of tumor (if malignant). The focus should be on what is causing them concern and presenting honest information from a caring perspective.

The radiology report of a patient who has had a chest x-ray shows consolidation in a segment of the patient's left lung. This is typical of which type of pneumonia?

Lobar Lobar pneumonia manifests as consolidation in a segment or an entire lobe of the lung. Bronchopneumonia manifests as diffusely scattered patches around the bronchi. While lobar pneumonia is generally bacterial, the pattern of lung involvement does not necessarily indicate the etiology.

Transtracheal Oxygen Delivery (TTO)

Long-term delivery of O2 directly into lungs Small flexible catheter is passed into trachea through small incision Avoids irritation that nasal prongs cause; is more comfortable Flow rates prescribed for rest, activity

A patient has been diagnosed with oral and laryngeal cancer. He completed a course of radiation, and it is 2 days since he underwent a total laryngectomy. The patient had been very anxious about his surgery. Which medications does the nurse expect to find on his home medication list?

Lorazepam Lorazepam is a short-acting antianxiety medication that would be the most appropriate choice for this patient. Amitriptyline is a tricyclic antidepressant that would not be used specifically for this patient's anxiety. Although diazepam is an effective medication for anxiety, it is more likely to cause respiratory depression; the location of this tumor makes diazepam not the best choice for anxiety. Ketorolac is an NSAID and should not be used before surgery. Ketorolac should be used with caution, or not at all, if the patient is taking medication for anxiety.

Which antianxiety drug has the shortest duration of action?

Lorazepam Lorazepam is an antianxiety drug of short duration and has a low number of respiratory side effects. Diazepam is also an antianxiety drug, but it is not a short-duration drug and causes respiratory depression. Morphine and cevimeline are not antianxiety drugs. Morphine is used for pain management of neck and head cancers. Cevimeline is used for xerostomia; it acts as a saliva stimulant.

Emphysemalveoli

Loss of lung elasticity and hyperinflation of lung. Dyspnea; need for increased respiratory rate. Air trapping caused by loss of elastic recoil in alveolar walls, overstretching and enlargement of alveoli into bullae, collapse of small airways (bronchioles)

Pleural fricition rub characteristics:

Loud, rough, grating, scratching sounds caused by the inflamed surfaces of the pleura rubbing together; often associated with pain on deep inspirations. Heard in lateral lung fields.

What would ABGs look like in a patient that is not getting enough oxygen?

Low PO2 Acidotic

Rhonchus (rhonchi) characteristics:

Lower-pitched, coarse, continuous snoring sounds Arise from the large airways

Physical Assessment/Clinical Manifestations of head and neck cancer

Lumps in mouth, throat, neck Difficulty swallowing Color changes in mouth or tongue Oral lesion or sore that does not heal in 2 weeks Persistent, unilateral ear pain, oral bleeding Numbness of mouth, lips, or face Change in fit of dentures Hoarseness or change in voice quality Persistent/recurrent sore throat Shortness of breath Anorexia and weight loss Burning sensation when drinking citrus or hot liquids

Which diagnostic study is used to differentiate normal tissue from a diseased one?

Magnetic resonance imaging (MRI) MRI is used to differentiate normal tissue from diseased tissue. Computed tomography helps to analyze a tumor's exact location. Positron emission tomography and single-photon emission computerized tomography help to locate additional tumor sites.

Nursing Management Planning for Asthma

Maintain >80% of personal best PEFR Have minimal symptoms Maintain acceptable activity levels Few or no adverse effects No recurrent exacerbations of asthma or decreased incidence of asthma attacks Adequate knowledge to participate in and carry out management

Oral hygiene

Maintain standard precautions during the procedure because its the mouth. Avoid using glycerin swabs or mouthwash that contains alcohol to clean the mouth because these products dry the mouth, change its pH, and promote bacterial growth Help the patient rinse their mouth with normal saline every 4 hours while awake or prn.

Which nursing action has the highest priority when caring for a patient with laryngeal trauma?

Maintaining a patent airway Maintaining a patent airway remains the nursing priority until the trauma to the larynx has healed. Pain and bleeding management and communication are secondary priorities to a patent airway.

A nurse is caring for a patient who has undergone laryngectomy. Upon suctioning the tracheostomy tube, the nurse notices a slight thickening of the secretions. Which measures should the nurse take to prevent thickening of secretions? Select all that apply.

Maintaining adequate hydration through an intravenous or enteral route and humidifying the inspired gases prevents drying and thickening of secretions. This allows easy suction of secretions. Changing the patient's position helps to relieve dyspnea in the patient. Changing the tracheostomy tube does not help, because the secretions are too copious to be removed. A normal saline bolus is not recommended, because it may cause infection.

Suctioning

Maintains patent airway, promotes gas exchange Assess the need in patients who cannot cough adequately Done through nose or mouth

The nurse is caring for a patient who is in fixed occlusion for a jaw fracture. The most important reason for the nurse to provide oral care with an electronic irrigation system (WaterPik) several times daily is to prevent which condition?

Mandibular infection Treatment delay, poor oral care, and tooth infection may contribute to mandibular bone infection, which may require antibiotic therapy and sometimes surgical débridement of the infected bone. Oral care can help to prevent dental caries, gingivitis, and dry oral mucous membranes, but these do not prolong treatment for this patient.

A nurse has performed tracheal suctioning on a patient who experienced increasing dyspnea prior to a procedure. When applying the nursing process, how can the nurse best evaluate the outcomes of this intervention? a) Percuss the patient's lungs and thorax. b) Determine whether the patient can now perform forced expiratory technique (FET). c) Measure the patient's oxygen saturation. d) Have the patient perform incentive spirometry.

Measure the patient's oxygen saturation. The patient's response to suctioning is usually determined by performing chest auscultation and by measuring the patient's oxygen saturation. FET, incentive spirometry, and percussion are not normally used as evaluative techniques.

The patient has been prescribed anti-infective azithromycin for community-acquired pneumonia. What information should the nurse include when educating the patient? Select all that apply.

Medication may cause diarrhea. Take all medication even if symptoms subside Notify provider if symptoms are present after completion of antibiotics.

The patient has been prescribed anti-infective azithromycin for community-acquired pneumonia. What information should the nurse include when educating the patient? Select all that apply.

Medication may cause diarrhea. Take all medication even if symptoms subside Notify provider if symptoms are present after completion of antibiotics Azithromycin may cause diarrhea as a side effect. It is important that the patient take the entire prescription of azithromycin even if symptoms subside to help eradicate the organism and prevent development of resistant bacterial strains. If symptoms are still present after completion of the antibiotics, the provider should be notified. Azithromycin does not cause constipation. The patient should notify the health care provider if there is no improvement of symptoms within 3 days, not 24 hours.

Which is considered the priority in treatment planning for patients with head and neck cancers?

Normal lifestyle and functional ability must be preserved. Preservation of normal function without compromising long-term effectiveness of treatment is a priority in patients with head and neck cancers, especially to decrease problems with swallowing/aspiration and speech to maximize quality of life. The likelihood of a cure with radiation is dependent on the extent of the disease and sensitivity to the therapy. Chemotherapy may actually be used alone or in combination with radiation and even surgical therapy. Nonsurgical management is not limited to palliative expectations in tumors identified early in a curable stage.

IV PUSH

NEVER GIVE POTASSIUM SUPPLEMENTS BY THE IM, SUBCUTANEOUS, OR _____ _______ ROUTES

A patient comes to the emergency department (ED) complaining of recurrent epistaxis. The nurse knows that recurrent episodes of epistaxis can be caused by which of these problems? Select all that apply.

Nasal tumors, some anatomic malformations of the nose, foreign bodies inserted into the nose, and facial trauma can result in epistaxis. Using nasal decongestant sprays too frequently can also cause nose bleeds. Fever of unknown origin does not cause nasal bleeding. Low humidity, not high humidity, also can increase one's risk of epistaxis.

I

Nasoethmoid complex fracture

Why is a nasogastric tube inserted during surgery?

Nasogastric tube intubation is performed during surgery to remove the stomach contents for the first 24 to 48 hours because of the reduction or absence of peristalsis. A tracheostomy tube or an endotracheal tube is used to provide mechanical ventilation. A nasogastric tube is also used to provide food to the patients via the nose; keep in mind that feedings are not given during surgery.

Which surgical procedure is performed if a patient's lymph nodes, sternocleidomastoid muscle, jugular vein, and 11th cranial nerve have to be removed?

Neck dissection Neck dissection includes removal of the lymph nodes, sternocleidomastoid muscle, jugular vein, 11th cranial nerve, and surrounding soft tissues. Tracheostomy helps to safeguard the airway. Total laryngectomy separates the upper airway from the throat and esophagus, and a permanent laryngectomy stoma is created. In partial laryngectomy, a part of the larynx or voice box is removed.

Which statement about the precautions of caring for a hospitalized patient iwth TB are true

Negative airflow rooms are required for these patients, health care workers must wear a N95 mask or high efficiency particulate air mask

Magnesium

Normal level: 1.3 to 2.1 mg/dL critical for skeletal muscle contraction, carbohydrate metabolism, ATP formation, vitamin activation, cell growth

The nurse is caring for a patient after extensive head and neck surgery and notes a small area of bright-red blood on the dressing, which is bigger 30 minutes later. Which nursing action is important to take?

Notify the Rapid Response Team Patients who have undergone extensive head and neck surgery are at increased risk for carotid rupture, which can be life-threatening. The nurse should notify the Rapid Response Team. Reinforcing the dressing does not help stop bleeding. Applying pressure could rupture the artery. Pressure should be applied in the event of a rupture.

Postoperative Care after Rhinoplasty

Observe for edema and bleeding Check vital signs every 4 hours Change drip pad as needed

Which group of individuals should be encouraged to receive the pneumococcal vaccine as an important health promotion and maintenance intervention?

Older adults with a chronic health problem Individuals older than age 65 and those with chronic health problems should be encouraged to receive PPV 23 to prevent pneumonia. Since pneumonia often follows influenza among older adults, these individuals should also be encouraged to receive the seasonal influenza vaccination yearly. Although many individuals who develop ventilator-associated pneumonia (VAP) are older individuals with chronic illnesses, VAP in a younger individual is not a primary indication for pneumococcal vaccine.

Ensuring air warming and humidification

On an ongoing basis, assess for a fine mist emerging from the tracheostomy collar or T-piece during inspiration and expiration. Ensure adequate hydration.

What is a key difference between seasonal influenza and pandemic influenza?

Pandemic influenza has the potential to spread globally because of its highly infectious nature in humans Mutated animal and bird viruses can be highly infectious to humans and spread globally very quickly because humans have no natural resistance to the mutated virus. Both seasonal and pandemic influenza are caused by viruses. Although there is the potential to develop a monovalent vaccine to a given mutated virus, widespread prophylactic vaccination is not realistic as a preventive measure. People over age 50 with chronic illnesses and those who are immunocompromised should receive a yearly flu vaccine for the seasonal variety.

Where do you find the Carina and what is significant about it?

Part of the trachea above the division of the two bronchi. This is where we should stop when performing tracheal suctioning.

Which patients are at risk for developing Health-care acquired pneumonia?

Patient with gram-negative colonization fo the mouth

The medical-surgical unit has one negative airflow room. Which of these four newly arrived patients should the charge nurse admit to this room?

Patient with possible pulmonary tuberculosis who currently has hemoptysis A patient with possible tuberculosis should be admitted to the negative airflow room to prevent airborne transmission of tuberculosis. A patient with bacterial pneumonia does not require a negative airflow room but should be placed in Droplet Precautions. A patient with neutropenia should be in a room with positive airflow. The patient with a right empyema who also has a chest tube and a fever should be placed in Contact Precautions but does not require a negative airflow room.

Which of these patients should the charge nurse assign to the LPN/LVN working on the medical-surgical unit?

Patient with pulmonary tuberculosis who is receiving multiple medications The LPN/LVN scope of practice includes medication administration, so a patient receiving multiple medications can be managed appropriately by an LPN/LVN. Stridor is an indication of respiratory distress; this patient needs to be managed by the RN. A patient in the immediate postoperative period requires frequent assessments by the RN to watch for deterioration. A patient with a thick-sounding voice and difficulty swallowing is at risk for deterioration and needs careful assessment and monitoring by the RN.

Pharynx and Larynx change to age-rationales

Patient's voice may be soft and difficult to understand.

The nurse is performing an admission assessment on a 90-year-old patient and notes confusion with poor orientation to person, place, and time. The patient's daughter tells the nurse that this isn't normal. Which initial action by the nurse is correct?

Perform a detailed respiratory assessment including lung sounds, pulse oximetry, and temperature In older patients, a frequent first indication of pneumonia is a change in mental status due to hypoxemia. The nurse should first perform a respiratory assessment and then notify the provider of the findings. Antibiotics are not indicated unless an assessment and tests indicate an infection is present. Lab work may be ordered by the provider as part of the ongoing evaluation of this patient. Nurses should listen to family members' reports about the usual status of patients and respond if a patient is not acting normally.

The nurse is performing an admission assessment on a 90-year-old client and notes confusion with poor orientation to person, place, and time. The client's daughter tells the nurse that this isn't normal. Which initial action by the nurse is correct

Perform a detailed respiratory assessment including lung sounds, pulse oximetry, and temperature.

A patient reports severe coughing for the last few weeks. The patient reports coughing long and hard with vomiting. What disease does the nurse suspect that this patient may have?

Pertussis Symptoms of pertussis include severe coughing "fits" that can last several minutes. During this time, the patient may turn red and vomit. The patient is frequently exhausted by the coughing. Anthrax is characterized by fever, fatigue, mild chest pain, and a dry, harsh cough. Pneumonia patients experience fever, chills, headache, chest pain, and sputum production. Coccidioidomycosis is a fungal infection, and symptoms include fever, cough, chest pain, and night sweats

A patient is taking phenylephrine nasal spray for nasal congestion. What should the nurse be sure to include when discussing side effects with the patient?

Phenylephrine is a nasal spray decongestant used in the treatment of rhinitis and sinusitis. Use of this nasal spray for more than three days can cause rebound nasal congestion. Ipratropium bromide is an anticholinergic nasal spray that can cause nasal dryness and irritation. Cromolyn spray can cause occasional burning or nasal irritation. Budesonide is a corticosteroid nasal spray that causes nasal burning and stinging.

The nurse is preparing to perform tracheostomy care on a patient with a newly inserted tracheostomy tube. Which of the following actions, if preformed by the nurse, indicates the need for further review of the procedure? a) Dries and reinserts the inner cannula or replaces it with a new disposable inner cannula b) Places clean tracheostomy ties, and removes soiled ties after the new ties are in place c) Puts on clean gloves; removes and discards the soiled dressing in a biohazard container d) Cleans the wound and the plate with a sterile cotton tip moistened with hydrogen peroxide

Places clean tracheostomy ties, and removes soiled ties after the new ties are in place For a new tracheostomy, two people should assist with tie changes. The other actions, if performed by the nurse during tracheostomy care, are correct.

Pleural fricition rub association:

Pleurisy Tuberculosis Pulmonary infarction Pneumonia Lung cancer

Which complication of pneumonia creates pain that increases on inspiration because of inflammation of the parietal pleural

Pleuritic chest pain

A patient has sustained a traumatic brain injury and is mechanically ventilated. Which statement indicates a safety issue and should be addressed by the nurse?

Poor oral hygiene can lead to respiratory crisis from secretions Many patients with traumatic brain injury (TBI) cannot perform their oral care unassisted and will potentially experience buildup of secretions and drying of the mucous membranes if they are mouth-breathing. Large amounts of accumulated and dried secretions can actually contribute to airway obstruction. Vocal cord damage is not usually directly associated with TBI. Tracheotomy is not always needed with TBI. Although a cough reflex is important to protect the patient's airway, it does not negate the need for good oral hygiene.

In the event of a new SARS outbreak what is the nurses primary role

Prevent the spread of infection to other employees and patients

Neck Trauma

Priority is to assess for and maintain patent airway Assess For other injuries (e.g., cardiovascular, respiratory, intestinal, neurologic damage) Carotid artery (Carotid Angiogram) and esophagus For cervical spine injuries (prevent excessive neck movement) Obstruction can occur from initial injury or resultant swelling.

Nasoseptoplasty or Submucous Resection (SMR)

Procedure to straighten a deviated septum when chronic "stuffy" nose, snoring, sinusitis, ect.

hypernatremia (due to the increase water loss from sweating)

would you expect hyponatremia or hypernatremia in a patient suffering from heat stroke?

A nurse is caring for a patient with community-acquired pneumonia. The patient's oxygen saturation is 88% on room air. The patient is writhing in pain and cries out, "It hurts so bad to take a deep breath. I can't even cough it hurts so bad." Understanding the patient's condition, what is the nurse's priority intervention for this patient?

Provide patient with supplemental oxygen The patient in the described scenario is experiencing impaired gas exchange, a potential life-threatening condition. The nurse's priority intervention is to provide the patient with supplemental oxygen. Encouraging the patient to deep breathe and cough, administering the ordered opioid analgesic medication, and instructing the patient on splinting the chest when breathing are all appropriate nursing interventions; however, these are not the priority interventions for this patient.

Which intervention will be most effective in preventing complications from xerostomia in a patient undergoing treatment for neck cancer?

Providing fluoride treatment A patient with xerostomia is at increased risk of tooth decay. Fluoride is the most effective treatment to prevent tooth decay and decrease the spread of tooth decay. Analgesic drugs are painkillers, used to provide relief from pain, so they will not prevent tooth decay. Tooth extraction involves removal of an infected tooth to prevent the spread of infection, but tooth extraction has no role in preventing tooth decay. A root canal is provided to a patient after tooth decay, which involves replacement of infected pulp (soft center of the tooth) with artificial pulp that helps keep the tooth in place. It does not prevent tooth decay.

A patient who is taking isoniazid and rifampin to treat tuberculosis reports reddish-orange urine. Which action by the nurse is correct?

Reassure the patient that this is an expected drug side effect

Which points does the nurse include when educating an older patient and family about pneumonia prevention? Select all that apply.

Receiving an annual influenza vaccine Decreasing exposure to air pollutants Avoiding crowded public places Avoiding dehydration Since pneumonia often follows influenza, an annual vaccination for the flu is important. It may also be beneficial to repeat the pneumonia vaccine if it has been more than 5 years since vaccination. Individuals with pneumonia or who are at risk for pneumonia are at increased risk for respiratory problems when exposed to air pollutants. Crowded public places should be avoided, especially during cold and flu season because of the risk of exposure to causative organisms. Dehydration, especially in older adults, will increase the difficulty of adequate bronchial hygiene. Although monitoring vital signs may be beneficial, blood pressure is probably less critical than monitoring for the presence of a fever

Susceptibility to Infection change to age-rationales

Regular pulmonary hygiene and overall fitness help maintain maximal functioning of the respiratory system and prevent illness.

Lungs change to age-physiologic change

Residual volume increases Vital capacity decreases Efficiency of oxygen and carbon dioxide exchange decreases Elasticity changes

a. Signs of hepatitis b. Flu-like symptoms c. Low neutrophil count e. Ocular pain or blurred vision Rationale: Rifabutin (Mycobutin) may be prescribed for a client with active Myobacterium avium complex (MAC) disease and tuberculosis. It inhibits mycobacterial DNA-dependent RNA polymerase and suppresses protein synthesis. Side effects include rash, gastrointestinal disturbances, neutopenia (Low neutrophil count), red-orange-colored body secretions, uveitis (blurred vision and eye pain), myositis, arthralgia, hepatitis, chest pain with dyspnea, and flu-like syndrome. Vitamin B6 deficiency and numbness and tingling in the extremities is associated with the use of isoniazid (INH). Ethambutol (Myambutol) also causes peripheral neuritis.

Rifabutin (Mycobutin) is prescribed for a client with active mycobacterium avium complex (MAC) disease and tuberculois. For which of the following side effects of the medication should the nurse monitor? (Select all that apply) a. Signs of hepatitis b. Flu-like symptoms c. Low neutrophil count d. Vitamin B6 deficiency e. Ocular pain or blurred vision f. Tingling and numbness of the fingers

The client is postoperative for a total laryngectomy and has recovered from anesthesia. The client's respirations are 32 breaths/minute, blood pressure is 102/58, and pulse rate is 104 beats/minute. Pulse oximetry is 90%. The client is receiving humidified oxygen. To aid in the client's respiratory status, the nurse places the client in which of the following positions.

Semi- Fowler's The client is in respiratory distress. The best position for the client who has a tracheostomy and recovered from anesthesia is semi-Fowler's.

A patient is diagnosed with a septal deviation. What common clinical manifestations will the nurse document relevant to this condition after assessment? Select all that apply.

Septal deviation is the deviation of a normal straight nasal septum. Epistaxis, facial pain, and nasal congestion are clinical manifestations of septal deviation. Ecchymosis is a clinical manifestation of nasal fracture. Discolored purulent nasal drainage is a clinical manifestation of acute sinusitis.

AFTER suctioning the artificial airway

Suction the mouth. Never use oral suctin equipment for suctioning an artificial airway, because this can introduce oral bacteria into the lungs.

hypokalemia

Serum potassium level below 3.5 mEq/L, which can be life threatening because every body system is affected -important to assess respiratory status q2h (falls precaution) -drug therapy: IV or PO potassium replacement -nutrition therapy: bananas, broccoli, oranges, & spinach -causes: excessive use of diuretics, water intoxication, significant diarrhea

The home care nurse is visiting a patient newly discharged home after a lobectomy. What would be most important for the home care nurse to assess? a) The family's willingness to care for the patient b) Resumption of the patient's ADLs c) Signs and symptoms of respiratory complications d) Nutritional status and fluid balance

Signs and symptoms of respiratory complications The nurse assesses the patient's adherence to the postoperative treatment plan and identifies acute or late postoperative complications. All options presented need assessment, but respiratory complications are the highest priority because they affect the patient's airway and breathing.

What does the presents of leaking CSF indicate?

Skull/ facial fracture

What is the Epworth Sleepiness Scale (ESS) used to assess?

Sleep Apnea The Epworth Sleepiness Scale (ESS) is used for the assessment of sleep apnea. The ESS scale is not used to assess patients who are having neck trauma, laryngeal trauma, or vocal cord paralysis. These conditions are not related to sleep disorders but are associated with disorders of the larynx.

Which intervention does the nurse anticipate when a patient is experiencing airway obstruction due to the tongue falling back or excessive secretions?

Slight extension of the head and neck Extending the patient's head and neck may actually be sufficient to alleviate the partial obstruction caused by the tongue falling back in the throat; this may especially help to align the neck to decrease the accumulation of any secretions in the throat. Placing the patient in prone position will make the situation worse. A nasogastric tube will not assist in removing secretions. The patient should first be repositioned, which may eliminate the need for intubation.

What are the symptoms of sleep apnea? Select all that apply.

Snores heavily Waking up tired Daytime sleepiness Snoring, waking up tired, and sleepiness during the daytime are symptoms of sleep apnea. Sleep apnea causes upper airway obstruction, and is unassociated with neck and chest pain.

Which actions by a patient will relieve dry mouth? Select all that apply.

Sucking on ice chips, chewing sugarless gum, and sucking on sugarless candy will help in managing dry mouth in the patient. Eating bland foods will aid in swallowing and reduce irritation. Rinsing the mouth with salt water helps to heal oral mucositis and relieve irritation.

Chest Tube Chamber 3

Suction control of system

The nurse is observing a new graduate nurse during suctioning of a tracheostomy patient. Which action by the graduate nurse would require intervention?

Suction is not applied while inserting the catheter; intermittent suction is applied as the catheter is being withdrawn. Preoxygenation is also required before suctioning, and suction time should be limited to 10 seconds or less. A patient is at risk for hypoxemia after suctioning. Therefore it is imperative to monitor the patient's oxygen status before, during, and after suctioning. The catheter should be rotated during withdrawal

Which procedure involves removal of the hyoid bone?

Supraglottic partial laryngectomy Supraglottic partial laryngectomy involves removal of the hyoid bone. Laser surgery, transoral cordectomy, and vertical laryngectomy are not associated with the removal of the hyoid bone. Laser surgery reduces the tumor or destroys a tumor completely through laryngoscope. Transoral cordectomy involves cutting off a tumor through laryngoscope. Vertical laryngectomy involves removal of one true cord, one false cord, and one-half of the thyroid cartilage.

T/F: a patient who is allergic to bananas could possibly develop a latex allergy

TRUE

A patient with chronic obstructive pulmonary disease (COPD) has coarse crackles throughout the lung fields and a chronic, nonproductive cough. Which nursing intervention will be most effective?

Teach the patient to use the Flutter airway clearance device.

The nurse is caring for a patient who has unilateral vocal cord paralysis after an injury to the neck. Which has the highest priority when developing a discharge teaching plan for this patient?

Teaching the patient to tuck the chin down and tilt the forehead forward while swallowing Patients with unilateral vocal cord paralysis have an increased risk for aspiration while swallowing so they should be taught how to minimize this. The other items may be included in a comprehensive teaching plan but are not critical.

The nurse is reading a pathologic report on a patient who has esophageal cancer. Which letter informs the nurse that the cancer has spread to another part of the body?

The "M" in TNM staging to evaluate the extent of cancer stands for metastasis, which indicates the spread of cancer beyond its site of origin. "T" stands for tumor, which is the site of origin. "N" stands for lymph node involvement. "D" is not a letter used in TNM staging.

YES, DON'T TAKE BP IN SAME ARM AS PICC LINE

The CP is taking the patients BP on the right arm. The patient has A PICC line to the right arm. Does the CP need further teaching about where to take the BP in this patient

A patient with allergic rhinitis has been advised to use beclomethasone nasal spray to relieve the symptoms. What instructions should be given to this patient about the use of corticosteroid nasal spray? Select all that apply.

The corticosteroid nasal spray should be used on a regular basis to obtain maximum benefit. In this case, the patient should take nasal spray twice daily, as ordered. Use of the nasal spray should be discontinued if nasal infection occurs because it may suppress the immune system and aggravate the infection. Nasal passages should be cleared before using the spray to ensure that the medicine reaches the target area. The spray should not be used on an as-needed basis because overuse can worsen symptoms. The spray does not need to be started two weeks before the pollen season begins.

a. Respiratory acidosis

The nurse is reviewing the arterial blood gas results for a 25-year-old trauma patient who has new onset of shortness of breath and demonstrates shallow and irregular respirations. The pH is 7.26. What imbalances does the nurse suspect this patient has? a. Respiratory acidosis b. Respiratory alkalosis c. Metabolic acidosis d. Metabolic alkalosis

The nurse is caring for a patient who comes to the clinic because of a cough. What symptom of the cough will lead the nurse to believe that the health care practitioner will order testing for pertussis?

The patient reports that the cough has lasted more than 3 week An individual who reports a cough that lasts longer than 3 weeks should be tested for pertussis. The pertussis cough is not accompanied with wheezing, does not have green or yellow sputum, and is not caused by a "tickle" (drainage).

d. Presence or absence of gag reflex

The patient returns to the unit after bronchoscopy. In addition to respiratory staus assessment, which assessment does the nurse make in order to prevent aspiration? a. Presence of pain or soreness in throat b. Time and amount of last oral fluid intake c. Type and location of chest pain d. Presence or absence of gag reflex

The nurse takes an admission history on a patient with possible asthma who has new-onset wheezing and shortness of breath. Which information may indicate a need for a change in therapy?

The patient takes propranolol (Inderal) for hypertension.

The nurse teaches a patient who has asthma about peak flow meter use. Which action by the patient indicates that teaching was successful?

The patient uses albuterol (Proventil) metered dose inhaler (MDI) for peak flows in the yellow zone.

A patient arrives at the clinic with chills, a runny nose, sore throat, purulent sputum, and a cough. The nurse auscultates crackles in the left lung base. Which medication will the nurse expect the primary health care provider to prescribe for this patient?

The patient with influenza experiences chills, rhinorrhea, sore throat, purulent sputum, cough, and pneumonia. Zanamivir is a neuraminidase inhibitor used to treat influenza and is likely to be prescribed by the health care provider. This medication acts by preventing the budding and spreading of the influenza virus to other cells. Cetirizine is a second-generation antihistamine used to treat rhinitis and sinusitis. Fluticasone and ciclesonide are corticosteroids used to treat rhinitis or sinusitis.

An asthmatic patient comes to a clinic with exacerbation of asthma. The primary health care provider examines the patient for sinusitis. How would the nurse correlate asthma with sinusitis?

The postnasal drip in sinusitis may cause bronchoconstriction leading to an exacerbation of asthma. The triggers of asthma and sinusitis are different. The triggers that cause sinusitis may not necessarily trigger an asthma attack. The allergic reaction in sinusitis may not be a triggering factor for asthma. The breathing difficulty associated with sinusitis does not trigger asthma.

The nurse is planning to perform tracheostomy care on a patient. In what order does the nurse perform the procedure? Place options in order of importance.

The procedure must first be explained to the patient. The proper equipment can be placed at the bedside. If the equipment is placed first, the patient might be frightened. Position the patient in semi-Fowler's position. Next put on goggles and clean gloves and auscultate chest sounds before beginning the procedure to determine if suctioning is needed. Next change the tracheostomy cannula. After the tracheostomy has been changed and dressing applied, tracheostomy ties can be changed.

c. Pink and frothy

The respiratory therapist consults with and reports to the nurse on the sputum production of several respiratory patients. The patient producing which kind of sputum needs priority attention? a. Thick and yellow b. Watery mucoid c. Pink and frothy d. Rust-colored

Which nursing interventions should the nurse conduct while providing home care to a patient with a stoma after a tracheostomy? Select all that apply.

The stoma should be covered with a soft cloth to prevent infection, and the patient's water intake should be increased to prevent dryness, especially in dry weather. Because patients who have undergone a cordectomy lose the ability to taste and smell, the nurse should serve colorful food to the patient in an attempt to generate interest in food. The nurse should clean the area around the stoma at least daily and should not let the patient take a bath or shower without wearing a plastic collar, which prevents entry of water into the stoma

The nurse is caring for a patient who has undergone neck surgery. Which type of food should the nurse avoid while feeding the patient orally?

Thin, watery fluids are difficult to swallow and can lead to aspiration in the patient; these foods should be avoided. Bland foods are tolerable and provide less irritation or inflammation if accidentally aspirated. Pureed foods and thickened liquids should be given to the patient because they can be swallowed easily.

A client comes to the emergency department with a sore throat. Examination reveals redness and swelling of the pharyngeal mucous membranes. Which diagnostic test does the nurse expect will be requested first?

Throat Culture.

A patient comes to the emergency department with a sore throat. Examination reveals redness and swelling of the pharyngeal mucous membranes. Which diagnostic test does the nurse expect will be requested first?

Throat culture A throat culture is important for distinguishing a viral infection from a group A beta-hemolytic streptococcal infection. A chest x-ray or TB skin test are not indicated by the symptoms given. A CBC might be indicated to evaluate infection and dehydration, but would not be the first action.

A patient presents with epistaxis. Which interventions are appropriate to control the bleeding? Select all that apply.

To control epistaxis, the patient should be reassured and kept quiet. In epistaxis, approximately 90% of nosebleeds occur in the anterior portion of the nasal cavity and can be easily visualized. The patient should be made to sit, leaning slightly forward, with the head tilted forward. Direct pressure should be applied by pinching the entire soft lower portion of the nose against the nasal septum for 10 to 15 minutes. If bleeding does not stop within 15 to 20 minutes, consult the health care provider. Saline nasal sprays should not be used because these can dislodge the clot that is needed to stop the bleeding. Nose blowing will also remove the clot, which could lead to further bleeding.

Trach Complications

Tracheomalacia Tracheal Stenosis TEF Trachea-innominate artery fistula Tube Obstruction Tube Dislodgement & accidental decannulation Pneumothorax Subcutaneous emphysema

Cancer of the Nose & Sinuses

Tumors rare, benign or malignant Seen with exposure to dust from wood, textiles, leather, flour, nickel, chromium, mustard gas, radium, cigarettes. Slow onset, resembles sinusitis Lymph enlargement often occurs on side with tumor mass Surgical removal is treatment; may be combined with radiation (IMRT)

b. Primary bronchi

Upon performing a lung sound assessment of the anterior chest, the nurse hears moderatley loud sounds on inspiration that are equal in length with expiration. In what area is this lung sound considered normal? a. Trachea b. Primary bronchi c. Lung fields d. Larynx

What is the most suitable position in which the nurse should place the patient with epistaxis?

Upright and leaning forward position Upright and leaning forward is the best position for a patient with epistaxis because it prevents blood from entering the airway. The prone position (lying face down) will increase nasal bleeding. A supine position (flat on the back) creates pressure on the head, which will cause bleeding. A semi-Fowler's position (sitting at 30°-45°) will not reduce bleeding.

The nurse notes that a patient has a disorder that contraindicates drug therapy that the physician has just prescribed for symptomatic relief of allergic rhinitis. Which disorder does the nurse report to the physician as a possible contraindication

Urinary retention

A new patient arrives in the medical-surgical unit with a flap after a total laryngectomy. The flap appears dusky in color. What is the nurse's first action?

Use a Doppler to assess flow to the area A complete assessment of the area including Doppler activity of major feeding vessels needs to be completed, and the surgeon must be notified because the patient may have to be returned to the operating room immediately. Neither hot nor cold packs (nor anything, for that matter) should be applied to the flap site. The site is delicate and should not be massaged.

A client tells the nurse that after 3 weeks of multidrug therapy to treat tuberculosis (TB), the symptoms seem to have resolved. What does the nurse tell this client?

You will need to continue therapy for atleast 6 months

Which nursing interventions are focused on preventing the spread of severe acute respiratory syndrome (SARS) caused by coronaviruses? Select all that apply.

Using strict airborne isolation techniques Handwashing before and after all patient care Using Contact Precautions with people suspected to have SARS Disinfecting contaminated surfaces and equipment Since the SARS virus is spread via airborne droplets from infected people through sneezing, coughing, and talking, strict Airborne Precautions are essential. Hand hygiene and the use of gloves decrease the likelihood of spread to the mucous membranes, nose, and mouth and contamination of surfaces outside the patient's room. Individuals suspected to have SARS should be placed in Contact Precautions until a definitive diagnosis is made. Diagnosis is confirmed by the manifestation of symptoms and the use of a rapid SARS test within 2 days after symptoms begin. All equipment and surfaces that potentially have been contaminated must be disinfected by an individual wearing gloves. Although careful monitoring of the occurrence of SARS is important, preventing its spread is the initial focus to decrease the likelihood of a widespread epidemic.

A patient with a history of frequent and recurrent episodes of tonsillitis now reports a severe sore throat with pain that radiates behind the ear and difficulty swallowing. The nurse suspects the patient may have peritonsillar abscess. On physical assessment which deviated structure supports the nurses supposition

Uvula

Vagal nerve stimulation is possible during suctioning, what to do?

Vagal stimulation results in bad things for heart. If vagal stimulation occurs, stop suctioning immediately and oxygenate the patient manually with 100% oxygen.

Which factors can contribute to a acute pharyngitis

Viruses, irritants, bacteria, alcohol

Disorders of the Larynx

Vocal cord paralysis Laryngeal trauma Dyspnea, Aphonia, Hoarse, Subcutaneous Emphysema, Hemoptysis

Disorders of the Larynx

Vocal cord paralysis Vocal cord nodules and polyps Laryngeal trauma

The nurse is giving discharge instructions to an adult patient diagnosed with the flu, the patient says, "I'm generally pretty healthy, but I am concerned because my wife has several serious chronic health problems. What can I do to protect her from getting my flu." what does the nurse instruct the patient to do?

Wash hands thoroughly after sneezing, coughing or blowing nose, Avoid kissing, hugging, close face to face proximity or hand holding, If there is no tissue immediately available cough or sneeze into our upper sleeve, use disposable tissues rather than cloth handkerchiefs and immediately dispose of tissues.

The nurse manager at a long-term care facility is planning care for a patient who is receiving radiation therapy for laryngeal cancer. Which of these tasks will be best to delegate to a nursing assistant?

Washing the skin with soap and water Personal hygiene is within the scope of practice of the nursing assistant. Throat-numbing lozenges should not be administered by nursing assistants because they are medication and administering medication is out of the scope of practice. Assessment is a complex task that must be completed by licensed nursing staff. Educating the patient is the responsibility of licensed nursing staff and is an ongoing part of the patient's care.

Chest Tube Chamber 2

Water seal prevents air from re-entering patient's pleural space

After lobectomy for lung cancer, a client receives a chest tube connected to a disposable chest drainage system. The nurse observes that the drainage system is functioning correctly when she notes tidal movements or fluctuations in which compartment of the system as the client breathes? a) Water-seal chamber b) Collection chamber c) Suction control chamber d) Air-leak chamber

Water-seal chamber Fluctuations in the water-seal compartment are called tidal movements and indicate normal function of the system as the pressure in the tubing changes with the client's respirations. The air-leak meter — not chamber — detects air leaking from the pleural space. The collection chamber connects the chest tube from the client to the system. Drainage from the tube drains into and collects in a series of calibrated columns in this chamber. The suction control chamber provides the suction, which can be controlled to provide negative pressure to the chest.

d. Oxygen saturation in the red blood cells

What is a pulse oximeter used to mesure? a. Oxygen perfusion in the extremities b. Pulse and perfusion in the extremities c. Generalized tissue perfusion d. Oxygen saturation in the red blood cells

c. Sitting up, leaning forward on the overbed table

What is the best position for the patient to assume fo a thoracentesis? a. Side-lying, affected side exposed, head slightly raised b. Lying flat with arm on affected side across the chest c. Sitting up, leaning forward on the overbed table d. Prone position with arms above the head

b. Soft sound, long inspiration, short quiet expiration

What is the characteristic of normal lung sounds that should be heard throughout the lung fields? a. Short inspiration, long expiration, loud, harsh b. Soft sound, long inspiration, short quiet expiration c. Mixed sounds of harsh and soft, long inspiration and long expiration d. Loud, long inspiration and short, loud expiration

perform hand hygiene

What is the primary intervention the nurse can implement to prevent a central catheter-related bloodstream infection (CR-BSI) for a patient having IV therapy?

a. Symmetry of chest movement b. Rate, rhythm, and depth of respirations c. Use of accessory muscles for breathing d. Comparison of the anteroposterior diameter with the lateral diameter

What observations does the nurse make when performing a general assessment of the patient's lungs and thorax? (Select all that apply) a. Symmetry of chest movement b. Rate, rhythm, and depth of respirations c. Use of accessory muscles for breathing d. Comparison of the anteroposterior diameter with the lateral diameter e. Measurement of the length of the chest f. Assessment of chest expansion and respiratory excursion

a. Blood-tinged sputum Rationale: Sputum may be blood-tinged secondary to bleeding from the tumor

When caring for a client with lung cancer, the nurse should expect which of the following assessment findings? a. Blood-tinged sputum b. Decreased tactile fremitus c. Resonance with percussion d. Peripheral edema

d. Clubbing of fingernails and a barrel-shaped chest

Which assessment finding is an objective sign of chronic oxygen deprivation? a. Continuous cough productive of clear sputum b. Audible inspiratory and expiratory wheeze c. Chest pain that increases with deep inspiration d. Clubbing of fingernails and a barrel-shaped chest

a. 45-year-old man who breeds and raises racing pigeons

Which patient has an increased risk for problems of the respiratory system? a. 45-year-old man who breeds and raises racing pigeons b. 25-year-old woman who enjoys body surfing in the ocean c. 68-year-old woman who does needlework for relaxation d. 56-year-old man who ties flies for trout fishing

c. Soft, low rustling; like the wind in the trees

Which sounds in the smaller bronchioles and the alveoli indicate normal lung sounds? a. Harsh, hollow, and tubular blowing b. Nothing; normally no sounds are heard c. Soft, low rustling; like wind in the trees d. Flat and dull tones with a moderate pitch

b. Unexplained restlessness

While caring for a patient who had a routine surgical procedure, he nurse suspects that the patient may be having decreased tissue perfusion. Which assessment finding is considered the earliest sign of decreased oxygenation? a. Cyanosis b. Unexplained restlessness c. Cool, clammy skin d. Paleness, shortness of breath

A nurse is cleaning the inner cannula of a patient who is using a tracheostomy tube. Which order should the nurse follow while cleaning the cannula?

While cleaning the inner cannula, the nurse should immerse the cannula in a sterile cleansing solution first. Then, the outer and inner sides of the cannula should be cleaned using a tube brush or pipe cleaner. The cannula should then be rinsed in a sterile saline solution and dried. The nurse should then insert the cleaned inner cannula into the outer cannula with its curved part downward before locking it in place.

D. Presence of crackles in the lungs

While monitoring a patient who has fluid overload, the nurse would be most concerned about which assessment finding? A. Bounding pulse B. Neck vein distention C. Pitting edema in the feet D. Presence of crackles in the lungs

c. Assess the patient for air hunger or pain at the end of inhalation and exhalation

While percussing the patient's chest and lung fields, the nurse notes a high, loud, musical, drumlike sound similar to tapping a cheek that is puffed out with air. What is the nurse's priority action? a. Document this expected finding using words like, "high", "loud", and "hollow. b. Immediately notify the physician because the patient has an airway obstruction c. Assess the patient for air hunger or pain at the end of inhalation and exhalation d. Palpate for crackling sensation underneath the skin or for localized tenderness

What is the most frequent and annoying side effect that typically begins a few weeks after initiating radiation therapy in a patient with head and neck cancer?

Xerostomia is defined as dry mouth resulting from reduced or absent saliva flow. It is the most frequent and annoying side effect of radiation therapy and typically begins few weeks after initiating radiation therapy in a patient with head and neck cancer. Depression is not specifically associated with radiation therapy. Frozen shoulder is caused by removal of or damage to the spinal accessory nerve and sternocleidomastoid muscles. Copious blood-tinged secretions may be seen in a patient who has undergone surgical therapy.

An active 45 wear old school teacher with COPD taking prednisone asks if it is necessary to get the flue shot. What is the best response by the nurse

Yes, flue shots are highly recommended for patients with chronic illness and or patients with chronic illness or patients receiving immunotherapy

It will be wet, cold, pain, swelling, redness To intervene= stop infusion, pull out IV, change site Apply warm compress to that area

You are taking care of a patient that has maintenance IVF infusing. You assess the patient's IV site and you notice that the patient developed and IV infiltration. What findings do you expect to see? What will the patient's complaints be? What should you do?

A young adult patient refuses an influenza vaccine, saying, "I'm healthy and won't get that sick if I get the flu." Which is the best response by the nurse?

You may spread the disease to people who are more at risk for severe symptoms." Young children, older adults, and those with underlying chronic conditions are at risk for pneumonia and death if they become ill with influenza. Patients who refuse the influenza vaccine should be told that they are putting others at risk. Pandemic influenzas typically originate from mutated animal and bird viruses, and prevention is handled separately from seasonal influenza—pandemic influenza vaccines are typically stockpiled and not part of general influenza vaccination. Antiviral medications are useful when given 24-48 hours after onset of symptoms, but usually shorten rather than cure the disease.

A patient with asthma reports diarrhea and vomiting. Which drug should be used with caution?

Zanamivir

intravenous (IV)

_____________ therapy is the most common invasive therapy administered to hospitalized patients

To prevent infection when infusing an intermittent "piggyback" line, which intervention does the nurse implement? a. Backpriming the secondary container from the primary line b. Detaching and capping the secondary line after use c. Using a new secondary container with each drug infused d. Using sterile gloves when administering medication

a. Backpriming the secondary container from the primary line Rational: The backpriming method allows multiple drugs to be infused through the same secondary set. This method allows the primary and secondary sets to remain connected together as an infusion system and allows the nurse to adhere to the Infusion Nurses Society (INS) standards of practice. The client is at increased risk for infection whenever the catheter is disconnected from the tubing. Sterile gloves are not necessary for IV administration of medication.

Which infusion device does the nurse select for the older adult client with a medical diagnosis of "dehydration"? a. Cassette pump b. Elastomeric balloons c. Volumetric controller d. Syringe pump

a. Cassette pump Rational: An older adult client who has dehydration will require a large fluid volume that is accurately measured by using a cassette pump during the infusion. Volumetric controllers count drops for administered volume and are inherently inaccurate because of variation in drop size. A syringe pump is accurate but not appropriate for a large volume. Elastomeric balloons are used to deliver intermittent medications.

When an IV pump alarms because of pressure, what action does the nurse take first? a. Check for kinking of the catheter. b. Flush the catheter with a thrombolytic enzyme. c. Get a new infusion pump. d. Remove the IV catheter.

a. Check for kinking of the catheter. Rational: Fluid flow through the infusion system requires that pressure on the external side be greater than pressure at the catheter tip. Fluid flow can be slowed for many reasons. A common reason, and one that is easy to correct, is a kinked catheter. If this is not the cause of the pressure alarm, the nurse may have to ascertain whether a clot has formed inside the catheter lumen, or if the pump is no longer functional.

A client is receiving an infusion of amiodarone (Cordarone), and the nurse notes that the client's arm has begun to blister around the IV site. This manifestation is consistent with which condition? a. Extravasation b. Infiltration c. Infection d. Phlebitis

a. Extravasation Rational: Certain medications, including amiodarone, vancomycin, and ciprofloxacin, are venous irritants that can cause tissue sloughing and necrosis if the IV infiltrates. The other three complications are possible with any infusion and are not specific to amiodarone.

A nurse is changing the administration set on a client's central venous catheter. Which intervention is most important for the nurse to complete? a. Have the client hold his breath during the set change. b. Keep the slide clamp on the catheter extension open. c. Position the client in a high Fowler's position. d. Position in the client in a semi-Fowler's position.

a. Have the client hold his breath during the set change. Rational: An air embolus is less likely to form if the exit site is lower than the level of the heart, and if pressure in the thoracic cavity is greater when the disconnection occurs. Having the client perform the Valsalva maneuver and maintain it during disconnection and reconnection helps maintain higher intrathoracic pressure. The slide clamp on the catheter extension should be kept clamped. The client should be placed in the flat position when administration sets are changed.

A nursing administrator is concerned about the incidence of complications related to IV therapy, including bloodstream infection. Which action by the administrator would have the biggest impact on decreasing complications? a. Investigate initiating a dedicated IV team. b. Require inservice education for all RNs. c. Limit IV starts to the most experienced nurses. d. Perform quality control testing on skin preparation products.

a. Investigate initiating a dedicated IV team. Rational: The Centers for Disease Control and Prevention (CDC) recommends having a dedicated IV team to reduce complications, save money, and improve client satisfaction and outcomes. In-service education would always be helpful, but it would not have the same outcomes as an IV team. Limiting IV starts to the most experienced nurses does not allow newer nurses to gain this expertise. The quality of skin preparation products is only one aspect of IV insertion that could contribute to infection.

The RN is working with an experienced LPN (licensed practical nurse) who has been assigned several clients receiving IV therapy. What actions guide the RN in delegating aspects of IV therapy to the LPN? (Select all that apply.) a. Look up and read the State Nurse Practice Act. b. Check facility policy regarding LPNs and IV therapy. c. Ask the LPN what he or she is comfortable performing. d. Supervise the LPN when performing IV therapy. e. Divide the clients up between the two of them.

a. Look up and read the State Nurse Practice Act. b. Check facility policy regarding LPNs and IV therapy. Rational: The State Nurse Practice Act will have the information the RN needs, and in some states, LPNs are able to perform specific aspects of IV therapy. However, in a client care situation, it may be difficult and time-consuming to find it and read what LPNs are permitted to do, so another good solution would be for the nurse to check facility policy and follow it.

The nurse preparing to insert an IV on an older adult client notices that the client's skin is extremely fragile. Which action by the nurse is best? a. Use a blood pressure cuff to cause the vein to distend. b. Slap the skin vigorously to cause the vein to rise. c. Place a gauze pad under the tourniquet before tightening. d. Avoid the use of a tourniquet if the vein is already hard.

a. Use a blood pressure cuff to cause the vein to distend. Rational: The skin of older adults is often fragile, and a tourniquet may leave an ecchymotic area after the IV insertion. One option for fragile skin is to inflate a blood pressure cuff to a reading just slightly less than the client's diastolic pressure. Tapping the skin lightly may help distend a vein, but avoid slapping vigorously. Gauze padding would not prevent bruising. Veins that are already distended may be cannulated without using a tourniquet, but they must be assessed first, and hard or cordlike veins need to be avoided.

The nurse finishes administering an intermittent medication through a Groshong catheter. What is the nurse's next action? a. Clamping the catheter b. Flushing the line with saline c. Flushing with heparin d. Removing the access needle

b. Flushing the line with saline Rational: The Groshong catheter is a type of midline catheter. After intermittent use, the catheter is to be flushed with saline. The manufacturer's instructions state that the catheter should not be clamped to maintain the integrity of the catheter valve. If a heparin flush is ordered, it is given after the catheter has been flushed with saline. The access needle is used for implanted ports.

The nurse is preparing to administer an infusion of dopamine (Intropin) using a smart pump. After programming the pump and attaching the IV to the client, what action by the nurse is most important? a. Start the infusion as ordered. b. Hand-calculate the infusion rate. c. Ensure that the pump is plugged in. d. Place a "time tape" on the IV bag.

b. Hand-calculate the infusion rate. Rational: Using a smart pump does not relieve the nurse of the responsibility of ensuring that the rate is correct. Pumps can malfunction or can be programmed incorrectly, and concentrations of solution can change and differ from the pump's drug library. The nurse must hand-calculate the rate before starting the infusion, then must ensure that the pump is plugged into an electrical source. "Time tapes" on the sides of IV bags are no longer used to show approximate volume infused.

The nurse wants to find written standards for IV therapy. The nursing manager suggests that the nurse investigate publications from which resource? a. IV Therapy Nursing Society b. Infusion Nurses Society c. Nurse's State Board of Nursing d. Hospital's IV solutions vendor

b. Infusion Nurses Society The Infusion Nurses Society publishes guidelines and standards related to IV therapy and offers a national certification examination. The State Board of Nursing publishes legal information related to nursing practice, and the solutions vendor would have written information pertaining only to specific products. The IV Therapy Nursing Society does not exist, and the other organizations listed do not provide standards and guidelines related to IV therapy.

A client who has just had an IV started in the right cephalic vein tells the nurse that the wrist and the hand below the IV site feel like "pins and needles." Which action by the nurse is best? a. Document the finding and continue to monitor the IV site. b. Check for the presence of a strong blood return. c. Discontinue the IV and restart it at another site. d. Elevate the extremity above the level of the heart.

c. Discontinue the IV and restart it at another site. Rational: The sensation that the client has described is related to the IV needle touching the nerve or possibly transecting the nerve. This can lead to loss of function and potential permanent disability in the distal extremity. It is considered an emergency and the IV must be discontinued.

The nurse has just performed an IV start on a client. After the catheter has been threaded its full length in the client's vein, which action does the nurse perform next? a. Secure the IV with a securement device or tape. b. Dispose of the IV needle in the sharps container. c. Engage the safety mechanism of the IV catheter d. Note the date and time of the dressing application over the insertion site.

c. Engage the safety mechanism of the IV catheter Rational: A federal law enacted in 2000 requires health care facilities to use IV catheters with an engineered safety mechanism to prevent needle sticks, which can be a source of contamination by bloodborne pathogens. This priority action would help keep the nurse safe. Securing the IV and dating/timing the dressing are also important actions, but engaging the safety mechanism comes first. After engaging the safety mechanism, safely dispose of the needle in the sharps container.

methemoglobinemia

conversion of normal hemoglobin to methemoglobin

hypertonic infusion

corrects fluid, electrolyte, and acid-base imbalances by moving water out of body's cells, into bloodstream Ex: parenteral nutrition, 3% NS, 5% NS

cricothyriodotomy

creation of a temporary airway by making a small opening in the throat between the thyroid cartilage and the cricoid cartilage

Which client is the best candidate to receive hypodermoclysis for IV therapy? a. Client requiring 4000 mL normal saline in 24 hours b. Client with an extensive burn injury c. Client with allergy to hyaluronidase d. Client receiving pain management

d. Client receiving pain management Rational: Subcutaneous therapy (hypodermoclysis) involves the slow infusion of isotonic fluids into the client's subcutaneous tissue. Most often, it is used in hospices for pain management. It should not be used if fluid replacement needs exceed 3000 mL/day. To be used, the client must have sufficient areas of intact skin. Hyaluronidase is frequently used to help absorb the fluid during therapy.

When assessing a client's peripheral IV site, the nurse notices edema and tenderness above the site. What action does the nurse take first? a. Apply cold compresses to the IV site. b. Elevate the extremity on a pillow. c. Flush the catheter. d. Stop the infusion of IV fluids.

d. Stop the infusion of IV fluids. Rational: Infiltration occurs when the needle dislodges partially or completely from the vein. Signs of infiltration include edema and tenderness above the site. The nurse should stop the infusion and remove the catheter. Cold compresses and elevation of the extremity can be done after the catheter is discontinued to increase client comfort. Alternatively, warm compresses may be prescribed by institutional policy and may help speed circulation to the area.

After undergoing a left thoracotomy, a client has a chest tube in place. When caring for this client, the nurse must: a) encourage coughing and deep breathing. b) milk the chest tube every 2 hours. c) clamp the chest tube once every shift. d) report fluctuations in the water-seal chamber.

encourage coughing and deep breathing. When caring for a client who's recovering from a thoracotomy, the nurse should encourage coughing and deep breathing to prevent pneumonia. Fluctuations in the water-seal chamber are normal. Clamping the chest tube could cause a tension pneumothorax. Chest tube milking is controversial and should be done only to remove blood clots that obstruct the flow of drainage.

dupuytren's contracture

flexion contracture of the fourth and fifth fingers -- caused by thickening of the palmer fascia

A community health nurse is preparing a community education class on bioterrorism and the use of inhalation anthrax. When preparing to discuss the manifestations of the fulminant stage of the infection, what manifestation does the nurse include in the teaching?

hypotension Inhalation anthrax infection has two stages: prodromal (early) and fulminant (late). Hypotension may occur in the fulminant stage of inhalation anthrax infection. Fever, fatigue, and dry cough occur in the prodromal stage of inhalation anthrax infection.

COPD complications

hypoxemia/tissue anoxia acidosis respiratory infections cardiac failure, especially cor pulmonale cardiac dysrhythmias

Paget's

if a patient has a pathological fracture of unknown origin, the nurse should look at their history to see if they have ________ disease

peripherally inserted central catheter (PICC)

inserted through a peripheral vein, preferably in the upper arm and advanced until the tip of the catheter reaches a large vein above the heart -- long term (3 months or longer)

nontunneled percutaneous central venous catheter

inserted through subclavian vein in upper chest or jugular veins in neck

thoracentesis

is the needle aspiration of pleural fluid or air from the pleural space for diagnostic or management purposes.

Oxygen therapy at 4L/min as need (if have pt that has mild respiratory acidosis and history of COPD, you don't need to fix ABG, they live in that state, that's how they compensate with it, if you gave oxygen you would make worse)

nurse is reviewing the standing orders for pt who was admitted for evaluation of chest pain. Pt has history of COPD and his lab results and assessment reveal that he has mild respiratory acidosis which order would quesition?

tunneled central venous catheter

portion lies in subcutaneous tunnel -- used for frequent and long-term infusion therapy

A patient with nasal congestion, fever, and cough has been using over-the-counter medications for a week without improvement. The patient exhibits tenderness to percussion over the sinuses and referred pain to the back of the head. These findings may indicate which condition?

rhinosinusitis Prolonged upper respiratory symptoms can indicate that a sinus infection has developed. Tenderness to percussion over the sinuses and referred pain to the back of the head are common manifestations of rhinosinusitis. Manifestations of rhinitis include headache, nasal irritation, sneezing, nasal congestion, rhinorrhea, and itchy, watery eyes. The patient with pharyngitis has throat soreness and dryness, throat pain, odynophagia, difficulty swallowing, and may have a fever. Tonsillitis is manifested by a sudden sore throat, fever, muscle aches, chills, and dysphagia. The tonsils are visibly swollen and red.

short peripheral catheters

superficial veins of dorsal surface of hand and forearm are used (dwell for 72 to 96 hr)

The nurse is providing teaching to a patient who has been diagnosed with bacterial rhinosinusitis. Which instruction does the nurse include when teaching this patient about his diagnosis?

"Be sure to complete the full course of antibiotics." Treatment for bacterial rhinosinusitis includes the use of broad-spectrum antibiotics. Facial pain that is worse when bending forward is a common manifestation of rhinosinusitis. Decongestants are commonly prescribed for rhinosinusitis. Fluids should be increased unless the patient has other medical conditions that require fluid restriction.

A nurse is teaching a patient how to manage fatigue induced by radiation therapy. What statement by the patient indicates a correct understanding of the lesson?

"Because I have the most energy in the morning, I will plan my errands during this time" indicates that the patient understands the importance of doing activities that are most important to them and to rest during periods of low energy. Fatigue is a common side effect of radiation therapy and usually begins a few weeks into therapy. "I will walk three to four hours every day to increase my level of energy" indicates that the patient does not understand that scheduling activities for a period of three to four hours is excessive and does not allow time for adequate rest periods. It is important that patients suffering from radiation-induced fatigue identify support systems as a means of assistance. Avoiding requests for help would be counter to this teaching. Continuous engagement in activity would not provide periods of much needed rest for a patient who is fatigued from radiation therapy.

A patient admitted for sleep apnea asks the nurse, "Why does it seem like I wake up every 5 minutes?" What is the nurse's best response?

"Because your body isn't getting rid of carbon dioxide. This is what stimulates your body to wake up and breathe." During sleep, the muscles relax and the tongue and neck structures are displaced with the tongue falling back, causing an upper airway obstruction. This obstruction leads to apnea and increased levels of carbon dioxide. Respiratory acidosis stimulates neural centers in the brain, and the patient awakens, takes a deep breath, and goes back to sleep. After the patient returns to sleep, the cycle may be repeated as often as every 5 minutes as the airway is reobstructed. Too much carbon dioxide, not a lack of oxygen, is the trigger that causes the patient to awaken and breathe. Technically the patient is not choking. Telling the patient he or she isn't really awakening that often minimizes the patient's concern and is not accurate. The patient may be awakening every 5 minutes as the cycle repeats.

A nurse is teaching a client about using an incentive spirometer. Which statement by the nurse is correct? a) "Don't use the incentive spirometer more than 5 times every hour." b) "You need to start using the incentive spirometer 2 days after surgery." c) "Breathe in and out quickly." d) "Before you do the exercise, I'll give you pain medication if you need it."

"Before you do the exercise, I'll give you pain medication if you need it." The nurse should assess the client's pain level before the client does incentive spirometry exercises and administer pain medication as needed. Doing so helps the client take deeper breaths and help prevents atelectasis. The client should breathe in slowly and steadily, and hold his breath for 3 seconds after inhalation. The client should start doing incentive spirometry immediately after surgery and aim to do 10 incentive spirometry breaths every hour.

A patient has received packing for a posterior nosebleed. In reviewing the patient's orders, which order does the nurse question?

"Give ibuprofen 800 mg every 8 hours as needed for pain." Ibuprofen is contraindicated in a patient with a nosebleed because NSAIDs inhibit clotting. At least initially, bedrest is suggested because significant amounts of blood may have been lost owing to a posterior nosebleed; elevation of the head of the bed is recommended for patient comfort and to facilitate drainage of secretions. Humidified air and humidified oxygen, if oxygen is ordered, are recommended because dryness of the nasal mucosa is a cause of epistaxis (nosebleed). Any patient who is admitted for epistaxis needs suction at the bedside in the event of further bleeding.

A patient's mother asks what is the most important thing she will need to know to care for her son who is having an inner maxillary fixation (IMF) completed as an outpatient. What does the nurse tell her?

"Make sure he always has wire cutters with him." It is extremely important that the patient always have wire cutters in the event of emesis, so the wires can be cut to prevent aspiration. Remind the patient to contact the surgeon as soon as possible if the wires have been cut, so that fixation can be re-established. Antiemetics like promethazine, ondansetron, and prochlorperazine are prescribed by a health care provider on an as-needed basis only for nausea. Good nutrition ensuring adequate protein intake for healing must be maintained. A specific dental liquid diet will be reviewed with the patient and significant others before surgery. Dental hygiene will be maintained with an irrigation device like a WaterPik or SoniCare, not with a brush.

A patient who has undergone a partial laryngectomy for neck cancer has been a heavy smoker for many years and tells the nurse that it will be easy to avoid smoking now that this has occurred. How does the nurse respond?

"Tell me about any times you have tried to quit smoking in the past." Studies are mixed when correlating confidence about attempts to quit smoking and successfully quitting smoking. An important initial assessment should be to identify how difficult the patient perceives smoking cessation to be. Asking the patient to describe previous attempts will help to determine this perceived difficulty. Getting patients to participate in a smoking cessation group will increase the chances of success; excessive confidence may be detrimental because smokers who are confident that they can quit are less likely to join a group. The nurse should not reinforce excessive confidence and should also not discourage the patient.

The nurse is preparing to assist the health care provider with the removal of a patient's chest tube. Which of the following instructions will the nurse correctly give the patient? a) "Exhale forcefully while the chest tube is being removed." b) "During the removal of the chest tube, do not move because it will make the removal more painful." c) "When the tube is being removed, take a deep breath, exhale, and bear down." d) "While the chest tube is being removed, raise your arms above your head."

"When the tube is being removed, take a deep breath, exhale, and bear down." When assisting in the chest tube's removal, instruct the patient to perform a gentle Valsalva maneuver or to breathe quietly. The chest tube is then clamped and quickly removed. Simultaneously, a small bandage is applied and made airtight with petrolatum gauze covered by a 4 × 4-inch gauze pad and thoroughly covered and sealed with nonporous tape. The other options are incorrect instructions for the patient.

A nurse is caring for a patient with chronic obstructive pulmonary disease (COPD). The patient's 52-year-old caregiver asks the nurse if she should receive an annual influenza vaccination. What is the nurse's best response?

"Yes, you should receive the influenza vaccination by injection and should receive it every year. Yearly vaccination is recommended for those older than 50 years as well as those who care for those with chronic conditions. The live attenuated influenza vaccine (LAIV) by intranasal spray is recommended only for those who are healthy and only for those age 49 years or younger.

A co-worker tells the nurse that she will not get the flu shot because she believes it is better to develop her own immunity to the flu. What does the nurse tell this co-worker?

"You are putting your patients at increased risk for serious respiratory illness." All people who provide direct care to patients should get the influenza vaccine to prevent the spread of influenza to patients who are at risk for serious respiratory illness. The flu vaccine does not cause influenza symptoms. Antiviral medications are only effective if given early and do not cure influenza. Even young, relatively healthy individuals can have severe influenza.

A 65-year-old patient with chronic obstructive pulmonary disease (COPD) asks the nurse about the best way to prevent pneumonia. What is the nurse's best response?

"You should get the pneumococcal polysaccharide vaccine." Older patients with chronic lung disease should receive at least one PPV23 vaccine to prevent pneumonia. Prophylactic antibiotics are not widely used because of the increased risk of bacterial resistance. Making an appointment with the provider at the first sign of infection and staying away from large groups of people may be recommended, but are not the most important.

A patient who has received radiation therapy for laryngeal cancer 2 weeks prior asks the nurse when the hoarseness will improve. Which answer by the nurse is correct?

"You will need to continue voice rest for 2-4 more weeks. Radiation therapy may cause worsening of hoarseness, and this may persist for 4-6 weeks. Patients will need to maintain strict voice rest until this symptom clears. Gargling with mouthwash is recommended to treat the discomfort caused by tissue dryness. Persistent hoarseness within 4-6 weeks of radiation therapy does not indicate the spread of cancer. Radiation therapy causes some permanent changes to tissues but not hoarseness.

Comlications: Tube Dislodgement and Accidental Decannulation Nursing Interventions!:

(Can occur when the tube system is not secure) Tube dislodgment in the first 72 hours after surgery is an emergency cause the tracheostomy tract has not matured and replacement is dificult. Tube may end up in subcutaneous tissue instead of trachea. -Secure tube in place with tracheostomy ties; do not remove trach ties until new ties in place -Be sure tube (and obturator) of same size is in patient room at all times -May attempt to re-insert tube +Perform if after 72 hours of initial placement +Extend patient's neck and open tissues of the stoma +Insert obturator into the tracheostomy tube, then quickly and gently re-insert tube and remove obturator

Pneumothorax

(air in the chest cavity) can develop during the tracheotomy procedure if the chest cavity is entered. Usually occurs in apex of lungs.

The nurse would monitor which comorbidity in the patient treated for an asthma exacerbation with methylprednisolone (SoluMedrol)? 1. Diabetes mellitus 2. Hyperlipidemia 3. Hypothyroidism 4. Raynaud's phenomenon

1 RATIONALE: Hyperglycemia or increased blood glucose level is an adverse effect of methylprednisolone, so the patient with diabetes mellitus should be monitored for elevations in blood sugar. Methylprednisolone will not affect elevated cholesterol, hypothyroidism, or Raynaud's phenomenon.

Serum potassium 5.7mEq (would have less than 7.35, 21-28 is normal bicarb so would have less bicarb, serum potassium is high and is suppose to be high with acidosis (hyperkalemia)

Nurse is evaluating the lab work of pt who has uncontrolled metabolic acidosis. Which outcome would result from this condition?

A nurse has taught the technique of pursed-lip breathing to a patient. During a return demonstration, what patient action requires correction? 1. Puffing of cheeks while exhaling air 2. Slow and deep inhalation through the nose 3. Slow exhalation through pursed lips, as if whistling 4. Exhalation time three times as long as inhalation time

1 RATIONALE: In pursed-lip breathing, the patient should avoid puffing of the cheeks while exhaling the air. Puffing of the cheeks makes the technique less effective. Slow and deep inhalation, slow exhalation through pursed lips as if whistling and exhalation time thrice as long as inhalation time are correct techniques of pursed-lip breathing.

The patient has been receiving oxygen by nasal cannula. The nurse suspects the patient is experiencing oxygen toxicity after noting which finding? 1. Restlessness 2. Tachypnea 3. Diminished lung sounds 4. Oxygen saturation 98%

1 RATIONALE: Oxygen toxicity is rare; signs and symptoms include restlessness, confusion, and substernal chest pain. Tachypnea, decreased lung sounds, and low oxygen saturation are not consistent with oxygen toxicity.

The nurse is assessing a patient with a respiratory problem. Which findings indicate the patient is suffering from pulmonary fibrosis? 1. Normal percussion 2.Prolonged expiration 3. Fremitus over affected area 4. Egophony over effusion

1 RATIONALE: Patients with pulmonary fibrosis have normal percussion findings. Inspection would reveal tachypnea and palpation would show movement. Auscultation shows crackles or sounds like Velcro being pulled apart. Prolonged expiration occurs with asthma and fremitus over affected area occurs with pneumonia. Egophony over effusion often occurs with pleural effusion.

c. Decreased breath sounds Rationale: The greatest risk following a throacentesis is development of a pneumothorax. Decreased breath sounds may indicate a pneumothorax and is the priority assessment finding.

A nurse is caring for a client who has just had a thoracentesis. Which of the following is the priority assessment finding? a. Hemoptysis b. Insertion site pain c. Decreased breath sounds d. Temperature of 37.3 (100 deg F)

A patient hospitalized with chronic obstructive pulmonary disease (COPD) is being discharged home on O2 therapy. Which instruction should the nurse include in the discharge teaching?

O2 use can improve the patient's prognosis and quality of life.

The nurse notices clear nasal drainage in a patient newly admitted with facial trauma, including a nasal fracture. What should the nurse do first? 1. Test the drainage for the presence of glucose 2. Suction the nose to maintain airway clearance 3. Document the findings and continue monitoring 4. Apply a drip pad and reassure the patient that this is norma

1 RATIONALE: Clear nasal drainage suggests leakage of cerebrospinal fluid (CSF). The drainage should be tested for the presence of glucose, which would indicate the presence of CSF. Suctioning should not be done. Documenting the findings and monitoring are important after notifying the health care provider. A drip pad may be applied, but the patient should not be reassured that this is normal.

The nurse is preparing a patient for video-assisted thoracotomy surgery (VATS). The nurse knows that this type of procedure: 1 Is a minimally invasive surgical approach 2 Provides a real-time one-dimensional video 3 Is contraindicated in patients with chest trauma 4 Can only be used for patients in previously good health

1 RATIONALE: VATS is a minimally invasive surgical approach to the chest cavity to diagnose and treat diseases of the pleura, pulmonary masses and nodules, mediastinal masses, and interstitial lung disease. VATS involves a real-time two-dimensional video. VATS is a good option for debilitated patients and those with limited respiratory reserve. It is being used also for patients with chest trauma.

A patient with chronic obstructive pulmonary disease (COPD) is suspected to have developed cor pulmonale. The nurse recognizes that which test result helps confirm the diagnosis? 1 Large pulmonary vessels on chest X-ray 2 Decreased pressure found in a right heart catheterization 3 Left-sided heart enlargement on echocardiogram 4 Decreased B-type natriuretic peptide (BNP) levels

1 RATIONALE: Cor pulmonale is a cardiac complication of COPD resulting from pulmonary hypertension. Due to pulmonary hypertension, the pulmonary vessel may appear enlarged in a chest X-ray. There may be increased pressure found in a right heart catheterization due to pulmonary hypertension. Cor pulmonale is usually associated with right-sided heart enlargement, as there is increased pressure in the blood vessels of lungs. The BNP levels are increased due to the stretching of the right ventricle.

A patient began taking antitubercular drugs a week ago. The nurse reviews the patient's medical record and learns that the patient has a 10-year history of consuming one standard drink of alcohol three times a week. The patient states, "In the last week, my urine turned orange and I am very worried about it." How should the nurse respond? 1 Inform the patient that it is one of the side effects of the antitubercular drug rifampin (Rifadin). 2 Recognize that the tuberculosis may have spread to the liver, and further medical consultation is required. 3 Recognize that the liver may be damaged due to alcohol, and so a liver function test should be performed. 4 Instruct the patient to stop taking antitubercular drugs immediately and consult the primary health care provider.

1 RATIONALE: A nurse should be aware of some of the common side effects of antitubercular drugs like rifampin, one of which is orange discoloration of body fluids such as urine, sweat, tears, and sputum. It may also cause hepatitis. Liver damage can lead to jaundice, which usually presents as yellowish discoloration of urine and sclera. However it is highly unlikely that tuberculosis has spread to the liver. The alcohol intake of the patient is within normal limits, and so it is not correct to say that alcohol may have damaged the liver. It is also inappropriate to advise the patient to stop taking antitubercular drugs.

The nurse is caring for an 83-year-old patient and is reviewing laboratory results. The PaO2 is 76 mm Hg. What nursing action is appropriate for this finding? 1 Nothing, as this is a normal finding for this patient. 2 Notify respiratory therapy to prepare for intubation. 3 Notify the primary health care provider immediately. 4 Administer oxygen via nasal cannula at 2 L/minute

1 RATIONALE: Age affects normal arterial blood gas (ABG) values. Elderly people have a lower PaO2 level, with a decrease in approximately 10 mm Hg per decade. Normal ABG values are ranges for normal, healthy adults. It is important to establish a baseline for the individual because abnormal values become "normal" for some individuals. Notifying either respiratory therapy or the primary health care provider is not necessary at this time. Administering oxygen is not necessary at this time.

Which inhaler should the nurse be prepared to administer to the patient at the onset of an asthma attack? 1 Albuterol (Proventil) 2 Fluticasone/Salmeterol (Advair) 3 Fluticasone (Flovent) 4 Salmeterol (Serevent)

1 RATIONALE: Albuterol is a short-acting bronchodilator that should be given first when the patient experiences an asthma attack. Fluticasone/salmeterol, fluticasone, and salmeterol are not short-acting bronchodilators and will not relieve the patient's symptoms of an acute asthma exacerbation.

The nurse is caring for a patient admitted to the hospital with pneumonia. Upon assessment, the nurse notes a temperature of 101.4° F, a productive cough with yellow sputum, and a respiratory rate of 20. Which most appropriate nursing diagnosis is based upon this assessment? 1. Hyperthermia related to infectious illness 2. Ineffective thermoregulation related to chilling 3. Ineffective breathing pattern related to pneumonia 4. Ineffective airway clearance related to thick secretions

1 RATIONALE: Because the patient has spiked a temperature and has a diagnosis of pneumonia, the logical nursing diagnosis is hyperthermia related to infectious illness. There is no evidence of a chill, and the patient's breathing pattern is within normal limits at 20 breaths/minute. There is no evidence of ineffective airway clearance from the information given because the patient is expectorating sputum.

A patient with chronic obstructive pulmonary disorder is considered for lung transplantation. The patient had melanoma and hepatitis A one year ago. The patient previously smoked three cigarettes per day but has not smoked in the past year. The nurse considers the patient's history and concludes what about the patient's eligibility for lung transplantation? 1. The patient is eligible for lung transplantation. 2. Because of the history of smoking, the patient is not eligible. 3. Because of the history of hepatitis A, the patient is not eligible. 4. Because of the history of cancer one year ago, the patient is not eligible.

1 RATIONALE: Chronic obstructive pulmonary disease is one of the indications for lung transplantation. There are some absolute contraindications for lung transplantation, including but not limited to being a current smoker, chronic active hepatitis B or C, history of cancer except skin cancer, poor nutritional status, and HIV. This patient is eligible for lung transplantation. The patient is not a current smoker, has hepatitis A, and had melanoma, which is a skin cancer. Therefore there are no contraindications for lung cancer.

To ease pleuritic pain caused by pneumonia, what nursing interventions should be performed? 1. Instructing the patient to splint the chest when coughing 2. Offering the patient an incentive spirometer every 4 hours 3. Instructing the patient in how to perform abdominal breathing 4. Encouraging the patient use shallow breathing during episodes of pain

1 RATIONALE: Pleuritic chest pain is triggered by chest movement and is particularly severe during coughing and deep breathing. Splinting the chest wall will reduce movement and thus ease discomfort during coughing. Use of an incentive spirometer and practicing abdominal breathing may help increase respiratory efficiency, remove secretions, and increase oxygenation, but it will not ease pleuritic pain. The patient should not be encouraged to breathe shallowly, because this will increase the risk for atelectasis and decrease oxygenation.

The nurse is caring for the patient with a productive cough. The nurse collects a sputum specimen for an acid-fast bacillus (AFB) smear. What collection time by the nurse is most appropriate? 1. 6 AM 2. 12 noon 3. 6 PM 4. 9 PM

1 RATIONALE: The correct answer is 6 AM because if the patient has a productive cough, early morning is the ideal time to collect sputum specimens for an AFB smear because secretions collect during the night. Twelve noon, 6 PM, and 9PM are incorrect because all of these times are afternoon or evening hours and the amount of secretions for the specimen may not be optimal.

The nurse expects to find which pathological change in a patient with pulmonary arterial hypertension? 1 Increased pulmonary vascular resistance 2 Decreased pulmonary vascular resistance 3 Respiratory acidosis with failure to compensate 4 Increased crackles upon auscultation of the lower posterior and lateral lungs

1 RATIONALE: The most common cause of pulmonary arterial hypertension (PAH) is increased pulmonary vascular resistance due to primary disease or as a secondary complication of respiratory, cardiac, autoimmune, hepatic, or connective tissue disorders. Increased pulmonary vascular resistance is caused by vasoconstriction, remodeling (vessel wall thickening), and thrombosis. PAH causes right ventricular hypertrophy and eventually right-sided heart failure. PAH does not cause decreased pulmonary vascular resistance or respiratory acidosis. Increased crackles in any area of the lungs would be present in left-sided heart failure, not right-sided heart failure.

The nurse would determine that levofloxacin (Levaquin) therapy has not been effective after noting which indicator? 1. White blood cell count 14,000/mm3 2. Temperature 99.7° F 3. Increased respiratory rate 4. Adventitious lung sounds

1 RATIONALE: The normal white blood cell (WBC) count is 5000 to 10,000/mm3. An elevated WBC count above the normal range indicates persistence of the infection during treatment with an antibiotic. Lung sounds and respiratory rate are not indicators of the efficacy of antibiotic treatment. A low grade fever also may signify that the infection is persisting; however, the white blood cell count is the most reliable indicator of active infection.

The nurse is teaching a patient how to use a hand-held nebulizer. Which guideline is correct? 1 Sit in an upright position during the treatment. 2 Take short, shallow breaths while inhaling the medication. 3 Rinse the nebulizer equipment under running water once a week. 4 During the treatment, breathe in and hold the breath for five seconds.

1 RATIONALE: The patient is placed in an upright position that allows for most efficient breathing to ensure adequate penetration and deposition of the aerosolized medication. The patient must breathe slowly and deeply through the mouth and hold inspirations for two or three seconds. Deep diaphragmatic breathing helps ensure deposition of the medication. Instruct the patient to breathe normally in between these large forced breaths to prevent alveolar hypoventilation and dizziness. After the treatment instruct the patient to cough effectively. An effective home-cleaning method is to wash the nebulizer equipment daily in soap and water, rinse it with water, and soak it for 20 to 30 minutes in a 1:1 white vinegar-water solution, followed by a water rinse and air drying.

A 46-year-old patient who has undergone total left-knee arthroplasty complains of shortness of breath and slight chest pain. Temperature is 98°F, blood pressure 140/86 mmHg, respirations 30, and oxygen saturation 92% on room air. The nurse suspects that the patient is experiencing which condition? 1 Pulmonary embolus 2 Unstable angina 3 Chronic obstructive pulmonary disease (COPD) exacerbation 4 Pneumonia

1 RATIONALE: The patient presents the classic symptoms of pulmonary embolus: acute onset of symptoms, tachypnea, shortness of breath, and chest pain. Unstable angina would present with chest pain occurring at rest; COPD exacerbation would present with wheezing, cough, and shortness of breath. Pneumonia would be evident if the patient had a fever, elevated white blood cell count, and a productive cough with yellow, green, or rust-colored sputum.

The nurse is caring for a patient with obstructive pulmonary disease who had tachycardia, tachypnea, and restlessness. The patient now becomes very lethargic and has a normal respiratory rate. How does the nurse interpret this? 1.Worsening of the condition 2. Stabilization of the condition 3. Improvement in the condition 4. A more relaxed and resting patient

1 RATIONALE: The patient's condition has deteriorated as evidenced by lethargy and decreased respiratory rate. The elevated carbon dioxide levels have affected the central nervous system, causing lethargy, which may progress to coma. The patient has become exhausted and is unable to maintain the compensatory mechanisms needed to maintain acid-base balance. This patient is not improving nor is more relaxed and resting. This patient's condition is not stabilizing.

A male patient with chronic obstructive pulmonary disease (COPD) becomes dyspneic at rest. His baseline blood gas results are PaO2 70 mm Hg, PaCO2 52mm Hg, and pH 7.34. What updated patient assessment requires the nurse's priority intervention? 1. Arterial pH 7.26 2. PaCO2 50 mm Hg 3. Patient in tripod position 4. Increased sputum expectoration

1 RATIONALE: The patient's pH shows acidosis that supports an exacerbation of COPD along with the worsening dyspnea. The PaCO2 has improved from baseline, the tripod position helps the patient's breathing, and the increase in sputum expectoration will improve the patient's ventilation.

When caring for a patient with pertussis, the nurse will prioritize 1. Administering antibiotic therapy 2. Having the patient use an antihistamine at night 3. Teaching the patient how to use a bronchodilator 4. Instructing the patient to use cough suppressants

1 RATIONALE: Treatment for pertussis is antibiotics, usually macrolides (erythromycin, azithromycin [Zithromax]), to minimize symptoms and prevent spread of the disease. Cough suppressants and antihistamines should not be used, because they are ineffective and may induce coughing episodes. Corticosteroids and bronchodilators are not useful in reducing symptoms.

The nurse is reviewing gender, cultural differences, and ethnic differences of patients with obstructive lung disease. The nurse knows that: 1. Whites have the highest incidence of cystic fibrosis. 2. Before puberty, more girls are affected with asthma than boys. 3. Male African Americans have the highest mortality rates from asthma than all ethnic/gender groups. 4. African Americans and Hispanics have a higher incidence of chronic obstructive pulmonary disease (COPD).

1 RATIONALE: Whites have the highest incidence of cystic fibrosis. Before puberty, boys are affected more with asthma than girls. Female African Americans have the highest mortality rates from asthma than all ethnic/gender groups. Whites have the highest incidence of (COPD), even though higher rates of smoking occur in other ethnic groups.

What is the most common sign during an initial assessment that alerts the nurse that the patient has chronic obstructive pulmonary disease? 1. Barrel chest 2. Sunken Chest 3. Hyperventilation 4. Circumoral cyanosis

1 RATIONALE: The patient with chronic obstructive pulmonary disease (COPD) develops a barrel chest over time because trapped air enlarges the lungs and thoracic cavity, thereby reducing chest flexibility. Sunken chest, also known as funnel chest or pectus excavatum, is not related to COPD. Hyperventilation is not characteristically seen with COPD. Instead, the patient usually displays persistent dyspnea on exertion, with or without a chronic cough. Circumoral cyanosis is a bluish discoloration of the skin surrounding the mouth. It is usually an indication of a severely diminished level of oxygen and respiratory distress. Circumoral cyanosis can result from a variety of respiratory diseases and may be a late sign of the COPD disease process.

A nurse is caring for a pediatric patient who has experienced a cough, clear and watery sputum, headache, and muscle aches for the past two weeks. The nurse auscultates wheezes during expiration. There is no other abnormality found. The patient's parent asks why an antibiotic has not been prescribed. How should the nurse respond? 1. Explain that antibiotics are not required for the patient. 2. Advise the parent to see another health care provider for a second opinion. 3. Explain that the child needs anticancer treatment, and antibiotics will not help. 4. Explain that antibiotics will be prescribed if the cough persists for two more days.

1 RATIONALE: The symptoms and signs indicate that the patient may have acute bronchitis, which is a viral disorder. Therefore the nurse should explain to the father that antibiotics will not help in viral infections. If they are prescribed, antibiotics may cause side effects and may also lead to antibiotic-resistance. It is incorrect to advise the father to see another healthcare provider who will do the same. It is also inappropriate to tell him that his child needs anticancer treatment. Acute bronchitis is a self-limiting disorder, and cough may last up to three weeks. Informing the father that antibiotics will be prescribed if the cough persists for two more days is not correct.

When initially teaching a patient the supraglottic swallow following a radical neck dissection, with which food or fluid should the nurse begin? 1. Cola 2. Applesauce 3. French fries 4. White grape juice

1 RATIONALE: When learning the supraglottic swallow, it may be helpful to start with carbonated beverages because the effervescence provides clues about the liquid's position. Nonpourable pureed foods, such as applesauce, would decrease the risk of aspiration, but carbonated beverages are the better choice with which to start. French fries would not be easy to swallow when learning the supraglottic swallow. Thin, watery fluids should be avoided because they are difficult to swallow and increase the risk of aspiration.

Which lung cancer diagnosis is associated with the most rapid growth rate? 1 Small cell carcinoma 2 Squamous cell carcinoma 3 Large cell carcinoma 4 Adenocarcinoma

1 RATIONALE; Small cell carcinoma is the most malignant form of lung cancer, with a very rapid growth rate. Squamous cell carcinoma has a slow growth rate, owing to its tendency to not metastasize. Large cell carcinoma is highly metastatic via the lymphatics and blood, but its growth rate is not as rapid as small cell. Adenocarcinoma is the most common type of lung cancer, a non-small cell lung cancer.

The nurse is observing a new graduate nurse during suctioning of a tracheostomy patient. Which action by the graduate nurse would require intervention? 1 Applying suction while inserting the catheter. 2 Limiting the suction time to 10 seconds or less. 3 Assessing the patient's SpO2 and heart rate and rhythm before the procedure. 4 Providing preoxygenation for a minimum of 30 seconds before the procedure.

1 RATIONALE: Suction is NOT applied while inserting the catheter; intermittent suction is applied as the catheter is being withdrawn. Preoxygenation is also required before suctioning, and suction time should be limited to 10 seconds or less. A patient is at risk for hypoxemia after suctioning. Therefore it is imperative to monitor the patient's oxygen status before, during, and after suctioning. The catheter should be rotated during withdrawal.

A patient reports shortness of breath one day after a cholecystectomy. On examination there is dullness on percussion on the right side of the chest, and breath sounds are also decreased in this region. The nurse recognizes that the most probable reason for the assessment findings is what? 1. Atelectasis 2. Pneumonia 3. Pneumothorax 4. Tension pneumothorax

1 RATIONALE: Atelectasis is the most common complication seen after thoracic or abdominal surgery. In this condition the alveoli are collapsed, and there is no air in them. On examination the affected area is dull when percussion is done, and the breath sounds in the affected area are decreased. Therefore this patient most probably has atelectasis. Pneumonia can have similar findings, but it is highly unlikely to occur one day after surgery. In both pneumothorax and tension pneumothorax, the affected area is hyperresonant.

When teaching the patient about reducing the risks of lung cancer, what is the most important topic for the nurse to address? 1. Smoking cessation 2. Drinking more fluids 3. Eating more grains 4. Using steam inhalation

1 RATIONALE: Lung cancer and the risks associated with it can be reduced by smoking cessation, and the patient should be counseled to discontinue use of tobacco and avoid secondhand smoke as much as possible. Drinking more fluids, eating more grains, and using steam inhalation are general measures to promote health and do not have much impact on reducing risks of lung cancer.

The nurse is monitoring a patient following lung transplantation for manifestations of acute rejection, such as which of the following? Select all that apply. 1. Fatigue 2. High fever 3. Low-grade fever 4. Productive cough 5. Oxygen desaturation

1,3,5 RATIONALE: Acute rejection is fairly common in lung transplantation, typically occurring in the first 5 to 10 days after surgery. Low-grade fever, fatigue, dyspnea, dry cough, and oxygen desaturation are signs of rejection. High fever and productive cough are not signs of acute rejection.

Pulmonary rehabilitation (PR) is an evidence-based intervention that includes many disciplines working together to individualize treatment of the symptomatic chronic obstructive pulmonary disease (COPD) patient. What is PR designed to do? Select all that apply. 1. Reduce symptoms 2. Improve quality of life 3. Reduce effort by teaching to inhale when pushing 4. Enhance rest by teaching to exhale with relaxation 5. Provide a "last ditch" effort for patients with COPD

1,2 RATIONALE: PR is an evidence-based intervention that includes many disciplines working together to individualize treatment of the symptomatic COPD patient. PR is designed to reduce symptoms and improve quality of life. PR is an effective intervention to improve exercise capacity and decrease hospitalizations, anxiety, and depression. An energy-saving tip is to exhale when pushing, pulling, or exerting effort during an activity and inhale during rest. PR should no longer be viewed as a "last ditch" effort for patients with severe COPD.

A patient with a nasal fracture is scheduled for a fracture reduction surgery in two weeks. What preoperative instructions should a nurse give to the patient? Select all that apply. 1. Avoid nonsteroidal antiinflammatory drugs (NSAIDs). 2. Refrain from smoking. 3. Limit exercising. 4. Avoid ice application on the fractured area. 5. Do not consume alcoholic drinks for one week before the surgery

1,2 RATIONALE: The nurse should instruct the patient to avoid intake of NSAIDs at least two weeks before the surgery to reduce the risk of bleeding. The patient should avoid smoking, as smoking cessation promotes better wound healing. Avoiding exercise is a postoperative instruction and helps to prevent bleeding. Applying ice is a postoperative instruction to decrease swelling. Alcohol tends to increase swelling and should be avoided after the operation.

The nurse is caring for a patient on positive pressure ventilation. What nursing interventions help to maintain cardiac output in this patient? Select all that apply. 1 Administer crystalloid fluids. 2 Administer colloid solutions. 3 Administer packed red blood cells. 4 Administer metoprolol (Lopressor). 5 Increase peak end expiratory pressure (PEEP)

1,2 RATIONALE: If cardiac output falls, it may be necessary to administer crystalloid fluids or colloid solutions to expand the volume and maintain hemodynamic stability. Packed red blood cells are used to increase tissue perfusion. Metoprolol (Lopressor) reduces cardiac contractility. PEEP has to be lowered to maintain cardiac output.

While recording respirations of a patient, what assessment findings would a nurse report as paradoxic breathing? Select all that apply. 1. Chest moving inward during inspiration 2. Abdomen moving inward during inspiration 3. Dyspnea on lying down 4. Dyspnea upon standing 5. Use of pursed-lip breathing

1,2 RATIONALE: Normally, the thorax and abdomen move outward on inspiration and inward on exhalation. During paradoxic breathing, the abdomen and chest move inward during inspiration due to maximal use of the accessory muscles of respiration. Dyspnea on standing and lying down are not paradoxic breathing. Pursed-lip breathing helps to prevent collapse of the airways and is also not paradoxic breathing.

The nurse is caring for a patient with stage III laryngeal cancer that is scheduled to undergo a total laryngectomy. To assist the patient to reestablish the ability to speak, which voice rehabilitation measures should be taken pre- and postoperatively? Select all that apply. 1. Preoperatively, a speech therapist should be consulted. 2. Transesophageal puncture can be considered at the time of laryngectomy. 3. Speaking tracheostomy tubes can be placed. 4. Radiation therapy can be planned. 5. Sign language can be used.

1,2 RATIONALE: A speech therapist should be asked to meet the patient to educate and discuss options for speech and voice restoration after surgery. Transesophageal puncture is the most common and practical voice restoration option and can be performed at the time of laryngectomy. Speaking tracheostomy tubes and radiation therapy are not useful for patients who do not have vocal cords. Sign language education would help in communication but not in restoring voice.

On auscultation of patient's lungs, which breath sounds would the nurse consider normal? Select all that apply. 1. Loud, high-pitched sounds resembling air blowing through a hollow pipe 2. Soft, low-pitched, gentle, rustling sounds heard over all lung areas except the major bronchi 3. Medium-pitched sounds heard anteriorly over the mainstem bronchi on either side of the sternum 4. The patient repeats the phrase "ninety-nine," and the words are easily understood and are clear and loud through the chest. 5 The patient whispers "one-two-three," and the almost inaudible voice is transmitted clearly and distinctly.

1,2,3 RATIONALE Bronchial, vesicular, and bronchovesicular sounds are normal breath sounds. Bronchial sounds are loud and high-pitched and resemble air blowing through a hollow pipe. Vesicular sounds are soft, low-pitched, gentle, rustling sounds heard over all lung areas except the major bronchi. Bronchovesicular sounds are medium-pitched sounds heard anteriorly over the mainstem bronchi on either side of the sternum. Bronchophony is an abnormal breath sound and is considered positive (abnormal) if the patient repeats the phrase "ninety-nine," and the words are easily understood and are clear and loud. Whispered pectoriloquy is also an abnormal breath sound and is considered positive (abnormal) when the patient whispers "one-two-three," and the almost inaudible voice is transmitted clearly and distinctly.

When teaching patients about preventing occupational lung diseases, what instructions should the nurse include? Select all that apply. 1 Use effective ventilation systems. 2 Ensure periodic inspection of workplaces. 3 Wear masks for high-risk occupations. 4 Avoid exposure to affected coworkers. 5 Use steam inhalation directly after working in a dusty environment.

1,2,3 RATIONALE Occupational lung diseases result from inhaled dust or chemicals and can be prevented by using effective ventilation systems, periodically inspecting workplaces to ensure safety, and wearing masks for high-risk occupations. It is not necessary to avoid coworkers who have been infected with lung diseases, and steam inhalation is not an effective deterrent.

A patient presents with acute exacerbation of asthma. The nurse expects that which strategies will be included in the treatment plan? Select all that apply. 1 Administration of 100% oxygen 2 Nebulization with short-acting β2-adrenergic agonists (SABAs) 3 Intravenous administration of corticosteroids 4 Administration of sedatives 5 Administration of antibiotics

1,2,3 RATIONALE: Acute exacerbation of asthma may be life-threatening and needs immediate intervention. Administering 100% oxygen helps to relieve hypoxia and improve tissue oxygenation. Nebulization with SABA helps to relax the airways and promote airflow. Corticosteroids are administered to blunt the hyperactive immune response. Sedatives should be avoided as they may depress the respiratory center and worsen dyspnea. Antibiotics are not administered unless there are symptoms of pneumonia.

The patient is receiving 3 L of oxygen (O2) via nasal cannula. Which action by the nurse is most appropriate? Select all that apply. 1 Assesses the bubble-through humidifier if humidity is used. 2 Assures that the patient is wearing the nasal cannula correctly 3 Adjusts humidification according to patient comfort. 4 Realizes that humidification is never needed.

1,2,3 RATIONALE: Assessing the bubble-through humidifier if humidity is used, assuring that the patient is wearing the nasal cannula correctly, and adjusting humidification according to patient comfort are correct because oxygen (O2) obtained from cylinders or wall systems is dry. Dry O2 has an irritating effect on mucous membranes and dries secretions. A common device used for humidification when the patient has a cannula or a mask is a bubble-through humidifier. It is important for the nurse to assess the bubble-through humidifier if humidity is used to make sure the humidification is on. This adds to the comfort of the patient. The nurse assesses the patient to make sure the nasal cannula is worn correctly for optimal effect. The cannula can become easily dislodged. Humidification is adjusted according to the patient's comfort level. When oxygen levels are 1-4 L the use of humidification may not be the preference of all patients. Realizing that humidification is never needed is incorrect because the use of humidification is patient preference.

A patient is admitted to the hospital with respiratory failure. Which drugs should the nurse anticipate to be advised for the patient? Select all that apply. 1 Diltiazem (Cardizem) for atrial fibrillation 2 Metoprolol (Lopressor) for atrial fibrillation 3 Azithromycin (Zithromax) to treat infections 4 Lorazepam (Ativan) for pulmonary congestion 5 Fentanyl (Sublimaze) for sedation and diuresis

1,2,3 RATIONALE: Diltiazem (Cardizem) and metoprolol (Lopressor) relieve pulmonary congestion in atrial fibrillation by decreasing heart rate and improving cardiac output. Azithromycin (Zithromax) is an antibiotic used to treat infections. Lorazepam (Ativan) is a benzodiazepine and does not improve pulmonary congestion. Fentanyl (Sublimaze) is a good opioid analgesic used to relieve pain, but it does not cause sedation or diuresis.

The nurse makes a nursing diagnosis of "impaired gas exchange" for a patient with pneumonia based upon which physical-assessment findings? Select all that apply. 1 SpO2 of 85% 2 PaO2 65 mm Hg 3 Absent breath sounds in right lung lobes 4 Presence of thick yellow mucus 5 Respiratory rate 24 breaths/minute

1,2,3 RATIONALE: Impaired gas exchange is evidenced by low oxygen saturation and elevated PaCO2 with absent breath sounds. Yellow mucus would indicate clearance of secretions. An increased respiratory rate does not imply impaired gas exchange.

The nurse is caring for the patient with a pulmonary embolism. Which factor(s) are associated with a pulmonary embolism (PE)? Select all that apply. 1. Pregnancy 2. Pelvic surgery 3. Immobility 4. Herbal therapy

1,2,3 RATIONALE: Pregnancy, pelvic surgery, and immobility are major risk factors associated with a pulmonary embolism. Risk factors among many for PE include immobility, pelvic surgery, pregnancy, oral contraceptives and hormone therapy. Herbal therapy is incorrect because herbal therapy is not associated with the development of a pulmonary embolism.

A patient has benzodiazepines (lorazepam [Ativan]) and opioids (morphine) ordered to decrease anxiety, agitation, and pain. What are the nursing roles in managing this patient? Select all that apply. 1 Monitoring cardiac function 2 Monitoring respiratory function 3 Ensuring regular sedation holiday 4 Using opioids aggressively as they reduce the length of hospital stay 5 Increasing the dose of morphine if respiration is depressed

1,2,3 RATIONALE: The nurse has to monitor patients closely for cardiopulmonary depression when giving benzodiazepines such as lorazepam (Ativan) and opioids (morphine). Patients receiving these agents are best managed by following an evidence-based protocol that includes a regular "sedation holiday" for ongoing assessment. Sedative and analgesic agents may have a prolonged effect in critically ill patients, delay weaning from mechanical ventilation, and contribute to increased length of stay. If respirations become depressed, then the dose of morphine has to be reduced.

A patient presents with epistaxis. Which interventions are appropriate to control the bleeding? Select all that apply. 1. Reassure the patient and keep him quiet. 2. Place the patient in a sitting position with his head tilted forward. 3. Apply direct pressure by pinching the entire soft lower portion of the nose. 4. Administer saline nasal sprays to relieve congestion. 5. Ask the patient to blow his nose to remove all the collected blood.

1,2,3 RATIONALE: To control epistaxis, the patient should be reassured and kept quiet. In epistaxis, approximately 90% of nosebleeds occur in the anterior portion of the nasal cavity and can be easily visualized. The patient should be made to sit, leaning slightly forward, with the head tilted forward. Direct pressure should be applied by pinching the entire soft lower portion of the nose against the nasal septum for 10 to 15 minutes. If bleeding does not stop within 15 to 20 minutes, consult the health care provider. Saline nasal sprays should not be used as these can dislodge the clot that is needed to stop the bleeding. Nose blowing will also remove the clot, which could lead to further bleeding.

The nurse is caring for the patient with pulmonary hypertension. Which treatment(s) are appropriate? Select all that apply. 1. Vasodilators 2. Diuretics 3. Anticoagulants 4. Thrombolytics

1,2,3 RATIONALE: Vasodilators, diuretics, and anticoagulants are correct because they are included in drug treatments for pulmonary hypertension. Vasodilators are especially important in the treatment of pulmonary hypertension as it will aid in reducing the right ventricular workload by dilating pulmonary vessels. Diuretics decrease plasma volume and thereby reduce myocardial workload. Anticoagulants also are used, especially if the case is severe, as it works to prevent in situ thrombus formation and venous thrombosis. Warfarin would be given to keep the international normalized ratio (INR) in the 2 to 3 range. Thrombolytic therapy is not an appropriate drug treatment and would be used if the condition causes right ventricle hypertrophy, resulting in cor pulmonale.

A nurse is educating a patient regarding hypercapnic respiratory failure. What information should the nurse include in her teaching? Select all that apply. 1. It is a ventilatory failure with insufficient carbon dioxide (CO2) removal. 2. There is evidence of the body's inability to compensate for acidemia. 3. Arterial CO2 (PaCO2) is greater than 45 mm Hg in combination with an arterial pH of less than 7.35. 4. The pH is at a level where a further increase may lead to severe acid-base imbalance. 5. There is inadequate oxygen transfer between the alveoli and the pulmonary capillaries.

1,2,3 RATIONALE: Hypercapnic respiratory failure is a ventilatory failure due to inadequate removal of CO2 from the body. The body may be unable to compensate for the acidemia caused by increased CO2 levels. As a result, the PaCO2 would be greater than 45 mm Hg in combination with acidemia (arterial pH less than 7.35). If the pH decreases further, it may lead to severe acid-base imbalance. Inadequate oxygen transfer between the alveoli and the pulmonary capillaries is a pathophysiological phenomenon during hypoxemic respiratory failure.

A patient with chronic obstructive pulmonary disease (COPD) is advised to use oxygen therapy at home. A nurse provides discharge instructions about how to prevent respiratory infection. What should be included in the teaching? Select all that apply. 1. Change the nasal cannula every 2 weeks. 2. Wash the nasal cannula twice a week. 3. Use mouthwash several times a day. 4. Retain secretions that are coughed out. 5. Clean the oxygen concentrator cabinet every week

1,2,3 RATIONALE: The strategies to reduce infection while using oxygen therapy at home include changing the cannula every 2 weeks, washing the cannula twice a week with liquid soap, and using a mouthwash several times a day. The nasal cannula may become contaminated with repeated use and should be changed every week. It should also be cleaned twice a week to remove the particulate material and moisture. Frequent use of mouthwash helps to keep the oral cavity clean and prevent infection. Removing the secretions that are coughed out reduces the risk of infection. The oxygen concentrator cabinet should be cleaned every day, not weekly

When caring for a patient with tuberculosis, what measures should the nurse instruct the patient to take to avoid the spread of infection? Select all that apply. 1. Cover the nose and mouth with a tissue while coughing and sneezing. 2. Throw used tissues in a paper bag and dispose with the trash. 3. Carefully wash hands after handling sputum and soiled tissues. 4. Wear a standard isolation mask when outside the patient's room. 5. Get out of bed and move freely about the hospital to keep up strength. 6. Drink plenty of water and maintain an erect posture.

1,2,3,4 RATIONALE: In order to prevent the spread of infection, patients with tuberculosis should be encouraged to cover the nose and mouth with tissues while coughing and sneezing, to throw used tissues in a paper bag and dispose of them with the trash, to carefully wash hands after handling sputum and soiled tissues, and to wear a standard isolation mask while moving out of their room. Increasing the frequency of prolonged visits to other parts of the hospital is not advisable, as it can increase the chances of infection spread; instead, such visits should be limited. Drinking plenty of water and maintaining erect posture have no effect on controlling infection.

A patient who has bronchiectasis asks the nurse, "What conditions would warrant a call to the clinic?" Which of these would the nurse include in the answer? Select all that apply. 1. Chest pain 2. Fever and chills 3. Blood clots in the sputum 4. Increased sputum production 5. Cough with no sputum production

1,2,3,4 RATIONALE: Teach the patient and caregiver to recognize significant clinical manifestations to report to the health care provider. These manifestations include increased sputum production, bloody sputum, increasing dyspnea, fever, chills, and chest pain. The patient will not have a nonproductive cough.

A patient is being admitted with a diagnosis of pertussis. The nurse knows which of the following? Select all that apply. 1. The cough may last from 6 to 8 weeks. 2. This is a highly contagious respiratory tract infection. 3. Treatment usually includes antibiotics with macrolides. 4. Cough suppressants and antihistamines should not be used. 5. Corticosteroids and bronchodilators are very useful in reducing symptoms. 6. Lifetime immunity results from one vaccination of diphtheria, pertussis, or tetanus (DPT).

1,2,3,4 RATIONALE: Pertussis is very contagious, and strict respiratory (droplet) precautions need to be instituted. Macrolide antibiotics, like erythromycin and azithromycin, usually are given to minimize symptoms and prevent the spread of the infection. The cough can last 6 to 8 weeks, but cough suppressants and antihistamines should not used because they are ineffective and can aggravate coughing episodes. Pertussis is caused by a gram-negative bacillus called Bordella pertussis. Corticosteroids and bronchodilators are not useful in reducing symptoms. Lifetime immunity does not result from one vaccination of DPT.

A patient is diagnosed with allergic rhinitis. Which instructions should the nurse include when teaching the patient about ways to avoid allergens? Select all that apply. 1. Wash your bedding in hot water weekly. 2. Remove pets from the interior of your home. 3. Ventilate closed rooms and open doors. 4.Wear a mask while vacuuming at home 5. Maintain high humidity levels at home

1,2,3,4 RATIONALE: Washing bedding in hot water weekly, removing pets from the interior of the home, ventilating closed rooms and opening doors, and wearing a mask while vacuuming are necessary to avoid allergens and will help to prevent allergic rhinitis. Maintaining high humidity levels at home will increase the chance of exposure to allergens and is not recommended.

When educating a patient about managing sinusitis without pharmacological interventions, which instructions should the nurse include? Select all that apply. 1. Use a steam inhaler. 2. Sleep with your head elevated. 3. Restrict fluid intake. 4. Avoid exposure to smoke. 5. Apply a cold compress on your cheeks.

1,2,4 RATIONALE: In the case of sinusitis, steam inhalation helps to promote drainage of secretions. Sleeping with the head elevated helps to drain the sinuses and reduce congestion. Smoke is an irritant and will worsen the symptoms of sinusitis. Adequate fluid intake will decrease the symptoms of sinusitis. Applying a cold compress on the cheeks is not recommended, as this worsens the symptoms. A hot compress on the cheeks will help.

The nurse is discharging a patient with chronic obstructive lung disease (COPD) who will be attending an outpatient pulmonary rehabilitation (PR) program. The nurse knows that components of this type of program generally include which of the following? Select all that apply. 1. Education 2. Exercise training 3; Smoking reduction 4. Nutrition counseling 5. On-site breathing treatments 6. Attendance by at least one family member or significant other

1,2,4 RATIONALE: Nutrition counseling definitely is needed in a PR program to teach the patient healthy nutrition. Education is also necessary, as well as exercise training. Smoking cessation and not reduction is needed. On-site breathing treatments are not offered. A family member or significant other is not a required component of these programs.

When teaching a patient with asthma about ways to reduce the severity of asthma and asthma attacks, which measures should be included? Select all that apply. 1 Avoid food irritants. 2 Avoid animals with fur. 3 Go out in the cold air for a walk. 4 Identify personal triggers. 5 Use nonsteroidal antiinflammatory drugs (NSAIDs)

1,2,4 RATIONALE: Patients with asthma should be taught to avoid food irritants and animals with fur. Identifying personal triggers can help to avoid them. Going out in the cold air and the use of NSAIDs are not recommended, because they can precipitate an asthma attack.

The nurse is teaching a class about smoking cessation. Select the respiratory-related symptoms associated with cigarette smoking. Select all that apply. 1. Chronic cough 2. Decreased sense of taste 3. Decreased sputum production 4. Paralysis of the cilia inside the lungs 5. Increased function of alveolar macrophages

1,2,4 RATIONALE: The effects of cigarette smoking on the respiratory system include development of a chronic cough, paralysis of the cilia, decreased sense of taste and smell, increased sputum production (not decreased), and decreased (not increased) function of alveolar macrophages.

While obtaining a health history for a patient with suspected tuberculosis, the nurse expects the patient to report which signs or symptoms? Select all that apply. 1. Night sweats 2. Fatigue 3. Chest tightness 4. Fever 5. Dyspnea

1,2,4 RATIONALE: The patient with tuberculosis presents with night sweats, fatigue, and fever. Chest tightness and dyspnea are not present in the patient with tuberculosis.

A nurse is caring for a patient with asthma. What instructions should the nurse provide to the patient to help prevent triggers of bronchoconstriction? Select all that apply. 1 Avoid wearing perfumes. 2 Avoid exposure to strong odors. 3 Do not drink cold water. 4 Avoid exposure to cold air. 5 Avoid people with certain infections.

1,2,4 RATIONALE: The different triggers for asthmatic attack include perfumes, strong odors, and cold air. They cause reflex bronchoconstriction, and the patient should avoid them to prevent asthma attacks. Cold water and bacteria do not trigger asthma.

The nurse is caring for the patient with chronic obstructive pulmonary disease (COPD). Which of the following are appropriate patient goals? Select all that apply. 1.Prevention of disease progression 2.Relief of symptoms 3.Healing of damaged lung tissue 4.Ability to perform activities of daily living (ADLs) 5. Improved quality of life

1,2,4,5 RATIONALE: The overall goals are that the patient with COPD will have (1) prevention of disease progression, (2) ability to perform ADLs and improved exercise tolerance, (3) relief from symptoms, (4) no complications related to COPD, (5) knowledge and ability to implement a long-term treatment regimen, and (6) overall improved quality of life. Lung tissue does not regenerate so "healing" is not a realistic goal. Patients need to know that symptoms can be managed, but COPD cannot be cured.

A patient in respiratory distress has thick and viscous secretions that are difficult to expel. Which nursing interventions can help to overcome this? Select all that apply. 1 Adequate fluid intake (2 to 3 L/day) 2 Oxygen given by aerosol mask 3 Acetylcysteine (Mucomyst) without a bronchodilator 4 Hydration through intravenous (IV) fluids 5 Aerosols of sterile normal saline, administered through a nebulizer

1,2,5 RATIONALE: Adequate fluid intake (2 to 3 L/day) keeps secretions thin and easier to remove. Oxygen may also be given by aerosol mask to thin secretions and facilitate their removal. Aerosols of sterile normal saline, administered by a nebulizer, may be used to liquefy secretions. Mucolytic agents such as nebulized acetylcysteine (Mucomyst) should be mixed with a bronchodilator, because they cause bronchoconstriction. If the patient is unable to take sufficient fluids orally, IV hydration is used with thorough assessment of patient's cardiac and renal status.

A patient reports a headache, nasal congestion, and fever for the past three days. A nurse examines the patient's nose and sinus areas thoroughly. What findings would suggest that the patient has sinusitis? Select all that apply. 1 Hyperemic and edematous mucosa and hyperemia 2 Tenderness over the sinuses 3 Clear nasal discharge 4 Nosebleed 5 Swollen turbinates

1,2,5 RATIONALE: Clinical findings that indicate sinusitis include hyperemic and edematous mucosa, tenderness over the involved sinuses, and enlarged turbinates. The inflammation results in increased blood supply to the affected area, which leads to hyperemic and edematous mucosa. Due to the inflammation, there may be tenderness over the involved frontal and/or maxillary sinuses. The turbinates may enlarge due to congestion. Clear nasal discharge is not a sign of sinusitis. Patients with sinusitis usually have a purulent nasal discharge. Nosebleed is not a manifestation of sinusitis as it is associated with epistaxis.

The nurse checks for which abnormal physical assessment findings consistent with cor pulmonale? Select all that apply. 1. Jugular vein distention 2. Pedal edema 3. Crackles 4. Wheezing 5. Hepatomegaly

1,2,5 RATIONALE: Cor pulmonale is a right-sided heart failure caused by resistance to right ventricular outflow caused by lung disease. With failure of the right ventricle, the blood emptying into the right atrium and ventricle would be slowed, leading to jugular vein distension, hepatomegaly, and pedal edema. Crackles would be associated with left-sided heart failure. Wheezing is not present in either type of heart failure.

The nurse is caring for a patient with a pleural effusion. What nursing interventions are appropriate when preparing this patient for a thoracentesis? Select all that apply. 1 Ensure that the informed consent was signed. 2 Position patient upright with elbows on an overbed table with feet supported. 3 Instruct the patient to cough vigorously during the procedure. 4 Instruct the patient not to eat anything 4 hours before the procedure. 5 Obtain chest x-ray after the procedure.

1,2,5 RATIONALE: For a thoracentesis, the nurse should ensure that the patient's informed consent was signed. The patient should be positioned upright with elbows on an overbed table and feet supported. This position gives appropriate access for needle insertion. A chest x-ray is obtained after the procedure to rule out a pneumothorax. The patient should be instructed not to talk or cough during the procedure, as it can cause injury by displacement of the needle. NPO status, or withholding food and drink, is not required for thoracentesis.

To promote airway clearance in a patient with pneumonia, what should the nurse instruct the patient to do? Select all that apply. 1. Maintain adequate fluid intake 2. Splint the chest when coughing 3. Maintain a 30-degree elevation 4. Maintain a semi-Fowler's position 5. Instruct patient to cough at end of exhalation

1,2,5 RATIONALE: Maintaining adequate fluid intake liquefies secretions, allowing easier expectoration. The nurse should instruct the patient to splint the chest while coughing. This will reduce discomfort and allow for a more effective cough. Coughing at the end of exhalation promotes a more effective cough. The patient should be positioned in an upright sitting position (high-Fowler's) with head slightly flexed.

Complications of pneumonia occur more frequently in older patients. The nurse knows that potential complications include which of the following? Select all that apply. 1 Sepsis 2 Pleurisy 3 Bronchitis 4 Encephalitis 5 Pleural effusion 6 Congestive heart disease

1,2,5 RATIONALE: Pleurisy, or inflammation of the pleura, and sepsis with bacteria in the blood stream can result from pneumonia, as well as pleural effusion (fluid in the pleural space). Bronchitis and encephalitis are not complications. Congestive heart failure is not known directly as a complication of pneumonia

A senior nurse is teaching nursing students to auscultate for adventitious sounds. One of the students auscultates a pleural friction rub. What are the common conditions in which a pleural friction rub is present? Select all that apply. 1. Pleurisy 2. Pneumonia 3. Asthma 4. Bronchitis 5. Pulmonary infarct

1,2,5 RATIONALE: The most common conditions presenting with pleural rub are pleurisy, pneumonia, and pulmonary infarct. Pleural rub is caused by the rubbing together of the two layers of the lungs. Asthma and bronchitis present with wheezes and do not manifest as pleural rub.

A patient comes to the emergency department (ED) complaining of recurrent epistaxis. The nurse knows that recurrent episodes of epistaxis can be caused by which of the following? Select all that apply. 1.Nasal tumors 2.Facial trauma 3. High humidity 4.Fever of unknown origin 5. Foreign bodies inserted into nares 6. Overuse of nasal decongestant sprays

1,2,5,6 RATIONALE: Nasal tumors, some anatomic malformations of the nose, foreign bodies inserted into the nose, and facial trauma can result in epistaxis. Using nasal decongestant sprays too frequently can also cause nose bleeds. Fever of unknown origin does not cause nasal bleeding. Low humidity, not high humidity, also can increase one's risk of epistaxis.

The nurse is completing a respiratory assessment on a recently admitted patient who uses oxygen at home. What information does the nurse need to obtain and document on the patient? Select all that apply. 1. Liter flow 2. Home safety practices 3. Method of payment for oxygen services 4. What agency supplies the oxygen services 5. Method and effectiveness of administration 6. Fraction of inspired oxygen concentration (FiO2)

1,2,5,6 RATIONALE: How the patient administers the oxygen and its effectiveness, home use, liter flow and FiO2 value need to be determined and documented. It is also important to find out what home safety practices are followed. What oxygen service the patient uses and the patient's method of payment are not necessary to determine in the admission assessment.

A patient is diagnosed with a pneumothorax, and the health care provider has inserted a chest tube with chest drainage system. The nurse who is monitoring the system finds that there is no bubbling. The nurse checked all the connections and found no problems. What is the most probable reason for the absence of bubbling? Select all that apply. 1. Suctioning is not present. 2. Suction pressure is very high. 3. Suction pressure is very low. 4. Pleural air leak is too large to be drained. 5. There is collection of blood in pleural space.

1,3,4 RATIONALE: It is important for a nurse to keep the water at the appropriate level in a suction chamber. If there is no bubbling seen, it indicates that there is no suction, or suction pressure is not enough, or the pleural leak is too large to be drained by the given suction pressure. The nurse should therefore revise the suction pressure. Collection of blood in pleural space would be evident by the type of drainage.

The patient with a tracheostomy is receiving humidified air. Which rationale(s) for the patient's use of humidification are correct? Select all that apply. 1 Prevents formation of mucous plugs 2 Prevents lower airway heat loss 3 Warms secretions 4 Moisturizes secretions

1,3,4 RATIONALE: Preventing formation of mucous plugs, warming secretions, and moisturizing secretions are correct because humidification is essential to prevent retention of tenacious secretions and formation of mucous plugs. Humidified air compensates for the loss of the upper airway to warm and moisturize secretions. Preventing lower airway heat loss is incorrect because humidified air compensates for the loss of the upper airway to warm and moisturize secretions, not lower airway. Additionally, it actively warms, and therefore does not prevent heat loss.

A nurse observes a patient using a dry powder inhaler device. The nurse should correct which patient actions? Select all that apply. 1. The patient breathes into the inhaler. 2. The patient performs deep and quick breathing. 3. The patient inhales more than 1 puff with each inspiration. 4. The patient shakes the medicine before using it. 5. The patient holds the breath for more than 10 seconds

1,3,4 RATIONALE: When using a dry powder inhaler, the patient should not breathe into the inhaler as this affects the dosing. Inhaling more than one puff with each inspiration may cause waste of the medication. The patient should not shake the medicine before using it as it can alter the dosing. Deep and quick breathing is the proper technique as it ensures that the medicine moves deep into the lungs. The patient should be encouraged to hold the breath beyond 10 seconds to help in penetration of the dry powder.

A nurse is monitoring a patient with respiratory failure. What are the possible complications of hypoxemia and hypoxia which the nurse should be aware of? Select all that apply. 1 Fall in blood pressure (BP) 2 Hyperdynamic state of liver 3 Hyperdynamic state of heart 4 Metabolic alkalosis and cell death 5 Permanent brain damage

1,3,5 RATIONALE: BP and cardiac output can fall, and vasoactive or inotropic agents are often less effective in an acidotic environment due to hypoxemia and hypoxia. A cardiovascular hyperdynamic state may occur due to catecholamine release, which is associated with the physiologic stress response. Permanent brain damage may occur if the hypoxia is severe and prolonged. Hypoxemia does not lead to a hyperdynamic state of the liver. If hypoxia or hypoxemia is severe, the cells shift from aerobic to anaerobic metabolism. The waste product of anaerobic metabolism is lactic acid. When the body does not have enough sodium bicarbonate to buffer the lactic acid, metabolic acidosis, tissue and cellular dysfunction, and cell death may occur.

A nurse is caring for a patient with pneumonia. The nurse is most likely to auscultate what breath sounds when assessing the patient's lungs? Select all that apply. 1 Egophony 2 Wheezes 3 Bronchophony 4 Stridor 5 Whispering pectoriloquy

1,3,5 RATIONALE: Pneumonia will present with egophony, bronchophony, and whispering pectoriloquy. Egophony is a test to assess breath sounds. It is positive when the patient is asked to pronounce "E" but instead says "A." In bronchophony, the patient is asked to repeat "ninety-nine" several times in a row. If the words are easily understood and are clear and loud, it indicates an abnormal finding. In pectoriloquy, the patient is asked to whisper "one-two-three." If the whisper is heard clearly and distinctly, it indicates an abnormal finding. Wheezes are heard in asthma when there is bronchoconstriction. Stridor is heard in laryngeal diseases due to the obstruction of the larynx or trachea.

A patient is diagnosed with pulmonary embolism. What nursing actions are appropriate for this patient? Select all that apply. 1. Administer oxygen therapy as prescribed. 2. Keep the patient on bed rest in a supine position. 3. Maintain an IV line for medications and fluid therapy. 4. Instruct the patient not to cough or perform deep breathing. 5. Monitor the patient for complications of anticoagulant therapy

1,3,5 RATIONALE: Pulmonary embolism requires prompt therapy for a good prognosis. Oxygen therapy should be administered as prescribed. An IV line should be maintained for medications and fluid therapy. Anticoagulants and fibrinolytics may have adverse effects, and the nurse should monitor the patient for side effects. The patient should be placed in a semi-Fowler's position to assist in breathing. The patient should be encouraged to cough and perform deep breathing exercises for better pulmonary function.

A patient with allergic rhinitis has been advised to use beclomethasone (Beconase) nasal spray to relieve his symptoms. What instructions should be given to this patient about the use of corticosteroid nasal spray? Select all that apply. 1. Use the spray twice daily, as ordered. 2. Use the spray as needed. 3. Discontinue use if nasal infection occurs. 4.Start two weeks before pollen season begins. 5. Clear nasal passages before using the spray.

1,3,5 RATIONALE: The corticosteroid nasal spray should be used on a regular basis to obtain maximum benefit. In this case, the patient should take nasal spray twice daily, as ordered. Use of the nasal spray should be discontinued if nasal infection occurs, as it may suppress the immune system and aggravate the infection. Nasal passages should be cleared before using the spray to ensure that the medicine reaches the target area. The spray should not be used on an as-needed basis, as overuse can worsen symptoms. The spray does not need to be started two weeks before the pollen season begins.

A patient is scheduled for a biopsy through bronchoscopy. What nursing interventions are appropriate for this patient? Select all that apply. 1 Ensure that an informed consent has been signed. 2 Assess blood urea nitrogen (BUN) and serum creatinine levels. 3 Administer sedative, if prescribed. 4 Instruct patient to remove all metal from the body. 5 Instruct patient to be on NPO status for 6-12 hours before the test.

1,3,5 RATIONALE: The nurse should ensure the consent form has been signed and administer sedatives if prescribed. For a bronchoscopy, the patient is required to be on NPO for 6-12 hours before the procedure. Renal function is not related to bronchoscopy, so assessment of BUN and serum creatinine levels is not needed. Removing metals from the body may not be necessary, as the procedure does not involve x-rays.

What should be the overall nursing goal for a patient with acute respiratory failure? Select all that apply. 1 No dyspnea 2 Normal bowel sounds 3 Maintenance of the airway 4 Normal blood urea nitrogen and creatinine 5 Effective cough and ability to clear secretions

1,3,5 RATIONALE: The overall goals for the patient with acute respiratory failure are absence of dyspnea, maintenance of the airway, effective cough and ability to clear secretions, normal ABG values and normal breath sounds. Absence of dyspnea indicates normal breathing and adequate oxygenation. Maintenance of the airway is important for normal oxygenation and tissue perfusion. Effective coughing and the ability to clear secretions help to keep the airway patent. Normal bowel sounds are not a goal for respiratory failure, as they are related to the function of the gastrointestinal tract. Blood urea nitrogen and serum creatinine are markers of renal function, not respiratory function.

The nurse is assessing a patient who was admitted from a nursing home with suspected tuberculosis (TB). Which of these are initial manifestations of tuberculosis? Select all that apply. 1. Anorexia 2. Dyspnea 3. Night sweats 4. Hemoptysis 5. Low-grade fever 6. Unexplained weight loss

1,3,5,6 RATIONALE: Active TB disease may manifest initially with constitutional symptoms such as fatigue, malaise, anorexia, unexplained weight loss, low-grade fevers, and night sweats. Dyspnea is a late symptom that may signify considerable pulmonary disease or a pleural effusion. Hemoptysis, which occurs in less than 10% of patients with TB, is also a late symptom.

he nurse is caring for a patient with blunt abdominal injuries caused by a motor vehicle accident. The nurse understands that pain in the patient can cause a ventilation/perfusion (V/Q) mismatch leading to respiratory failure. What pathophysiological processes may be involved in V/Q mismatch caused by pain? Select all that apply. 1 The pain interferes with chest and abdominal wall movement. 2 The pain decreases muscle tension. 3 The pain causes systemic vasodilation. 4 The pain activates the stress response. 5 The pain produces generalized muscle rigidity

1,4,5 RATIONALE: Pain interferes with chest and abdominal wall movement and compromises ventilation. It also activates the stress response. Pain results in generalized muscle rigidity. All these activities increase oxygen consumption and carbon dioxide production, increasing the ventilation demands. All these conditions result in limited airflow to the alveoli but have no effect on blood flow to the gas exchange units. The consequence of the imbalance is V/Q mismatch. Pain does not decrease muscle tension; it increases the muscle tension, causing muscle rigidity and an increase in oxygen demand. Pain also causes vasoconstriction rather than vasodilation.

A patient with asthma has been advised to undergo a pulmonary function test. What actions should the nurse take to prepare the patient for the procedure? Select all that apply. 1, Explain the procedure to the patient. 2. Withhold liquids for 2 hours before the test. 3. Instruct patient to wear comfortable shoes. 4. Avoid scheduling the test immediately after a meal. 5. Avoid administration of inhaled bronchodilator 6 hours before the procedure.

1,4,5 RATIONALE: When preparing the patient for a pulmonary function test , the nurse should explain the procedure to the patient. The test should not be scheduled immediately after meals. The nurse should not administer an inhaled bronchodilator to the patient 6 hours before the procedure. It is not necessary to withhold liquids before the test, and the patient does not need to wear any particular type of shoes.

A patient with chronic obstructive pulmonary disease (COPD) reports waking up frequently from sleep. The patient smokes cigarettes before going back to sleep. What instructions should the nurse give to the patient to promote sleep? Select all that apply. 1 Do not smoke between periods of sleep. 2 Ask the health care provider about using diazepam (Valium) to induce sleep. 3 Ask the health care provider about using β2-agonists to relieve insomnia. 4 Do not use oxygen therapy at night. 5 Use saline nasal sprays before sleep if experiencing postnasal drip.

1,5 RATIONALE: Cigarette smoking aggravates sleep disturbance so the nurse should discourage this habit. The postnasal drip associated with COPD causes disturbed sleep and can be managed by using saline nasal sprays before going to sleep and upon awakening. Use of diazepam should be discouraged as it may suppress respiration. Use of β2-agonists will aggravate insomnia. If the patient is prescribed oxygen therapy, it should be continued as it helps to relieve insomnia.

A patient presents pain and difficulty swallowing, swelling in the throat, difficulty in opening the mouth, and a history of tonsillectomy performed 20 days ago. What should be the order of the treatment?

1. Antibiotic therapy 2. Incision and drainage 3. Additional antibiotic therapy 4. Intravenous (IV) opioids and IV steroids 5. Percutaneous needle aspiration and drainage of the abscess 6. Hospitalization, if the patient is showing slow response to the drugs Pain and difficulty swallowing, swelling, and difficulty in opening the mouth are manifestations of peritonsillar abscess (PTA). Percutaneous needle aspiration and drainage of the abscess is performed, and antibiotic therapy is administered to control the infection. IV opioids are administered to relieve the pain and IV steroids are administered to reduce the swelling. If the infection does not respond to antibiotic therapy, hospitalization is advised. Incision and drainage are performed, along with additional antibiotic therapy to control the infection. To avoid recurrence, tonsillectomy may be performed.

Respiratory Assessment includes...

1. Rate, pattern, depth 2. Work of breathing (subjective and objective) 3. Behavior and LOC 4. Lung sounds (auscultation) 5. Skin and Mucosa 6. Vital Sign and Labs

If a client is to receive an entire 250-mL bag of saline over the next 4 hours and the drop rate of the IV tubing chamber is 15 drops/mL, at what drop rate per minute will the nurse set this IV? ____________ drops/min

16 Drops per minute = volume × drop factor ÷ total minutes 250 × 15 = 15.625 4 (hours) × 60 (minutes/hour)

The nurse understands that in normal lungs the volume of blood perfusing the lungs each minute (4 to 5 L) is approximately equal to the amount of gas that reaches the alveoli each minute (4 to 5 L). In a perfectly matched system, each portion of the lung would receive 1 mL of air (ventilation) for each 1 mL of blood flow (perfusion). What would be the V/Q in this case? Record your answer using a whole number.

1:1 --> V/Q = 1 A match of ventilation and perfusion would result in a V/Q ratio of 1:1, which is expressed as V/Q = 1. When the match is not 1:1, a V/Q mismatch occurs.

The nurse provides home care instructions to a patient who has emphysema. Which statement by the patient indicates correct understanding of the teaching? 1. "I'll get a dehumidifier to use at home." 2. "I'll report any change in the color of my sputum." 3. "When I feel short of breath, I'll increase my oxygen flow rate." 4. "The correct procedure for pursed-lip breathing is to inhale twice as long as I exhale."

2 RATIONALE: A change in the color of the patient's sputum from clear to yellow or green may indicate an infection and therefore should be reported to the patient's healthcare provider. Patients with emphysema should have humidified air in the home to prevent drying of the respiratory tract. The rate of oxygen delivery should not be increased over that recommended by the primary healthcare provider because of the risk of decreased respiratory drive. The correct procedure for pursed-lip breathing is to exhale twice as long as the breather inhales.

A patient diagnosed with acute respiratory distress syndrome is being mechanically ventilated with 12 cm of positive-end expiratory pressure (PEEP). Upon assessment, the nurse notes deterioration of vital signs and absent breath sounds in the right lung field. What is the most likely cause for this finding? 1. Deterioration of the disease 2. Pneumothorax 3. Decreased cardiac output 4. Obstructed endotracheal tube

2 RATIONALE: A complication of PEEP may be a pneumothorax as a result of overdistention of the alveoli. If deterioration of the disease were the cause, both lung sounds would be decreased equally. Decreased cardiac output would affect vital signs, but not breath sounds. An obstructed endotracheal tube would affect both lung fields.

The nurse is suctioning the patient's tracheostomy. Which occurrence is the first priority consideration by the nurse? 1. Heart rate increases from a baseline of 65 to 70 2. Heart rate decreases from a baseline of 65 to 44 3. SpO2 decreases from 100% to 92% 4. SpO2 decreases from 99% to 90%

2 RATIONALE: A heart rate decrease by 20 or more beats from baseline is an indication to immediately discontinue suctioning through the tracheostomy. A heart rate increase from baseline by 40 or more beats is an indication to immediately discontinue suctioning through the tracheostomy. The heart rate only increases by 5 beats and is not a reason, by itself, to discontinue suctioning. A decrease in SpO2 less than 90% is an indication to discontinue suctioning through the tracheostomy.

While teaching a 45-year-old patient with asthma about the appropriate use of a peak flow meter, the nurse instructs the patient to notify the health care provider immediately if which situation occurs? 1 Wheezing is improved moderately with the use of a bronchodilator. 2 Less than 50% of the patient's personal best is achieved. 3 The short-acting bronchodilator is being used every three to four days. 4 Peak flow measurements remain unchanged after exercise.

2 RATIONALE: Achieving less than 50% of the patient's personal best on the peak flow meter indicates a medical emergency related to poor gas exchange and air flow. The patient should notify the health care provider immediately. Wheezing should be improved with a bronchodilator. Short acting bronchodilators used every one to two days indicate the need for additional asthma treatment. Peak flow measurements should not decrease following exercise if asthma is well-controlled.

When can airborne infection isolation for a patient with pulmonary tuberculosis (TB) be discontinued? 1. Once isoniazid drug therapy has been initiated 2. After three consecutive acid-fast bacillus (AFB) smears are negative 3. After effective instruction on the use of a high-efficiency particulate air (HEPA) mask 4. When two consecutive negative x-ray results are confirmed

2 RATIONALE: Airborne infection isolation is indicated for the patient with pulmonary or laryngeal TB until the patient is noninfectious (defined as effective drug therapy, clinical improvement, and three negative AFB smears). Therapy must be deemed effective. Teaching the patient to properly use the HEPA mask isn't criteria to terminate isolation. Chest x-rays do not meet criteria to terminate isolation.

After assisting the health care provider with a bedside thoracentesis, the nurse should continue to assess the patient for signs and symptoms of what? 1 Bronchospasm 2 Pneumothorax 3 Pulmonary edema 4 Respiratory acidosis

2 RATIONALE: Because thoracentesis involves the introduction of a catheter into the pleural space, there is a risk of pneumothorax. Thoracentesis does not carry a significant potential for causing bronchospasm, pulmonary edema, or respiratory acidosis.

hich strategy by the nurse would be most helpful in treating a patient with pneumonia and asthma who is experiencing chills? 1 Apply heat to the patient's posterior neck 2 Provide a blanket to the patient 3 Encourage the patient to bathe in hot water 4 Administer acetaminophen (Tylenol) 650 mg as prescribed

2 RATIONALE: Chills often occur in cycles and last for 10 to 30 minutes at a time. They usually signal the onset of an increase in temperature. For this reason, the nurse should provide a light blanket for comfort, but avoid overheating the patient with heat on the neck or bathing in hot water. Acetaminophen would be used if the patient became febrile following the chills.

A patient is complaining of a history of sinus infections off and on since childhood. The nurse knows that with chronic sinusitis: 1 Severe pain occurs 2 Diagnosis is difficult 3 Nasal drainage is purulent 4 Fever starts before the infection

2 RATIONALE: Chronic sinusitis is hard to diagnose because symptoms are often nonspecific. Severe pain and purulent nasal drainage do not occur. Patients are rarely febrile.

The nurse is assessing a patient who may have manifestations of chronic obstructive pulmonary disease (COPD). Which of these is a clinical manifestation of early COPD? 1 Dyspnea at rest 2 A chronic, intermittent cough 3 The presence of chest breathing 4 Production of copious amounts of sputum

2 RATIONALE: Clinical manifestations of COPD typically develop slowly. A chronic intermittent cough, which is often the first symptom to develop, later may be present every day as the disease progresses. Typically dyspnea is progressive, usually occurs with exertion, and is present every day. Dyspnea at rest and chest breathing are manifestations of late COPD. The cough may be unproductive of mucus.

A patient has a known allergy to dogs and is prescribed cromolyn spray (NasalCrom). What should the nurse instruct the patient regarding the use of the spray? 1 Use the spray daily. 2 Use the spray 15 minutes before contact with a dog. 3 Use the spray 15 minutes after contact with a dog. 4 Use the spray only when symptoms of rhinitis show up.

2 RATIONALE: Cromolyn is a mast cell stabilizer which inhibits the secretion of histamine and other inflammatory mediators. It is used to prevent symptoms of rhinitis and should be used 10 to 15 minutes before likely exposure to a known allergen. It is not recommended for daily use. Using the spray after allergen exposure would not be beneficial. Using the spray after the symptoms appear would also not help the patient.

The nurse is monitoring a patient who is having a thoracentesis for recurrent pleural effusion. Which of these assessment findings would be of most concern? 1. Removal of 1000 mL of pleural fluid 2. Restlessness and sudden complaint of dyspnea 3. SpO2 reading of 96% while on 2 L/minute of oxygen 4. Patient complaint of pressure at the needle insertion site

2 RATIONALE: During and after a thoracentesis, monitor the patient's vital signs and pulse oximetry and observe the patient for any manifestations of respiratory distress, which may indicate a possible complication, such as pneumothorax or pulmonary edema. It is not unusual to remove up to 1000 to 1200 mL of pleural fluid at one time. The SpO2 reading of 96% and patient complaint of pressure at the needle insertion site are not abnormal findings.

When teaching a patient about chronic obstructive pulmonary disease (COPD) rehabilitation, what strategy should the nurse teach the patient as essential to perform for energy conservation? 1. Complete inactivity 2. Exercise training 3. Reduced water intake 4. Reduced food intake

2 RATIONALE: Exercise training leads to energy conservation, which is an important component in COPD rehabilitation. Complete inactivity may alleviate symptoms acutely but is not helpful in the long term, because the patient needs to learn effective ways to improve muscle function. It is also important to reduce dyspnea by exercise training. Reduced water and food intake is not advisable; instead, increased water and food intake is essential to maintain energy and to loosen the secretions.

The nurse is working in the hospital during the flu season and knows that: 1 The onset is insidious 2 Generalized myalgia occurs 3 Vomiting and diarrhea result 4 Nuchal rigidity starts before headache

2 RATIONALE: Generalized myalgia or body aches are common with flu symptoms. The onset of flu is abrupt and not insidious. Anorexia occurs, but not vomiting and diarrhea. Nuchal rigidity is impaired neck flexion resulting from muscle spasms of the neck and is related to meningeal irritation.

The nurse is assessing a patient with a diagnosis of restrictive lung disease caused by an extrapulmonary condition. Which finding indicates the patient may be suffering from a chest wall problem? 1 Brain tumor 2 Obesity hypoventilation 3 Paralysis of lower extremities 4 Recent changes in level of consciousness (LOC)

2 RATIONALE: Pickwickian syndrome is an obesity-ventilation condition of the chest wall that is a cause of extrapulmonary restrictive lung disease. A brain tumor and recent changes in LOC are related to a CNS cause. Paralysis of lower extremities is related to a neuromuscular system problem, and the patient may be suffering from Guillian-Barré Syndrome.

The patient had abdominal surgery yesterday. Today the lung sounds in the lower lobes have decreased. The nurse knows this could be because of what occurring? 1. Pain 2. Atelectasis 3. Pneumonia 4. Pleural effusion

2 RATIONALE: Postoperatively there is an increased risk for atelectasis from anesthesia as well as restricted breathing from pain. Without deep breathing to stretch the alveoli, surfactant secretion to hold the alveoli open is not promoted. Pneumonia will occur later after surgery. Pleural effusion occurs because of blockage of lymphatic drainage or an imbalance between intravascular and oncotic fluid pressures, which is not expected in this case.

What is the most appropriate nursing intervention for a patient learning about cetirizine (Zyrtec)? 1. Warn the patient to avoid driving or making important decisions 2. Prepare the patient to expect little to no side effects 3. Instruct the patient that urinary hesitancy is a common side effect 4. Explain to the patient that alcohol intake has an additive depressive effect

2 RATIONALE: Preparing the patient to expect little to no side effects is correct because the appropriate nursing intervention for a patient taking a second-generation antihistamine, such as cetirizine, includes informing the patient of the paucity of side effects when compared with first-generation antihistamines. Avoiding drinking or making important decisions, expecting urinary hesitancy, or explaining that alcohol has an additive depressive effect are teaching interventions applying to first generation antihistamines, not second-generation.

Which task can the registered nurse (RN) delegate to an unlicensed assistive personnel (UAP) in the care of a stable patient who has a tracheostomy? 1 Assessing the need for suctioning 2 Suctioning the patient's oropharynx 3 Assessing the patient's swallowing ability 4 Maintaining appropriate cuff inflation pressure

2 RATIONALE: Providing the individual has been trained in correct technique, UAP may suction the patient's oropharynx. Assessing the need for suctioning should be performed by an RN or licensed practical nurse, whereas swallowing assessment and the maintenance of cuff inflation pressure should be performed solely by the RN.

A patient presents to the emergency department with sudden-onset wheezing and coughing with progressive respiratory distress. What condition or diagnosis does the nurse recognize? 1. Acute bronchitis 2. An asthma attack 3. Pulmonary edema 4. Congestive heart failure

2 RATIONALE: Sudden onset of coughing and wheezing are the initial and most obvious symptoms of an acute asthma attack. An asthma attack may begin mild but progress to respiratory distress and arrest if it goes untreated. Acute bronchitis is irritation and inflammation of the mucous membrane lining of the respiratory tract, usually caused by an infectious agent. Pulmonary edema is fluid accumulation in the lungs due to heart failure or lung injury. The fluid collection impairs gas exchange and may result in respiratory failure. Congestive heart failure, or heart failure, is a condition in which the heart cannot pump effectively. Fluid may accumulate in the lungs. Edema may develop in the lower extremities, and shortness of breath may also occur with increasing frequency and severity.

The family of a patient with newly diagnosed tuberculosis is tested for infection with Mycobacterium tuberculosis. The patient's wife, who has a history of alcoholism, has had two negative Mantoux (PPD) tests. Both of their children have positive Mantoux results. The nurse recognizes that a course of preventive treatment with isoniazid will be required for which family member(s)? 1. The spouse only 2. Both children only 3. The spouse and the children 4. Neither the spouse nor the children

2 RATIONALE: The children would be given isoniazid (INH) because of exposure and conversion from a negative to a positive Mantoux result. The wife would not be treated because she has not converted to positive and may be susceptible to INH hepatitis, a susceptibility that increases with age. She may also be susceptible to hepatotoxicity because of her age and history of alcoholism. The children definitely require preventive treatment with INH.

The nurse caring for a patient with respiratory failure finds that the patient has hypoxemia due to ventilation-perfusion (V/Q) mismatch. What is the primary intervention that the nurse should implement in this case? 1 Administer antibiotics as ordered. 2 Give oxygen therapy as ordered. 3 Administer bronchodilators as ordered. 4 Give antisecretary agents as ordered

2 RATIONALE: The first intervention for any hypoxemia patient is to administer oxygen. Oxygen therapy can reverse hypoxemia caused by V/Q mismatch, because not all gas exchange units are affected. Antibiotics improve V/Q mismatch only in infective conditions of the lung. Bronchodilators improve V/Q mismatch only if bronchospasm is the underlying cause. Anti-secretary agents improve V/Q mismatch only if increased bronchial secretions are the underlying cause.

Before discharge, the nurse discusses nutrition with the patient with emphysema and pneumonia. The nurse instructs the patient to: 1. Eat three large meals per day 2. Rest for 30 minutes before eating 3. Increase intake of hot foods 4. Exercise before meals to increase appetite

2 RATIONALE: The patient with emphysema should conserve energy to eat and should rest for at least 30 minutes before eating to increase energy needed to eat. The patient should consume five to six small meals per day, avoid hot foods, and exercise after eating to conserve energy.

CHART/EXHIBITION 1 A patient is concerned that he or she may have asthma. The nurse assesses the severity of the symptoms. Based on this information, how would the nurse classify the patient's asthma? Refer to chart. 1 Intermittent severity 2 Mild persistent 3 Moderate persistent 4 Severe persistent

2 RATIONALE: This patient's asthma would be classified as "mild persistent" severity. The patient reports having symptoms more than twice a week, but not daily, wakes up with symptoms at night three or four times a month, uses a short-acting beta agonist more than twice a week, but not daily, and reports a minor limitation in normal activity. The lung function assessment shows FEV1 over 80% and normal FEV1/FVC.

A patient has been prescribed topical decongestants for sinusitis. What information should the nurse include when teaching the patient about this medication? 1 Use these medications for at least 10 days. 2 Use these medications for a maximum of 3 days. 3 Use these medications for at least 1 month. 4 Use these medications for at least a week

2 RATIONALE: Topical decongestants should be used for no more than 3 days to avoid rebound congestion by vasodilatation. If used longer than 3 days, the patient's symptoms may be made worse.

A nurse finds that the tracheostomy tube of a patient is dislodged. The nurse is unable to replace the tube and calls for assistance. Until assistance arrives, the nurse should perform what intervention? 1. Cover the stoma with sterile gauze. 2. Put the patient in a semi-Fowler position to relieve dyspnea. 3. Ventilate the patient with a bag-mask. 4. Remove the retention sutures.

2 RATIONALE: When the dislodged tracheostomy tube cannot be replaced single-handedly, the nurse should put the patient in the semi-Fowler position. This position helps to prevent dyspnea until assistance arrives. The tracheostomy stoma should be covered only if a bag-mask ventilation needs to be performed, which is done only in case of respiratory arrest. The retention sutures help to reinsert the tube and should not be removed.

An asthmatic patient is in acute respiratory distress. The nurse auscultates the lungs and notes cessation of inspiratory wheezing. How does the nurse interpret this finding? 1 The patient has developed a pneumothorax 2 There is worsening airway inflammation and bronchoconstriction 3 Airflow has now improved through the bronchioles 4 A mucous plug has developed within a main stem bronchus

2 RATIONALE: When the patient in respiratory distress has inspiratory wheezing that then ceases, it is an indication of airway obstruction and requires emergency action to restore the airway.. A pneumothorax would be evidenced by absent breath sounds. Absence of wheezing does not correlate with improved airflow if the patient is also in current respiratory distress. A mucus plug would result in crackles in the lungs

A patient whose tracheostomy was inserted 30 minutes ago is recovering in the postanesthesia recovery unit when the patient coughs and expels the tracheostomy tube. How should the nurse respond? 1. Suction the tracheostomy opening 2. Maintain the airway with a sterile hemostat 3. Use an Ambu bag and mask to ventilate the patient 4. Insert the tracheostomy tube obturator into the stoma

2 RATIONALE: As long as the patient is not in acute respiratory distress after dislodging the tracheostomy tube, the nurse should use a sterile hemostat to maintain an open airway until a sterile tracheostomy tube can be reinserted into the tracheal opening. If the patient is in respiratory distress, the nurse will use an Ambu bag and mask to ventilate the patient temporarily. The tracheostomy is an open surgical wound that has not had time to mature into a stoma.

The nurse determines that a patient is experiencing the most common adverse effect of albuterol (ProAir HFA) after noting which sign? 1. Headache 2.Tachycardia 3. Diarrhea 4. Oral candidiasis

2 RATIONALE: Tachycardia is a common adverse effect of the use of inhaled beta 2 adrenergic agonists because of its stimulant effect. Headache, diarrhea, and oral candidiasis are not associated adverse effects of albuterol. TEST-TAKING TIP: Choose the best answer for questions asking for a single answer. More than one answer may be correct, but one answer may contain more information or more important information than another answer.

The health care provider has prescribed intravenous (IV) vancomycin for a patient with pneumonia. Which action should the nurse perform first? 1. Obtain a full set of vital signs 2. Obtain sputum cultures for sensitivity 3. Draw a blood specimen to evaluate the white blood cell count 4. Administer the antibiotic over at least 60 minutes

2 RATIONALE: The nurse should ensure that the sputum for culture and sensitivity has been sent to the laboratory before administering the antibiotic. It is important that the organisms be correctly identified (in the culture) before their numbers are affected by the antibiotic; the test also will determine whether the proper antibiotic has been prescribed (sensitivity testing). Vital signs and white blood cell count measurement can be assessed following the obtainment of sputum cultures. Timing of antibiotic administration should be based upon the institution's policy.

A patient sustains an injury to the nose. Surgery is scheduled to repair the deviated nasal septum. The nurse expects to assess what patient symptoms? Select all that apply. 1 Nasal swelling 2 Nasal congestion 3 Nosebleeds 4 Nasal breathing difficulty 5 Redness on the nose

2,3,4 RATIONALE: Common signs that are suggestive of a nasal septal deviation are nasal congestion, frequent nosebleeds, and difficulty in breathing through the nose. As a response to injury, blood circulation increases, causing congestion and frequent nosebleeds. A deviated septum may obstruct the air pathway, leading to difficulty in breathing through the nose. Nasal swelling and redness do not indicate deviated nasal septum but may indicate other injuries to the nose.

The nurse is caring for a patient with respiratory failure. What information should the nurse include when explaining the disease condition to the family members? Select all that apply. 1 Respiratory failure is inevitable after cardiac failure. 2 Respiratory failure is categorized as hypoxemic or hypercapnic. 3 Respiratory failure is insufficient oxygen transfer into the blood. 4 Respiratory failure is inadequate carbon dioxide removal from the lungs. 5 Respiratory failure is a disease that presents with various respiratory symptoms.

2,3,4 RATIONALE: Respiratory failure is classified as hypoxemic or hypercapnic. Respiratory failure results when gas exchanging functions are inadequate, i.e., insufficient oxygen is transferred to the blood or inadequate carbon dioxide is removed from the lungs. Although respiratory failure may be secondary to cardiac failure, not all cases of respiratory failure are secondary to cardiac failure. Respiratory failure is not a disease but a symptom of an underlying pathologic condition affecting lung function, oxygen delivery, cardiac output, or the baseline metabolic state.

A nurse is monitoring a patient with stiff lungs due to acute respiratory distress syndrome (ARDS). What compensatory clinical events may the nurse expect in this patient? Select all that apply. 1 Hypoventilation 2 Respiratory alkalosis 3 Increase in cardiac output 4 Increase in respiratory rate and a decrease in tidal volume 5 Accumulation of carbon dioxide

2,3,4 RATIONALE: The breathing pattern of stiff lung increases carbon dioxide removal, producing respiratory alkalosis. Cardiac output increases in response to hypoxemia, a compensatory effort to increase pulmonary blood flow. Hypoxemia and the stimulation of juxtacapillary receptors in the stiff lung parenchyma initially cause an increase in respiratory rate and a decrease in tidal volume. Hypoventilation occurs after compensatory failure. Carbon dioxide does not accumulate; instead, there may be increased removal of carbon dioxide.

A patient is recovering from acute respiratory distress syndrome (ARDS). What goals should a nurse have for this patient? Select all that apply. 1. Arterial oxygen (PaO2) above normal limits 2. Oxygen saturation (SaO2) greater than 90% 3. Patent airways 4. PaO2 within normal limits 5. Clear lungs

2,3,4,5 RATIONALE: The goals for a patient recovering from ARDS are SaO2 greater than 90%, patent airways, PaO2 within normal limits for age or baseline values on room air, and clear lungs on auscultation. SaO2 above 90% and PaO2 within normal limits indicate satisfactory arterial oxygenation. Airways should be kept patent and clear off any secretions. Clear lungs indicate that there are no secretions obstructing the airway, and the lungs are functional. PaO2 above normal levels means that the patient is hyperventilating. STUDY TIP: Regular exercise, even if only a 10-minute brisk walk each day, aids in reducing stress. Although you may have been able to enjoy regular sessions at the health club or at an exercise class several times a week, you now may have to cut down on that time without giving up a set schedule for an exercise routine. Using an exercise bicycle that has a book rack on it at home, the YMCA, or a health club can help you accomplish two goals at once. You can exercise while beginning a reading assignment or while studying notes for an exam. Listening to lecture recordings while doing floor exercises is another option. At least a couple of times a week, however, the exercise routine should be done without the mental connection to school; time for the mind to unwind is necessary, too.

The laryngoscopy report of a patient indicates the presence of a cancerous mass over the vocal cords. The patient is to be treated with radiation therapy. How can a nurse help the patient cope with the side effects of radiation therapy? Select all that apply. 1 Encourage alcohol consumption. 2 Encourage increased fluid intake. 3 Administer pilocarpine hydrochloride. 4 Discourage use of artificial saliva. 5 Encourage intake of soft food.

2,3,5 RATIONALE: Dry mouth is the most common side effect of radiation therapy. The nurse should instruct the patient to increase fluid intake, as hydration will help the patient to relieve symptoms. Due to radiation therapy, the patient's saliva decreases in volume and becomes thick. Pilocarpine hydrochloride is helpful in increasing saliva production. Soft and bland food causes less irritation to the oral mucosa and should be encouraged if the patient has oral mucositis. Alcohol cessation should be encouraged following the diagnosis of cancer to avoid complications. Artificial saliva also helps in keeping the mouth hydrated in xerostomia.

A patient with an acute attack of asthma is in a state of panic. Which nursing measures help to relieve the panic? Select all that apply. 1. Use sedation. 2. Encourage pursed-lip breathing. 3. Utilize the "talking down" technique. 4. Utilize a "walking down" technique. 5. Be calm, quiet, and reassuring

2,3,5 RATIONALE: Pursed-lip breathing keeps the airways open, slows down the respiratory rate, and encourages deep breathing. "Talking down" is a technique that helps to calm the patient. A calm, quiet, and reassuring nurse helps to pacify the patient. Use of sedatives should not be encouraged as they may cause respiratory depression. There is no technique called "walking down."

A patient is in respiratory distress. Which positions of the patient indicate severity of the distress? Select all that apply. 1 Ability of the patient to breathe while in a lying position indicates moderate distress. 2 Ability of the patient to breathe only in sitting position indicates mild distress. 3 Ability of the patient to breathe only in sitting position indicates severe distress. 4 Ability of the patient to breathe only in a lying position indicates severe distress. 5 Ability of the patient to breathe while lying down but preferring to sit indicates moderate distress.

2,3,5 RATIONALE: The patient's position is an indication of the effort associated with the work of breathing. If the patient is unable to breathe unless he is sitting upright, severe distress is present. Moderate distress is exhibited if the patient is able to lie down but prefers to sit. If the patient is able to lie down, the distress is mild.

A patient is seen at the clinic with a sore throat, and states "I can hardly swallow." The examination of the throat reveals a red and edematous pharynx with patchy exudates. The patient has a temperature of 101º F (38.3º C). The nurse anticipates that the collaborative management will include which of the following? Select all that apply. 1.Antiviral agents to treat influenza. 2.Treatment with antibiotics starting as soon as possible. 3. A throat culture or rapid strep antigen test. 4.Treatment with nystatin (Mycostatin) liquid. 5.Supportive care, including cool, bland liquids.

2,3,5 RATIONALE: These signs and symptoms are indicative of bacterial pharyngitis . A throat culture or rapid strep antigen test will be performed first. Penicillin is the drug of choice for bacterial pharyngitis. Other antibiotics (amoxicillin, azithromycin [Zithromax], cephalosporins) also may be used. Supportive care includes drinking cool, bland liquids and avoiding citrus juices, which may be irritating. The use of antiviral agents or nystatin (an antifungal drug) is not appropriate for bacterial pharyngitis.

A nurse observes a patient with asthma using a metered-dose inhaler. The nurse should correct which patient actions? Select all that apply. 1 The patient inspires slowly. 2 The patient holds the breath for 5 seconds. 3 The patient shakes the device before using it. 4 The patient presses the meter dose inhaler gently. 5 The patient coordinates the use of the inhaler with inspiration.

2,4 RATIONALE: While using a metered dose inhaler, the breath should be held for at least 10 seconds for better absorption of the drug. The metered dose inhaler should be pressed with adequate strength to deliver the required dose. Inspiring slowly, shaking the device before using it, and coordinating use of the inhaler with inspiration are the right techniques for using a metered dose inhaler

A patient with chronic obstructive pulmonary disease (COPD) is receiving oxygen therapy through a mask. Which nursing actions should the nurse perform to ensure proper care of the patient? Select all that apply. 1 Take a chest radiograph. 2 Choose the optimal oxygen device. 3 Assess the need to change intravenous fluids. 4 Assess the need to adjust the oxygen flow rate. 5 Monitor for signs of adverse effects of oxygen therapy.

2,4,5 RATIONALE: Choosing the optimal oxygen device, assessing the need for adjustment in oxygen flow rate, and monitoring signs of adverse effects of oxygen therapy are all duties that the nurse should perform when evaluating the response of the patient to oxygen therapy. Taking a chest radiograph and assessing the need to change intravenous fluids are not relevant to this situation.

A patient is being discharged from the emergency department after being treated for epistaxis. In teaching the family first aid measures in the event the epistaxis would recur, what measures should the nurse suggest? Select all that apply. 1 Tilt patient's head backwards 2 Apply ice compresses to the nose 3 Tilt head forward while lying down 4 Pinch the entire soft lower portion of the nose 5 Partially insert a small gauze pad into the bleeding nostril

2,4,5 RATIONALE: First aid measures to control epistaxis include tilting the patient's head backwards and placing the patient in a sitting position, leaning forward. Pinching the soft lower portion of the nose or inserting a small gauze pad into the bleeding nostril should stop the bleeding within 15 minutes. Tilting the head back or forward does not stop the bleeding, but rather allows the blood to enter the nasopharynx, which could result in aspiration or nausea/vomiting from swallowing blood. Lying down also will not decrease the bleeding.

A patient has been admitted to the emergency department after complaining of having difficulty breathing for several days. To detect late manifestations of inadequate oxygenation, the nurse would examine the patient for which of these? Select all that apply. 1 Mild hypertension 2 Use of accessory muscles 3 Apprehension and restlessness 4 Cyanosis and cool, clammy skin 5 Pausing for breath between sentences and words

2,4,5 RATIONALE: Late manifestations of inadequate oxygenation include use of accessory muscles, cyanosis and cool, clammy skin, and pausing for breath between sentences and words. Mild hypertension, apprehension, and restlessness are early manifestations of inadequate oxygenation

A patient with pneumonia is being treated at home and has reported fatigue to the nurse. What instructions should the nurse include when teaching the patient about care and recovery at home? Select all that apply. 1 Resume work to build strength. 2 Get adequate rest. 3 Restrict fluid intake. 4 Avoid alcohol and smoking. 5 Take every dose of the prescribed antibiotic.

2,4,5 RATIONALE: To ensure complete recovery after pneumonia, the patient should be advised to rest, avoid alcohol and smoking, and take every dose of the prescribed antibiotic. The patient should not resume work if feeling fatigued and should be encouraged to drink plenty of fluids during the recovery period.

A patient has been prescribed inhalational steroids. What information should the nurse include when explaining to the patient about inhalational steroids? Select all that apply. 1 They are used for acute respiratory infections. 2 They are not used for acute respiratory failure. 3 Prolonged use can cause adrenal insufficiency. 4 They exacerbate hypokalemia caused by diuretics. 5 They require 4 to 5 days for optimum therapeutic effects.

2,5 RATIONALE: Inhaled steroids are not used for acute respiratory failure because of delayed onset of action. Inhaled steroids require 4 to 5 days for optimum therapeutic effects. These are not used for respiratory infections, as they do not act on pathogenic organisms. Inhaled steroids do not cause adrenal insufficiency; prolonged intravenous (IV) use, however, can cause adrenal insufficiency. IV use of corticosteroids exacerbates hypokalemia caused by diuretics.

A patient with bacterial pneumonia has rhonchi and thick sputum. What is the nurse's most appropriate action to promote airway clearance? A. Assist the patient to splint the chest when coughing. B. Teach the patient about the need for fluid restrictions. C. Encourage the patient to wear the nasal oxygen cannula. D. Instruct the patient on the pursed lip breathing technique.

A. Assist the patient to splint the chest when coughing.

T-piece FiO2 delivered:

24-100% FiO2 with flow rates of at least 10 mL/min; provides high humidity

Venturi Mask FiO2 delievered:

24-50% FiO2 with usually 4-10 mL/min provides high humidity

A pediatric patient is diagnosed with pertussis. The cough is worse during the night and is accompanied by a loud, long, rasping, indrawn breath. Even though the child has completed a week's course of antibiotics and has received cough suppressants, the cough has worsened. The nurse determines that what is the likely reason the cough has worsened? 1. The child may have lung cancer. 2. It could be a side effect of the antibiotics. 3. It may be due to administration of cough suppressant. 4. It could be a fungal infection.

3

A nurse preparing educational information about lung cancer notes that the primary risk factor related to the development of lung cancer is 1 Genetics 2 Chewing tobacco 3 Cigarette smoking 4 Occupational exposure

3 As many as 90% of patients with lung cancer have a history of cigarette smoking. Cigarette smoke contains several organ-specific carcinogens. Genetics and occupational exposure are risks but not as high as cigarette smoking. Chewing tobacco is not associated with lung cancer; it is related to oral cancer. Text Reference - p. 535

A patient with a spontaneous pneumothorax has a chest tube that is attached to a closed drainage system. The drainage unit is not attached to suction. The water level in the water-seal chamber is fluctuating. What action should the nurse take? 1 Notifying the primary health care provider immediately 2 Decreasing the amount of water in the water-seal chamber 3 Continuing to monitor and document the respiratory status 4 Clamping the chest tube as close as possible to the insertion site

3 In a closed drainage system (not in a suction system) the fluid in the water-seal chamber rises when the patient inhales and falls when the patient exhales; this is a normal finding. The absence of fluctuations may indicate an obstruction in the system. The nurse must continually check the function of the closed drainage system and assess respiratory status at least every 4 hours. There is no need to notify the primary healthcare provider or decrease the amount of water in the water-seal chamber, because the chest tube system is functioning normally. The chest tube should not be clamped unless the nurse is directed to do so by the primary healthcare provider, because as doing so could cause a tension pneumothorax. Text Reference - p. 545

The patient is hospitalized with pneumonia. Which diagnostic test should be used to measure the efficiency of gas transfer in the lung and tissue oxygenation? 1 Thoracentesis 2 Bronchoscopy 3 Arterial blood gases 4 Pulmonary function tests

3 Arterial blood gases are used to assess the efficiency of gas transfer in the lung and tissue oxygenation, as is pulse oximetry. Thoracentesis is used to obtain specimens for diagnostic evaluation, remove pleural fluid, or instill medication into the pleural space. Bronchoscopy is used for diagnostic purposes, to obtain biopsy specimens, and to assess changes resulting from treatment. Pulmonary function tests measure lung volumes and airflow to diagnose pulmonary disease, monitor disease progression, evaluate disability, and evaluate response to bronchodilators. Text Reference - p. 491

A patient with a history of epilepsy is admitted to the hospital for treatment of fever and shortness of breath. The patient is diagnosed with pneumonia. On taking history, the nurse finds that the patient had an episode of seizures four days ago with profuse vomiting. What type of pneumonia does the patient have? 1. Hospital-associated pneumonia 2. Community-acquired pneumonia 3. Aspiration pneumonia 4. Opportunistic pneumonia

3 RATIONALE: A patient who has seizures is at risk of developing aspiration pneumonia. The gastric contents enter the respiratory tract during the seizure and damage the lung tissue. Therefore this is the most probable reason for the patient's symptoms. The history of the patient does not suggest any exposure to pneumonia in the community. The patient has never been in the hospital; therefore hospital-associated pneumonia is highly unlikely. The patient does not have a history of HIV, intake of immunosuppressive drugs, corticosteroids, or any disorders leading to immunosuppression. Therefore opportunistic pneumonia did not occur in this patient.

A patient experiencing severe wheezing arrives in the emergency department and is diagnosed with severe exacerbation of asthma. During the admission assessment, the nurse on the inpatient unit notes that the patient continues to struggle with breathing, however, there is an absence of wheezing. How should the nurse interpret the assessment findings? 1. The patient has improved as there is no wheezing. 2. The patient is hypoxic and needs oxygen therapy. 3. The patient has respiratory failure and needs mechanical ventilation. 4. The patient has retained secretions and needs chest physiotherapy.

3 RATIONALE: A silent chest or absence of wheezing in a patient who had been having severe wheezing indicates an impending respiratory failure. The patient may need mechanical ventilation to support respiration. It is a sign of severe obstruction and it is a life-threatening condition. It is not a sign of improvement. Oxygen therapy may not help the patient as there is an obstruction in the airway. Chest physiotherapy helps in removing secretions from the airway, but may not be helpful in patients who are at risk of respiratory failure.

Which clinical manifestation should the nurse expect to find during assessment of a patient admitted with pneumonia? 1. Hyperresonance on percussion 2. Vesicular breath sounds in all lobes 3. Increased vocal fremitus on palpation 4. Fine crackles in all lobes on auscultation

3 RATIONALE: A typical physical examination finding for a patient with pneumonia is increased vocal fremitus on palpation. Other signs of pulmonary consolidation include bronchial breath sounds, egophony, and crackles in the affected area. With pleural effusion, there may be dullness to percussion over the affected area.

The nurse determines that additional discharge teaching is needed for a patient with pneumonia when the patient states: 1 "I will seek medical attention if I develop a fever or productive cough." 2 "Breathing exercises may help prevent future infections." 3 "I should take antibiotics for all upper respiratory infections." 4 "I will take all medications as prescribed.

3 RATIONALE: Antibiotics are not indicated for all upper respiratory tract infections. It is important for the patient to continue with coughing and deep-breathing exercises for at least six weeks, until all of the infection has cleared from the lungs. The patient should take all medications as prescribed and seek medical attention for signs or symptoms of new infection.

The patient has been diagnosed with head and neck cancer. Along with the treatment for the cancer, what other treatment should the nurse expect? 1 Nasal packing 2 Epistaxis balloon 3 Gastrostomy tube 4 Peripheral skin care

3 RATIONALE: Because 50% of patients with head and neck cancer are malnourished before treatment begins, many patients need enteral feeding via a gastrostomy tube because the effects of treatment make it difficult to take in enough nutrients orally, whether surgery, chemotherapy, or radiation are used. Nasal packing could be used with epistaxis or with nasal or sinus problems. Peripheral skin care would not be expected because it is not related to head and neck cancer.

The nurse is caring for an adult patient with bronchiectasis. The nurse knows that the primary cause of this disease in adults is related to: 1. Adult-onset asthma 2. Heavy smoking for more than 20 years 3. Untreated or delayed treatment of bacterial lung infections 4. Genetic predisposition and family members with the same diagnosis

3 RATIONALE: Bronchiectasis is an obstructive lung disease, generally caused by bacterial lung infections that are either not treated or treatment is delayed, leading to inflammation, airway damage, and remodeling. It can even follow a single severe bacterial infection if treatment is not given or delayed. Adult-onset asthma with no infection has no specific relationship to acquiring this disease. A patient's smoking history or genetically related factors do not affect the cause of this condition.

The nurse is caring for a patient with pneumothorax at risk of developing respiratory failure. What is the earliest indication of respiratory failure that the nurse should be observant for? 1. Cyanosis 2. Difficulty in walking 3. Change in mental status 4. Early morning headache

3 RATIONALE: Changes is mental status are often the earliest sign of impending respiratory failure, even before arterial blood gas results are obtained. The brain is very sensitive to even the slightest of variations in oxygen and carbon dioxide levels and acid-base balance. The early manifestations suggestive of inadequate oxygen delivery to the brain may include restlessness, confusion, agitation, and combative behavior. Cyanosis is an unreliable indicator of hypoxemia and is a late sign of respiratory failure because it does not occur until hypoxemia is severe (arterial oxygen [PaO2] of 45 mm Hg or less). Difficulty in walking is unreliable and a late symptom of respiratory failure. A severe morning headache may suggest that hypercapnia occurred during the night and is usually seen in chronic respiratory insufficiency. TEST-TAKING TIP: Answer the question that is asked. Read the situation and the question carefully, looking for key words or phrases. Do not read anything into the question or apply what you did in a similar situation during one of your clinical experiences. Think of each question as being an ideal, yet realistic, situation.

A patient with acute viral rhinitis has asked the nurse about medications that can be taken to relieve symptoms. Which statement, by the patient, reflects a need for further instruction? 1. "Antibiotics won't help this cold to go away." 2. "I can use saline nose spray to relieve congestion." 3. "I will limit the usage of nasal decongestant spray to 10 days." 4. "I will not drive if I take an antihistamine as it may make me drowsy.

3 RATIONALE: Decongestant sprays should not be used for more than three days to prevent rebound nasal congestion. Antibiotics may not be effective if the patient still has the cold. Saline nasal spray can be used to relieve congestion. The patient should not drive if taking an antihistamine.

A patient suspected of having lung cancer has undergone lung biopsy through transthoracic needle aspiration (TTNA). What is the priority nursing action for this patient immediately following the procedure? 1. Allow the patient to take a rest. 2. Instruct the patient to do deep breathing. 3. Send the patient for a chest x-ray as prescribed. 4. Measure oxygen saturation levels

3 RATIONALE: Following a lung biopsy through TTNA, the patient should be sent for chest x-ray to rule out a pneumothorax, which is a common complication of the procedure. Only after the chest x-ray is done, can the patient can be told to rest or do deep breathing exercises. Oxygen saturation levels are generally monitored throughout the procedure.

After being hit by a baseball, a patient arrives in the emergency department with a possible nasal fracture. Which finding by the nurse is most important to report to the health care provider? A. Clear nasal drainage B. Complaint of nasal pain C. Bilateral nose swelling and bruising D. Inability to breathe through the nose

A. Clear nasal drainage

The nurse is caring for a patient suspected of having pneumonia. What instructions should the nurse provide to the patient prior to receiving a chest x-ray? 1. Instruct the patient to undress completely and put on a gown. 2. Instruct the patient to sign a consent form. 3. Instruct the patient to remove any metal between neck and waist. 4. Tell the patient to not have food 2 hours before the test.

3 RATIONALE: For the chest x-ray, the nurse should instruct the patient to remove any metal between neck and waist. The patient is not required to undress completely; as it is a chest x-ray, undressing to waist is sufficient. The test does not require the patient to sign a consent form, and there is no need to avoid food before the test.

Which observation by the nurse indicates possible hemorrhage in a patient who has just undergone rhinoplasty? 1 Lack of appetite 2 Sudden forgetfulness 3 Excessive swallowing 4 Complaint of dizziness

3 RATIONALE: Frequent swallowing is an indication of hemorrhage after rhinoplasty. Because blood flows down the posterior pharynx into the throat, the patient is forced to swallow frequently. Lack of appetite may be present, but forgetfulness and complaints of dizziness are not complications associated with this procedure.

What is the arterial oxygen/fraction of inspired oxygen (PaO2/FIO2, or P/F) ratio in acute respiratory distress syndrome (ARDS)? 1 Greater than 400 2 Greater than 300 3 Less than 200 4 Between 200 and 300

3 RATIONALE: In acute respiratory distress syndrome, the P/F ratio is less than 200. Under normal circumstances, the P/F ratio would be greater than 400 (e.g., 95/0.21 = 452). With the onset and progression of lung injury and impairment in oxygen delivery through the alveolar-capillary interface, the PaO2 may remain lower than expected despite increased FIO2. The P/F ratio is 200 to 300 in acute lung injury.

The nurse cares for a patient with emphysema. What change in the alveolar sacs is the pathophysiological change in the lungs most characteristic of this disease? 1. The alveolar sacs collapse. 2. The alveolar sacs retain CO2. 3. The alveolar sacs are overdistended. 4. The alveolar sacs become filled with fluid

3 RATIONALE: In emphysema the alveolar sacs lose elasticity, become distended with trapped air, and may rupture. This causes obstruction of the alveolar capillary bed and impairs gas exchange. Alveolar sacs do not collapse or become filled with fluid. However, as a result of the overdistention and impaired gas exchange, carbon dioxide will be retained, but this is not the correct answer option because it is not the characteristic of emphysema.

The nurse is aware of the value of using a mini-tracheostomy to facilitate suctioning when patients are unable to independently mobilize their secretions. For which patient is the use of a mini-tracheostomy indicated? 1. A patient whose recent ischemic stroke has resulted in the loss of the gag reflex 2. A patient who requires long-term mechanical ventilation as the result of a spinal cord injury 3. A patient whose increased secretions are the result of community-acquired pneumonia 4. A patient with a head injury who has developed aspiration pneumonia after the patient's family insisted on spoon-feeding the patient

3 RATIONALE: It is appropriate to suction a patient with pneumonia using a mini-tracheostomy if blind suctioning is ineffective or difficult. An absent or compromised gag reflex, long-term mechanical ventilation, and a history of aspiration contraindicates the use of a mini-tracheostomy.

A patient with allergic asthma has been prescribed omalizumab (Xolair). The patient requests that the medication be administered at home for convenience. Which nursing action is appropriate in this case? 1 Load the injection and provide it to the patient for self-administration. 2 Administer the injection to the patient on a home visit. 3 Explain to the patient that the medication should be administered strictly at the clinic. 4 Ask a family member to administer the medication at home.

3 RATIONALE: Omalizumab (Xolair) is a monoclonal antibody to IgE that decreases circulating free IgE levels. The drug prevents IgE from attaching to mast cells, preventing the release of chemical mediators that may exacerbate asthma. The medication can cause anaphylactic reaction and should be administered at a clinic that is well-equipped to handle emergencies. The nurse should not encourage self-administration of the medication at home, as the anaphylactic reaction can be life-threatening. The nurse may not be well equipped to handle emergencies at home, so the medication should not be administered during a home visit. A family member should not be asked to administer the medication for the same reason.

A nurse is caring for a patient diagnosed with acute respiratory distress syndrome. The nurse is aware that these patients often will require which intervention? 1 Peritoneal dialysis 2 Frequent suctioning 3 Mechanical ventilation 4 Creatinine and blood urea nitrogen (BUN) testing

3 RATIONALE: Patients with acute respiratory distress syndrome likely will require mechanical ventilation to support their respiratory status. Frequent suctioning is not required often, but some suctioning may be required. Peritoneal dialysis and creatinine and BUN testing might be necessary with some level of kidney failure, not respiratory compromise.

A 72-year-old patient is in the emergency department with a temperature of 101.4° F and a productive cough with rust-colored sputum. The nurse notifies the health care provider, understanding these findings are indicative of which condition? 1. Tuberculosis 2. Chronic heart failure (CHF) 3. Pneumonia 4. Pulmonary embolus

3 RATIONALE: Rust-colored sputum is indicative of pneumococcal pneumonia. Tuberculosis frequently presents with a dry cough. Pink frothy sputum would be present in CHF and pulmonary edema. A pulmonary embolus would produce hemoptysis.

A patient with chronic obstructive pulmonary disease (COPD) experiences dyspnea and has a forced expiratory volume (FEV1) of 70% of predicted value. The nurse expects that what will be included on the patient's treatment plan? 1. Use of inhalational budesonide (Pulmicort Flexhaler) 2. Use of long-acting theophylline (Theo-24) 3. Use of short-acting bronchodilator 4. Using a combination of fluticasone and salmeterol (Advair Diskus)

3 RATIONALE: Since the patient has FEV1 of 70% and is suffering from mild COPD, the patient may benefit from the use of short-acting bronchodilators. Budesonide, being a steroid, is not used for treating mild COPD. Use of theophylline in COPD is controversial and should be used only in patients who do not respond to other drugs. A combination of fluticasone and salmeterol is not required for treating mild COPD; this medication is prescribed to patients who have a FEV1 of less than 60%.

Barbiturates were given to an older man with reduced kidney function, and he nearly died as a result. What was the most likely reason for this near-fatality? Select one: A. Increased biological half-life of the drug B. Increased Kidney filtration C. Increased drug dosage D. Increased reabsorption of the drugs into the blood

A. Increased biological half-life of the drug

The nurse is assigned to care for a patient in the emergency department admitted with an exacerbation of asthma. The patient has received a β-adrenergic bronchodilator and supplemental oxygen. If the patient's condition does not improve, the nurse should anticipate what as the most likely next step in treatment? 1. Intravenous (IV) fluids 2.Biofeedback therapy 3.Systemic corticosteroids 4.Pulmonary function testing

3 RATIONALE: Systemic corticosteroids speed the resolution of asthma exacerbations and are indicated if the initial response to the β-adrenergic bronchodilator is insufficient. IV fluids may be used, but not to improve ventilation. Biofeedback therapy and pulmonary function testing may be used after recovery to assist the patient and monitor the asthma.

An asthmatic patient was prescribed theophylline (Theo-24). A nurse understands that the patient is at risk for tachycardia and seizures. In regards to safety, the nurse expects that what will be included on the patient's treatment plan? 1. Encourage the use of caffeine. 2. Monitor serum blood levels of adrenaline. 3. Monitor serum blood levels of theophylline. 4. Use diazepam (Valium) to prevent seizures.

3 RATIONALE: Tachycardia and seizures are known toxic effects of theophylline at higher blood levels. In addition, the drug has a narrow margin of safety. Therefore, monitoring blood levels of theophylline helps to reduce such toxic effects. Caffeine increases the toxic effects of theophylline. Monitoring plasma levels of adrenaline may not help as tachycardia is not associated with adrenaline in this case. Prophylactic use of diazepam may not prevent seizures as they are caused by the use of theophylline.

To promote airway clearance in a patient with pneumonia and asthma, the nurse instructs the patient to perform which action? 1. Wear supplemental oxygen at all times 2. Perform pursed-lip breathing 3. Sit upright while using the flutter device 4. Use the incentive spirometer 10 times per hour

3 RATIONALE: The flutter device is used to increase mucus production to promote airway clearance and gas exchange; it should be used while the patient is in an upright position. Supplemental oxygen may not be indicated depending on the oxygen saturation level. Pursed-lip breathing and the incentive spirometer will not promote airway clearance.

A patient has been admitted with a suspected lung abscess. During the assessment, the nurse is aware that the most common manifestation of a lung abscess is which of these? 1 Fever 2 Vomiting 3 Purulent sputum that has a foul odor and taste 4 Increased breath sounds on auscultation over the involved segment of lung.

3 RATIONALE: The most common manifestation of a lung abscess is cough-producing purulent sputum (often dark brown) that is foul smelling and foul tasting. Hemoptysis is common, especially when an abscess ruptures into a bronchus. Other common manifestations are fever, chills, prostration, night sweats, pleuritic pain, dyspnea, anorexia, and weight loss. Physical examination of the lungs indicates dullness to percussion and decreased breath sounds on auscultation over the involved segment of lung. Vomiting is not a manifestation of a lung abscess.

The nurse determines that a 61-year-old patient with chronic bronchitis has a nursing diagnosis of "impaired gas exchange," after noting an oxygen saturation of 88%. What is an appropriate intervention to add to the care plan? 1 Obtain an arterial blood gas (ABG) 2 Place the patient in the prone position to increase postural drainage 3 Sit the patient upright in a chair leaning slightly forward 4 Administer 6 L oxygen via nasal cannula

3 RATIONALE: The patient with chronic bronchitis can engage in better gas exchange in an upright position leaning slightly forward. Once the patient's oxygen increases, the nurse may obtain an ABG, if requested by the health care provider. Placing the patient in a prone position would further impair gas exchange. Six liters of oxygen is too much oxygen for a patient with chronic bronchitis.

The nurse is caring for a child who is suspected of having cystic fibrosis (CF). The nurse knows that the gold standard of diagnosing this condition is: 1 Genetic test 2 Lung biopsy 3 The sweat chloride test 4 Test for pancreatic insufficiency

3 RATIONALE: The sweat chloride test is the most definitive test in children, but may not be conclusive in adults. Patients with CF secrete four times the normal amounts of sodium and chloride in their sweat. Genetic tests may be used if the sweat test is inconclusive. A lung biopsy is not used to diagnose this condition. Pancreatic insufficiency is related to CF but does not specifically diagnose the disease.

A patient who was in a motor vehicle accident is brought to the emergency department unconscious, and cardiopulmonary resuscitation (CPR) is performed. The patient responds well, and the condition improves. After several hours, the patient experiences dyspnea and becomes cyanotic. On examination the neck veins are distended, and the patient is tachycardic. The nurse expects that the immediate plan for treatment will include what intervention? 1. Oxygen administration 2. Pericardiocentesis 3. Needle decompression 4. Placing the patient in a side-lying position

3 RATIONALE: The symptoms and signs indicate that the patient has tension pneumothorax. This is a medical emergency in which air enters into the pleural space and does not come out. This leads to compression of the surrounding organs like the lung, heart, and large vessels. If not treated promptly, the patient may die. Therefore the patient requires immediate needle decompression followed by chest tube insertion with chest drainage system. Oxygen administration will not help this patient. Pericardiocentesis is done for patients with cardiac tamponade. Repositioning the patient to his side is not required in this case.

A patient has a chest tube inserted to treat a spontaneous pneumothorax. Which observation causes the nurse to conclude that the water-seal chamber of the closed chest drainage system is functioning properly? 1. Gentle bubbling in the suction chamber 2. Patient tolerating mild shortness of breath 3. Water-seal chamber level fluctuating with respirations 4. The presence of bloody drainage in the water-seal chamber

3 RATIONALE: The water-seal chamber level fluctuates with respirations as a result of the restoration of negative pressure within the thoracic cavity. Gentle bubbling in the suction chamber indicates a possible air leak. New-onset of shortness of breath in a patient with a chest tube requires further assessment. The water-seal chamber should not contain blood; this finding indicates that the chest tube drainage system may have been knocked onto its side and should be replaced.

The various events occurring in the exudative phase of acute respiratory distress syndrome (ARDS) are listed. Arrange these events in the sequential order of development. 1. Necrotic cells, protein, and fibrin form hyaline membranes that line the alveoli. 2. Fluid from the interstitial space crosses the alveolar membrane and enters the alveolar space. 3. Engorgement of the peribronchial and perivascular interstitial space produces interstitial edema. 4. Damage to alveolar type I and II cells is followed by atelectasis due to reduced surfactant synthesis.

3, 2, 4, 1 RATIONALE: The exudative phase of acute respiratory distress syndrome (ARDS) occurs within 1 to 7 days after the initial direct lung injury. Initially, engorgement of the peribronchial and perivascular interstitial space produces interstitial edema. Next, fluid from the interstitial space crosses the alveolar membrane and enters the alveolar space. Decreased synthesis of surfactant and inactivation of existing surfactant cause the alveoli to become unstable and collapse (atelectasis). Finally, necrotic cells, protein, and fibrin form hyaline membranes that line the alveoli.

A nurse is caring for a patient who has undergone laryngectomy. Upon suctioning the tracheostomy tube, the nurse notices a slight thickening of the secretions. Which measures should the nurse take to prevent thickening of secretions? Select all that apply. 1 Keep the patient in a semi-Fowler position. 2 Change the tracheostomy tube. 3 Maintain adequate fluid intake. 4 Humidify inspired gases. 5 Give a normal saline bolus through tracheostomy

3,4 RATIONALE: Maintaining adequate hydration through intravenous or enteral route and humidifying the inspired gases prevents drying and thickening of secretions. This allows easy suction of secretions. Changing the patient's position helps to relieve dyspnea in the patient. Changing the tracheostomy tube does not help as the secretions are too copious to be removed. Normal saline bolus is not recommended as it may cause infection

The nurse is caring for the patient with chronic obstructive pulmonary disease who is undernourished and underweight. Which steps can the nurse take to improve the patient's nutritional status? Select all that apply. 1. Restrict fluid intake to 1 L/day 2. Have the patient drink fluid with meals 3. Provide a diet high in protein and calories 4. Provide five to six small meals per day 5. Avoid overfeeding the patient

3,4,5 RATIONALE: A diet high in calories and protein, moderate in carbohydrates, and moderate to high in fat is recommended and can be divided into five or six small meals a day. High-protein, high-calorie nutritional supplements can be offered between meals. Nonprotein calories should be divided evenly between fat and carbohydrate, but avoid overfeeding the patient. Fluid intake should be at least 3 L/day unless contraindicated by other medical conditions. Fluids should be taken between meals (rather than with them) to prevent excess stomach distention and to decrease pressure on the diaphragm.

The nurse attending to an older patient finds the patient coughing. On auscultation, the nurse finds retained pulmonary secretions in the lungs. What nursing interventions should the nurse perform to mobilize secretions? Select all that apply. 1. Advise bed rest for the patient. 2. Limit fluid intake. 3. Encourage the patient to cough. 4. Provide chest physiotherapy. 5. Provide humidified air.

3,4,5 RATIONALE: Retained pulmonary secretions increase the risk of respiratory failure. It is important to mobilize the secretions and clear the airway to facilitate ventilation. Effective coughing is an important measure to move up the secretions and relieve obstruction of the airway. Chest physiotherapy can be used in patients who produce copious amount of sputum and who have a collapsed lung. Humidified air helps to keep secretions liquefied and eases in coughing them out. The patient should not be limited to bed rest; ambulation should be done when possible, since it helps to expand the lungs and clear the secretions. Fluid intake should not be restricted. Adequate fluid intake is required to prevent the secretions from thickening.

The nurse is caring for a patient who had a recent lung transplant. The nurse knows which of the following? Select all that apply. 1 Acute rejection can occur 2 to 3 weeks after surgery. 2 Immunosupressive therapy is usually a two-drug regimen. 3 Accurate diagnosis of rejection is by transtracheal biopsy. 4 Cytomegalovirus (CMV) pneumonia is the most common opportunistic infection. 5 During the first year, viral pneumonia is the most common postoperative infection. 6 Lung transplant recipients usually receive higher levels of immunosuppressive therapy than other organ recipients.

3,4,6 RATIONALE: Cytomegalovirus (CMV) is the most common opportunistic infection. Because acute rejection is common, higher levels of immunosuppressive therapy than what other organ recipients receive are given. Diagnosis of rejection is confirmed by transtracheal biopsy. Immunosuppressive therapy is usually a three-drug regimen and acute rejection can occur in the first 5 to 10 postoperative days. If acute rejection is diagnosed, then high doses of corticosteroids are given for three days, followed by high doses of oral prednisone. If this does not work and rejection occurs, then antilymphocytic therapy may be useful. Bacterial pneumonia is the most common postoperative infection.

A nurse is caring for a patient with respiratory failure. Which medications should be included when planning the therapy for this patient? Select all that apply. 1 Nitroglycerin (Tridil) for bronchospasm 2 Albuterol (Ventolin) for pulmonary congestion 3 Methylprednisolone (Solu-Medrol) for inflammation and bronchospasm 4 Azithromycin (Zithromax) for pain, anxiety, and restlessness 5 Furosemide (Lasix) for pulmonary congestion caused by heart failure

3,5 RATIONALE: Corticosteroids like methylprednisolone (Solu-Medrol) may be used in conjunction with bronchodilating agents for bronchospasm and inflammation. The intravenous diuretic furosemide (Lasix) is used to decrease the pulmonary congestion caused by heart failure. Nitroglycerin (Tridil) will relieve pulmonary congestion caused by heart failure and does not relieve bronchospasm. Albuterol (Ventolin) is used in bronchospasm, not for pulmonary congestion. Azithromycin (Zithromax) is used for pulmonary infection, and not for pain, anxiety, and restlessness.

The patient is prescribed a high-flow oxygen delivery system. Which of the following are high-flow devices? Select all that apply. 1 Nasal cannula 2 Simple face mask 3 Venturi mask 4 Non-rebreather mask 5 Mechanical ventilator

3,5 RATIONALE: Venturi mask and mechanical ventilator are correct because oxygen (O2) delivery systems are classified as low- or high-flow systems. Venturi mask is a high-flow device that delivers fixed concentrations of O2 (e.g., 24%, 28%) independent of the patient's respiratory pattern. Mechanical ventilators are another example of high-flow O2 delivery system. Mechanical ventilation is a means of support until the patient can breathe on his or her own. A nasal cannula, simple face mask, and non-rebreather mask are low-flow devices that deliver O2 in concentrations that vary with the person's respiratory pattern. Because room air is mixed with O2, in low-flow systems, the percentage of O2 delivered to the patient is not as precise as with high-flow systems.

The clinic nurse teaches a patient with a 42 pack-year history of cigarette smoking about lung disease. Which information will be most important for the nurse to include? A. Options for smoking cessation B. Reasons for annual sputum cytology testing C. Erlotinib (Tarceva) therapy to prevent tumor risk D. Computed tomography (CT) screening for lung cancer

A. Options for smoking cessation

Bed rest is prescribed for a patient during the acute phase of respiratory failure. What is the rationale for the recommendation of bed rest and limitation of other activity in the plan of care? 1 To prevent further alveolar collapse 2 To decrease the basal metabolic rate 3 To promote the clearance of secretions 4 To reduce the cellular demand for oxygen

4 Respiratory failure interferes with ventilation and oxygenation. It is essential to reduce the body's need for oxygen at the cellular level. Bed rest is an essential and effective means of reducing the need for oxygen. Bed rest and limitation of activity do not prevent alveolar collapse, clear secretions, or decrease the basal metabolic rate. Text Reference - p. 1659

The health care provider has prescribed salmeterol (Serevent) for a patient with asthma. In reviewing the use of dry powder inhalers (DPIs) with the patient, what instructions should the nurse provide? 1 "Close lips tightly around the mouthpiece and breathe in deeply and quickly." 2 "To administer a DPI, you must use a spacer that holds the medicine so that you can inhale it." 3 "You will know you have correctly used the DPI when you taste or sense the medicine going into your lungs." 4 "Hold the inhaler several inches in front of your mouth and breathe in slowly, holding the medicine as long as possible."

4 The patient should be instructed to tightly close the lips around the mouthpiece and breathe in deeply and quickly to ensure the medicine moves down deeply into the lungs. Dry powder inhalers do not require spacer devices. The patient may not taste or sense the medicine going into the lungs. The inhaler should not be placed several inches in front of the mouth.

Which factor places a conscious patient at risk for pneumonia? 1 Effective postoperative pain management 2 Lying supine for two consecutive hours 3 Adequate cough and deep breathing exercises 4 Difficulty swallowing medication

4 The patient who has difficulty swallowing needs assistance in eating, drinking, and taking medication to prevent aspiration. Difficulty swallowing increases risk of aspiration. Treating postoperative pain effectively provides comfort, permitting the patient to cough and deep breathe and achieve optimum mobility. Lying supine for two consecutive hours alone does not place an otherwise healthy patient at risk for pneumonia but for the altered consciousness patient, repositioning should occur at least every two hours. Turning, coughing, and deep breathing exercises promote optimal oxygenation or perfusion and help prevent atelectasis. Text Reference - p. 547

A patient had an open reduction repair of a bilateral nasal fracture. The nurse plans to implement an intervention that focuses on both nursing and medical goals for this patient. Which intervention should the nurse implement? 1. Apply an external splint to the nose 2. Insert plastic nasal implant surgically 3. Humidify the air for mouth breathing 4. Maintain surgical packing in the nose

4 RATIONALE: A goal that is common to nursing and medical management of a patient after rhinoplasty is to prevent the formation of a septal hematoma and potential infections resulting from a septal hematoma. Therefore, the nurse helps to keep the nasal packing in the nose. The packing applies direct pressure to oozing blood vessels to stop postoperative bleeding. A medical goal includes realigning the fracture with an external or internal splint. The nurse helps maintain the airway by humidifying inspired air because the nose is unable to do so following surgery because it is swollen and packed with gauze.

The nurse is assessing a young male patient who came to the emergency department complaining of sudden shortness of breath. He has no other visible problems. The nurse notes that, upon auscultation, there are no breath sounds on the right upper lobe of the lung. The nurse suspects that the patient has which of these conditions? 1 Tension pneumothorax 2 Iatrogenic pneumothorax 3 Traumatic pneumothorax 4 Spontaneous pneumothorax

4 RATIONALE: A lack of breath sounds over a portion of the lung fields indicates the presence of a pneumothorax. A spontaneous pneumothorax typically occurs because of the rupture of small blebs (air-filled blisters) located on the apex of the lung. These blebs can occur in healthy, young individuals, especially tall, thin males. Tension pneumothorax occurs when air enters the pleural space but cannot escape. The continued accumulation of air in the pleural space causes increasingly elevated intrapleural pressures. Tension pneumothorax can occur with mechanical ventilation and resuscitative efforts. Iatrogenic pneumothorax can occur because of laceration or puncture of the lung during medical procedures. Traumatic pneumothorax can occur from either penetrating (open) or nonpenetrating (closed) chest trauma.

The nurse provides care to a trauma victim. Which clinical manifestation most suggests a pneumothorax? 1. Inspiratory crackles 2. Pronounced crackles 3. Dullness on percussion 4. Absence of breath sound

4 RATIONALE: A pneumothorax indicates that one of the lungs has collapsed and is not functioning. On auscultation no sounds of air movement will be heard. Because no air movement occurs with a pneumothorax, no breath sounds, including crackles, will be heard. Dullness may be a finding on percussion over the area of the pneumothorax, but an absence of breath sounds is the definitive finding.

The nurse is assessing a patient with a pneumothorax. The nurse expects to note which type of fremitus? 1. Normal fremitus 2. Decreased fremitus 3. Increased fremitus 4. Absent fremitus

4 RATIONALE: In a patient with pneumothorax, the nurse would find absent fremitus. Decreased fremitus may be found in pleural effusion when the hand is farther from the lung, and in barrel chest where the lung is hyperinflated. Increased fremitus is found in pneumonia, in lung tumors, with thick bronchial secretions, and above a pleural effusion. As the patient's voice moves through a dense tissue or fluid-filled lungs, the vibration is increased.

The nurse determines that the patient with chronic obstructive pulmonary disease (COPD) and diabetes mellitus is experiencing adverse effects of albuterol (Proventil) after noting which finding? 1 Blood sugar 139 mg/dL 2 Respiratory rate of 21/minute 3 Temperature of 99.1°F 4 Pulse rate of 102 beats/minutes

4 RATIONALE: Albuterol is a beta-2-agonist that sometimes can cause adverse cardiovascular effects. These would include tachycardia and angina. A pulse rate of 102 indicates that the patient is experiencing tachycardia as an adverse effect. This medication will not affect the blood sugar or the temperature. The respiratory rate is normal and does not indicate any adverse reaction is occurring.

The nurse is caring for a patient with a diagnosis of active tuberculosis (TB). The nurse knows that: 1. Directly observed therapy is used only in the initial phase 2. Drug therapy is in three phases (initial, interim, and continuation) 3. Liver function tests (LFTs) are initiated 14 days after the start of treatment 4. Alcohol is avoided because it increases the hepatotoxicity of isoniazid (INH)

4 RATIONALE: Alcohol must be avoided because it increases the hepatotoxicity of INH. Directly observed therapy must be continued through both phases in patients who are at risk for non-adherence to drug therapy. Drug therapy includes a two-phase process, with an initial and continuation phase. Baseline LFTs are done before treatment is begun and then monitored monthly

The nurse is reviewing the care of a patient with a pulmonary fungal infection. Which of these statements is true? 1 The patient will be placed on droplet isolation. 2 Opportunistic fungal infections occur in otherwise healthy people. 3 Pulmonary fungal infections are transmitted from person to person. 4 Amphotericin B is the standard therapy for treating serious systemic fungal infections.

4 RATIONALE: Amphotericin B remains the standard therapy for treating serious systemic fungal infections. These infections are not transmitted from person to person, and the patient does not have to be placed in isolation. Opportunistic fungal infections occur in immunocompromised patients (e.g., those being treated with corticosteroids, chemotherapy, and immunosuppressive drugs) and in patients with human immunodeficiency virus (HIV) and cystic fibrosis. Fungal infections are acquired by inhalation of spores.

A 71-year-old patient is admitted with acute respiratory distress related to cor pulmonale. Which nursing intervention is most appropriate during admission of this patient? 1 Perform a comprehensive health history with the patient to review prior respiratory problems. 2 Complete a full physical examination to determine the effect of the respiratory distress on other body functions. 3 Delay any physical assessment of the patient and review with the family the patient's history of respiratory problems. 4 Perform a physical assessment of the respiratory system and ask specific questions related to this episode of respiratory distress.

4 RATIONALE: Because the patient is having respiratory difficulty, the nurse should ask specific questions about this episode and perform a physical assessment of this system. Further history taking and physical examination of other body systems can proceed once the patient's acute respiratory distress is being managed.

The emergency department nurse is assessing a patient who has been having trouble breathing for the last few months. His wife tells the nurse that he worked in a coal mine for 25 years but stopped working there last year. The nurse will look for manifestations of which problem? 1. Pulmonary edema 2. Pulmonary fibrosis 3. Respiratory alkalosis 4. Hypersensitivity pneumonitis

4 RATIONALE: Black lung is caused by inhalation of large amounts of coal dust, an occupational hazard for underground coal miners. The inhaled substance is ingested by macrophages, which releases substances that cause cell injury and death. Fibrosis occurs as a result of tissue repair. Repeated exposure eventually results in diffuse pulmonary fibrosis (excess connective tissue). Pulmonary edema and respiratory alkalosis are not caused by inhaled coal dust.

A patient presenting with pneumonia scores 5 on the CURB-65 scale. What action should the nurse take? 1. Advise no treatment. 2. Advise treating at home. 3. Consider hospital admission. 4. Consider admission to an intensive care unit.

4 RATIONALE: CURB-65 is used in addition to clinical judgment in determining the severity of pneumonia and the need for advanced medical care. A patient scoring 5 on the CURB-65 scale means the condition is severe and needs advanced medical care. Hence, the nurse should consider admission to an intensive care unit. If the patient has symptoms of pneumonia, advising no treatment is not an option. Treating at home is advised when the score on CURB-65 scale is 0. Hospital admission is considered when the score on the CURB-65 scale is 1-2.

Which risk factor is an absolute contraindication to receiving lung transplantation? 1 Skin cancer 2 Former smoker 3 Reversible renal failure 4 Chronic active hepatitis B

4 RATIONALE: Chronic active hepatitis, types B and C, are absolute contraindications for lung transplantation. Malignancy, with the exception of skin cancer, is an absolute contraindication. Skin cancer is not an absolute contraindication to receiving lung transplantation. Current smoking is an absolute contraindication. Former smoking is not, however, an absolute contraindication to lung transplantation. Untreatable advanced dysfunction of another major organ system, such as the kidneys, would be an absolute contraindication to lung transplantation. Reversible renal failure is not a contraindication.

The nurse is planning to deflate the tracheostomy cuff of a patient. Which task is least helpful in preventing secretions from being aspirated during deflation? 1. Suction the tracheostomy tube 2. Have the patient cough up secretions 3. Suction the mouth 4. Deflate the cuff during inspiration

4 RATIONALE: Deflating the cuff during inspiration would be least helpful in preventing aspiration because the tracheostomy cuff should be deflated during the patient's expiration and exhaled gas helps propel secretions into the mouth. Suctioning the tracheostomy tube and then the mouth are important steps in preventing secretions from being aspirated during cuff deflation. Having the patient cough and therefore clear the lower airway before cuff deflation minimizes risk of aspiration. Suctioning the tracheostomy tube and then the mouth are important steps in preventing secretions from being aspirated during cuff deflation.

The nurse understands that one way to assess the degree of impairment in gas exchange is to measure the arterial oxygen/fraction of inspired oxygen (PaO2/FIO2, or P/F) ratio. What is the P/F ratio in acute lung injury (ALI)? 1. Greater than 400 2. Greater than 300 3. Less than 200 4. Between 200 and 300

4 RATIONALE: In ALI, the P/F ratio is between 200 and 300. This indicates compromised gas exchange through the alveoli. Under normal circumstances, when PaO2 is 85 to 100 mm Hg and FIO2 is 0.21, the P/F ratio would be greater than 400. The term acute respiratory distress syndrome (ARDS) is used when the P/F ratio is less than 200 (e.g., 80/0.8 = 100) and indicates refractory hypoxemia.

A patient with a respiratory condition asks "I know how air gets into my lungs. But how does it get out of them?" The nurse bases the answer on the knowledge that air moves out of the lungs because of 1 Contraction of the accessory abdominal muscles 2 Increased carbon dioxide and decreased oxygen in the blood 3 Stimulation of the respiratory muscles by the chemoreceptors 4 Increase in intrathoracic pressure as elastic recoil of the lungs occurs

4 RATIONALE: In contrast to inspiration, expiration is passive. Elastic recoil is the tendency for the lungs to relax after being stretched or expanded. The elasticity of lung tissue is because of the elastin fibers found in the alveolar walls and surrounding the bronchioles and capillaries. The elastic recoil of the chest wall and lungs allows the chest to passively decrease in volume. Intrathoracic pressure rises, causing air to move out of the lungs. Lungs do not expel air by use of abdominal muscles. An increase in carbon dioxide and decrease in oxygen in the blood do not promote expiration.

A patient with human immunodeficiency virus (HIV) infection has been admitted with pneumonia. The nurse anticipates that this patient will receive treatment for which type of pneumonia? 1 Aspiration pneumonia 2 Hospital-associated pneumonia 3 Community-acquired pneumonia 4 Opportunistic pneumonia caused by Pneumocystis jiroveci

4 RATIONALE: Individuals at risk for opportunistic pneumonia include those with altered immune responses, such as HIV infection. In addition to the risk of bacterial and viral pneumonia, the immunocompromised person may develop an infection from microorganisms that do not normally cause disease, such as P. jiroveci (formerly carinii ). The patient likely does not have aspiration pneumonia, hospital-associated pneumonia, or community-acquired pneumonia.

The nurse is teaching a patient stoma care after a tracheostomy. Discharge instructions should include: 1. No diving into water but swimming is okay. 2. Washing the area around the stoma at least three times a day. 3. No covering of the stoma while coughing so secretions can be excreted. 4. Wearing a Medic Alert bracelet or other form of emergency identification

4 RATIONALE: It is important to wear a Medic Alert bracelet or other form of identification so that emergency personnel are able to identify that the patient breathes from the neck. Swimming and diving are contraindicated. At least once a day, the area around the stoma needs to be cleansed with a moist cloth. The stoma needs to be covered when coughing to prevent the spread of mucous and secretions to others.

The nurse is caring for a patient with a diagnosis of idiopathic pulmonary arterial hypertension (IAPH). The nurse knows that: 1. IAPH occurs more often in males than females. 2. The mean diagnosis time between onset of symptoms and diagnosis is 5 to 10 years. 3. IAPH increases the work of the left ventricle and causes left-ventricular hypertrophy. 4. Functional classification of IAPH is measured by using the New York Heart Association Scale

4 RATIONALE: Patients are classified using the New York Heart Association functional classification. IAPH occurs more often in females than in males. The time between onset of symptoms and diagnosis is about two years and usually by the time the patient becomes symptomatic, the disease is in the advanced stages. IAPH increases the work of the right ventricle and causes right-ventricular hypertrophy (cor pulmonale).

The nurse is caring for a patient with a shunt due to acute respiratory distress syndrome (ARDS). Which nursing intervention is associated with better symptomatic relief for this patient? 1 Mechanical ventilation only 2 Bronchodilators along with corticosteroids 3 High fraction of inspired oxygen (FIO2) only 4 Mechanical ventilation and high FIO2

4 RATIONALE: Patients with ARDS having a shunt disorder are usually more hypoxemic than patients with ventilation-perfusion (V/Q) mismatch. They often require mechanical ventilation and a high FIO2 in combination to improve gas exchange. Bronchodilators and corticosteroids are not helpful for immediate relief because the patient does not have bronchospasm and inflammation.

A patient with emphysema is receiving oxygen at 1 L/min by way of nasal cannula. The nurse understands that this prescription is appropriate because: 1 The patient does not require more than 1 L of oxygen 2 High concentrations of oxygen may rupture the alveoli 3 Oxygen is the natural stimulus for breathing and not required 4 High concentrations of oxygen eliminate the respiratory drive

4 RATIONALE: Patients with emphysema become accustomed to a high level of carbon dioxide and low level of oxygen. This situation reverses the natural breathing stimulus. A low oxygen level then becomes the stimulus for breathing, and too much oxygen will eliminate the stimulus to breathe. There is not enough information to determine that the patient does not need more than 1 L of oxygen. A high concentrations of oxygen does not rupture alveoli. In healthy individuals, increased carbon dioxide, not oxygen, is the stimulus for breathing.

Which nursing intervention is used to optimize oxygenation and ventilation in a patient with acute respiratory failure? 1 Suction the patient every hour. 2 Avoid hyperventilating the patient. 3 Position the patient with the good lung up. 4 Provide adequate rest and recovery time between procedures.

4 RATIONALE: Providing adequate rest and recovery time between various procedures prevents desaturation and optimizes oxygenation. Suctioning is indicated when the patient's condition warrants it, not when it is scheduled regularly. Hyperventilation may be used before treatments, repositioning, interventions, and so forth to allow the patient to have a "reserve." Positioning for optimal oxygenation and ventilation is good lung down, not up.

The nurse teaches pursed-lip breathing to a patient with emphysema. The nurse explains that the primary reason for this technique is: 1. Conserving energy 2. Relieving chest pain 3. Promoting oxygen saturation 4. Promoting carbon dioxide elimination

4 RATIONALE: Pursed-lip breathing promotes elimination of carbon dioxide by increasing positive pressure within the alveoli and making it easier to expel air from the lungs. This procedure also helps the patient slow the breathing and increase the depth of respirations. The purpose of instructing the patient in pursed-lip breathing is not to conserve energy, reduce chest pain, or promote oxygen saturation, although when performed correctly it may result in improved oxygenation.

The nurse is caring for a patient who has had a left pneumonectomy. An appropriate nursing intervention for a patient postpneumonectomy is 1. Monitoring chest tube drainage and functioning 2. Positioning the patient on the nonoperative side 3. Auscultating frequently for lung sounds on the operative side 4. Encouraging range-of-motion exercises on the affected upper extremity

4 RATIONALE: Range of motion exercises performed on the affected upper extremity will prevent edema and encourage circulation to the lung space to promote healing. A patient who has had a pneumonectomy may have a clamped chest tube postoperatively, so there will not be any drainage. Fluid will gradually fill space where the lung has been removed. Position patient on operative side to facilitate expansion of remaining lung. There will not be lung sounds on the operative side as the entire lung has been removed.

A patient who has tuberculosis (TB) is being treated with combination drug therapy. The nurse explains that combination drug therapy is essential because: 1. It minimizes the required dosage of each of the medications 2. It helps reduce the unpleasant side effects of the medications 3. It shortens amount of time that the treatment regimen will be needed 4. It discourages the development of resistant strains of the TB organism

4 RATIONALE: Recommendations for the initial treatment of tuberculosis (TB) includes a four-drug regimen until drug susceptibility tests are available. After susceptibility is established, the regimen can be altered, but patients should still receive at least two drugs to prevent emergence of drug-resistance organisms. Dosage, side effects, and duration of the regimen are not reasons for combination drug therapy in a patient with TB.

A patient who has just undergone a laryngectomy wants to lie flat in bed. Which statement would be an incorrect explanation for placing the patient in semi-Fowler position? 1. Semi-Fowler position will limit tension on the suture lines 2. Semi-Fowler position will decrease edema 3. Semi-Fowler position will help maintain airway patency 4. Semi-Fowler position will enhance the ability to communicate

4 RATIONALE: Semi-Fowler position will not enhance one's ability to communicate. After a laryngectomy, the patient will need teaching about methods of communicating other than speaking, either temporarily or permanently. Semi-Fowler position will limit tension on the suture lines, decrease edema, and help maintain airway patency are all correct. Maintenance of a patent airway is essential after laryngectomy. This is achieved primarily by keeping the patient in semi-Fowler position postoperatively. Keeping the patient in semi-Fowler position decreases edema and limits tension on the suture lines. This is one of many correct explanations for keeping the patient in semi-Fowler position postlaryngectomy.

The nurse is measuring a patient's pulse oximetry and obtains an SpO2 reading of 88%, and then another reading of 87%. The patient is receiving oxygen per nasal cannula at 2 L/minute and states feeling short of breath. Previous readings were above 94%. What should the nurse do next? 1 Document the findings. 2 Increase the flow of oxygen delivery. 3 Check the pulse oximetry again in an hour. 4 Notify the health care provider for a possible arterial blood gas (ABG) analysis.

4 RATIONALE: The abbreviation SpO2 is used to indicate the oxygen saturation of hemoglobin as measured by pulse oximetry. If there is doubt about the accuracy of the SpO2 reading, obtain an ABG analysis to verify the results. The findings should not be documented as normal. The nurse should not increase oxygen flow without further assessment.

Infection can be a major hazard of O2 administration. Heated nebulizers present the highest risk. The most common organism found is: 1. Rickettsia prowazekii 2. Clostridium perfringens 3. Bordatella pertussis 4. Pseudomonas aeruginosa

4 RATIONALE: The constant use of humidity supports bacterial growth, with the most common organism being P. aeruginosa. Rickettsia prowazekii, Clostridium perfringens, and Bordatella pertussis are not the most common organisms found in this case.

The nurse evaluates that discharge teaching for a patient hospitalized with pneumonia has been effective when the patient makes which statement about measures to prevent a relapse? 1. "I will seek immediate medical treatment for any upper respiratory infections." 2. "I should continue to do deep-breathing and coughing exercises for at least 12 weeks." 3. "I will increase my food intake to 2400 calories a day to keep my immune system well." 4. "I must have a follow-up chest x-ray in six to eight weeks to evaluate the pneumonia's resolution."

4 RATIONALE: The follow-up chest x-ray will be done in six to eight weeks to evaluate pneumonia resolution. A patient should seek medical treatment for upper respiratory infections that persist for more than seven days. It may be important for the patient to continue with coughing and deep breathing exercises for six to eight weeks, not 12 weeks, until all of the infection has cleared from the lungs. Increased fluid intake, not caloric intake, is required to liquefy secretions.

The nurse concludes that interventions carried out to promote airway clearance in a patient admitted with asthma are successful on the basis of which finding? 1. Oxygen saturation 96% 2. Use of accessory muscles 3. Absence of wheezing 4. Clearance of mucous from the bronchi

4 RATIONALE: The issue is airway clearance, which is evaluated most directly as successful if the patient can engage in effective and productive coughing. Oxygen saturation would indicate gas exchange, not airway clearance. Use of accessory muscles indicates respiratory distress. The absence of wheezing does not always coincide with improved airway clearance, and may represent worsening bronchospasm

The patient seeks relief from the symptoms of an upper respiratory infection (URI) that has lasted for five days. Which patient assessment should the nurse use to help determine if the URI has developed into acute sinusitis? 1 Coughing 2 Fever, chills 3 Dust allergy 4 Maxillary pain

4 RATIONALE: The nurse should assess the patient for sinus pain or pressure as a clinical indicator of acute sinusitis. Coughing and fever are nonspecific clinical indicators of a URI. A history of an allergy that is likely to affect the upper respiratory tract is supportive of the sinusitis diagnosis, but is not specific for sinusitis.

The nurse is caring for a 73-year-old patient who underwent a left total knee arthroplasty. On the third postoperative day, the patient complains of shortness of breath, slight chest pain, and that "something is wrong." Temperature is 98.4° F, blood pressure 130/88, respirations 36/minute, and oxygen saturation 91% on room air. What action should the nurse take first? 1 Notify the health care provider 2 Administer a nitroglycerin tablet sublingually 3 Conduct a thorough assessment of the chest pain 4 Sit the patient up in bed as tolerated and apply oxygen

4 RATIONALE: The patient's clinical picture is most likely pulmonary embolus , and the first action the nurse should take is to assist with the patient's respirations. For this reason, the nurse should sit the patient up as tolerated and apply oxygen before notifying the health care provider. The nitroglycerin tablet would not be helpful and the oxygenation status is a bigger problem than the slight chest pain at this time.

The nurse is assisting the health-care provider (HCP) perform a diagnostic thoracentesis on a patient. The nurse positions the patient in what position? 1 Lying flat in the fetal position on the unaffected side 2 Sitting in bed with knees slightly flexed and feet flexed 3 Lying flat on the unaffected side with knees slightly flexed 4 Sitting upright with elbows on an over bed table and feet supported

4 RATIONALE: To appropriately locate the pleural space, the patient needs to be positioned sitting upright with elbows on an over bed table, with feet supported. Lying flat would not adequately expand the thorax and permit the provider to position the thoracentesis needle in the correct place. Sitting in bed would also not allow the HCP to visualize the pleural space for needle insertion.

A patient with a new laryngectomy is considering different methods of voice restoration. The nurse knows that which of these offers the best speech quality and patient satisfaction? 1. Esophageal speech 2. Intraoral electrolarynx 3. Neck type electrolarynx 4. Transesophageal puncture

4 RATIONALE: Transesophageal puncture, the most common voice rehabilitation method, offers the best speech quality with the highest patient satisfaction. Esophageal speech is used, but very few develop fluent speech. Neck type electrolarynx and intraoral electrolarynx are easy to use and common, but have a mechanical-sounding speech.

The nurse is completing tracheostomy care. Which of these is the best method for ensuring the fit of tracheostomy ties? 1 Have the respiratory therapist check the ties. 2 Ask the patient if the ties feel comfortable after tying them. 3 Place one finger underneath the ties to ensure they are not too tight around the neck. 4 Place two fingers underneath the ties to ensure they are not too tight around the neck.

4 RATIONALE: When securing tracheostomy ties , place two fingers underneath the ties to ensure that they are not too tight around the patient's neck. The respiratory therapist may not be trained in changing the ties, or may not check them accurately. The patient may not be able to identify if the ties are too tight. One finger beneath the tie is too tight.

When the nurse is explaining treatment to the families, for which patient would noninvasive positive pressure ventilation (NIPPV) be an appropriate intervention to promote oxygenation? 1. A patient whose cardiac output and blood pressure are unstable 2. A patient whose respiratory failure is because of a head injury with loss of consciousness 3. A patient with a diagnosis of cystic fibrosis who currently is producing copious secretions 4. A patient who is experiencing respiratory failure as a result of the progression of myasthenia gravis

4 RATIONALE: NIPPV is most effective in treating patients with respiratory failure resulting from chest wall and neuromuscular disease. It is not recommended in patients who are experiencing hemodynamic instability, decreased level of consciousness, or excessive secretions.

The nurse determines that the patient has understood medication instructions about the use of a metered dose inhaler (MDI) when the patient performs which action? 1. Inhales rapidly when activating the inhaler 2. Breathes through the nose with activation of the MDI. 3.Holds the MDI sideways to increase ease of use. 4.Waits one minute between each puff from the MDI.

4 RATIONALE: The patient should wait at least one minute in between puffs to increase medication dispersion throughout the lungs. The patient should inhale slowly, hold the MDI upright, and breathe through the mouth. STUDY TIP: Record the information you find to be most difficult to remember on 3" × 5" cards and carry them with you in your pocket or purse. When you are waiting in traffic or for an appointment, just pull out the cards and review again. This "found" time may add points to your test scores that you have lost in the past.

The nurse provides preprocedure teaching for a patient who is scheduled for bedside thoracentesis. What does the nurse explain is the primary purpose of thoracentesis? 1. Determining the stage of a lung tumor 2. Directly inspecting and examining the pleural space 3. Obtaining a specimen of pleural tissue for evaluation 4. Relieving an abnormal accumulation of fluid in the pleural space

4 RATIONALE: Thoracentesis involves the insertion of a large-bore needle into the pleural space to relieve an abnormal accumulation of fluid in the pleural space. The procedure can significantly relieve symptoms related to this fluid accumulation, such as shortness of breath and discomfort. Thoracentesis cannot reveal the stage of lung cancer or permit direct inspection and examination of the pleural space. It may provide a pleural fluid specimen but not a pleural tissue specimen

The nurse is teaching a patient how to self-administer ipratropium (Atrovent) via a metered dose inhaler (MDI). Which instruction given by the nurse is most appropriate to help the patient learn the proper inhalation technique? 1 "Avoid shaking the inhaler before use." 2 "Breathe out slowly before positioning the inhaler." 3 "Using a spacer should be avoided for this type of medication." 4 "After taking a puff, hold the breath for 30 seconds before exhaling."

4 RATIONALE: It is important to breathe out slowly before positioning the inhaler. This allows the patient to take a deeper breath while inhaling the medication, thus enhancing the effectiveness of the dose. The inhaler should be shaken well. A spacer may be used. Holding the breath after the inhalation of medication helps keep the medication in the lungs, but 30 seconds will not be possible for a patient with COPD. Text Reference - p. 573

Which statement made by the patient with chronic obstructive pulmonary disease (COPD) indicates a need for further teaching regarding the use of an ipratropium inhaler? 1 "I can rinse my mouth following the two puffs to get rid of the bad taste." 2 "I should wait at least one to two minutes between each puff of the inhaler." 3 "Because this medication is not fast acting, I cannot use it in an emergency if my breathing gets worse." 4 "If my breathing gets worse, I should keep taking extra puffs of the inhaler until I can breathe more easily."

4 The patient should not just keep taking extra puffs of the inhaler to make breathing easier. Excessive treatment could trigger paradoxical bronchospasm, which would worsen the patient's respiratory status. Rinsing the mouth after the puffs will eliminate a bad taste. Waiting one to two minutes between each puff will facilitate the effectiveness of the administration. Ipratropium is not used in an emergency for COPD. Text Reference - p. 589

When administering and reading the tuberculosis (TB) skin test, what measures should the nurse take? 1. Ensure that the injection is given subcutaneously. 2. Do not use a pen around the test area to mark the site 3. Include the reddened flat areas on the skin when measuring the induration. 4. Draw a diagram of the forearm and hand and label the injection sites in the patient's chart.

4 RATIONALE: When a skin test is administered for TB bacilli, the nurse should chart the site of administration by drawing a diagram of the forearm and hand and labeling the injection sites. The nurse should ensure that the injection is given intradermally. The nurse should circle the area with a pen and instruct the patient not to remove the mark. The diameter of the induration should be measured for reading the test. The reddened flat area is not included in the measurement.

A patient is diagnosed with allergy to molds. A nurse is educating the patient on management of rhinitis due to mold allergy. What measures would be helpful to the patient? Select all that apply. 1 Remove pets from the house. 2 Drape the windows well to limit light. 3 Keep plants inside the house in large numbers. 4 Keep closets and basements well lit. 5 Ensure good ventilation to allow ample airflow in the house.

4,5 RATIONALE: Darkness, dampness, and drafts promote the growth of mold spores. Keeping lights on in closets and basement would inhibit the growth of molds. Good ventilation allows for more airflow in the house which reduces humidity and development of molds. Removing pets is not an appropriate suggestion as pets do not carry mold. Draping windows will decrease light in the house and having plants inside the house will increase the humidity, both of which would promote mold growth.

An elderly patient is in end-stage chronic lung disease due to chronic obstructive pulmonary disease (COPD). What nursing interventions are appropriate for this patient? Select all that apply. 1 Encourage patient to perform chest exercises. 2 Elevate the foot of the bed. 3 Use benzodiazepines to reduce anxiety. 4 Immediately report any change in mental status. 5 Monitor specific and nonspecific signs of respiratory failure.

4,5 While caring for patients with end-stage chronic lung disease, the nurse must immediately report any change in mental status such as agitation or confusion. This may indicate the onset of rapid deterioration in clinical status and the need for mechanical ventilation. It is especially important to monitor specific and nonspecific signs of respiratory failure in patients with COPD because a small change can cause significant decomposition. Chest exercises are needed only for rehabilitation once the crisis has passed. Elevation of the foot of the bed increases respiratory distress; instead, head end elevation is useful. Although benzodiazepines reduce anxiety, they suppress the respiratory center.

The nurse cares for a patient who has just had a thoracentesis. Which assessment information obtained by the nurse is a priority to communicate to the health care provider? A. Oxygen saturation is 88%. B. Blood pressure is 145/90 mm Hg. C. Respiratory rate is 22 breaths/minute when lying flat. D. Pain level is 5 (on 0 to 10 scale) with a deep breath.

A. Oxygen saturation is 88%.

30. A nurse is planning the care of a client with bronchiectasis. What goal of care should the nurse prioritize? A) The patient will successfully mobilize pulmonary secretions. B) The patient will maintain an oxygen saturation level of 98%. C) The patients pulmonary blood pressure will decrease to within reference ranges. D) The patient will resume prediagnosis level of function within 72 hours.

A Feedback: Nursing management focuses on alleviating symptoms and helping patients clear pulmonary secretions. Pulmonary pressures are not a central focus in the care of the patient with bronchiectasis. Rapid resumption of prediagnosis function and oxygen saturation above 98% are unrealistic goals.

6. A nurse is evaluating the diagnostic study data of a patient with suspected cystic fibrosis (CF). Which of the following test results is associated with a diagnosis of cystic fibrosis? A) Elevated sweat chloride concentration B) Presence of protein in the urine C) Positive phenylketonuria D) Malignancy on lung biopsy

A Feedback: Gene mutations affect transport of chloride ions, leading to CF, which is characterized by thick, viscous secretions in the lungs, pancreas, liver, intestine, and reproductive tract as well as increased salt content in sweat gland secretions. Proteinuria, positive phenylketonuria, and malignancy are not diagnostic for CF.

10. A nurse is caring for a 6-year-old patient with cystic fibrosis. In order to enhance the childs nutritional status, what intervention should most likely be included in the plan of care? A) Pancreatic enzyme supplementation with meals B) Provision of five to six small meals per day rather than three larger meals C) Total parenteral nutrition (TPN) D) Magnesium, thiamine, and iron supplementation

A Feedback: Nearly 90% of patients with CF have pancreatic exocrine insufficiency and require oral pancreatic enzyme supplementation with meals. Frequent, small meals or TPN are not normally indicated. Vitamin supplements are required, but specific replacement of magnesium, thiamine, and iron is not typical.

40. A nurse is admitting a new patient who has been admitted with a diagnosis of COPD exacerbation. How can the nurse best help the patient achieve the goal of maintaining effective oxygenation? A) Teach the patient strategies for promoting diaphragmatic breathing. B) Administer supplementary oxygen by simple face mask. C) Teach the patient to perform airway suctioning. D) Assist the patient in developing an appropriate exercise program.

A Feedback: The breathing pattern of most people with COPD is shallow, rapid, and inefficient; the more severe the disease, the more inefficient the breathing pattern. With practice, this type of upper chest breathing can be changed to diaphragmatic breathing, which reduces the respiratory rate, increases alveolar ventilation, and sometimes helps expel as much air as possible during expiration. Suctioning is not normally necessary in patients with COPD. Supplementary oxygen is not normally delivered by simple face mask and exercise may or may not be appropriate.

5. A patient with emphysema is experiencing shortness of breath. To relieve this patients symptoms, the nurse should assist her into what position? A) Sitting upright, leaning forward slightly B) Low Fowlers, with the neck slightly hyperextended C) Prone D) Trendelenburg

A Feedback: The typical posture of a person with COPD is to lean forward and use the accessory muscles of respiration to breathe. Low Fowlers positioning would be less likely to aid oxygenation. Prone or Trendelenburg positioning would exacerbate shortness of breath.

The nurse in the cardiac care unit is caring for a patient who has developed acute respiratory failure. The nurse knows that which medication is being used to decrease this patient's pulmonary congestion and agitation? A. Morphine sulfate B. Albuterol (Ventolin) C. Azithromycin (Zithromax) D. Methylprednisolone (Solu-Medrol)

A RATIONALE: For a patient with acute respiratory failure related to the heart, morphine is used to decrease pulmonary congestion as well as anxiety, agitation, and pain. Albuterol is used to reduce bronchospasm. Azithromycin is used for pulmonary infections. Methylprednisolone is used to reduce airway inflammation and edema. TEST-TAKING TIP: Read every word of each question and option before responding to the item. Glossing over the questions just to get through the examination quickly can cause you to misread or misinterpret the real intent of the question.

D. Oral rehydration therapy with a solution containing glucose and electrolytes

A 25-year-old student has been taken to an urgent care clinic because of dehydration. She says she has had "the flu," with vomiting and diarrhea "all night" and has had very little to eat or drink. She says the GI symptoms have subsided, but she feels weak. The nurse expects which type of rehydration to occur? A. IV fluid replacement B. Oral rehydration therapy with tea C. Oral rehydration therapy with water D. Oral rehydration therapy with a solution containing glucose and electrolytes

The nurse is assigned the care of a patient with a chest tube. The nurse should ensure that which of the following items is kept at the patient's bedside? a) An incentive spirometer b) A bottle of sterile water c) A set of hemostats d) An Ambu bag

A bottle of sterile water It is essential that the nurse ensure that a bottle of sterile water is readily available at the patient's bedside. If the chest tube and drainage system become disconnected, air can enter the pleural space, producing a pneumothorax. To prevent the development of a pneumothorax, a temporary water seal can be established by immersing the chest tube's open end in a bottle of sterile water. There is no need to have an Ambu bag, incentive spirometer, or a set of hemostats at the bedside.

A patient is admitted to an emergency department with injuries of the face and nose. A nurse notices a clear, pink-tinged discharge from the nostrils of the patient, even after controlling the nasal bleed. What could be the cause of the discharge?

A clear and pink-tinged discharge from the nose even after control of nasal bleeding suggests a cerebrospinal fluid (CSF) leak. It is an emergency situation and can lead to life-threatening complications. Skull fracture is manifested as ecchymosis of the eyes. There is no clear discharge in the event of a septal deviation or epistaxis.

a. Airborne Rationale: Tuberculosis is a respiratory infection that is spread through the air and requires airborne isolation.

A client diagnosed with active tuberculosis is hospitalized. Which of the following isolation precautions should the nurse plan to implement? a. Airborne b. Neutopenic c. Contact d. Droplet

c. Causes orange discoloration of seat, tears, urine, and feces Rationale: Rifampin should be taken exactly as directed. Doses should not be doubled or skipped. The client should not stop therapy until directed to do so by a physician. The medication should be administered on an empty stomach unless it causes gastrointestinal upset, and hen it may be taken with food. Antacids, if prescribed, should be taken at least 1 hour before the medication. Rifampin causes orange-red discoloration of body secretions and will stain soft contact lenses permanently.

A client has been started on long-term therapy with rifampin (Rifadin). A nurse teaches the client that the medication a. Should always be taken with food or antacids b. Should be double-dosed if one dose is forgotten c. Causes orange discloration of sweat, tears, urine, and feces d. May be discontinued independently if symptms are gone in 3 months

b. Peripheral neuritis Rationale: Isoniazid (INH) is an antitubercular medication. A common side effect of isoniazid is peripheral neuritis, manifested by numbness, tingling, and paresthesias in the extremities. This can be minimized with pyridoxine (vitamin B6) intake. Options a, c, & d are incorrect.

A client has been taking isoniazid (INH) for 1 1/2 months. The client complains to a nurse about numbness, paresthesias, and tingling in the extremities. The nurse interprets that the client is experiencing: a. Hypercalcemia b. Peripheral neuritis c. Small blood vessel spasm d. Impaired peripheral circulation

a. Coffee,cola, and chocolate Rationale: theophylline (Theo-24) is a methylxanthine bronchodilator. The nurse teaches the client to limit the intake of xanthine-containing foods while taking this medication. These foods include coffee, cola, and chocolate

A client has begun therapy with theophylline (Theo-24). A nurse plans to teach the client to limit the intake of which of the following while taking this medication? a. Coffee, cola, and chocolate b. Oysters, lobster, and shrimp c. Melons, oranges, and pineapple d. Cottage cheese, cream cheese, and dairy creamers

c. Chest pain that occurs suddenly Rationale: the most common initial symptom in pulmonary embolism is chest pain that is sudden in onset. The next most commonly reported symptom is dyspnea, which is accompanied by an increased respiratory rate. Other typical symptoms of pulmonary embolism include apprehension and restlessness, tachycardia, cough, and cyanosis.

A client has experienced pulmonary embolism. A nurse assesses for which symptom, which is most commonly reported? a. Hot, flushed feeling b. Sudden chills and fever c. Chest pain that occurs suddenly d. Dyspnea when deep breaths are taken

c. Arterial blood gases Rationale: Arterial blood gases will provide important information regarding serum oxygen saturation and the acid-base balance of the client's blood. This information will evaluate if the mechanical ventilator is providing adequate oxygenation to maintain lung function.

A client in acute respiratory failure is receiving mechanical ventilation. Which of the following is the priority assessment the nurse should use to evaluate the effectiveness of the mechanical ventilation? a. Blood pressure b. Breath sounds c. Arterial blood gases d. Heart rate

b. Report yellow eyes or skin immediately Rationale: Isniazid (INH) is hepatotoxic, and therefore the client is taught to report signs and symptoms of hepatitis immediately, which include yellow skin and sclera. For the same reason, alcohol should be avoided during therapy. The client should avoid intake of swiss cheese, fish such as tuna, and foods containing tyramine because they may cause a reaction characterized by redness and itching of the skin, flushing, sweating, tachycardia, headache, or lightheadedness. The client can avoid developing peripheral neuritis by increasing the intake of pyridoxine (vitamin B6) during the course of isoniazid therapy.

A client is to begin a 6-month course of therapy with isoniazid (INH). A nurse plans to teach the client to: a. Use alcohol in small amounts only b. Report yellow eyes or skin immediately c. Increas intak of swiss or aged cheeses d. Avoid vitamin supplements during therapy

a. Positive Rationale: The client with human immunodeficiency virus (HIV) infection is considered to have positive results on Mantoux skin testing with an area larger than 5 mm of induration. The client without HIV is positive with an induration larger than 10 mm. The client with HIV is immunosuppressed, making a smaller area of induration positive for this type of client. It is possible for the client infected with HIV to have false-negative readings because of the immunosuppression factor. Options b, c, & d are incorrect interpretations.

A client who is humanimmunodeficiency virus-positive has had a Mantoux skin test. The nurse notes a 7-mm area of induration at the site of the skin test. The nurse interprets the results as: a. Positive b. Negative c. Inconclusive d. Indicating the need for repeat testing

a. Apply supplemental oxygen Rationale: According to the airway, breathing, circulation priority-setting framework, the greatest risk to this client is respiratory compromise. The first action would be to apply supplemental oxygen.

A client who is postoperative develops an acute onset of severe chest pain that is worse with inspiration. The client is anxious and tachypneic. The nurse should anticipate taking which of the following actions first? a. Apply supplemental oxygen b. Auscultate lung sounds c. Administer pain medication d. Initiate heparin therapy

d. pH 7.30, PO2 80 mm Hg, PaCO2 55 mm Hg, HCO3 -22 mEq/L Rationale: These ABG values indicate respiratory acidosis. The pH is below 7.35, and the PaCO2 is greater than 45 mm Hg, which indicates respiratory acidosis.

A client who is postoperative is hypoventilating secondary to general anesthesia effects and incisional pain. Which of the following ABG values support the nurse's suspicion of respiratory acidosis? a. pH 7.50, PO2 95 mm Hg, PaCO2 25 mm Hg, HCO3 -22 mEq/L b. pH 7.50, PO2 87 mm Hg, PaCO2 35 mm Hg, HCO3 -30 mEq/L c. pH 7.30, PO2 90 mm Hg, PaCO2 35 mm Hg, HCO3 -20 mEq/L d. pH 7.30, PO2 80 mm Hg, PaCO2 55 mm Hg, HCO3 -22 mEq/L

c. Paradoxical chest movement Rationale: Flail chest results from multiple rib fractures. This results in a "floating" section of the ribs. Because this section is unattached to the rest of the boy rib cage, this segment results in paradoxical chest movement. This means that the force of inspiration pulls the fractured segment inward, while the rest of the chest expands. Similarly, during exhalation, the segment balloons outward while the rest of the chest moves inward. This is a characteristic sign of flail chest.

A client with a chest injury has suffered flail chest. A nurse assesses the client fo which most distinctivesign of flail chest? a. Cyanosis b. Hypotension c. Paradoxical chest movement d. Dyspnea, especially on exhalation

a. Dyspnea Rationale: Histoplasmosis s an opportunistic fungal infection that can occur in the client with acquired immunodeficiency syndrome (AIDS). The infection begins as a respiratory infection and can progress to diseminated infection. Typical signs and symptoms include fever, dyspnea, cough, and weight loss. Enlargement of the client's lymph nodes, liver, and spleen may occur as well.

A client with acquired immunodeficiency syndrome has histoplasmosis. A nurse assesses the client for which of the following signs and symptoms? a. Dyspnea b. Headache c. Weight gain d. Hypothermia

c. Urine andother secretions will be orange in color Rationale: Rifampin will turn the urine and other secretions orange.

A client with pulmonary tuberculosis is being discharged with a prescription for rifampin (Rifadin). The nurse should plan to include which of the following in the client's discharge? a. Ringing in the ears is expected b. Purified protein derivative skin test results will improve in 4 months c. Urine and other secretions will be orange in color d. Medication should be taken with meals

b. Liver enzyme levels Rationale: Isoniazid (INH) therapy can cause an elevation of hepatic enzyme levels and hepatitis. Therefore, liver enzyme levels are monitored when therapy is initiated and during the first 3 months of therapy.They may be monitored longer in the client who is older than 50 or abuses alcohol. The laboratory tests in a, c, & d are not necessary.

A client with tuberculosis is being started on antituberculosis therapy with isoniazid (INH). Before giving the client the first dose, a nurse ensures that which of the following baseline studies has been completed? a. Electrolyte levels b. Liver enzyme levels c. Serum creatinine level d. Coagulation times

A patient has undergone neck dissection surgery and has a skin flap and a closed surgical drain. The nurse has been monitoring drainage hourly on the first postoperative day and notes a decrease in amount from 20-30 mL hourly to less than 5 mL in 1 hour. The nurse suspects which cause for this decrease in drainage?

A clot obstructing the drain A sudden decrease in drainage from a wound drain can indicate a clot obstructing drainage from the wound site. Most drains are sutured in place and it is not likely the drain is dislodged. Edema is not a likely cause. Granulation tissue does not begin to form on the first postoperative day.

central line INCREASED ICP, RESPIRATORY CONDITION (PULMONARY EDEMA)

A code rescue was called for a patient who was hypoxic and had altered mental status. The patient was a "difficult stick" and staff was unable to get IV access. Considering this an emergent situation what access would be most appropriate at this time?Are there any contraindications?

d. A cough with the expectoration of mucoid sputum Rationale: One of the first pulmonary symptoms of tuberculosis is a slight cough with the expectoration of mucoid sputum. Options a, b, & c are late symptoms and signify cavitation and extensive lung involvement.

A community health nurse is conducting an educational session with community members regarding tuberculosis. The nurse tells the group that one of the first symptoms associated with tuberculosis is: a. Dyspnea b. Chest pain c. A bloody, productive cough d. A cough with the expectoration of mucoid sputum

b. Produtive cough with green sputum Rationale: A productive cough with green sputum indicates an infection. This should be reported to the primary care provider.

A nurse is caring for a client who has COPD. Which of the following findings should the nurse report to the primary care provider? a. An oxygen saturation of 89% b. Productive cough with green sputum c. Clubbing of fingers d. Use of pursed-lip breathing with exertion

b. Cyanotic lips Rationale: Cyanosis of the lips indicates that the client is not efficiently oxygenating the blood. This finding should be reported to the primary care provider.

A nurse is assessing a client with emphysema. The nurse should report which of the following client assessment findings? a. Fatigue b. Cyanotic lips c. Barrel-shaped chest d. Crackles in posterior chest

d. Increased respiratory rate Rationale: The earliest detectable sign of acute respiratory distress syndrome is an increased respiratory rae, which can begin from 1 to 96 hours after the initial insult to the body. This is followed by increasing dyspnea, air hunger, retraction of accessory muscles, and cyanosis. Breath sounds may be clear or consist of fine inspiratory crackles or diffuse coarse crackles. a, b, & c would all occur later than an increased respiratory rate.

A nurse is assessing a client with multiple trauma who is at risk for developing acute respiratory distress syndrome. The nurse assesses for which earliest sign of acute respiratory distress syndrome? a. Bilateral wheezing b. Inspiratory crackles c. Intercostal retractions d. Increased respiratory rate

d. Pain, especially with inspiration Rationale: Rib fractures are a common injury, especially in the older client, and result from a blunt injury or a fall. Typical signs and symptoms include pain and tenderness localized at the fracture site and exacerbated by inspiration and palpation, shallow respirations, splinting or guarding the chest protectively to minimize chest movement, and possible bruising at the fracture site. Paradoxical respirations are seen with flail chest.

A nurse is assessing the respiratory status of a client who has suffered a fractured rib. The nurse would expect to note which of the following? a. Slow deep respirations b. Rapid deep respirations c. Paradoxical respirations d. Pain, especially with inspiration

c. Bronchospasm Rationale: If a biopsy was performed during a bronchoscopy, blood-streaked sputum is expected for several hours. Frank blood indicates hemrrhage. A dry cough may be expected. The client should be assessed for signs of complications, which would include cyanosis, dyspnea, stridor, bronchospasm, hemoptysis, hypotension, tachycardia, and dysrhythmias. Hematuria is unrealted to this procedure.

A nurse is caring for a client aftera bronchoscopy and biopsy. Which of the following signs, if noted in the client, should be reported immediately to the physician? a. Dry cough b. Hematuria c. Bronchospasm d. Blood-streaked sputum

c. Temperature of 38.8 C (101.8 F) Rationale: A temperature elevation is expected with bacterial pneumonia

A nurse is caring for a client diagnosed with bacterial pneumonia. In this situation, the nurse should expect which of the following assessment findings? a. Nonproductive cough b. SaO2 96% c. Temperature of 38.8 C (101.8 F) d. Bradypnea

c. Pair of padded clamps Rationale: The nurse should plan to have a pair of padded clamps in the event that the tubing becomes disconnected.

A nurse is caring for a client following the insertion of a chest tube. The nurse should plan to have which of the following items in the client's room? a. Extra drainage system b. Suture removal set c. Pair of padded clamps d. Nonadherent pads

b. Hyperinflated chest noted on the chest x-ray Rationale: Clinical manifestations of COPD include hypoxemia, hypercapnia, dyspnea on exertion and at rest, oxygen desaturation with exercise, and the use of accessory muscles of respiration. Chest x-rays reveal a hyperinflated chest and a flattened diaphragm if the disease is advanced. Eliminate "a" because hypercapnia would be noted in a person with COPD. Eliminate "c" because oxygen desaturation would occur. Eliminate "d" because it would be a flattened diaphragm that would be noted.

A nurse is caring for a client hospitalized with acute exacerbation of chronic obstructive pulmonary disease. Which of the ollowing would the nurse expect to note on assessment of this client? a. Hypocapnia b. A hyperinflated chest noted on the chest x-ray c. Increased oxygen saturation with exercise d. A widened diaphragm noted n the chest x-ray

b. Nonrebreather mask Rationale: A nonrebreather mask is made up of a reservoir bag from which the client obtains the oxygen, a one-way valve to prevent exhaled air from entering the reservoir bag, and exhalation ports with flaps that prevent room air from entering the mask. This delivers greater than 90% FIO 2 (Fraction of inspired oxygen), which provides the highest level of oxygen.

A nurse is caring for a client in respiratory distress. Which of the following devices should the nurse use to provide the highest level of oxygen via a low-flow system? a. Nasal cannula b. Nonrebreather mask c. Simple face mask d. Partial rebreather mask

d. Tracheal deviation to the unaffected side Rationale: A pneumothorax will cause the trachea to deviate to the unaffected side.

A nurse is caring for a client in the emergency department following chest trauma. Which of the following findings should the nurse recognize as indicating a tension pneumothorax? a. Collapsed neck veins on the affected side b. collapsed neck veins on the unaffected side c. Tracheal deviation to the affected side d. Tracheal deviation to the unaffected side

c. Artificial airway cuff leak Rationale: An artificial airway cuff leak interferes with oxygenation and causes the low pressure alarm to sound.

A nurse is caring for a client receiving mechanical ventilation. The low pressure alarm sounds. Which of the following should the nurse recognize as a cause of the alarm? a. Excess secretions b. Kinks in the tubing c. Artificial airway cuff leak d. Biting on the endotracheal tube

d. Administer IV heparin at 1,300 units per hr. Rationale: A client with a pulmonary embolism is at greatest risk for respiratory arrest related to extension of the clot. Administration of heparin will prevent further clot formation; therefore this is the highest priority.

A nurse is caring for a client who was just diagnosed with a pulmonary embolism. Which of the following interventions is the highest priority? a. Provide for a quiet environment b. Encourage use of incentive spirometer every 1 to 2 hr. c. Apply electrodes for continuous cardiac monitoring d. Administer IV heparin at 1,300 units per hr.

a. Increase in respiratory rate Rationale: An increase in respiratory rate indicates increased work of breathing and the need for improvement in oxygen delivery.

A nurse is caring for a client with adult respiratory distress syndrome (ARDS). Which of the following assessment findings indicates that the client's work of breathing has worsened? a. Increase in respiratory rate b. Increase in oxygen saturation c. Decrease in carbon dioxide retention d. Decrease in adventitious breath sounds

d. Shortnes of breath Rationale: Dry cough and dyspnea are typical signs and symptoms of pulmonary sarcoidosis. Others include night sweats, fever, weight loss, and skin nodules. The shortness of breath appears earlier than other symptoms.

A nurse is giving discharge instructions to a client with pulmonary sarcoidosis. The nurse concludes that the client understands the information if the client reports which of the following early signs of exacerbation? a. Fever b. Fatigue c. Weight loss d. Shortness of breath

d. Sitting on the side of the bed and leaning on the overbed table Rationale: Positions that will assist the client with emphysema with breathing include sitting up and leaning on an overbed table, sitting up and resting the elbows on the knees, and standing and leaning against a wall.

A nurse is instructing a hospitalized client with a diagnosis of emphysema about measures that will enhance the effectiveness of breathing during dyspneic periods. Which of the following positions will the nurse instruct the client to assume? a. Sitting up in bed b. Side-lying in bed c. Sitting in a recliner d. Sitting on the side of the bed and leaning on the overbed table

a. Holding at the puncture site for 5 min b. Aspirating the specimen into a heparinized syringe e. Performing the Allen test prior to obtaining the specimen Rationale: Holding pressure at the puncture site for 5 minutes is necessary to ensure adequate clotting and prevent bleeding. Aspirating the specimen into a heparinized syringe is required for an accurate sampling result. Performing the Allen test prior to obtaining the specimen prevents use of an artery that has insufficient blood flow, which can damage the hand. Inserting an air bubble into the syringe is not correct because it will not provide accurate results. Transporting the specimen to the lab within 30 min is not correct because it must be transported immediately.

A nurse is obtaining a blood sample for ABG determination from a client's radial artery. Which of the following interventions are correct when performing this procedure? (Select all that apply) a. Holding pressure at the puncture site for 5 min. b. Aspirating the specimen into a heparinized syringe c. Inserting an air bubble into the syringe before capping d. Transporting the specimen to the laboratory within 30 min e. Performing the Allen test prior to obtaining the specimen

b High-Fowler's poson with pillows supporting both arms Rationale: The client should be encouraged to sit upright leaning slightly forward with both arms supported on the overbed table to allow for better expansion of the chest.

A nurse is positioning a client with emphysema to promote effective breathing. The nurse should place the client in which of the following positions? a. Lateral position with a pillow over the chest to support the arm b. High-Fowler's position with arms supported on the overbed table c. Semi-Fowler's position with pillows supporting both arms d. Supine position with the head of the bed elevated 15 degrees

a. Assessing gag reflex Rationale: Using the safety and risk reduction priority-setting framework, the greatest risk to the client is aspiration due to a depressed gag reflex.

A nurse is preparing a client for discharge following a bronchoscopy. Which of the following is the priority assessment? a. Assessing gag reflex b. Percussing lung fields c. Auscultating heart sounds d. Palpating peripheral pulses

d. "I will take this medication every evening, even when I do not have symptoms." Rationale: Montelukast is used for prophylaxis of asthma exacerbation and should be taken on a daily basis in the evening.

A nurse is providing instruction to a client on how to use montelukast (Singulair) to treat chronic asthma. The nurse should recognize that the client understands the teaching when he states, a. "I will take this medication with each meal." b. "I will take this medication during my asthma attacks." c. " I will take this medication up to three times a day when I begin to wheeze." d. " I will take this medication every evening, even when I do not have symptoms."

c. Stop the procedure and reoxygenate the client Rationale: During suctioning, the nurse should monitor the client closely for side effects, including hypoxemia, cardiac irregularities such as a decrease in heart rate resulting from vgal stimulation, mucosal trauma, hypotension, and paroxysmal coughing. If side effects develop, especially cardiac irregularities, the procedure is stopped and the client is reoxygenated.

A nurse is suctioning fluids from a client through an endotracheal tube. During the suctioning procedure, the nurse notes on the monitor that the heart rate is decreasing. Which of the following is the appropriate nursing intervention? a. Continue to suction b. Notify the physician immediately c. Stop the procedure and reoxygenate the client d. Ensure that the suction is limited to 15 seconds

c. 10 seconds Rationale: Hypoxemia can be caused by prolonged suctioning, which stimulates the pacemaker cells in the heart. A vasovagal response may occur, causing bradycardia. The nurse must preoxygenate the client before suctioning and limit the suctioning pass to 10 seconds.

A nurse is suctioning fluids from a client via a tracheostomy tube. When suctioning, the nurse must limit the suctioning time to a maximum of: a. 1 minute b. 5 seconds c. 10 seconds d. 30 seconds

a. Mask Rationale: Silicosis results from chronic, excessive inhalation of particles of free crystalline silica dust. The client should wear a mask to limit inhalation of this substance, which can cause restrictive lung disease after years of exposure. Options b, c, & d are not necessary.

A nurse is taking the history of a client with silicosis. The nurse assesses whether the client wears which of the following items during periods of exposure to silica particles? a. Mask b. Gown c. Gloves d. Eye protection

c. Sputum culture Rationale: Tuberculosis is definitively diagnosed through culture and isolation of Mycobacterium tuberculosis. A presumptive diagnosis is made based on a tuberculin skin test, a sputum smear that is positive for acid-fast bacteria, a chest x-ray, and istological evidence of granulomatous disease on biopsy.

A nurse perform an admission assessment on a client with a diagnosis of tuberculosis. The nurse reviews the results of which diagnostic test that will confirm this diagnosis? a. Chest x-ray b. Bronchoscopy c. Sputum culture d. Tuberculin skin test

a. A client admitted with a closed-head injury who is lethargic Rationale: Increased intracranial pressure often follows a closed-head injury. Nasopharyngeal suctioning can further increase intracranial pressure and should be avoided.

A nurse receives an order from the primary care provider to perform nasopharyngeal suctioning for each of the following clients. The nurse should question the provider's order for which of the following clients? a. A client admitted with a closed-head injury who is lethargic b. A client admitted with a fractured femur who is in severe pain c. A client admitted with a ruptured appendix who has a temperature of 39 C (102.2 F) d. A client admitted with emphysema who has a respiratory rate of 36/min

b. Albuterol (Proventil) Rationale: Albuterol is a short-acting beta 2 adrenergic agonist, which acts quickly to produce bronchodilation during an acute asthma attack.

A nurse should plan to administer which of the following medications to a client during an acute asthma attack? a. Cromolyn sodium (Intal) b. Albuterol (Proventil) c. Fluticasone and salmeterol (Advair Discus) d. Prednisone (Deltasone)

c. Tachycardia Rationale: Tachycardia is a common side effect, especially if albuterol is used excessively.

A nurse should plan to monitor a client receiving albuterol (Proventil) for which of the following side effects? a. Hypokalemia b. Dyspnea c. Tachycardia d. Candidiasis

a. Total hip arthroplasty Rationale: The surgical procedure and decreased mobility of the affected extremity will place the client at risk for development of a pulmonary embolism.

A nurse should recognize that which of the following factors places a client at risk for a pulmonary embolus? a. Total hip arthroplasty b. Peripheral IV catheter c. Indwelling urinary catheter d. Laparoscopic cholecystectomy

b. Remove the suction catheter using a rotating motion Rationale: Rotating the suction catheter during withdrawal reduces the risk of tissue trauma.

A nurse should take which of the following action when providing endotracheal suctioning for a client who is in respiratory distress? a. Suction the client's endotracheal tube using clean technique b. Remove the suction catheter using a rotating motion c. Suction the oropharyngeal cavity prior to suctioning the endotracheal tube d. Suction the client's endotracheal tube every 2 hr.

a. Take the medication on an empty stomach Rationale: Diphenhydramine (Benadryl) has several uses, including antihistamine, antitussive, antidyskinetic, and sedative-hypnotic. Instructions for use include taking with food or milk to decrease gastrointestinal upset and using oral rinses or sugarless gum or hard candy to minimize dry mouth. Because the medication causes drowsiness, the client should avoid use of alcohol or central nervous system depressants, operating a car, or engaging in other activities requiring mental awareness during use.

A nurse teaches a client about the effects of diphenhydramine (Benadryl), which has been prescribed as a cough suppressant. The nurse determines that the client needs further instructions if the client states that he or she will: a. Take the medication on an empty stomach b. Avoid using alcohol while taking this medication c. Use sugarless gum, candy, or oral rinses to decrease dry mouth d. Avoid activities requiring mental alertness while taking this medication

A patient is to receive an oxygen concentration of 70%. What is the best way for the nurse to deliver this concentration? a) A Venturi mask b) An oropharyngeal catheter c) A partial rebreathing mask d) A nasal cannula

A partial rebreathing mask Partial rebreathing masks have a reservoir bag that must remain inflated during both inspiration and expiration. The nurse adjusts the oxygen flow to ensure that the bag does not collapse during inhalation. A high concentration of oxygen (50% to 75%) can be delivered because both the mask and the bag serve as reservoirs for oxygen. The other devices listed cannot deliver oxygen at such a high concentration.

A patient has been involved in a motor vehicle crash and has pain, swelling, and epistaxis. What is the priority nursing action in the care of this patient?

A patient can have edema, bleeding, nasal obstruction, and swelling of the nose due to trauma. Ice packs can promote vasoconstriction and reduce edema and bleeding. The nurse should apply ice packs at intervals of 10 to 20 minutes. The supine position can cause difficulty in breathing, so the nurse places the patient in an upright position to maintain the airway. The nurse will instruct the patient to avoid hot showers for the first 48 hours in order to reduce swelling. Aspirin is administered to reduce facial pain, but it can increase the clotting time and the risk of bleeding. The patient should be instructed to avoid the use of aspirin for the first 48 hours.

YES THEY NEED TO USE THE OTHER ARM, WILL CAUSE IT TO BE FALSE APTT

A patient has a Heparin drip infusing to the Right AC. Lab comes to draw blood for a PTT. The lab tech select the area of the right upper arm. Does the nurse need to intervene in this situation?

B. Hypokalemia

A patient has been having frequent liquid diarrhea for the last 24 hours. A stool sample was sent to the laboratory to confirm possible Clostridium difficile infection. The nurse should monitor the patient for which electrolyte imbalance? A. Dehydration B. Hypokalemia C. Hyponatremia D. Hypocalcemia

Can use midline catheter

A patient is being treated for acute pancreatitis and is NPO. The physician has ordered PPN (Peripheral Parenteral Nutrition). The nurse knows that this type of IV Solution must be administered which route?

Keep area dry, no excessive physical activity, no heavy lifting, cant use any types of crutches PICC lines may have for antibiotics, chemo, vasopressor agents, can be in for months or even years

A patient is being treated for osteomyelitis. Zosyn has been ordered for a 4 week period. The patient is concerned about his ADL's and asks the nurse what kinds of activities to avoid. What should nurse reply?

A patient with a radical neck dissection is refusing enteral feeding. Which factor should the nurse monitor in this patient?

A patient who has a radical neck dissection will have difficulty eating; enteral feeding is used to provide the required nutrients. If the patient refuses to take enteral feeding, he or she may have an inadequate supply of nutrients, which will result in weight loss. The nurse should closely monitor weight loss in any patient who refuses enteral feeding. Pulse rate, blood pressure, and respiratory rate are not affected if the patient refuses to take enteral feeding.

Reattempt correct needle placement. Should see blood return.

A patient will be receiving chemotherapy through a right chest port. The nurse accesses the port and checks for blood return. The nurse does not get blood return. What does the nurse do?

Which action should the nurse take first when a patient develops a nosebleed? A. Pinch the lower portion of the nose for 10 minutes. B. Pack the affected nare tightly with an epistaxis balloon. C. Obtain silver nitrate that will be needed for cauterization. D. Apply ice compresses over the patient's nose and cheeks.

A. Pinch the lower portion of the nose for 10 minutes.

The nurse is teaching a hospitalized client who is being discharged about how to care for a PICC line. Which client statement indicates a need for further education? A. "I can continue my 20-mile running schedule as I have for the past 10 years." B, "I can still go about my normal activities of daily living." C. "I have less chance of getting an infection because the line is not in my hand." D. "The PICC line can stay in for months."

A. "I can continue my 20-mile running schedule as I have for the past 10 years." Excessive physical activity can dislodge the PICC and should be avoided. Clients with PICCs should be able to perform normal activities of daily living. PICCs have low complication rates because the insertion site is in the upper extremity. The dry skin of the arm has fewer types and numbers of microorganisms, leading to lower rates of infection. PICC lines can be used long term (months).

The nurse completes discharge instructions for a patient with a total laryngectomy. Which statement by the patient indicates that additional instruction is needed? A. "I must keep the stoma covered with an occlusive dressing at all times." B. "I can participate in most of my prior fitness activities except swimming." C. "I should wear a Medic-Alert bracelet that identifies me as a neck breather." D. "I need to be sure that I have smoke and carbon monoxide detectors installed."

A. "I must keep the stoma covered with an occlusive dressing at all times."

Which statement by the patient indicates that the teaching has been effective for a patient scheduled for radiation therapy of the larynx? A. "I will need to buy a water bottle to carry with me." B. "I should not use any lotions on my neck and throat." C. "Until the radiation is complete, I may have diarrhea." D. "Alcohol-based mouthwashes will help clean oral ulcers."

A. "I will need to buy a water bottle to carry with me."

What would a nurse instruct a patient to do after administration of a sublingual medication? Select one: A. "Try not to swallow while the pill dissolves." B. "Chew the pill so it will dissolve faster." C. "Swallow frequently to get the best benefit." D. "Take a big drink of water and swallow the pill."

A. "Try not to swallow while the pill dissolves." Sublingual and buccal medications should not be swallowed but rather held in place so that complete absorption takes place.

The nurse is caring for a 65-year-old patient who has previously been diagnosed with hypertension. Which of the following blood pressure readings represents the threshold between high-normal blood pressure and hypertension? Select one: A. 140/90 mm Hg B. 160/100 mm Hg C. 145/95 mm Hg D. 150/100 mm Hg

A. 140/90 mm Hg Hypertension is the diagnosis given when the blood pressure is greater than 140/90 mm Hg. This makes the other options incorrect.

The nurse in the emergency department receives arterial blood gas results for four recently admitted patients with obstructive pulmonary disease. Which patient will require the most rapid action by the nurse? A. 22-year-old with ABG results: pH 7.28, PaCO2 60 mm Hg, and PaO2 58 mm Hg B. 34-year-old with ABG results: pH 7.48, PaCO2 30 mm Hg, and PaO2 65 mm Hg C. 45-year-old with ABG results: pH 7.34, PaCO2 33 mm Hg, and PaO2 80 mm Hg D. 65-year-old with ABG results: pH 7.31, PaCO2 58 mm Hg, and PaO2 64 mm Hg

A. 22-year-old with ABG results: pH 7.28, PaCO2 60 mm Hg, and PaO2 58 mm Hg

Which patient in the ear, nose, and throat (ENT) clinic should the nurse assess first? A. A 23-year-old who is complaining of a sore throat and has a muffled voice B. A 34-year-old who has a "scratchy throat" and a positive rapid strep antigen test C. A 55-year-old who is receiving radiation for throat cancer and has severe fatigue D. A 72-year-old with a history of a total laryngectomy whose stoma is red and inflamed

A. A 23-year-old who is complaining of a sore throat and has a muffled voice

The nurse is reviewing the medical records for five patients who are scheduled for their yearly physical examinations in September. Which patients should receive the inactivated influenza vaccination (select all that apply)? A. A 76-year-old nursing home resident B. A 36-year-old female patient who is pregnant C. A 42-year-old patient who has a 15 pack-year smoking history D. A 30-year-old patient who takes corticosteroids for rheumatoid arthritis E. A 24-year-old patient who has allergies to penicillin and cephalosporins

A. A 76-year-old nursing home resident B. A 36-year-old female patient who is pregnant D. A 30-year-old patient who takes corticosteroids for rheumatoid arthritis

You are the nurse planning an educational event for the nurses on a subacute medical unit on the topic of normal, age-related physiological changes. What phenomenon would you include in your teaching plan? Select one: A. A decrease in muscle mass and bone density B. A decrease in cognition, judgment, and memory C. The disappearance of sexual desire for both men and women D. An increase in sebaceous and sweat gland function in both men and women

A. A decrease in muscle mass and bone density Normal signs of aging include a decrease in the sense of smell, a decrease in muscle mass, a decline but not disappearance of sexual desire, and decreased sebaceous and sweat glands for both men and women. Cognitive changes are usually attributable to pathologic processes, not healthy aging.

A design firm is contracted to remodel a care facility. Which bathroom design component is most conducive to safety and quality of life for the older adult residents who will use them? Select one: A. A small independent light to remain lit in the bathroom at all times B. Bathrooms will include bathtubs rather than showers C. A single, rotating faucet installed at the sink to control water flow and temperature D. Throw rugs will be placed on the tile floors

A. A small independent light to remain lit in the bathroom at all times A small light that remains lit in a bathroom promotes safety. It is not necessary to exclude showers from all residents' rooms and clearly marked, separate hot and cold faucets should be used. Throw rugs constitute a fall risk.

The nurse reviews the medication administration record (MAR) for a patient having an acute asthma attack. Which medication should the nurse administer first? A. Albuterol (Ventolin) 2.5 mg per nebulizer B. Methylprednisolone (Solu-Medrol) 60 mg IV C. Salmeterol (Serevent) 50 mcg per dry-powder inhaler (DPI) D. Triamcinolone (Azmacort) 2 puffs per metered-dose inhaler (MDI)

A. Albuterol (Ventolin) 2.5 mg per nebulizer

Which statement should be incorporated into the restraint policy for residents of a long-term care facility? Select one: A. Alternatives should be explored before chemical and physical restraints are utilized. B. Physical restraints should only be used with verifiably agitated patients. C. Restraints should never be used. D. Restraints should only be used when one-to-one staff supervision is not possible.

A. Alternatives should be explored before chemical and physical restraints are utilized. While it is not realistic to categorically prohibit the use of restraints, it is important to first explore and exhaust all other options. Restraints exacerbate agitation and the decision to use restraints should not be driven by staffing considerations.

A client is being admitted to the burn unit from another hospital. The client has an intraosseous IV that was started 2 days ago, according to the client's medical record. What does the admitting nurse do first? A. Anticipate an order to discontinue the intraosseous IV and start an epidural IV. B. Call the previous hospital to verify the date. C. Immediately discontinue the intraosseous IV. D. Nothing; this is a long-term treatment.

A. Anticipate an order to discontinue the intraosseous IV and start an epidural IV. The intraosseous route should be used only during the immediate period of resuscitation and should not be used for longer than 24 hours. Alternative IV routes, such as epidural access, should then be considered for pain management. The nurse should know what to do in this client's situation without contacting the previous hospital. Other client data, such as the date and time that the burn occurred, should validate the date and time of insertion of the IV. Discontinuing the IV is not the priority in this situation—the client is in a precarious fluid balance situation. One IV access should not be stopped until another is established. This type of IV is not used for long-term therapy; an action must be taken.

The nurse is starting a peripheral IV catheter on a recently admitted client. What actions does the nurse perform before insertion of the line? (Select all that apply.) A. Apply povidone-iodine to clean skin, dry for 2 minutes. B. Clean the skin around the site. C. Prepare the skin with 70% alcohol or chlorhexidine. D. Shave the hair around the area of insertion. E. Wear clean gloves and touch the site only with fingertips after applying antiseptics.

A. Apply povidone-iodine to clean skin, dry for 2 minutes. B. Clean the skin around the site. C. Prepare the skin with 70% alcohol or chlorhexidine. Povidone-iodine (Betadine) is applied to the selected insertion site before insertion. The solution is allowed to dry, which takes about 2 minutes. The insertion site should be cleansed before the antiseptic skin preparations are completed. After soap and water cleansing, prepping with 70% alcohol or chlorhexidine is done. Clipping, rather than shaving, hair around the selected IV site is done; shaving is abrasive and makes the skin more vulnerable to infection (i.e., microbial invasion). The insertion site should not be palpated again after it has been prepped; this mistake is frequently made with IV starts.

A home health nurse makes a home visit to a 90-year-old patient who has cardiovascular disease. During the visit the nurse observes that the patient has begun exhibiting subtle and unprecedented signs of confusion and agitation. What should the home health nurse do? Select one: A. Arrange for the patient to see his primary care physician. B. Have a family member check in on the patient in the evening. C. Refer the patient to an adult day program. D. Increase the frequency of the patient's home care.

A. Arrange for the patient to see his primary care physician. In more than half of the cases, sudden confusion and hallucinations are evident in multi-infarct dementia. This condition is also associated with cardiovascular disease. Having the patient's home care increased does not address the problem, neither does having a family member check on the patient in the evening. Referring the patient to an adult day program may be beneficial to the patient, but it does not address the acute problem the patient is having, the nurse should arrange for the patient to see his primary care physician.

The nurse who is starting the shift finds a client with an IV that is leaking all over the bed linens. What does the nurse do initially? A. Assess the insertion site. B. Check connections. C. Check the infusion rate. D. Discontinue the IV and start another.

A. Assess the insertion site. Assessing the insertion site to check for patency is the priority. IV assessments typically begin at the insertion site and move "up" the line; that is, from the insertion site to the tubing, to the tubing's connection to the bag. Checking the IV connection is important, but is not the priority in this situation. Checking the infusion rate is not the priority. Discontinuing the IV to start another may be required, but it may be possible to "save" the IV, and the problem may be positional or involve a loose connection.

What does the nurse do to verify an order for a medication listed on a medication administration record (MAR)? Select one: A. Compare it with the original physician's order. B. Look up the drug in a textbook. C. Call the pharmacist for verification. D. Ask another nurse what the drug is.

A. Compare it with the original physician's order. In many institutions, the medication order is copied onto the patient's medication record. The nurse is responsible for checking that the medication order was transcribed correctly by comparing it with the original physician's order.

Which finding in a patient hospitalized with bronchiectasis is most important to report to the health care provider? A. Cough productive of bloody, purulent mucus B. Scattered rhonchi and wheezes heard bilaterally C. Respiratory rate 28 breaths/minute while ambulating in hallway D. Complaint of sharp chest pain with deep breathing

A. Cough productive of bloody, purulent mucus

A 93-year-old male patient with failure to thrive has begun exhibiting urinary incontinence. When choosing appropriate interventions, you know that various age-related factors can alter urinary elimination patterns in elderly patients. What is an example of these factors? Select one: A. Decreased muscle tone B. Decreased residual volume C. Increased bladder capacity D. Urethral stenosis

A. Decreased muscle tone Factors that alter elimination patterns in the older adult include decreased bladder capacity, decreased muscle tone, increased residual volumes, and delayed perception of elimination cues. The other noted phenomena are atypical.

A nurse has administered an intramuscular injection. What will the nurse do with the syringe and needle? Select one: A. Do not recap the needle and place it in a puncture-resistant container. B. Take off the needle and throw the syringe in the patient's trash can. C. Break off the needle, place it in the barrel, and throw it in the trash. D. Recap the needle and place it in a puncture-resistant container.

A. Do not recap the needle and place it in a puncture-resistant container. After use, needles and syringes are placed in a puncture-resistant container without being recapped. This prevents needlestick injuries, because most occur during recapping.

The nurse is caring for a patient who has a right-sided chest tube after a right lower lobectomy. Which nursing action can the nurse delegate to the unlicensed assistive personnel (UAP)? A. Document the amount of drainage every eight hours. B. Obtain samples of drainage for culture from the system. C. Assess patient pain level associated with the chest tube. D. Check the water-seal chamber for the correct fluid level.

A. Document the amount of drainage every eight hours.

A 47-year-old patient who has come to the physician's office for his annual physical is being assessed by the office nurse. The nurse who is performing routine health screening for this patient should be aware that one of the first physical signs of aging is what? Select one: A. Failing eyesight, especially close vision B. Having more frequent aches and pains C. Increasing loss of muscle tone D. Accepting limitations while developing assets

A. Failing eyesight, especially close vision Failing eyesight, especially close vision, is one of the first signs of aging in middle life. More frequent aches and pains begin in the "early" late years (between ages 65 and 79). Increase in loss of muscle tone occurs in later years (ages 80 and older). Accepting limitations while developing assets is socialization development that occurs in adulthood.

A nurse is administering an intramuscular injection of a viscous medication using the appropriate-gauge needle. What does the nurse need to know about needle gauges? Select one: A. Gauges range from 18 to 30, with 18 being the largest. B. The gauge will depend on the length of the needle. C. Ask the patient what size needle is preferred. D. All needles for parenteral injection are the same gauge.

A. Gauges range from 18 to 30, with 18 being the largest. The gauge is determined by the diameter of the needle and ranges from 18 to 30. As the diameter of the needle increases, the gauge number decreases (an 18-gauge needle is, therefore, larger than a 30-gauge needle). A viscous medication requires a larger-gauge needle for injection.

A patient in the clinic with cystic fibrosis (CF) reports increased sweating and weakness during the summer months. Which action by the nurse would be most appropriate? A. Have the patient add dietary salt to meals. B. Teach the patient about the signs of hypoglycemia. C. Suggest decreasing intake of dietary fat and calories. D. Instruct the patient about pancreatic enzyme replacements.

A. Have the patient add dietary salt to meals.

Gerontologic nursing is a specialty area of nursing that provides care for the elderly in our population. What goal of care should a gerontologic nurse prioritize when working with this population? Select one: A. Helping older adults use their strengths to optimize independence B. Helping older adults determine how to reduce their use of external resources C. Helping older adults promote social integration D. Helping older adults identify the weaknesses that most limit them

A. Helping older adults use their strengths to optimize independence Gerontologic nursing is provided in acute care, skilled and assisted living, community, and home settings. The goals of care include promoting and maintaining functional status and helping older adults identify and use their strengths to achieve optimal independence. Goals of gerontologic nursing do not include helping older adults "promote social integration" or identify their weaknesses. Optimal independence does not necessarily involve reducing the use of available resources.

What intervention should be included on a plan of care to prevent pressure ulcer development in healthcare settings? Select one: A. Implement a turning schedule every 2 hours. B. Use ring cushions for heels and elbows. C. Do not turn, use pressure-relieving support surface. D. Change position at least once each shift.

A. Implement a turning schedule every 2 hours. To protect patients at risk from the adverse effects of pressure, implement turning using an every-2-hour schedule in the healthcare setting. More frequent position changes may be necessary. Never use ring cushions or "donuts."

A resident of a long-term care facility has been experiencing pain associated with sciatica, a health problem that has not previously been present. Which intervention should the nurse implement first to help control this patient's pain? Select one: A. Implement nonpharmacologic measures B. Administer fentanyl or sustained-release oxycodone C. Provide morphine or codeine D. Prepare a dose of acetaminophen

A. Implement nonpharmacologic measures Nursing guidelines for older adults with pain include exploring nonpharmacologic means to manage pain first. If nonpharmacologic measures are unsuccessful, begin with the weakest type and dose of analgesic and gradually increase so that the patient's response can be evaluated. Morphine, codeine, fentanyl, and oxycodone should be used carefully in the older patient.

The nurse is providing care for an older adult man whose diagnosis of dementia has recently led to urinary incontinence. When planning this patient's care, what intervention should the nurse avoid? Select one: A. Indwelling catheter B. Scheduled toileting C. External condom catheter D. Incontinence pads

A. Indwelling catheter Indwelling catheters are avoided if at all possible because of the high incidence of urinary tract infections with their use. Intermittent self-catheterization is an appropriate alternative for managing reflex incontinence, urinary retention, and overflow incontinence related to an overdistended bladder. External catheters (condom catheters) and leg bags to collect spontaneous voiding are useful for male patients with reflex or total incontinence. Incontinence pads should be used as a last resort because they only manage, rather than solve, the incontinence.

On auscultation of a patient's lungs, the nurse hears low-pitched, bubbling sounds during inhalation in the lower third of both lungs. How should the nurse document this finding? A. Inspiratory crackles at the bases B. Expiratory wheezes in both lungs C. Abnormal lung sounds in the apices of both lungs D. Pleural friction rub in the right and left lower lobes

A. Inspiratory crackles at the bases

The care team has deemed the occasional use of restraints necessary in the care of a patient with Alzheimer's disease. What ethical violation is most often posed when using restraints in a long-term care setting? Select one: A. It threatens the patient's autonomy. B. It limits the patient's personal safety. C. It is not normally legal. D. It exacerbates the patient's disease process.

A. It threatens the patient's autonomy. Because safety risks are involved when using restraints on elderly confused patients, this is a common ethical problem, especially in long-term care settings. By definition, restraints limit the individual's autonomy. Restraints are not without risks, but they should not normally limit a patient's safety. Restraints will not affect the course of the patient's underlying disease process, though they may exacerbate confusion. The use of restraints is closely legislated, but they are not illegal.

A nurse is planning discharge teaching for an 80-year-old patient with mild short-term memory loss. The discharge teaching will include how to perform basic wound care for the venous ulcer on his lower leg. When planning the necessary health education for this patient, what should the nurse plan to do? Select one: A. Keep teaching periods short. B. Keep visual cues to a minimum to enhance the patient's focus. C. Provide a list of useful Web sites to supplement learning. D. Set long-term goals with the patient.

A. Keep teaching periods short. To assist the elderly patient with short-term memory loss, the nurse should keep teaching periods short, provide glare-free lighting, link new information with familiar information, use visual and auditory cues, and set short-term goals with the patient. The patient may or may not be open to the use of online resources.

A client who takes corticosteroids daily for rheumatoid arthritis requires insertion of an IV catheter to receive IV antibiotics for 5 days. Which type of IV catheter does the nurse teach the new graduate nurse to use for this client? A. Midline catheter B. Nontunneled percutaneous central catheter C. Peripherally inserted central catheter D. Short peripheral catheter

A. Midline catheter For a client with fragile veins (which occur with long-term corticosteroid use) and the need for a catheter for 5 days, the midline catheter is the best choice. Nontunneled central catheters usually are used for clients who require IV access for longer periods. Peripherally inserted central catheters usually are used for clients who require IV access for longer periods. A short peripheral catheter is likely to infiltrate before 5 days in a client with fragile veins, requiring reinsertion.

A client admitted to the ICU is expected to remain for 3 weeks. The nurse has orders to start an IV. Which vascular access device is best for this client? A. Midline catheter B. Peripherally inserted central catheter (PICC) C. Short peripheral catheter D. Tunneled central catheter

A. Midline catheter Midline catheters are used for therapies lasting from 1 to 4 weeks. PICCs are typically used when IV therapy is expected to last for months. Short peripheral catheters are allowed to dwell (stay in) for 72 to 96 hours, but they then require removal and insertion at another venous site. Tunneled central catheters must be inserted by a health care provider; the nurse typically is not qualified to start this type of IV.

A patient who is experiencing an asthma attack develops bradycardia and a decrease in wheezing. Which action should the nurse take first? A. Notify the health care provider. B. Document changes in respiratory status. C. Encourage the patient to cough and deep breathe. D. Administer IV methylprednisolone (Solu-Medrol).

A. Notify the health care provider.

The nurse administers prescribed therapies for a patient with cor pulmonale and right-sided heart failure. Which assessment would best evaluate the effectiveness of the therapies? A. Observe for distended neck veins. B. Auscultate for crackles in the lungs. C. Palpate for heaves or thrills over the heart. D. Review hemoglobin and hematocrit values.

A. Observe for distended neck veins.

What must a nurse do each time medications are administered to ensure that medication errors do not occur? Select one: A. Observe the three checks and five rights. B. Review information about classification of drugs. C. Verify the number of medications to be administered. D. Ask another nurse to double-check the medications.

A. Observe the three checks and five rights. Safety is of the utmost when preparing and administering drugs. The nurse observes the three checks and five rights each time medications are administered.

Which nursing action for a patient with chronic obstructive pulmonary disease (COPD) could the nurse delegate to experienced unlicensed assistive personnel (UAP)? A. Obtain oxygen saturation using pulse oximetry. B. Monitor for increased oxygen need with exercise. C. Teach the patient about safe use of oxygen at home. D. Adjust oxygen to keep saturation in prescribed parameters.

A. Obtain oxygen saturation using pulse oximetry.

An aide tells the nurse that an older adult patient has a pulse of 105 beats/min. This pulse rate should be taken seriously because of what? Select one: A. Older adults have poor cardiac reserves B. The patient is likely taking many medications C. The patient needs to be reassured that providers care D. Older adults usually have lower than normal pulse rates

A. Older adults have poor cardiac reserves Pulses greater than 100 are abnormal and should be taken seriously. Because of their poor cardiac reserves, older adults do not tolerate these pulse rates well for long periods. The scenario does not mention the patient's medications. Pulse rates on older adults often run in the 50-60 bpm range, but not always. While patients can benefit from reassurance, this is not the primary concern in this scenario.

You are the nurse caring for an elderly adult who is bedridden. What intervention would you include in the care plan that would most effectively prevent pressure ulcers? Select one: A. Post a turning schedule at the patient's bedside and ensure staff adherence. B. Slide, rather than lift, the patient when turning. C. Turn and reposition the patient a minimum of every 8 hours. D. Vigorously massage lotion into bony prominences.

A. Post a turning schedule at the patient's bedside and ensure staff adherence. A turning schedule with a signing sheet will help ensure that the patient gets turned and, thus, help prevent pressure ulcers. Turning should occur every 1 to 2 hours, not every 8 hours, for patients who are in bed for prolonged periods. The nurse should apply lotion to keep the skin moist, but should avoid vigorous massage, which could damage capillaries. When moving the patient, the nurse should lift, rather than slide, the patient to avoid shearing.

Your patient has been admitted for a liver biopsy because the physician believes the patient may have liver cancer. The family has told both you and the physician that if the patient is terminal, the family does not want the patient to know. The biopsy results are positive for an aggressive form of liver cancer and the patient asks you repeatedly what the results of the biopsy show. What strategy can you use to give ethical care to this patient? Select one: A. Promptly communicate the patient's request for information to the family and the physician. B. Obtain the results of the biopsy and provide them to the patient. C. Tell the patient that the biopsy results are not back yet in order temporarily to appease him. D. Tell the patient that only the physician knows the results of the biopsy.

A. Promptly communicate the patient's request for information to the family and the physician. Strategies nurses could consider include the following: not lying to the patient, providing all information related to nursing procedures and diagnoses, and communicating the patient's requests for information to the family and physician. Ethically, you cannot tell the patient the results of the biopsy and you cannot lie to the patient.

The nurse assesses a patient with chronic obstructive pulmonary disease (COPD) who has been admitted with increasing dyspnea over the last 3 days. Which finding is most important for the nurse to report to the health care provider? A. Respirations are 36 breaths/minute. B. Anterior-posterior chest ratio is 1:1. C. Lung expansion is decreased bilaterally. D. Hyperresonance to percussion is present.

A. Respirations are 36 breaths/minute.

Which action should the emergency department staff take first for an older patient who is demonstrating extreme confusion? Select one: A. Review the drugs being taken B. Order an ECG C. Check serum electrolyte levels D. Administer a stimulant

A. Review the drugs being taken The risk of adverse reactions to drugs is so high in older people that some health care providers suggest that any symptom in an older adult be suspected as being related to a drug until proven otherwise. If the patient or an accompanying person knows what drugs are being taken and the dosages, the cause of the dysfunction may be immediately apparent. No stimulant should be given until that information is available, as it might cause an interaction with adverse results. Serum electrolyte levels and an ECG may be needed but only after the drug information is known.

After a sudden decline in cognition, a 77-year-old man who has been diagnosed with vascular dementia is receiving care in his home. To reduce this man's risk of future infarcts, what action should the nurse most strongly encourage? Select one: A. Rigorous control of the patient's blood pressure and serum lipid levels B. Use of mobility aids to promote independence C. Adequate nutrition and fluid intake D. Activity limitation and falls reduction efforts

A. Rigorous control of the patient's blood pressure and serum lipid levels Because vascular dementia is associated with hypertension and cardiovascular disease, risk factors (e.g., hypercholesterolemia, history of smoking, diabetes) are similar. Prevention and management are also similar. Therefore, measures to decrease blood pressure and lower cholesterol levels may prevent future infarcts. Activity limitation is unnecessary and infarcts are not prevented by nutrition or the use of mobility aids.

Mrs. Harris is an 83-year-old woman who has returned to the community following knee replacement surgery. The community health nurse recognizes that Mrs. Harris has prescriptions for nine different medications for the treatment of varied health problems. In addition, she has experienced occasional episodes of dizziness and lightheadedness since her discharge. The nurse should identify which of the following nursing diagnoses? Select one: A. Risk for falls related to polypharmacy and impaired balance B. Disturbed thought processes related to adverse drug effects and hypotension C. Adult failure to thrive related to chronic disease and circulatory disturbance D. Risk for infection related to polypharmacy and hypotension

A. Risk for falls related to polypharmacy and impaired balance Polypharmacy and loss of balance are major contributors to falls in the elderly. This patient does not exhibit failure to thrive or disturbed thought processes. There is no evidence of a heightened risk of infection.

A patient is admitted to the emergency department complaining of sudden onset shortness of breath and is diagnosed with a possible pulmonary embolus. How should the nurse prepare the patient for diagnostic testing to confirm the diagnosis? A. Start an IV so contrast media may be given, B. Ensure that the patient has been NPO for at least 6 hours. C. Inform radiology that radioactive glucose preparation is needed. D. Instruct the patient to undress to the waist and remove any metal objects.

A. Start an IV so contrast media may be given.

A hospitalized patient asks the nurse for "some aspirin for my headache." There is no order for aspirin for this patient. What will the nurse do? Select one: A. State that an order from the doctor is legally required and check with the doctor. B. Ask the patient's family to bring some aspirin from home. C. Ask the patient's visitors if they have any aspirin for the patient. D. Go ahead and give the patient aspirin, a common self-prescribed drug.

A. State that an order from the doctor is legally required and check with the doctor. No medication may be given to a patient without a medication order from a physician (or a nurse practitioner in some states). The nurse would tell the patient an order from the physician is required.

The nurse is caring for a hospitalized older patient who has nasal packing in place to treat a nosebleed. Which assessment finding will require the most immediate action by the nurse? A. The oxygen saturation is 89%. B. The nose appears red and swollen. C. The patient's temperature is 100.1° F (37.8° C). D. The patient complains of level 8 (0 to 10 scale) pain.

A. The oxygen saturation is 89%.

The nurse is caring for a mechanically ventilated patient with a cuffed tracheostomy tube. Which action by the nurse would best determine if the cuff has been properly inflated? A. Use a manometer to ensure cuff pressure is at an appropriate level. B. Check the amount of cuff pressure ordered by the health care provider. C. Suction the patient first with a fenestrated inner cannula to clear secretions. D. Insert the decannulation plug before the nonfenestrated inner cannula is removed.

A. Use a manometer to ensure cuff pressure is at an appropriate level.

Following assessment of a patient with pneumonia, the nurse identifies a nursing diagnosis of ineffective airway clearance. Which assessment data best supports this diagnosis? A. Weak, nonproductive cough effort B. Large amounts of greenish sputum C. Respiratory rate of 28 breaths/minute D. Resting pulse oximetry (SpO2) of 85%

A. Weak, nonproductive cough effort

A young adult patient who denies any history of smoking is seen in the clinic with a new diagnosis of chronic obstructive pulmonary disease (COPD). It is most appropriate for the nurse to teach the patient about A. α1-antitrypsin testing. B. use of the nicotine patch. C. continuous pulse oximetry. D. effects of leukotriene modifiers.

A. α1-antitrypsin testing.

A patient with acute dyspnea is scheduled for a spiral computed tomography (CT) scan. Which information obtained by the nurse is a priority to communicate to the health care provider before the CT? a. Allergy to shellfish c. Respiratory rate of 30 b. Apical pulse of 104 d. O2 saturation of 90%

ANS: A Because iodine-based contrast media is used during a spiral CT, the patient may need to have the CT scan without contrast or be premedicated before injection of the contrast media. The increased pulse, low oxygen saturation, and tachypnea all indicate a need for further assessment or intervention but do not indicate a need to modify the CT procedure.

On auscultation of a patient's lungs, the nurse hears low-pitched, bubbling sounds during inhalation in the lower third of both lungs. How should the nurse document this finding? a. Inspiratory crackles at the bases b. Expiratory wheezes in both lungs c. Abnormal lung sounds in the apices of both lungs d. Pleural friction rub in the right and left lower lobes

ANS: A Crackles are low-pitched, bubbling sounds usually heard on inspiration. Wheezes are high-pitched sounds. They can be heard during the expiratory or inspiratory phase of the respiratory cycle. The lower third of both lungs are the bases, not apices. Pleural friction rubs are grating sounds that are usually heard during both inspiration and expiration.

The nurse assesses a patient with chronic obstructive pulmonary disease (COPD) who has been admitted with increasing dyspnea over the past 3 days. Which finding is important for the nurse to report to the health care provider? a. Respirations are 36 breaths/min. b. Anterior-posterior chest ratio is 1:1. c. Lung expansion is decreased bilaterally. d. Hyperresonance to percussion is present.

ANS: A The increase in respiratory rate indicates respiratory distress and a need for rapid interventions such as administration of O2 or medications. The other findings are common chronic changes occurring in patients with COPD.

14. A patient with acute dyspnea is scheduled for a spiral computed tomography (CT) scan. Which information obtained by the nurse is a priority to communicate to the health care provider before the CT? a. Allergy to shellfish b. Apical pulse of 104 c. Respiratory rate of 30 d. Oxygen saturation of 90%

ANS: A Because iodine-based contrast media is used during a spiral CT, the patient may need to have the CT scan without contrast or be premedicated before injection of the contrast media. The increased pulse, low oxygen saturation, and tachypnea all indicate a need for further assessment or intervention but do not indicate a need to modify the CT procedure. DIF: Cognitive Level: Apply (application) REF: 492 OBJ: Special Questions: Prioritization TOP: Nursing Process: Implementation MSC: NCLEX: Physiological Integrity

4. On auscultation of a patient's lungs, the nurse hears low-pitched, bubbling sounds during inhalation in the lower third of both lungs. How should the nurse document this finding? a. Inspiratory crackles at the bases b. Expiratory wheezes in both lungs c. Abnormal lung sounds in the apices of both lungs d. Pleural friction rub in the right and left lower lobes

ANS: A Crackles are low-pitched, bubbling sounds usually heard on inspiration. Wheezes are high-pitched sounds. They can be heard during the expiratory or inspiratory phase of the respiratory cycle. The lower third of both lungs are the bases, not apices. Pleural friction rubs are grating sounds that are usually heard during both inspiration and expiration. DIF: Cognitive Level: Understand (comprehension) REF: 487 | 489 TOP: Nursing Process: Assessment MSC: NCLEX: Physiological Integrity

12. A patient is admitted to the emergency department complaining of sudden onset shortness of breath and is diagnosed with a possible pulmonary embolus. How should the nurse prepare the patient for diagnostic testing to confirm the diagnosis? a. Start an IV so contrast media may be given. b. Ensure that the patient has been NPO for at least 6 hours. c. Inform radiology that radioactive glucose preparation is needed. d. Instruct the patient to undress to the waist and remove any metal objects.

ANS: A Spiral computed tomography (CT) scans are the most commonly used test to diagnose pulmonary emboli, and contrast media may be given IV. A chest x-ray may be ordered but will not be diagnostic for a pulmonary embolus. Preparation for a chest x-ray includes undressing and removing any metal. Bronchoscopy is used to detect changes in the bronchial tree, not to assess for vascular changes, and the patient should be NPO 6 to 12 hours before the procedure. Positron emission tomography (PET) scans are most useful in determining the presence of malignancy, and a radioactive glucose preparation is used. DIF: Cognitive Level: Apply (application) REF: 492 TOP: Nursing Process: Planning MSC: NCLEX: Physiological Integrity

20. The nurse assesses a patient with chronic obstructive pulmonary disease (COPD) who has been admitted with increasing dyspnea over the last 3 days. Which finding is most important for the nurse to report to the health care provider? a. Respirations are 36 breaths/minute. b. Anterior-posterior chest ratio is 1:1. c. Lung expansion is decreased bilaterally. d. Hyperresonance to percussion is present.

ANS: A The increase in respiratory rate indicates respiratory distress and a need for rapid interventions such as administration of oxygen or medications. The other findings are common chronic changes occurring in patients with COPD. DIF: Cognitive Level: Apply (application) REF: 482 OBJ: Special Questions: Prioritization TOP: Nursing Process: Assessment MSC: NCLEX: Physiological Integrity

A patient is scheduled for a computed tomography (CT) scan of the chest with contrast media. Which assessment findings should the nurse report to the health care provider before the patient goes for the CT (select all that apply)? a. Allergy to shellfish b. Patient reports claustrophobia c. Elevated serum creatinine level d. Recent bronchodilator inhaler use e. Inability to remove a wedding band

ANS: A, C Because the contrast media is iodine based and may cause dehydration and decreased renal blood flow, asking about iodine allergies (such as allergy to shellfish) and monitoring renal function before the CT scan are necessary. The other actions are not contraindications for CT of the chest, although they may be for other diagnostic tests, such as magnetic resonance imaging or pulmonary spirometry

A diabetic patient's arterial blood gas (ABG) results are pH 7.28; PaCO2 34 mm Hg; PaO2 85 mm Hg; HCO3 - 18 mEq/L. The nurse would expect which finding? a. Intercostal retractions c. Low oxygen saturation (SpO2 ) b. Kussmaul respirations d. Decreased venous O2 pressure

ANS: B Kussmaul (deep and rapid) respirations are a compensatory mechanism for metabolic acidosis. The low pH and low bicarbonate result indicate metabolic acidosis. Intercostal retractions, a low oxygen saturation rate, and a decrease in venous O2 pressure would not be caused by acidosis.

Which action is appropriate for the nurse to delegate to unlicensed assistive personnel (UAP)? a. Listen to a patient's lung sounds for wheezes or crackles. b. Label specimens obtained during percutaneous lung biopsy. c. Instruct a patient about how to use home spirometry testing. d. Measure induration at the site of a patient's intradermal skin test.

ANS: B Labeling of specimens is within the scope of practice of UAP. The other actions require nursing judgment and should be done by licensed nursing personnel.

The nurse observes a student who is listening to a patient's lungs. Which action by the student indicates a need to review respiratory assessment skills? a. The student compares breath sounds from side to side at each level. b. The student listens during the inspiratory phase, then moves the stethoscope. c. The student starts at the apices of the lungs, moving down toward the lung bases. d. The student instructs the patient to breathe slowly and deeply through the mouth.

ANS: B Listening only during inspiration indicates the student needs a review of respiratory assessment skills. At each placement of the stethoscope, listen to at least one cycle of inspiration and expiration. During chest auscultation, instruct the patient to breathe slowly and a little deeper than normal through the mouth. Auscultation should proceed from the lung apices to the bases, comparing opposite areas of the chest, unless the patient is in respiratory distress or will tire easily.

A patient admitted to the emergency department complaining of sudden onset shortness of breath is diagnosed with a possible pulmonary embolus. How should the nurse prepare the patient for diagnostic testing to confirm the diagnosis? a. Ensure that the patient has been NPO. b. Start an IV so contrast media may be given. c. Inform radiology that radioactive glucose preparation is needed. d. Instruct the patient to expect to inspire deeply and exhale forcefully.

ANS: B Spiral computed tomography scans are the most commonly used test to diagnose pulmonary emboli and contrast media may be given IV. Bronchoscopy is used to detect changes in the bronchial tree, not to assess for vascular changes, and the patient should be NPO 6 to 12 hours before the procedure. Positron emission tomography scans are most useful in determining the presence of malignancy and a radioactive glucose preparation is used. For spirometry, the patient is asked to inhale deeply and exhale as long, hard, and fast as possible.

The nurse observes that a patient with respiratory disease experiences a decrease in SpO2 from 93% to 88% while the patient is ambulating. What is the priority action of the nurse? a. Notify the health care provider. b. Administer PRN supplemental O2 . c. Document the response to exercise. d. Encourage the patient to pace activity.

ANS: B The drop in SpO2 to 85% indicates that the patient is hypoxemic and needs supplemental O2 when exercising. The other actions are also important, but the first action should be to correct the hypoxemia.

18. After the nurse has received change-of-shift report, which patient should the nurse assess first? a. A patient with pneumonia who has crackles in the right lung base b. A patient with possible lung cancer who has just returned after bronchoscopy c. A patient with hemoptysis and a 16-mm induration with tuberculin skin testing d. A patient with chronic obstructive pulmonary disease (COPD) and pulmonary function testing (PFT) that indicates low forced vital capacity

ANS: B Because the cough and gag are decreased after bronchoscopy, this patient should be assessed for airway patency. The other patients do not have clinical manifestations or procedures that require immediate assessment by the nurse. DIF: Cognitive Level: Apply (application) REF: 492 OBJ: Special Questions: Prioritization; Multiple Patients TOP: Nursing Process: Assessment MSC: NCLEX: Safe and Effective Care Environment

3. A diabetic patient's arterial blood gas (ABG) results are pH 7.28; PaCO2 34 mm Hg; PaO2 85 mm Hg; HCO3- 18 mEq/L. The nurse would expect which finding? a. Intercostal retractions b. Kussmaul respirations c. Low oxygen saturation (SpO2) d. Decreased venous O2 pressure

ANS: B Kussmaul (deep and rapid) respirations are a compensatory mechanism for metabolic acidosis. The low pH and low bicarbonate result indicate metabolic acidosis. Intercostal retractions, a low oxygen saturation rate, and a decrease in venous O2 pressure would not be caused by acidosis. DIF: Cognitive Level: Apply (application) REF: 479 TOP: Nursing Process: Assessment MSC: NCLEX: Physiological Integrity

22. Which action is appropriate for the nurse to delegate to unlicensed assistive personnel (UAP)? a. Listen to a patient's lung sounds for wheezes or rhonchi. b. Label specimens obtained during percutaneous lung biopsy. c. Instruct a patient about how to use home spirometry testing. d. Measure induration at the site of a patient's intradermal skin test.

ANS: B Labeling of specimens is within the scope of practice of UAP. The other actions require nursing judgment and should be done by licensed nursing personnel. DIF: Cognitive Level: Apply (application) REF: 15 OBJ: Special Questions: Delegation TOP: Nursing Process: Assessment MSC: NCLEX: Safe and Effective Care Environment

6. A patient with a chronic cough has a bronchoscopy. After the procedure, which intervention by the nurse is most appropriate? a. Elevate the head of the bed to 80 to 90 degrees. b. Keep the patient NPO until the gag reflex returns. c. Place on bed rest for at least 4 hours after bronchoscopy. d. Notify the health care provider about blood-tinged mucus.

ANS: B Risk for aspiration and maintaining an open airway is the priority. Because a local anesthetic is used to suppress the gag/cough reflexes during bronchoscopy, the nurse should monitor for the return of these reflexes before allowing the patient to take oral fluids or food. Blood-tinged mucus is not uncommon after bronchoscopy. The patient does not need to be on bed rest, and the head of the bed does not need to be in the high-Fowler's position. DIF: Cognitive Level: Apply (application) REF: 492 TOP: Nursing Process: Planning MSC: NCLEX: Physiological Integrity

1. A patient with acute shortness of breath is admitted to the hospital. Which action should the nurse take during the initial assessment of the patient? a. Ask the patient to lie down to complete a full physical assessment. b. Briefly ask specific questions about this episode of respiratory distress. c. Complete the admission database to check for allergies before treatment. d. Delay the physical assessment to first complete pulmonary function tests.

ANS: B When a patient has severe respiratory distress, only information pertinent to the current episode is obtained, and a more thorough assessment is deferred until later. Obtaining a comprehensive health history or full physical examination is unnecessary until the acute distress has resolved. Brief questioning and a focused physical assessment should be done rapidly to help determine the cause of the distress and suggest treatment. Checking for allergies is important, but it is not appropriate to complete the entire admission database at this time. The initial respiratory assessment must be completed before any diagnostic tests or interventions can be ordered. DIF: Cognitive Level: Apply (application) REF: 482 TOP: Nursing Process: Assessment MSC: NCLEX: Physiological Integrity

A patient is scheduled for a computed tomography (CT) of the chest with contrast media. Which assessment findings should the nurse immediately report to the health care provider (select all that apply)? a. Patient is claustrophobic. b. Patient is allergic to shellfish. c. Patient recently used a bronchodilator inhaler. d. Patient is not able to remove a wedding band. e. Blood urea nitrogen (BUN) and serum creatinine levels are elevated.

ANS: B, E Because the contrast media is iodine-based and may cause dehydration and decreased renal blood flow, asking about iodine allergies (such as allergy to shellfish) and monitoring renal function before the CT scan are necessary. The other actions are not contraindications for CT of the chest, although they may be for other diagnostic tests, such as magnetic resonance imaging (MRI) or pulmonary function testing (PFT). DIF: Cognitive Level: Analyze (analysis) REF: 492 TOP: Nursing Process: Implementation MSC: NCLEX: Physiological Integrity

A patient in metabolic alkalosis is admitted to the emergency department and pulse oximetry (SpO2 ) indicates that the O2 saturation is 94%. Which action should the nurse expect to take next? a. Complete a head-to-toe assessment. b. Administer an inhaled bronchodilator. c. Place the patient on high-flow oxygen. d. Obtain repeat arterial blood gases (ABGs).

ANS: C Although the O2 saturation is adequate, the left shift in the oxyhemoglobin dissociation curve will decrease the amount of O2 delivered to tissues, so high oxygen concentrations should be given. A head-to-toe assessment and repeat ABGs may be implemented later. Bronchodilators are not needed for metabolic alkalosis and there is no indication that the patient is having difficulty with airflow

After the nurse has received change-of-shift report, which patient should the nurse assess first? a. A patient with pneumonia who has crackles in the right lung base b. A patient with chronic bronchitis who has a low forced vital capacity c. A patient with possible lung cancer who has just returned after bronchoscopy d. A patient with hemoptysis and a 16-mm induration after tuberculin skin testing

ANS: C Because the cough and gag are decreased after bronchoscopy, this patient should be assessed for airway patency. The other patients do not have clinical manifestations or procedures that require immediate assessment by the nurse.

The nurse completes a shift assessment on a patient admitted in the early phase of heart failure. When auscultating the patient's lungs, which finding would the nurse most likely hear? a. Continuous rumbling, snoring, or rattling sounds mainly on expiration b. Continuous high-pitched musical sounds on inspiration and expiration c. Discontinuous, high-pitched sounds of short duration during inspiration d. A series of long-duration, discontinuous, low-pitched sounds during inspiration

ANS: C Fine crackles are likely to be heard in the early phase of heart failure. Fine crackles are discontinuous, high-pitched sounds of short duration heard on inspiration. Course crackles are a series of long-duration, discontinuous, low-pitched sounds during inspiration. Wheezes are continuous high-pitched musical sounds on inspiration and expiration.

The nurse teaches a patient about pulmonary spirometry testing. Which statement, if made by the patient, indicates teaching was effective? a. "I should use my inhaler right before the test." b. "I won't eat or drink anything 8 hours before the test." c. "I will inhale deeply and blow out hard during the test." d. "My blood pressure and pulse will be checked every 15 minutes."

ANS: C For spirometry, the patient should inhale deeply and exhale as long, hard, and fast as possible. The other actions are not needed. The administration of inhaled bronchodilators should be avoided 6 hours before the procedure.

A patient who has a history of chronic obstructive pulmonary disease (COPD) was hospitalized for increasing shortness of breath and chronic hypoxemia (SaO2 levels of 89% to 90%). In planning for discharge, which action by the nurse will be most effective in improving compliance with discharge teaching? a. Have the patient repeat the instructions immediately after teaching. b. Accomplish the patient teaching just before the scheduled discharge. c. Arrange for the patient's caregiver to be present during the teaching. d. Start giving the patient discharge teaching during the admission process.

ANS: C Hypoxemia interferes with the patient's ability to learn and retain information, so having the patient's caregiver present will increase the likelihood that discharge instructions will be followed. Having the patient repeat the instructions will indicate that the information is understood at the time, but it does not guarantee retention of the information. Because the patient is likely to be distracted just before discharge, giving discharge instructions just before discharge is not ideal. The patient is likely to be anxious and even more hypoxemic than usual on the day of admission, so teaching about discharge should be postponed.

A patient with a chronic cough is scheduled to have a bronchoscopy with biopsy. Which intervention will the nurse implement directly after the procedure? a. Encourage the patient to drink clear liquids. b. Place the patient on bed rest for at least 4 hours. c. Keep the patient NPO until the gag reflex returns. d. Maintain the head of the bed elevated 90 degrees.

ANS: C Risk for aspiration and maintaining an open airway is the priority. Because a local anesthetic is used to suppress the gag and cough reflexes during bronchoscopy, the nurse should monitor for the return of these reflexes before allowing the patient to take oral fluids or food. The patient does not need to be on bed rest, and the head of the bed does not need to be in the high-Fowler's position.

Using the illustrated technique, the nurse is assessing for which finding in a patient with chronic obstructive pulmonary disease (COPD)? a. Hyperresonance c. Reduced excursion b. Tripod positioning d. Accessory muscle use

ANS: C The technique for palpation for chest excursion is shown in the illustrated technique. Reduced chest movement would be noted on palpation of a patient's chest with COPD. Hyperresonance would be assessed through percussion. Accessory muscle use and tripod positioning would be assessed by inspection. DIF: Cognitive Level: Understand (comprehension) REF: 467 TOP: Nursing Process: Assessment MSC: NCLEX: Physiological Integrity

8. While caring for a patient with respiratory disease, the nurse observes that the patient's SpO2 drops from 93% to 88% while the patient is ambulating in the hallway. What is the priority action of the nurse? a. Notify the health care provider. b. Document the response to exercise. c. Administer the PRN supplemental O2. d. Encourage the patient to pace activity.

ANS: C The drop in SpO2 to 85% indicates that the patient is hypoxemic and needs supplemental oxygen when exercising. The other actions are also important, but the first action should be to correct the hypoxemia. DIF: Cognitive Level: Apply (application) REF: 480 OBJ: Special Questions: Prioritization TOP: Nursing Process: Implementation MSC: NCLEX: Physiological Integrity

17. A patient in metabolic alkalosis is admitted to the emergency department, and pulse oximetry (SpO2) indicates that the O2 saturation is 94%. Which action should the nurse take next? a. Administer bicarbonate. b. Complete a head-to-toe assessment. c. Place the patient on high-flow oxygen. d. Obtain repeat arterial blood gases (ABGs).

ANS: C Although the O2 saturation is adequate, the left shift in the oxyhemoglobin dissociation curve will decrease the amount of oxygen delivered to tissues, so high oxygen concentrations should be given. Bicarbonate would worsen the patient's condition. A head-to-toe assessment and repeat ABGs may be implemented. However, the priority intervention is to give high-flow oxygen. DIF: Cognitive Level: Apply (application) REF: eTable 26-1 OBJ: Special Questions: Prioritization TOP: Nursing Process: Implementation MSC: NCLEX: Physiological Integrity

7. The nurse completes a shift assessment on a patient admitted in the early phase of heart failure. When auscultating the patient's lungs, which finding would the nurse most likely hear? a. Continuous rumbling, snoring, or rattling sounds mainly on expiration b. Continuous high-pitched musical sounds on inspiration and expiration c. Discontinuous, high-pitched sounds of short duration heard on inspiration d. A series of long-duration, discontinuous, low-pitched sounds during inspiration

ANS: C Fine crackles are likely to be heard in the early phase of heart failure. Fine crackles are discontinuous, high-pitched sounds of short duration heard on inspiration. Rhonchi are continuous rumbling, snoring, or rattling sounds mainly on expiration. Course crackles are a series of long-duration, discontinuous, low-pitched sounds during inspiration. Wheezes are continuous high-pitched musical sounds on inspiration and expiration. DIF: Cognitive Level: Apply (application) REF: 489 TOP: Nursing Process: Assessment MSC: NCLEX: Physiological Integrity

9. The nurse teaches a patient about pulmonary function testing (PFT). Which statement, if made by the patient, indicates teaching was effective? a. "I will use my inhaler right before the test." b. "I won't eat or drink anything 8 hours before the test." c. "I should inhale deeply and blow out as hard as I can during the test." d. "My blood pressure and pulse will be checked every 15 minutes after the test."

ANS: C For PFT, the patient should inhale deeply and exhale as long, hard, and fast as possible. The other actions are not needed with PFT. The administration of inhaled bronchodilators should be avoided 6 hours before the procedure. DIF: Cognitive Level: Apply (application) REF: 493-495 TOP: Nursing Process: Planning MSC: NCLEX: Physiological Integrity

10. The nurse observes a student who is listening to a patient's lungs who is having no problems with breathing. Which action by the student indicates a need to review respiratory assessment skills? a. The student starts at the apices of the lungs and moves to the bases. b. The student compares breath sounds from side to side avoiding bony areas. c. The student places the stethoscope over the posterior chest and listens during inspiration. d. The student instructs the patient to breathe slowly and a little more deeply than normal through the mouth.

ANS: C Listening only during inspiration indicates the student needs a review of respiratory assessment skills. At each placement of the stethoscope, listen to at least one cycle of inspiration and expiration. During chest auscultation, instruct the patient to breathe slowly and a little deeper than normal through the mouth. Auscultation should proceed from the lung apices to the bases, comparing opposite areas of the chest, unless the patient is in respiratory distress or will tire easily. If so, start at the bases (see Fig. 26-7). Place the stethoscope over lung tissue, not over bony prominences. DIF: Cognitive Level: Apply (application) REF: 486 TOP: Nursing Process: Assessment MSC: NCLEX: Safe and Effective Care Environmen

The nurse analyzes the results of a patient's arterial blood gases (ABGs). Which finding would require immediate action? a. The bicarbonate level (HCO3 - ) is 31 mEq/L. b. The arterial oxygen saturation (SaO2 ) is 92%. c. The partial pressure of CO2 in arterial blood (PaCO2 ) is 31 mm Hg. d. The partial pressure of oxygen in arterial blood (PaO2 ) is 59 mm Hg.

ANS: D All the values are abnormal, but the low PaO2 indicates that the patient is at the point on the oxyhemoglobin dissociation curve where a small change in the PaO2 will cause a large drop in the O2 saturation and a decrease in tissue oxygenation. The nurse should intervene immediately to improve the patient's oxygenation.

Which assessment finding indicates that the nurse should take immediate action for an older patient? a. Weak cough effort c. Dry mucous membranes b. Barrel-shaped chest d. Bilateral basilar crackles

ANS: D Crackles in the lower half of the lungs indicate that the patient may have an acute problem such as heart failure. The nurse should immediately accomplish further assessments, such as O2 saturation, and notify the health care provider. A barrel-shaped chest, hyperresonance to percussion, and a weak cough effort are associated with aging. Further evaluation may be needed, but immediate action is not indicated. An older patient has a less forceful cough and fewer and less functional cilia. Mucous membranes tend to be drier.

The nurse admits a patient who has a diagnosis of an acute asthma attack. Which statement indicates that the patient may need teaching regarding medication use? a. "I have not had any acute asthma attacks during the past year." b. "I became short of breath an hour before coming to the hospital." c. "I've been taking Tylenol 650 mg every 6 hours for chest wall pain." d. "I've been using my albuterol inhaler more frequently over the last 4 days."

ANS: D The increased need for a rapid-acting bronchodilator should alert the patient that an acute attack may be imminent and that a change in therapy may be needed. The patient should be taught to contact a health care provider if this occurs. The other data do not indicate any need for additional teaching. DIF: Cognitive Level: Apply (application) REF: 460 TOP: Nursing Process: Assessment MSC: NCLEX: Physiological Integrity

The nurse prepares a patient with a left-sided pleural effusion for a thoracentesis. How should the nurse position the patient? a. High-Fowler's position with the left arm extended b. Supine with the head of the bed elevated 30 degrees c. On the right side with the left arm extended above the head d. Sitting upright with the arms supported on an over bed table

ANS: D The upright position with the arms supported increases lung expansion, allows fluid to collect at the lung bases, and expands the intercostal space so that access to the pleural space is easier. The other positions would increase the work of breathing for the patient and make it more difficult for the health care provider performing the thoracentesis.

The nurse palpates the posterior chest while the patient says "99" and notes absent fremitus. Which action should the nurse take next? a. Palpate the anterior chest and observe for barrel chest. b. Encourage the patient to turn, cough, and deep breathe. c. Review the chest x-ray report for evidence of pneumonia. d. Auscultate anterior and posterior breath sounds bilaterally.

ANS: D To assess for tactile fremitus, the nurse should use the palms of the hands to assess for vibration when the patient repeats a word or phrase such as "99." After noting absent fremitus, the nurse should then auscultate the lungs to assess for the presence or absence of breath sounds. Absent fremitus may be noted with pneumothorax or atelectasis. The vibration is increased in conditions such as pneumonia, lung tumors, thick bronchial secretions, and pleural effusion. Turning, coughing, and deep breathing is an appropriate intervention for atelectasis, but the nurse needs to first assess breath sounds. Fremitus is decreased if the hand is farther from the lung or the lung is hyperinflated (barrel chest). The anterior of the chest is more difficult to palpate for fremitus because of the presence of large muscles and breast tissue.

15. The nurse analyzes the results of a patient's arterial blood gases (ABGs). Which finding would require immediate action? a. The bicarbonate level (HCO3-) is 31 mEq/L. b. The arterial oxygen saturation (SaO2) is 92%. c. The partial pressure of CO2 in arterial blood (PaCO2) is 31 mm Hg. d. The partial pressure of oxygen in arterial blood (PaO2) is 59 mm Hg.

ANS: D All the values are abnormal, but the low PaO2 indicates that the patient is at the point on the oxyhemoglobin dissociation curve where a small change in the PaO2 will cause a large drop in the O2 saturation and a decrease in tissue oxygenation. The nurse should intervene immediately to improve the patient's oxygenation. DIF: Cognitive Level: Apply (application) REF: eTable 26-1 OBJ: Special Questions: Prioritization TOP: Nursing Process: Assessment MSC: NCLEX: Physiological Integrity

Which of the following diagnostic and assessment findings from among the patients on a geriatric medical unit most warrants further investigation? Select one: A. A 78-year-old male's stomach pH is increased. B. A 78-year-old man has recently developed urinary incontinence. C. An 81-year-old woman's glomerular filtration rate (GFR) is low D. A 71-year-old male client's echocardiogram reveals slight left ventricular hypertrophy.

B. A 78-year-old man has recently developed urinary incontinence.

16. When assessing the respiratory system of an older patient, which finding indicates that the nurse should take immediate action? a. Weak cough effort b. Barrel-shaped chest c. Dry mucous membranes d. Bilateral crackles at lung bases

ANS: D Crackles in the lower half of the lungs indicate that the patient may have an acute problem such as heart failure. The nurse should immediately accomplish further assessments, such as oxygen saturation, and notify the health care provider. A barrel-shaped chest, hyperresonance to percussion, and a weak cough effort are associated with aging. Further evaluation may be needed, but immediate action is not indicated. An older patient has a less forceful cough and fewer and less functional cilia. Mucous membranes tend to be drier. DIF: Cognitive Level: Apply (application) REF: 489 OBJ: Special Questions: Prioritization TOP: Nursing Process: Assessment MSC: NCLEX: Physiological Integrity

11. A patient who has a history of chronic obstructive pulmonary disease (COPD) was hospitalized for increasing shortness of breath and chronic hypoxemia (SaO2 levels of 89% to 90%). In planning for discharge, which action by the nurse will be most effective in improving compliance with discharge teaching? a. Start giving the patient discharge teaching on the day of admission. b. Have the patient repeat the instructions immediately after teaching. c. Accomplish the patient teaching just before the scheduled discharge. d. Arrange for the patient's caregiver to be present during the teaching.

ANS: D Hypoxemia interferes with the patient's ability to learn and retain information, so having the patient's caregiver present will increase the likelihood that discharge instructions will be followed. Having the patient repeat the instructions will indicate that the information is understood at the time, but it does not guarantee retention of the information. Because the patient is likely to be distracted just before discharge, giving discharge instructions just before discharge is not ideal. The patient is likely to be anxious and even more hypoxemic than usual on the day of admission, so teaching about discharge should be postponed. DIF: Cognitive Level: Apply (application) REF: 484 TOP: Nursing Process: Planning MSC: NCLEX: Physiological Integrity

21. Using the illustrated technique, the nurse is assessing for which finding in a patient with chronic obstructive pulmonary disease (COPD)? a. Hyperresonance b. Tripod positioning c. Accessory muscle use d. Reduced chest expansion

ANS: D The technique for palpation for chest expansion is shown in the illustrated technique. Reduced chest movement would be noted on palpation of a patient's chest with COPD. Hyperresonance would be assessed through percussion. Accessory muscle use and tripod positioning would be assessed by inspection. DIF: Cognitive Level: Understand (comprehension) REF: 486 TOP: Nursing Process: Assessment MSC: NCLEX: Physiological Integrity

19. The laboratory has just called with the arterial blood gas (ABG) results on four patients. Which result is most important for the nurse to report immediately to the health care provider? a. pH 7.34, PaO2 82 mm Hg, PaCO2 40 mm Hg, and O2 sat 97% b. pH 7.35, PaO2 85 mm Hg, PaCO2 45 mm Hg, and O2 sat 95% c. pH 7.46, PaO2 90 mm Hg, PaCO2 32 mm Hg, and O2 sat 98% d. pH 7.31, PaO2 91 mm Hg, PaCO2 50 mm Hg, and O2 sat 96%

ANS: D These ABGs indicate uncompensated respiratory acidosis and should be reported to the health care provider. The other values are normal or close to normal. DIF: Cognitive Level: Apply (application) REF: 479 OBJ: Special Questions: Prioritization TOP: Nursing Process: Implementation MSC: NCLEX: Physiological Integrity

5. The nurse palpates the posterior chest while the patient says "99" and notes absent fremitus. Which action should the nurse take next? a. Palpate the anterior chest and observe for barrel chest. b. Encourage the patient to turn, cough, and deep breathe. c. Review the chest x-ray report for evidence of pneumonia. d. Auscultate anterior and posterior breath sounds bilaterally.

ANS: D To assess for tactile fremitus, the nurse should use the palms of the hands to assess for vibration when the patient repeats a word or phrase such as "99." After noting absent fremitus, the nurse should then auscultate the lungs to assess for the presence or absence of breath sounds. Absent fremitus may be noted with pneumothorax or atelectasis. The vibration is increased in conditions such as pneumonia, lung tumors, thick bronchial secretions, and pleural effusion. Turning, coughing, and deep breathing is an appropriate intervention for atelectasis, but the nurse needs to first assess breath sounds. Fremitus is decreased if the hand is farther from the lung or the lung is hyperinflated (barrel chest).The anterior of the chest is more difficult to palpate for fremitus because of the presence of large muscles and breast tissue. DIF: Cognitive Level: Apply (application) REF: 486 TOP: Nursing Process: Assessment MSC: NCLEX: Physiological Integrity

A patient is admitted with symptoms of periorbital and facial edema, swelling of the hands and feet, bilateral crackles in the lungs, and reddish-brown urine. The patient reports having had a fever and sore throat 10 days prior to developing symptoms. The nurse suspects that this patient may have which condition?

Acute glomerulonephritis One complication of streptococcal pharyngitis is acute glomerulonephritis, which manifests about 7-10 days after the throat infection and is characterized by edema, fluid overload, and hematuria. Patients with streptococcal pharyngitis who do not improve with antibiotic therapy may have HIV and should be tested. A peritonsillar abscess is characterized by pain, swelling, and fever of the affected tonsil. Rheumatic fever is characterized by tremors, rash, and cardiovascular symptoms.

When caring for a patient with head and neck cancer following a total laryngectomy 12 hours ago, which potential complications are important for the nurse to address? Select all that apply.

Airway obstruction and inadequate oxygenation Comfort and nonverbal communication Wound breakdown and hemorrhage Significant potential complications after surgery for head and neck cancer include wound breakdown, airway obstruction/compromise, pain management, and adequate nonverbal communication. This is an extremely stressful time for patients and their families; attentiveness to these aspects of the recovery process can prevent complications and delayed recovery. Once the patient is past the early postoperative period, preparation for teaching about radiation therapy will be needed if it is a recommended treatment. The nurse will not teach a patient about chemotherapy unless this is the prescribed course of treatment. It is also not a priority during the early postoperative period.

The nurse reviews the medication administration record (MAR) for a patient having an acute asthma attack. Which medication should the nurse administer first?

Albuterol (Ventolin HFA) 2.5 mg per nebulizer

Which is not a cause of sleep apnea?

Alcohol Alcohol does not contribute to sleep apnea. Obesity, which is a result of unhealthy lifestyle practices, contributes to sleep apnea. Smoking is another major factor that leads to sleep apnea. Anatomical abnormalities, such as a short neck, also contributes to sleep apnea.

The nurse is teaching a patient about isoniazid (INH) and rifampin (RIF) drug therapy for tuberculosis (TB). The nurse instructs that while on these medications, the patient should avoid consuming which food?

Alcohol Isoniazid and rifampin can damage the liver, so alcohol should be avoided for the duration of the medication regimen, which can be 6 months to 2 years. Consuming foods high in tyramine while on these drugs can cause a severe increase in blood pressure. However, not all dairy products need to be avoided; only aged cheeses are high in tyramine. Red meat and eggs are not high in tyramine and can be consumed freely.

Which two factors in combination are the greatest risk factors for head and neck cancer?

Alcohol and tobacco use The combination of alcohol and tobacco use is one of the greatest risk factors for head and neck cancer. Chronic laryngitis and voice abuse in combination are not the greatest risk factors; however, each one individually is a risk factor for head and neck cancer. No large, randomized, controlled studies have identified a relationship between marijuana use and head and neck cancer. Exposure to industrial chemicals may increase a person's risk. Poor oral hygiene is a risk factor, as is chewing tobacco; however, no studies have reported that a combination of the two will lead to increased risk. The same cancer-causing agents in smoking tobacco may be present in smokeless (chewing) tobacco.

A patient is being evaluated for laryngeal cancer. Besides tobacco use, which aspect of the patient's history is important for the nurse to assess?

Alcohol consumption The two most important risk factors for head and neck cancer are tobacco and alcohol use, especially in combination. Dietary habits may need to be assessed in the chronic alcohol user as part of a comprehensive health evaluation, but do not necessarily contribute to development of cancer. Patients who have severe gastroesophageal reflux disease (GERD) have an increased risk for head and neck cancers. Shortness of breath may be a symptom in a patient with head and neck cancer.

The nurse understands which symptom to be a hallmark subjective sign of lung disease? A) Cough B) Dyspnea C) Chest pain D) Sputum production

Answer: A Rationale: Cough is a main sign of lung disease. Dyspnea (difficulty in breathing or breathlessness) is a subjective perception and varies among patients. A patient's feeling of dyspnea may not be consistent with the severity of the presenting problem. Sputum production may be associated with coughing and indicate an acute or chronic lung condition. Chest pain can occur with other health problems, as well as with lung problems.

The nurse is caring for a patient admitted for treatment of neck and throat cancer. Which intervention should the nurse perform? A) Encourage hydration with water. B) Feed the patient if coughing occurs. C) Encourage the patient to sit in a chair for meals. D) Encourage the patient to drink juice to address thirst.

Answer: C Rationale: Several interventions are necessary to reduce the risk of aspiration. Having the patient sit upright to eat is an important initial step to reduce aspiration. Other interventions include encouraging liquids that are "thick." Avoiding thin liquids like juice, water, and fruits that produce juice are important strategies to reduce aspiration risks. Coughing may be a sign of difficulty with swallowing or aspiration and requires additional assessment.

Which finding by the nurse for a patient with a nursing diagnosis of impaired gas exchange will be most useful in evaluating the effectiveness of treatment? A. Even, unlabored respirations B. Pulse oximetry reading of 92% C. Respiratory rate of 18 breaths/minute D. Absence of wheezes, rhonchi, or crackles

B. Pulse oximetry reading of 92%

A family member of a patient who has been diagnosed with severe acute respiratory syndrome (SARS) asks the nurse why the patient is not receiving an antibiotic. How does the nurse respond to this family member?

Antibiotics are not effective because SARS is caused by a virus. SARS is a viral infection and antibiotics are not useful for treating this disease. Patients are provided with supportive care to allow their immune systems to fight the disease. Antibiotics are given only when a secondary infection is present.

Excerise Tolerance change due to age-nursing interventions

Assess for subtle manifestations of hypoxia.

35. A nurse is caring for a patient who has been hospitalized with an acute asthma exacerbation. What drugs should the nurse expect to be ordered for this patient to gain underlying control of persistent asthma? A) Rescue inhalers B) Anti-inflammatory drugs C) Antibiotics D) Antitussives

B Feedback: Because the underlying pathology of asthma is inflammation, control of persistent asthma is accomplished primarily with regular use of anti-inflammatory medications. Rescue inhalers, antibiotics, and antitussives do not aid in the first-line control of persistent asthma.

A new ICU nurse is observed by her preceptor entering a patient's room to suction the tracheostomy after performing the task 15 minutes before. What should the preceptor educate the new nurse to do to ensure that the patient needs to be suctioned? a) Have the patient cough. b) Have the patient inform the nurse of the need to be suctioned. c) Auscultate the lung for adventitious sounds. d) Assess the CO2 level to determine if the patient requires suctioning.

Auscultate the lung for adventitious sounds. When a tracheostomy or endotracheal tube is in place, it is usually necessary to suction the patient's secretions because of the decreased effectiveness of the cough mechanism. Tracheal suctioning is performed when adventitious breath sounds are detected or whenever secretions are obviously present. Unnecessary suctioning can initiate bronchospasm and cause mechanical trauma to the tracheal mucosa.

Prevent aspiration (10)

Avoid consecutive swallowing Don't hurry Be awake Tuck chin Thicken liquids Stop coughing Dry swallow Sit upright Avoid foods that create thin liquid Suction PRN

3. A nurse is caring for a young adult patient whose medical history includes an alpha1-antitrypsin deficiency. This deficiency predisposes the patient to what health problem? A) Pulmonary edema B) Lobular emphysema C) Cystic fibrosis (CF) D) Empyema

B Feedback: A host risk factor for COPD is a deficiency of alpha1-antitrypsin, an enzyme inhibitor that protects the lung parenchyma from injury. This deficiency predisposes young patients to rapid development of lobular emphysema even in the absence of smoking. This deficiency does not influence the patients risk of pulmonary edema, CF, or empyema.

38. An asthma nurse educator is working with a group of adolescent asthma patients. What intervention is most likely to prevent asthma exacerbations among these patients? A) Encouraging patients to carry a corticosteroid rescue inhaler at all times B) Educating patients about recognizing and avoiding asthma triggers C) Teaching patients to utilize alternative therapies in asthma management D) Ensuring that patients keep their immunizations up to date

B Feedback: Asthma exacerbations are best managed by early treatment and education, including the use of written action plans as part of any overall effort to educate patients about self-management techniques, especially those with moderate or severe persistent asthma or with a history of severe exacerbations. Corticosteroids are not used as rescue inhalers. Alternative therapies are not normally a high priority, though their use may be appropriate in some cases. Immunizations should be kept up to date, but this does not necessarily prevent asthma exacerbations.

13. A nurse is developing the teaching portion of a care plan for a patient with COPD. What would be the most important component for the nurse to emphasize? A) Smoking up to one-half of a pack of cigarettes weekly is allowable. B) Chronic inhalation of indoor toxins can cause lung damage. C) Minor respiratory infections are considered to be self-limited and are not treated. D) Activities of daily living (ADLs) should be clustered in the early morning hours.

B Feedback: Environmental risk factors for COPD include prolonged and intense exposure to occupational dusts and chemicals, indoor air pollution, and outdoor air pollution. Smoking cessation should be taught to all patients who are currently smoking. Minor respiratory infections that are of no consequence to the person with normal lungs can produce fatal disturbances in the lungs of the person with emphysema. ADLs should be paced throughout the day to permit patients to perform these without excessive distress.

39. An asthma educator is teaching a patient newly diagnosed with asthma and her family about the use of a peak flow meter. The educator should teach the patient that a peak flow meter measures what value? A) Highest airflow during a forced inspiration B) Highest airflow during a forced expiration C) Airflow during a normal inspiration D) Airflow during a normal expiration

B Feedback: Peak flow meters measure the highest airflow during a forced expiration.

22. An older adult patient has been diagnosed with COPD. What characteristic of the patients current health status would preclude the safe and effective use of a metered-dose inhaler (MDI)? A) The patient has not yet quit smoking. B) The patient has severe arthritis in her hands. C) The patient requires both corticosteroids and beta2-agonists. D) The patient has cataracts.

B Feedback: Safe and effective MDI use requires the patient to be able to manipulate the device independently, which may be difficult if the patient has arthritis. Smoking does not preclude MDI use. A modest loss of vision does not preclude the use of an MDI and a patient can safely use more than one MDI.

21. A nurse is reviewing the pathophysiology of cystic fibrosis (CF) in anticipation of a new admission. The nurse should identify what characteristic aspects of CF? A) Alveolar mucus plugging, infection, and eventual bronchiectasis B) Bronchial mucus plugging, inflammation, and eventual bronchiectasis C) Atelectasis, infection, and eventual COPD D) Bronchial mucus plugging, infection, and eventual COPD

B Feedback: The hallmark pathology of CF is bronchial mucus plugging, inflammation, and eventual bronchiectasis. Commonly, the bronchiectasis begins in the upper lobes and progresses to involve all lobes. Infection, atelectasis, and COPD are not hallmark pathologies of CF.

24. An admitting nurse is assessing a patient with COPD. The nurse auscultates diminished breath sounds, which signify changes in the airway. These changes indicate to the nurse to monitor the patient for what? A) Kyphosis and clubbing of the fingers B) Dyspnea and hypoxemia C) Sepsis and pneumothorax D) Bradypnea and pursed lip breathing

B Feedback: These changes in the airway require that the nurse monitor the patient for dyspnea and hypoxemia. Kyphosis is a musculoskeletal problem. Sepsis and pneumothorax are atypical complications. Tachypnea is much more likely than bradypnea. Pursed lip breathing can relieve dyspnea.

26. A nurses assessment reveals that a client with COPD may be experiencing bronchospasm. What assessment finding would suggest that the patient is experiencing bronchospasm? A) Fine or coarse crackles on auscultation B) Wheezes or diminished breath sounds on auscultation C) Reduced respiratory rate or lethargy D) Slow, deliberate respirations

B Feedback: Wheezing and diminished breath sounds are consistent with bronchospasm. Crackles are usually attributable to other respiratory or cardiac pathologies. Bronchospasm usually results in rapid, inefficient breathing and agitation.

The admissions department at a local hospital is registering an elderly man for an outpatient diagnostic test. The admissions nurse asks the man if he has an advanced directive. The man responds that he does not want to complete an advance directive because he does not want anyone controlling his finances. What would be appropriate information for the nurse to share with this patient? Select one: A. "Advance directives are not legal documents, so you have nothing to worry about." B. "Advance directives are limited only to health care instructions and directives." C. "Your finances cannot be managed without an advance directive." D. "Advance directives are implemented when you become incapacitated, and then you will use a living will to allow the state to manage your money."

B. "Advance directives are limited only to health care instructions and directives." An advance directive is a formal, legally endorsed document that provides instructions for care (living will) or names a proxy decision maker (durable power of attorney for health care) and covers only issues related specifically to health care, not financial issues. They do not address financial issues. Advance directives are implemented when a patient becomes incapacitated, but financial issues are addressed with a durable power of attorney for finances, or financial power of attorney.

A young adult female patient with cystic fibrosis (CF) tells the nurse that she is not sure about getting married and having children some day. Which initial response by the nurse is best? A. "Are you aware of the normal lifespan for patients with CF?" B. "Do you need any information to help you with that decision?" C. "Many women with CF do not have difficulty conceiving children." D. "You will need to have genetic counseling before making a decision."

B. "Do you need any information to help you with that decision?"

What statement by a patient would indicate that a nurse had successfully implemented a teaching/learning strategy to prevent injury in the home? Select one: A. "I will turn off the outside lights and lock the doors every night." B. "I am going to remove all those throw rugs on the floor." C. "Well, I always let the boys play in the bathtub; they love it." D. "Do you think it would be best for me to buy a gun?"

B. "I am going to remove all those throw rugs on the floor." Nurses must evaluate the effectiveness of their interventions to promote safety and prevent injury. If the expected patient outcomes have been met and evaluative criteria satisfied, the patient should be able to correctly identify real and potential unsafe environmental situations and implement safety measures in the environment.

The nurse provides dietary teaching for a patient with chronic obstructive pulmonary disease (COPD) who has a low body mass index (BMI). Which patient statement indicates that the teaching has been effective? A. "I will drink lots of fluids with my meals." B. "I can have ice cream as a snack every day." C. "I will exercise for 15 minutes before meals." D. "I will decrease my intake of meat and poultry."

B. "I can have ice cream as a snack every day."

A nurse is teaching an older adult at home about taking newly prescribed medications. Which of the following would be included? Select one: A. "Don't worry if the label comes off; just look at the shapes." B. "I have written the names of your drugs with times to take them." C. "You won't forget a medication if you count them every day." D. "You can identify your medications by their color."

B. "I have written the names of your drugs with times to take them." Teach patients the names of drugs rather than distinguishing drugs by color. Manufacturers may vary the color of generic drugs, and the visual changes associated with aging may make it more difficult to identify medications by color. Medications should not be identified by counting or by shapes.

The nurse provides discharge instructions to a patient who was hospitalized for pneumonia. Which statement, if made by the patient, indicates a good understanding of the instructions? A. "I will call the doctor if I still feel tired after a week." B. "I will continue to do the deep breathing and coughing exercises at home." C. "I will schedule two appointments for the pneumonia and influenza vaccines." D. "I'll cancel my chest x-ray appointment if I'm feeling better in a couple weeks."

B. "I will continue to do the deep breathing and coughing exercises at home."

Despite the wishes of her family and the recommendations of the care team, a 70-year-old client with a diagnosis of congestive heart failure, but who is otherwise healthy, wants to have a no-code order in place. Which of the following statements by the care team most clearly prioritizes the patient's autonomy? Select one: A. "If this is what is best for everyone then we need to go ahead with the order." B. "If that's what she wants, then ultimately we're obliged to respect her wishes." C. "Provided it can be demonstrated that she has a potentially poor prognosis, we should certainly consider doing this." D. "It's best that social work get involved at this point to reconcile the family's and the patient's wishes."

B. "If that's what she wants, then ultimately we're obliged to respect her wishes." Answer D most clearly prioritizes the patient's individual freedom, preference, and rights in this case, and these considerations would override the family's or the care team's conflicting interests.

Nurse M is employed in an assisted living facility and is privy to many of the changes that accompany the aging process. An older female resident of the facility has expressed a fear that her decreased mobility will make her increasingly dependent on her daughter. How can the nurse best respond to the resident's concerns? Select one: A. "This is a normal part of the aging process and you don't necessarily need to fear it." B. "Many older adults have similar concerns. We can work together to keep you independent as long as possible." C. "There are treatments and drugs that we can explore which might prevent this from happening." D. "I'm sure this is very stressful for you, but it's fortunate that you are not experiencing severe pain or illness."

B. "Many older adults have similar concerns. We can work together to keep you independent as long as possible." Answer C validates the client's concerns and expresses the possibility of addressing the issue without downplaying it or providing unrealistic promises. Answer A provides an unrealistic promise of prevention, while answers B and D downplay the severity and significance of the client's concerns.

The nurse teaches a patient about the transmission of pulmonary tuberculosis (TB). Which statement, if made by the patient, indicates that teaching was effective? A. "I will avoid being outdoors whenever possible." B. "My husband will be sleeping in the guest bedroom." C. "I will take the bus instead of driving to visit my friends." D. "I will keep the windows closed at home to contain the germs."

B. "My husband will be sleeping in the guest bedroom."

Which instruction should the nurse include in an exercise teaching plan for a patient with chronic obstructive pulmonary disease (COPD)? A. "Stop exercising if you start to feel short of breath." B. "Use the bronchodilator before you start to exercise." C. "Breathe in and out through the mouth while you exercise." D. "Upper body exercise should be avoided to prevent dyspnea."

B. "Use the bronchodilator before you start to exercise."

The nurse monitors a patient after chest tube placement for a hemopneumothorax. The nurse is most concerned if which assessment finding is observed? A. A large air leak in the water-seal chamber B. 400 mL of blood in the collection chamber C. Complaint of pain with each deep inspiration D. Subcutaneous emphysema at the insertion site

B. 400 mL of blood in the collection chamber

The nurse receives change-of-shift report on the following four patients. Which patient should the nurse assess first? A. A 23-year-old patient with cystic fibrosis who has pulmonary function testing scheduled B. A 46-year-old patient on bed rest who is complaining of sudden onset of shortness of breath C. A 77-year-old patient with tuberculosis (TB) who has four antitubercular medications due in 15 minutes D. A 35-year-old patient who was admitted the previous day with pneumonia and has a temperature of 100.2° F (37.8° C)

B. A 46-year-old patient on bed rest who is complaining of sudden onset of shortness of breath

A nurse is caring for an 86-year-old female patient who has become increasingly frail and unsteady on her feet. During the assessment, the patient indicates that she has fallen three times in the month, though she has not yet suffered an injury. The nurse should take action in the knowledge that this patient is at a high risk for what health problem? Select one: A. Pelvic dysplasia B. A hip fracture C. Tearing of a meniscus or bursa D. A femoral fracture

B. A hip fracture The most common fracture resulting from a fall is a fractured hip resulting from osteoporosis and the condition or situation that produced the fall. The other listed injuries are possible, but less likely than a hip fracture.

The nurse receives a change-of-shift report on the following patients with chronic obstructive pulmonary disease (COPD). Which patient should the nurse assess first? A. A patient with loud expiratory wheezes B. A patient with a respiratory rate of 38/minute C. A patient who has a cough productive of thick, green mucus D. A patient with jugular venous distention and peripheral edema

B. A patient with a respiratory rate of 38/minute

After the nurse has received change-of-shift report, which patient should the nurse assess first? A. A patient with pneumonia who has crackles in the right lung base B. A patient with possible lung cancer who has just returned after bronchoscopy C. A patient with hemoptysis and a 16-mm induration with tuberculin skin testing D. A patient with chronic obstructive pulmonary disease (COPD) and pulmonary function testing (PFT) that indicates low forced vital capacity

B. A patient with possible lung cancer who has just returned after bronchoscopy

When caring for a patient who is hospitalized with active tuberculosis (TB), the nurse observes a student nurse who is assigned to take care of a patient. Which action, if performed by the student nurse, would require an intervention by the nurse? A. The patient is offered a tissue from the box at the bedside. B. A surgical face mask is applied before visiting the patient. C. A snack is brought to the patient from the unit refrigerator. D. Hand washing is performed before entering the patient's room.

B. A surgical face mask is applied before visiting the patient.

You are the nurse caring for an elderly patient who is being treated for community-acquired pneumonia. Since the time of admission, the patient has been disoriented and agitated to varying degrees. Appropriate referrals were made and the patient was subsequently diagnosed with dementia. What nursing diagnosis should the nurse prioritize when planning this patient's care? Select one: A. Social isolation related to dementia B. Acute confusion related to dementia C. Hopelessness related to dementia D. Risk for infection related to dementia

B. Acute confusion related to dementia Acute confusion is a priority problem in patients with dementia, and it is an immediate threat to their health and safety. Hopelessness and social isolation are plausible problems, but the patient's cognition is a priority. The patient's risk for infection is not directly influenced by dementia.

After 2 months of tuberculosis (TB) treatment with isoniazid (INH), rifampin (Rifadin), pyrazinamide (PZA), and ethambutol, a patient continues to have positive sputum smears for acid-fast bacilli (AFB). Which action should the nurse take next? A. Teach about treatment for drug-resistant TB treatment. B. Ask the patient whether medications have been taken as directed. C. Schedule the patient for directly observed therapy three times weekly. D. Discuss with the health care provider the need for the patient to use an injectable antibiotic.

B. Ask the patient whether medications have been taken as directed.

Following a laryngectomy a patient coughs violently during suctioning and dislodges the tracheostomy tube. Which action should the nurse take first? A. Cover stoma with sterile gauze and ventilate through stoma. B. Attempt to reinsert the tracheostomy tube with the obturator in place. C. Assess the patient's oxygen saturation and notify the health care provider. D. Ventilate the patient with a manual bag and face mask until the health care provider arrives.

B. Attempt to reinsert the tracheostomy tube with the obturator in place.

A patient with acute shortness of breath is admitted to the hospital. Which action should the nurse take during the initial assessment of the patient? A. Delay the physical assessment to first complete pulmonary function tests. B. Briefly ask specific questions about this episode of respiratory distress. C.Complete the admission database to check for allergies before treatment. D. Delay the physical assessment to first complete pulmonary function tests.

B. Briefly ask specific questions about this episode of respiratory distress.

A client is admitted to the cardiothoracic surgical ICU after cardiac bypass surgery. The client is still sedated on a ventilator and has an arterial catheter in the right wrist. What assessment does the nurse make to determine patency of the client's arterial line? A. Blood pressure B. Capillary refill and pulse C. Neurologic function D. Questioning the client about the pain level at the site

B. Capillary refill and pulse Capillary refill and pulse should be assessed to ensure that the arterial line is not occluding the artery. Blood pressure and neurologic function are not pertinent to the client's arterial line. Although the client's comfort level is important with an arterial line, it is not a determinant of patency of the line.

The nurse is admitting clients to the same-day surgery unit. Which insertion site for routine peripheral venous catheters does the nurse choose most often? A. Back of the hand for an older adult B. Cephalic vein of the forearm C. Lower arm on the side of a radical mastectomy D. Subclavian vein

B. Cephalic vein of the forearm For same-day surgery, the cephalic or basilic vein allows insertion of a larger IV catheter while allowing movement of the arm without impairing intravenous flow. Peripheral venous catheters should never be inserted into the back of the hand in an older adult because the veins are brittle. Peripheral venous catheters should never be inserted into the lower arm on the same side as a radical mastectomy because they interfere with limited circulation. Catheters are typically inserted into the subclavian vein by the health care provider, not by the nurse.

A client who is receiving IV antibiotic treatments every 6 hours has an intermittent IV set that was opened and begun 20 hours ago. What action does the nurse take? A. Change the set immediately. B. Change the set in about 4 hours. C. Change the set in the next 12 to 24 hours. D. Nothing; the set is for long-term use.

B. Change the set in about 4 hours. Because both ends of the set are being manipulated with each dose, standards of practice dictate that the set should be changed every 24 hours, so the set should be changed in about 4 hours. It is not necessary to change out the set immediately, but it must be changed before the next 12 to 24 hours.

The nurse is administering a drug to a client through an implanted port. Before giving the medication, what does the nurse do to ensure safety? A. Administer 5 mL of a heparinized solution. B. Check for blood return. C. Flush the port with 10 mL of normal saline. D. Palpate the port for stability.

B. Check for blood return. Before a drug is given through an implanted port, it is critical that the nurse check for blood return. If no blood return is observed, the drug should be held until patency is reestablished. Ports are flushed with heparin or saline after, rather than before, use. The port is palpated for stability, but this action alone does not ensure the client's safety.

The nurse is documenting peripheral venous catheter insertion for a client. What does the nurse include in the note? (Select all that apply.) A. Client's name and hospital number B. Client's response to the insertion C. Date and time inserted D. Type and size of device E. Type of dressing applied F. Vein used for insertion

B. Client's response to the insertion C. Date and time inserted D. Type and size of device E. Type of dressing applied F. Vein used for insertion The client's ability to adapt to interventions, such as IV insertion, should be noted when the intervention is performed. The date and time of the insertion are important data. IV sites need to be routinely monitored and changed at prescribed intervals per facility policy. It is important to note the device used (often the brand name is given), as well as all specifics such as needle or cannula length, gauge, and material (Teflon). It is necessary to describe the dressing applied, and the vein used should be noted. The client's name and hospital number should be on the medical record, but the nurse makes certain that the information is recorded in the correct medical record.

The nurse checking an IV fluid order questions its accuracy. What does the nurse do first? A. Asks the charge nurse about the order B. Contacts the health care provider who ordered it C. Contacts the pharmacy for clarification D. Starts the fluid as ordered, with plans to check it later

B. Contacts the health care provider who ordered it The nurse is responsible for accuracy and has the duty to verify the order with the health care provider who ordered it. Although the nurse can consult the charge nurse, this is not the definitive action that the nurse should take. Contacting the pharmacy is not the definitive action that the nurse should take. Giving (or starting) the fluid when the order is questionable is not appropriate.

The nurse obtains the following assessment data on an older patient who has influenza. Which information will be most important for the nurse to communicate to the health care provider? A. Fever of 100.4° F (38° C) B. Diffuse crackles in the lungs C. Sore throat and frequent cough D. Myalgia and persistent headache

B. Diffuse crackles in the lungs

The nurse is to administer a unit of whole blood to a postoperative client. What does the nurse do to ensure the safety of the blood transfusion? A. Asks the client to both say and spell his or her full name before starting the blood transfusion B. Ensures that another qualified health care professional checks the unit before administering C. Checks the blood identification numbers with the laboratory technician at the blood bank at the time it is dispensed D. Makes certain that an IV solution of 0.9% normal saline is infusing into the client before starting the unit

B. Ensures that another qualified health care professional checks the unit before administering To ensure safety, blood must be checked by two qualified health care professionals, usually two registered nurses. Administering an incorrectly matched unit of blood creates great consequences for the client and is considered to be a sentinel event. It requires a great amount of follow-up and often changing of policies to improve safety. The Joint Commission requires that the client provide two identifiers, but they are the name and date of birth or some other identifying data, depending on the facility; saying and spelling the name is only one identifier. Although a check is provided at the blood bank, this is not the one that is done before administration to the client. Clients do need to have normal saline running with blood, but this is not considered to be part of the safety check before administration of blood and blood products.

A patient admitted with right leg thrombophlebitis is to be discharged from an acute-care facility. Following treatment with a heparin infusion, the nurse notes that the patient's leg is pain-free, without redness or edema. Which step of the nursing process does this reflect? Select one: A. Diagnosis B. Evaluation C. Analysis D. Implementation

B. Evaluation The nursing actions described constitute evaluation of the expected outcomes. The findings show that the expected outcomes have been achieved. Analysis consists of considering assessment information to derive the appropriate nursing diagnosis. Implementation is the phase of the nursing process where the nurse puts the care plan into action. This nurse's actions do not constitute diagnosis.

Which assessment finding in a patient who has received omalizumab (Xolair) is most important to report immediately to the health care provider? A. Pain at injection site B. Flushing and dizziness C. Peak flow reading 75% of normal D. Respiratory rate 22 breaths/minute

B. Flushing and dizziness

An adult patient has requested a "do not resuscitate" (DNR) order in light of his recent diagnosis with late stage pancreatic cancer. The patient's son and daughter-in-law are strongly opposed to the patient's request. What is the primary responsibility of the nurse in this situation? Select one: A. Contact a social worker or mediator to intervene. B. Honor the request of the patient. C. Perform a "slow code" until a decision is made. D. Temporarily withhold nursing care until the physician talks to the family.

B. Honor the request of the patient. The nurse must honor the patient's wishes and continue to provide required nursing care. Discussing the matter with the physician may lead to further communication with the family, during which the family may reconsider their decision. It is not normally appropriate for the nurse to seek the assistance of a social worker or mediator. A "slow code" is considered unethical.

The nurse is revising an agency's recommended central line catheter-related bloodstream infection prevention (CR-BSI) bundle. Which actions decrease the client's risk for this complication? (Select all that apply.) A. During insertion, draping the area around the site with a sterile barrier B. Immediately removing the client's venous access device (VAD) when it is no longer needed C. Making certain that observers of the insertion are instructed to look away during the procedure D. Thorough hand hygiene (i.e., no quick scrub) before insertion E. Using chlorhexidine for skin disinfection

B. Immediately removing the client's venous access device (VAD) when it is no longer needed D. Thorough hand hygiene (i.e., no quick scrub) before insertion E. Using chlorhexidine for skin disinfection As soon as the VAD is deemed unnecessary, it should be removed to reduce the risk for infection. Thorough handwashing is a key factor in insertion and maintenance of a central line device; quick handwashing is not sufficient. Chlorhexidine is recommended for skin disinfection because it has been shown to have the best outcomes in infection prevention. During the insertion, the whole body (head to toe) of the client is draped with a sterile barrier; draping only the area around the site will increase risk for infection. Looking away will not reduce the risk for infection. Reducing the number of people in the room and having everyone wear a mask will help reduce the risk for infection.

An elderly patient has come in to the clinic for her twice-yearly physical. The patient tells the nurse that she is generally enjoying good health, but that she has been having occasional episodes of constipation over the past 6 months. What intervention should the nurse first suggest? Select one: A. Increase carbohydrate intake and reduce protein intake. B. Increase daily intake of water. C. Take herbal laxatives, such as senna, each night at bedtime. D. Reduce the amount of stress she currently experiences.

B. Increase daily intake of water. Constipation is a common problem in older adults and increasing fluid intake is an appropriate early intervention. This should likely be attempted prior to recommending senna or other laxatives. Stress reduction is unlikely to wholly resolve the problem and there is no need to increase carbohydrate intake and reduce protein intake.

Older people have many altered reactions to disease that are based on age-related physiological changes. When the nurse observes physical indicators of illness in the older population, that nurse must remember which of the following principles? Select one: A. The same physiological processes that indicate serious health care problems in a younger population indicate mild disease states in the elderly. B. Indicators that are useful and reliable in younger populations cannot be relied on as indications of potential life-threatening problems in older adults. C. Potential life-threatening problems in the older adult population are not as serious as they are in a middle-aged population. D. Middle-aged people do not react to disease states the same as a younger population does.

B. Indicators that are useful and reliable in younger populations cannot be relied on as indications of potential life-threatening problems in older adults. Physical indicators of illness that are useful and reliable in young and middle-aged people cannot be relied on for the diagnosis of potential life-threatening problems in older adults. Option A is incorrect because a potentially life-threatening problem in an older person is more serious than it would be in a middle-aged person because the older adult does not have the physical resources of the middle-aged person. Physical indicators of serious health care problems in a young or middle-aged population do not indicate disease states that are considered "mild" in the elderly population. It is true that middle-aged people do not react to disease states the same as a younger population, but this option does not answer the question.

You are writing a care plan for an 85-year-old patient who has community-acquired pneumonia and you note decreased breath sounds to bilateral lung bases on auscultation. What is the most appropriate nursing diagnosis for this patient? Select one: A. Poor ventilation related to acute lung infection B. Ineffective airway clearance related to tracheobronchial secretions C. Pneumonia related to progression of disease process D. Immobility related to fatigue

B. Ineffective airway clearance related to tracheobronchial secretions Nursing diagnoses are not medical diagnoses or treatments. The most appropriate nursing diagnosis for this patient is "ineffective airway clearance related to copious tracheobronchial secretions." "Pneumonia" and "poor ventilation" are not nursing diagnoses. Immobility is likely, but is less directly related to the patient's admitting medical diagnosis and the nurse's assessment finding.

The clinic nurse makes a follow-up telephone call to a patient with asthma. The patient reports having a baseline peak flow reading of 600 L/minute and the current peak flow is 420 L/minute. Which action should the nurse take first? A. Tell the patient to go to the hospital emergency department. B. Instruct the patient to use the prescribed albuterol (Proventil). C. Ask about recent exposure to any new allergens or asthma triggers. D. Question the patient about use of the prescribed inhaled corticosteroids.

B. Instruct the patient to use the prescribed albuterol (Proventil).

A patient is receiving 35% oxygen via a Venturi mask. To ensure the correct amount of oxygen delivery, which action by the nurse is most important? A. Teach the patient to keep mask on at all times. B. Keep the air entrainment ports clean and unobstructed. C. Give a high enough flow rate to keep the bag from collapsing. D. Drain moisture condensation from the oxygen tubing every hour.

B. Keep the air entrainment ports clean and unobstructed.

A patient with a chronic cough has a bronchoscopy. After the procedure, which intervention by the nurse is most appropriate? A. Elevate the head of the bed to 80 to 90 degrees. B. Keep the patient NPO until the gag reflex returns. C. Place on bed rest for at least 4 hours after bronchoscopy. D. Notify the health care provider about blood-tinged mucus.

B. Keep the patient NPO until the gag reflex returns.

Which action is appropriate for the nurse to delegate to unlicensed assistive personnel (UAP)? A. Listen to a patient's lung sounds for wheezes or rhonchi. B. Label specimens obtained during percutaneous lung biopsy. C. Instruct a patient about how to use home spirometry testing. D. Measure induration at the site of a patient's intradermal skin test.

B. Label specimens obtained during percutaneous lung biopsy.

Nurses provide many interventions to prevent falls in healthcare settings. Which of the following would be an appropriate fall-prevention intervention? Select one: A. Keep bed in the high position. B. Lock wheels on beds and wheelchairs. C. Apply restraints to all confused patients. D. Keep side rails up at all times.

B. Lock wheels on beds and wheelchairs. Locking wheels on beds and wheelchairs prevents them from rolling and precipitating a fall. Beds should be kept in low positions with the side rails down in most situations; restraints should be applied only as a last resort.

The nurse is admitting a patient diagnosed with an acute exacerbation of chronic obstructive pulmonary disease (COPD).What is the best way for the nurse to determine the appropriate oxygen flow rate? A. Minimize oxygen use to avoid oxygen dependency. B. Maintain the pulse oximetry level at 90% or greater. C. Administer oxygen according to the patient's level of dyspnea. D. Avoid administration of oxygen at a rate of more than 2 L/minute.

B. Maintain the pulse oximetry level at 90% or greater.

Gerontological nursing will become an increasingly important profession, compared with in the past, because: Select one: A. A greater number of people are surviving the previously hazardous period of infancy B. More people are spending a longer time span in old age C. More people will be presenting with the same health care challenges D. More elderly are living in increasingly squalid living conditions

B. More people are spending a longer time span in old age More people are achieving and spending longer periods of time in old age than ever before in history. Declines in living conditions, increased prevalence and incidence of the same health problems, and higher survival rates during infancy do not account for the increased importance of gerontological nursing.

A nurse has begun creating a patient's plan of care shortly after the patient's admission. It is important that the wording of the chosen nursing diagnoses falls within the taxonomy of nursing. Which organization is responsible for developing the taxonomy of a nursing diagnosis? Select one: A. Joint Commission B. NANDA C. National League for Nursing (NLN) D. American Nurses Association (ANA)

B. NANDA NANDA International is the official organization responsible for developing the taxonomy of nursing diagnoses and formulating nursing diagnoses acceptable for study. The ANA, NLN, and Joint Commission are not charged with the task of developing the taxonomy of nursing diagnoses.

The nurse prevents and treats pressure ulcers in the at-risk population of immobile elders. Which of the following interventions will best treat a stage 1 pressure ulcer? Select one: A. Thoroughly clean and irrigate the lesion. B. Off-load the area under pressure. C. Cover with a hydrogel sheet like Vigilon. D. Clean with normal saline three times a day.

B. Off-load the area under pressure. With hyperemia the redness of the skin can disappear quickly if pressure is removed. Vigilon is a high moisture content dressing appropriate for open lesion. The skin should be kept clean and dry using lotions to keep the skin soft.

The emergency department nurse is evaluating the effectiveness of therapy for a patient who has received treatment during an asthma attack. Which assessment finding is the best indicator that the therapy has been effective? A. No wheezes are audible. B. Oxygen saturation is >90%. C. Accessory muscle use has decreased. D. Respiratory rate is 16 breaths/minute.

B. Oxygen saturation is >90%.

A patient is scheduled for a computed tomography (CT) of the chest with contrast media. Which assessment findings should the nurse immediately report to the health care provider (select all that apply)? A. Patient is claustrophobic. B. Patient is allergic to shellfish. C. Patient recently used a bronchodilator inhaler. D. Patient is not able to remove a wedding band. E. Blood urea nitrogen (BUN) and serum creatinine levels are elevated.

B. Patient is allergic to shellfish. E. Blood urea nitrogen (BUN) and serum creatinine levels are elevated.

A patient with idiopathic pulmonary arterial hypertension (IPAH) is receiving nifedipine (Procardia). Which assessment would best indicate to the nurse that the patient's condition is improving? A. Blood pressure (BP) is less than 140/90 mm Hg. B. Patient reports decreased exertional dyspnea. C. Heart rate is between 60 and 100 beats/minute. D. Patient's chest x-ray indicates clear lung fields.

B. Patient reports decreased exertional dyspnea.

A patient with a pleural effusion is scheduled for a thoracentesis. Which action should the nurse take to prepare the patient for the procedure? A. Start a peripheral IV line to administer the necessary sedative drugs. B. Position the patient sitting upright on the edge of the bed and leaning forward. C. Obtain a large collection device to hold 2 to 3 liters of pleural fluid at one time. D. Remove the water pitcher and remind the patient not to eat or drink anything for 6 hours.

B. Position the patient sitting upright on the edge of the bed and leaning forward.

A terminally ill patient you are caring for is complaining of pain. The physician has ordered a large dose of intravenous opioids by continuous infusion. You know that one of the adverse effects of this medicine is respiratory depression. When you assess your patient's respiratory status, you find that the rate has decreased from 16 breaths per minute to 10 breaths per minute. What action should you take? Select one: A. Stimulate the patient in order to increase respiratory rate. B. Report the decreased respiratory rate to the physician. C. Allow the patient to rest comfortably. D. Decrease the rate of IV infusion.

B. Report the decreased respiratory rate to the physician. End-of life issues that often involve ethical dilemmas include pain control, "do not resuscitate" orders, life-support measures, and administration of food and fluids. The risk of respiratory depression is not the intent of the action of pain control. Respiratory depression should not be used as an excuse to withhold pain medication for a terminally ill patient. The patient's respiratory status should be carefully monitored and any changes should be reported to the physician.

When administering a proton pump inhibitor to a patient with gastroesophageal reflux disease (GERD), the nurse notes that the patient has great difficulty swallowing the enteric-coated pill. What should the nurse do when administering this medication to the patient in the future? Select one: A. Provide an herbal alternative that also reduces stomach acid production B. Reposition the patient and provide more fluid when giving the pill C. Crush the pill and mix with applesauce D. Split the pill in two parts and give each separately

B. Reposition the patient and provide more fluid when giving the pill Since enteric-coated pills should not be crushed or split, the nurse's best alternative is to reposition the patient and provide more fluid to aid with swallowing. It would be inappropriate to provide a nonpharmacologic alternative to the prescribed medication.

Which statement is true about the special needs of older adults receiving IV therapy? A. Placement of the catheter on the back of the client's dominant hand is preferred. B. Skin integrity can be compromised easily by the application of tape or dressings. C. To avoid rolling the veins, a greater angle of 25 degrees between the skin and the catheter will improve success with venipuncture. D. When the catheter is inserted into the forearm, excess hair should be shaved before insertion.

B. Skin integrity can be compromised easily by the application of tape or dressings. Skin in older adults tends to be thin. Tape or dressings used with IV therapy can compromise skin integrity. Placement on the back of the dominant hand is contraindicated because hand movement can increase the risk of catheter dislodgement. An angle smaller than 25 degrees is required for venipuncture success in older adults. This technique is less likely to puncture through the older adult client's vein. Clipping the hair around the insertion site typically is necessary only for younger men.

You have been referred to the care of an extended care resident who has been diagnosed with a stage III pressure ulcer. You are teaching staff at the facility about the role of nutrition in wound healing. What would be the best meal choice for this patient? Select one: A. Eggs, hash browns, coffee, and an apple B. Steak, baked potato, spinach and strawberry salad C. Skim milk, oatmeal, and whole wheat toast D. Whole wheat macaroni with cheese

B. Steak, baked potato, spinach and strawberry salad The patient should be encouraged to eat foods high in protein, carbohydrates and vitamins A, B, and C. A meal of steak, baked potato, spinach and strawberry salad best exemplifies this dietary balance.

Which nursing action could the registered nurse (RN) working in a skilled care hospital unit delegate to an experienced licensed practical/vocational nurse (LPN/LVN) caring for a patient with a permanent tracheostomy? A. Assess the patient's risk for aspiration. B. Suction the tracheostomy when needed. C. Teach the patient about self-care of the tracheostomy. D. Determine the need for replacement of the tracheostomy tube.

B. Suction the tracheostomy when needed.

A patient is admitted to the emergency department with an open stab wound to the left chest. What is the first action that the nurse should take? A. Position the patient so that the left chest is dependent. B. Tape a nonporous dressing on three sides over the chest wound C. Cover the sucking chest wound firmly with an occlusive dressing. D. Keep the head of the patient's bed at no more than 30 degrees elevation.

B. Tape a nonporous dressing on three sides over the chest wound

A patient with cystic fibrosis (CF) has blood glucose levels that are consistently between 180 to 250 mg/dL. Which nursing action will the nurse plan to implement? A. Discuss the role of diet in blood glucose control. B. Teach the patient about administration of insulin. C. Give oral hypoglycemic medications before meals. D. Evaluate the patient's home use of pancreatic enzymes.

B. Teach the patient about administration of insulin.

A patient with chronic obstructive pulmonary disease (COPD) has rhonchi throughout the lung fields and a chronic, nonproductive cough. Which nursing intervention will be most effective? A. Change the oxygen flow rate to the highest prescribed rate. B. Teach the patient to use the Flutter airway clearance device. C. Reinforce the ongoing use of pursed lip breathing techniques. D. Teach the patient about consistent use of inhaled corticosteroids.

B. Teach the patient to use the Flutter airway clearance device.

The child of a nursing home resident complains to the nurse that his mother is required to get out of bed and do simple morning exercises when she would rather sleep. Which of the following rationales is the most appropriate for the nurse to state to the relative? Select one: A. The exercise is needed for preventing pressure ulcers. B. The exercise is needed for promoting circulation. C. The nurse's aide can make the bed more easily. D. The exercise is needed for psychological health.

B. The exercise is needed for promoting circulation. All of these choices are valid reasons for asking a patient to get out of bed, but A is the best one. Without exercise, the resident is likely to develop varicose veins and stasis ulcers. A bed-ridden patient can be turned to prevent pressure ulcers, and making the bed is possible, though more difficult, while the patient is in it. Psychological health is aided by exercise, but that can be promoted in other ways (such as getting enough sleep) as well.

The nurse is providing care for a 90-year-old patient whose severe cognitive and mobility deficits result in the nursing diagnosis of risk for impaired skin integrity due to lack of mobility. When planning relevant assessments, the nurse should prioritize inspection of what area? Select one: A. The patient's elbows B. The patient's heels C. The patient's knees D. The soles of the patient's feet

B. The patient's heels Full inspection of the patient's skin is necessary, but the coccyx and the heels are the most susceptible areas for skin breakdown due to shear and friction.

When patients are pulled up in bed rather than lifted, they are at increased risk for the development of a decubitus ulcer. What is the name given to the factor responsible for this risk? Select one: A. ischemia B. shearing force C. friction D. necrosis of tissue

B. shearing force A shearing force results when one layer of tissue slides over another layer. Patients who are pulled rather than lifted when being moved up in bed or from bed to chair to stretcher are at risk for injury from shearing forces.

Nasal Polyps

Benign, grapelike clusters of mucous membranes and connective tissue Large polyps can obstruct airway Treatment: inhaled steroids, polypectomy

Nasal Polyps

Benign, grapelike clusters of mucous membranes and connective tissue Large polyps may obstruct nasal airway Manifestations: Obstructed nasal breathing Increased nasal discharge Change in voice quality Most managed with inhaled steroids Polypectomy treatment of choice

Nasal Polyps

Benign, grapelike clusters of mucous membranes and connective tissue Large polyps may obstruct nasal airway Most managed with inhaled steroids Polypectomy treatment of choice

7. A school nurse is caring for a 10-year-old girl who is having an asthma attack. What is the preferred intervention to alleviate this clients airflow obstruction? A) Administer corticosteroids by metered dose inhaler B) Administer inhaled anticholinergics C) Administer an inhaled beta-adrenergic agonist D) Utilize a peak flow monitoring device

C Feedback: Asthma exacerbations are best managed by early treatment and education of the patient. Quick-acting beta-adrenergic medications are the first used for prompt relief of airflow obstruction. Systemic corticosteroids may be necessary to decrease airway inflammation in patients who fail to respond to inhaled beta-adrenergic medication. A peak flow device will not resolve short-term shortness of breath.

19. A nurse is providing discharge teaching for a client with COPD. When teaching the client about breathing exercises, what should the nurse include in the teaching? A) Lie supine to facilitate air entry B) Avoid pursed lip breathing C) Use diaphragmatic breathing D) Use chest breathing

C Feedback: Inspiratory muscle training and breathing retraining may help improve breathing patterns in patients with COPD. Training in diaphragmatic breathing reduces the respiratory rate, increases alveolar ventilation, and, sometimes, helps expel as much air as possible during expiration. Pursed-lip breathing helps slow expiration, prevents collapse of small airways, and controls the rate and depth of respiration. Diaphragmatic breathing, not chest breathing, increases lung expansion. Supine positioning does not aid breathing.

32. A patients severe asthma has necessitated the use of a long-acting beta2-agonist (LABA). Which of the patients statements suggests a need for further education? A) I know that these drugs can sometimes make my heart beat faster. B) Ive heard that this drug is particularly good at preventing asthma attacks during exercise. C) Ill make sure to use this each time I feel an asthma attack coming on. D) Ive heard that this drug sometimes gets less effective over time.

C Feedback: LABAs are not used for management of acute asthma symptoms. Tachycardia is a potential adverse effect and decreased protection against exercise-induced bronchospasm may occur with regular use.

11. A patient arrives in the emergency department with an attack of acute bronchiectasis. Chest auscultation reveals the presence of copious secretions. What intervention should the nurse prioritize in this patients care? A) Oral administration of diuretics B) Intravenous fluids to reduce the viscosity of secretions C) Postural chest drainage D) Pulmonary function testing

C Feedback: Postural drainage is part of all treatment plans for bronchiectasis, because draining of the bronchiectatic areas by gravity reduces the amount of secretions and the degree of infection. Diuretics and IV fluids will not aid in the mobilization of secretions. Lung function testing may be indicated, but this assessment will not relieve the patients symptoms.

12. A nurse is completing a focused respiratory assessment of a child with asthma. What assessment finding is most closely associated with the characteristic signs and symptoms of asthma? A) Shallow respirations B) Increased anterior-posterior (A-P) diameter C) Bilateral wheezes D) Bradypnea

C Feedback: The three most common symptoms of asthma are cough, dyspnea, and wheezing. There may be generalized wheezing (the sound of airflow through narrowed airways), first on expiration and then, possibly, during inspiration as well. Respirations are not usually slow and the childs A-P diameter does not normally change.

A patient comes to the emergency room presenting with dyspnea, tachycardia, violent agitation, tracheal deviation, neck vein distension, and hyperresonance to percussion. Which condition should the nurse suspect? A. Hemothorax B. Flail chest C. Tension pneumothorax D. Cardiac tamponade

C RATIONALE: Tension pneumothorax is the result of increased air in the pleural space and causes shifting of bodily organs and an increase in intrathoracic pressure. The patient usually presents with cyanosis, air hunger, violent agitation, tracheal deviation, neck vein distension, and hyperresonance to percussion. Hemothorax is an accumulation of blood in the pleural space, and the patient usually presents with dyspnea, diminished breath sounds, dullness to percussion, and shock, depending on blood loss. Flail chest is a fracture of two or more ribs, and the patient presents with paradoxical movement of the chest wall and respiratory distress. Cardiac tamponade occurs when blood collects in the pericardial sac, and the patient presents with muffled, distant heart sounds, hypotension, neck vein distension, and increased central venous pressure.

The nurse teaches a patient about pulmonary function testing (PFT). Which statement, if made by the patient, indicates teaching was effective? A. "I will use my inhaler right before the test." B. "I won't eat or drink anything 8 hours before the test." C. "I should inhale deeply and blow out as hard as I can during the test." D. "My blood pressure and pulse will be checked every 15 minutes after the test."

C. "I should inhale deeply and blow out as hard as I can during the test."

The nurse is inserting a peripheral IV catheter. Which client statement is of greatest concern during this procedure? A. "I hate having IVs started." B. "It hurts when you are inserting the line." C. "My hand tingles when you poke me." D. "My IV lines never last very long."

C. "My hand tingles when you poke me." The client's statement about a tingling feeling indicates possible nerve puncture. To avoid further nerve damage, the nurse should stop immediately, remove the IV catheter, and choose a new site. The other statements indicate a need for client teaching, but are not indicators of immediate complications of catheter insertion—pain at the insertion site is common, and IV sites that "never last very long" should be addressed with teaching about the importance of proper protection of the site.

The nurse provides preoperative instruction for a patient scheduled for a left pneumonectomy for cancer of the lung. Which information should the nurse include about the patient's postoperative care? A. Positioning on the right side B. Bed rest for the first 24 hours C. Frequent use of an incentive spirometer D. Chest tube placement with continuous drainage

C. Frequent use of an incentive spirometer

A client who used to work as a nurse asks, "Why is the hospital using a 'fancy new IV' without a needle? That seems expensive." How does the nurse respond? A. "OSHA, a government agency, requires us to use this new type of IV." B. "These systems are designed to save time, not money." C. "They minimize health care workers' exposure to contaminated needles." D. "They minimize clients' exposure to contaminated needles."

C. "They minimize health care workers' exposure to contaminated needles." Needleless IVs were designed to protect health care personnel from exposure to contaminated needles. The Occupational Safety and Health Administration (OSHA) requires the use of devices with engineered safety mechanisms only. It does not mandate that they be needleless. Saving time and money is not the purpose of the needleless IV, and it was not designed to protect clients from exposure to contaminated needles.

You are providing care for a patient who has a diagnosis of pneumonia attributed to Streptococcus pneumonia infection. Which of the following aspects of nursing care would constitute part of the planning phase of the nursing process? Select one: A. Auscultate chest q4h. B. Administer oral fluids q1h and PRN. C. Achieve SaO2 ≥ 92% at all times. D. Avoid overexertion at all times.

C. Achieve SaO2 ≥ 92% at all times. The planning phase entails specifying the immediate, intermediate, and long-term goals of nursing action, such as maintaining a certain level of oxygen saturation in a patient with pneumonia. Providing fluids and avoiding overexertion are parts of the implementation phase of the nursing process. Chest auscultation is an assessment.

A patient seen in the asthma clinic has recorded daily peak flows that are 75% of the baseline. Which action will the nurse plan to take next? A. Increase the dose of the leukotriene inhibitor. B. Teach the patient about the use of oral corticosteroids. C. Administer a bronchodilator and recheck the peak flow. D. Instruct the patient to keep the next scheduled follow-up appointment.

C. Administer a bronchodilator and recheck the peak flow.

An elderly woman diagnosed with osteoarthritis has been referred for care. The patient has difficulty ambulating because of chronic pain. When creating a nursing care plan, what intervention may the nurse use to best promote the patient's mobility? Select one: A. Encourage the patient to push through the pain in order to gain further mobility. B. Motivate the patient to walk in the afternoon rather than the morning. C. Administer an analgesic as ordered to facilitate the patient's mobility. D. Have another person with osteoarthritis visit the patient.

C. Administer an analgesic as ordered to facilitate the patient's mobility. At times, mobility is restricted because of pain, paralysis, loss of muscle strength, systemic disease, an immobilizing device (e.g., cast, brace), or prescribed limits to promote healing. If mobility is restricted because of pain, providing pain management through the administration of an analgesic will increase the patient's level of comfort during ambulation and allow the patient to ambulate. Motivating the patent or having another person with the same diagnosis visit is not an intervention that will help with mobility. The patient should not be encouraged to "push through the pain."

While caring for a patient with respiratory disease, the nurse observes that the patient's SpO2 drops from 93% to 88% while the patient is ambulating in the hallway. What is the priority action of the nurse? A. Notify the health care provider. B. Document the response to exercise. C. Administer the PRN supplemental O2. D. Encourage the patient to pace activity.

C. Administer the PRN supplemental O2.

A client is seen in the ER with pain, redness, and warmth of the right lower arm. The client was in the ER last week after an accident at work. On the day of the injury, the client was in the ER for 12 hours receiving IV fluids. On close examination, the nurse notes the presence of a palpable cord 1 inch in length and streak formation. How does the nurse classify this client's phlebitis? A. Grade 1 B. Grade 2 C. Grade 3 D. Grade 4

C. Grade 3 Grade 1: only erythema with or without pain Grade 2: erythema WITH pain and/or edema Grade 3: erythema WITH pain and/or edema and streak formation with a palpable cord. Grade 4: erythema WITH pain and/or edema and streak formation with a palpable cord than 1 inch, and purulent drainage. No purulent drainage is present in this client, and the palpable cord is 1 inch in length.

A nurse is teaching medication safety to a group of healthy older adults at a community center. The nurse explains that due to the high frequency of food-drug interactions, the clients should avoid taking their medications with which of the following? Select one: A. Orange juice B. Milk C. Grapefruit juice D. Carbonated beverages

C. Grapefruit juice

A recent nursing graduate is aware of the differences between nursing actions that are independent and nursing actions that are interdependent. A nurse performs an interdependent nursing intervention when performing which of the following actions? Select one: A. Providing mouth care to a patient who is unconscious following a cerebrovascular accident B. Auscultating a patient's apical heart rate during an admission assessment C. Administering an IV bolus of normal saline to a patient with hypotension D. Providing discharge teaching to a postsurgical patient about the rationale for a course of oral antibiotics

C. Administering an IV bolus of normal saline to a patient with hypotension Although many nursing actions are independent, others are interdependent, such as carrying out prescribed treatments, administering medications and therapies, and collaborating with other health care team members to accomplish specific, expected outcomes and to monitor and manage potential complications. Irrigating a wound, administering pain medication, and administering IV fluids are interdependent nursing actions and require a physician's order. An independent nursing action occurs when the nurse assesses a patient's heart rate, provides discharge education, or provides mouth care.

An elderly patient is brought to the emergency department with a fractured tibia. The patient appears malnourished, and the nurse is concerned about the patient's healing process related to insufficient protein levels. What laboratory finding would the floor nurse prioritize when assessing for protein deficiency? Select one: A. Cortisol B. Hemoglobin C. Albumin D. Bilirubin

C. Albumin Serum albumin is a sensitive indicator of protein deficiency. Albumin levels of less than 3 g/mL are indicative of hypoalbuminemia. Altered hemoglobin levels, cortisol levels, and bilirubin levels are not indicators of protein deficiency.

A nurse will conduct an influenza vaccination campaign at an extended care facility. The nurse will be administering intramuscular (IM) doses of the vaccine. Of what age-related change should the nurse be aware when planning the appropriate administration of this drug? Select one: A. An older patient has a higher risk of bleeding after an IM injection than a younger patient. B. An older patient has more subcutaneous tissue and less durable skin than a younger patient. C. An older patient has less subcutaneous tissue and less muscle mass than a younger patient. D. An older patient has more superficial and tortuous nerve distribution than a younger patient.

C. An older patient has less subcutaneous tissue and less muscle mass than a younger patient. When administering IM injections, the nurse should remember that in an older patient, subcutaneous fat diminishes, particularly in the extremities. Muscle mass also decreases. There are no significant differences in nerve distribution or bleeding risk.

An older individual is touring an assisted living facility with his family in order to find a place to live after the death of his spouse and issues with mobility. Which characteristic of the facility is most in need of modification? Select one: A. Aromatherapy is used in the facility to provide a pleasant scent environment B. The temperature of common areas is kept between 75°F and 77°F C. Area rugs are placed in front of each sink in residents' washrooms to ensure warmth D. There is more tile than carpet throughout the facility and carpets are glued to the floor

C. Area rugs are placed in front of each sink in residents' washrooms to ensure warmth Area rugs constitute a fall risk and should not be used. The temperature, the use of aromatherapy, and the prioritization of tile over carpet are all conducive to a safe and pleasant environment.

When administering a medication to a client, the nurse needs to identify the client. Which of the following methods of identification should the nurse perform? Select one: A. Check the client's ID bracelet and scan the bar code. B. Check the client's name on the medication administration record. C. Ask the the client's full name and date of birth. D. Check the client's name with a family member

C. Ask the the client's full name and date of birth.

A patient with a tracheostomy has a new order for a fenestrated tracheostomy tube. Which action should the nurse include in the plan of care in collaboration with the speech therapist? A. Leave the tracheostomy inner cannula inserted at all times. B. Place the decannulation cap in the tube before cuff deflation. C. Assess the ability to swallow before using the fenestrated tube. D. Inflate the tracheostomy cuff during use of the fenestrated tube.

C. Assess the ability to swallow before using the fenestrated tube.

The nurse completes a shift assessment on a patient admitted in the early phase of heart failure. When auscultating the patient's lungs, which finding would the nurse most likely hear? A. Continuous rumbling, snoring, or rattling sounds mainly on expiration B. Continuous high-pitched musical sounds on inspiration and expiration C. Discontinuous, high-pitched sounds of short duration heard on inspiration Correct D. A series of long-duration, discontinuous, low-pitched sounds during inspiration

C. Discontinuous, high-pitched sounds of short duration heard on inspiration

A patient with chronic obstructive pulmonary disease (COPD) has poor gas exchange. Which action by the nurse would be most appropriate? A. Have the patient rest in bed with the head elevated to 15 to 20 degrees. B. Ask the patient to rest in bed in a high-Fowler's position with the knees flexed. C. Encourage the patient to sit up at the bedside in a chair and lean slightly forward. D. Place the patient in the Trendelenburg position with several pillows behind the head.

C. Encourage the patient to sit up at the bedside in a chair and lean slightly forward.

The nursing instructor is explaining critical thinking to a class of first-semester nursing students. When promoting critical thinking skills in these students, the instructor should encourage them to do which of the following actions? Select one: A. Weigh each of the potential negative outcomes in a situation. B. Disregard input from people who do not have to make the particular decision. C. Examine and analyze all available information. D. Set aside all prejudices and personal experiences when making decisions.

C. Examine and analyze all available information.

A patient who is taking rifampin (Rifadin) for tuberculosis calls the clinic and reports having orange discolored urine and tears. Which is the best response by the nurse? A. Ask if the patient is experiencing shortness of breath, hives, or itching. B. Ask the patient about any visual abnormalities such as red-green color discrimination. C. Explain that orange discolored urine and tears are normal while taking this medication. D. Advise the patient to stop the drug and report the symptoms to the health care provider.

C. Explain that orange discolored urine and tears are normal while taking this medication.

A nurse is administering an oral medication to an older adult client. The client states, "The pill I always take is green. I don't take an orange pill." Which of the following nursing responses is appropriate? Select one: A. Let me explain the purpose of the medication." B. This is the medication that your doctor wants you to take." C. I will check your medication orders again." D. Sometimes the same pill comes in a different color."

C. I will check your medication orders again."

An elderly female patient who is bedridden is admitted to the unit because of a pressure ulcer that can no longer be treated in a community setting. During your assessment of the patient, you find that the ulcer extends into the muscle and bone. At what stage would document this ulcer? Select one: A. I B. II C. IV D. III

C. IV Stage III and IV pressure ulcers are characterized by extensive tissue damage. In addition to the interventions listed for stage I, these advanced draining, necrotic pressure ulcers must be cleaned (débrided) to create an area that will heal. Stage IV is an ulcer that extends to underlying muscle and bone. Stage III is an ulcer that extends into the subcutaneous tissue. With this type of ulcer, necrosis of tissue and infection may develop. Stage I is an area of erythema that does not blanch with pressure. Stage II involves a break in the skin that may drain.

A patient has recently been diagnosed with type 2 diabetes. The patient is clinically obese and has a sedentary lifestyle. How can the nurse best begin to help the patient increase his activity level? Select one: A. Have a family member ensure the patient follows a suggested exercise plan. B. Construct an exercise program and have the patient follow it. C. Identify barriers with the patient that inhibit his lifestyle change. D. Set up appointment times at a local fitness center for the patient to attend.

C. Identify barriers with the patient that inhibit his lifestyle change. Nurses cannot expect that sedentary patients are going to develop a sudden passion for exercise and that they will easily rearrange their day to accommodate time-consuming exercise plans. The patient may not be ready or willing to accept this lifestyle change. This is why it is important that the nurse and patient identify barriers to change.

A client is to receive an IV solution of 5% dextrose and 0.45% normal saline at 125 mL/hr. Which system provides the safest method for the nurse to accurately administer this solution? A. Controller B. Glass container C. Infusion pump D. Syringe pump

C. Infusion pump Infusion pumps are used for drugs or fluids under pressure. They accurately measure the volume of fluid being infused. A controller is a stationary, pole-mounted electronic device that uses a sensor to monitor fluid flow and detect when flow has been interrupted. Because controllers rely completely on gravity to create fluid flow and do not create pressure, they do not ensure infusion but only control the drip rate. A glass container is necessary to use only with IV solutions that may cling to the plastic bag; this is not an issue with this solution. A syringe pump does not hold sufficient volume to be practical in this situation.

A group of residents in a skilled nursing facility are sitting outside in the garden enjoying a hot summer day. What primary concern does the nurse recognize for these residents? Select one: A. Lack of motivation to get out of the sun B. Effects of certain medications on body temperature C. Lack of thirst perception D. Lack of energy and related depression

C. Lack of thirst perception Thirst perception declines with age, and so older persons are less aware of their fluid needs. This can be dangerous in hot weather. Natural sunlight would help with energy and depression. The residents may need assistance to get out of the sun. The effects of medications on body temperature may or may not be a concern since it would depend upon the resident and the medications being taken.

The nurse notes that a patient has incisional pain, a poor cough effort, and scattered rhonchi after a thoracotomy. Which action should the nurse take first? A. Assist the patient to sit upright in a chair. B. Splint the patient's chest during coughing. C. Medicate the patient with prescribed morphine. D. Observe the patient use the incentive spirometer.

C. Medicate the patient with prescribed morphine.

Falls, which are a major health problem in the elderly population, occur from multifactorial causes. When implementing a comprehensive plan to reduce the incidence of falls on a geriatric unit, what risk factors should nurses identify? Select one or more: A. Chronic Disease diagnosis B. C. Medication effects D. Overdependence on assistive devices E. Ineffective coping mechanisms

C. Medication effects Causes of falls are multifactorial. Both extrinsic factors, such as changes in the environment or poor lighting, and intrinsic factors, such as physical illness, neurologic changes, or sensory impairment, play a role. Mobility difficulties, medication effects, foot problems or unsafe footwear, postural hypotension, visual problems, and tripping hazards are common, treatable causes. Overdependence on assistive devices and ineffective use of coping strategies have not been shown to be factors in the rate of falls in the elderly population.

A patient with chronic obstructive pulmonary disease (COPD) has a nursing diagnosis of imbalanced nutrition: less than body requirements. Which intervention would be most appropriate for the nurse to include in the plan of care? A. Encourage increased intake of whole grains. B. Increase the patient's intake of fruits and fruit juices. C. Offer high-calorie snacks between meals and at bedtime. D. Assist the patient in choosing foods with high vegetable and mineral content.

C. Offer high-calorie snacks between meals and at bedtime.

Which of the following statements is true of the older adult population? Select one: A. Most older adults live in nursing homes. B. Older adults are not interested in sex. C. Old age begins at 65 years of age. D. Incontinence is not a part of aging.

C. Old age begins at 65 years of age. D. Incontinence is not a part of aging. Although a common myth of aging is that bladder problems are common, in actuality incontinence is not a normal part of aging and requires medical attention.

Which client does the charge nurse on a medical-surgical unit assign to the LPN/LVN? A. Cardiac client who has a diltiazem (Cardizem) IV infusion being titrated to maintain a heart rate between 60 and 80 beats/min B. Diabetic client admitted for hyperglycemia who is on an IV insulin drip and needs frequent glucose checks C. Older client admitted for confusion who has a heparin lock that needs to be flushed every 8 hours D. Postoperative client receiving blood products after excessive blood loss during surgery

C. Older client admitted for confusion who has a heparin lock that needs to be flushed every 8 hours The older client admitted for confusion with a heparin lock is the most stable and requires basic monitoring of the IV site for common complications such as phlebitis and local infection, which would be familiar to an LPN/LVN. The cardiac client with a diltiazem (Cardizem) IV infusion, the diabetic client on an IV insulin drip, and the postoperative client receiving blood products all are not stable and will require ongoing assessments and adjustments in IV therapy that should be performed by an RN.

A patient in metabolic alkalosis is admitted to the emergency department, and pulse oximetry (SpO2) indicates that the O2 saturation is 94%. Which action should the nurse take next? A. Administer bicarbonate. B. Complete a head-to-toe assessment. C. Place the patient on high-flow oxygen. D. Obtain repeat arterial blood gases (ABGs).

C. Place the patient on high-flow oxygen.

A nurse who is caring for patient with a tracheostomy tube in place has just auscultated rhonchi bilaterally. If the patient is unsuccessful in coughing up secretions, what action should the nurse take? A. Encourage increased incentive spirometer use. B. Encourage the patient to increase oral fluid intake. C. Put on sterile gloves and use a sterile catheter to suction. D. Preoxygenate the patient for 3 minutes before suctioning.

C. Put on sterile gloves and use a sterile catheter to suction.

A nurse assesses an area of pale white skin over a patient's coccyx. After turning the patient on her side, the skin becomes red and feels warm. What should the nurse do about these assessments? Select one: A. Immediately report to the physician that the patient has a pressure ulcer. B. Implement nursing interventions for Altered Skin Integrity. C. Recognize that this is ischemia, followed by reactive hyperemia. D. Document the presence of a pressure ulcer and develop a care plan.

C. Recognize that this is ischemia, followed by reactive hyperemia. Blanching of skin over an area under pressure results from ischemia. When pressure is relieved, reactive hyperemia follows and the skin is red and feels warm. Reactive hyperemia is not a stage I pressure ulcer.

An occupational health nurse works at a manufacturing plant where there is potential exposure to inhaled dust. Which action, if recommended by the nurse, will be most helpful in reducing the incidence of lung disease? A. Treat workers with pulmonary fibrosis. B. Teach about symptoms of lung disease. C. Require the use of protective equipment. D. Monitor workers for coughing and wheezing.

C. Require the use of protective equipment.

A patient newly diagnosed with asthma is being discharged. The nurse anticipates including which topic in the discharge teaching? A. Use of long-acting b-adrenergic medications B. Side effects of sustained-release theophylline C. Self-administration of inhaled corticosteroids D. Complications associated with oxygen therapy

C. Self-administration of inhaled corticosteroids

While conducting a health assessment with an older adult, the nurse notices it takes the person longer to answer questions than is usual with younger patients. What should the nurse do? Select one: A. Realize that the patient has some dementia. B. Stop asking questions so as not to confuse the patient. C. Slow the pace and allow extra time for answers. D. Ask a family member to answer the questions.

C. Slow the pace and allow extra time for answers. Cognition does not change appreciably with aging. It is normal for the older adult to take longer to respond and react. The nurse should slow the pace of care and allow older patients extra time to answer questions or complete activities.

A patient who has a right-sided chest tube following a thoracotomy has continuous bubbling in the suction-control chamber of the collection device. Which action by the nurse is most appropriate? A. Document the presence of a large air leak. B. Notify the surgeon of a possible pneumothorax. C. Take no further action with the collection device. D. Adjust the dial on the wall regulator to decrease suction.

C. Take no further action with the collection device.

The clinic nurse is teaching a patient with acute sinusitis. Which interventions should the nurse plan to include in the teaching session (select all that apply)? A. Decongestants can be used to relieve swelling. Correct B. Blowing the nose should be avoided to decrease the nosebleed risk. C. Taking a hot shower will increase sinus drainage and decrease pain. D. Saline nasal spray can be made at home and used to wash out secretions. E. You will be more comfortable if you keep your head in an upright position.

C. Taking a hot shower will increase sinus drainage and decrease pain. D. Saline nasal spray can be made at home and used to wash out secretions. E. You will be more comfortable if you keep your head in an upright position.

The health care provider writes an order for bacteriologic testing for a patient who has a positive tuberculosis skin test. Which action should the nurse take? A. Teach about the reason for the blood tests. B. Schedule an appointment for a chest x-ray. C. Teach about the need to get sputum specimens for 2 to 3 consecutive days. D. Instruct the patient to expectorate three specimens as soon as possible.

C. Teach about the need to get sputum specimens for 2 to 3 consecutive days.

Which action by the nurse will be most effective in decreasing the spread of pertussis in a community setting? A. Providing supportive care to patients diagnosed with pertussis B. Teaching family members about the need for careful hand washing C. Teaching patients about the need for adult pertussis immunizations D. Encouraging patients to complete the prescribed course of antibiotics

C. Teaching patients about the need for adult pertussis immunizations

You are the nurse caring for an elderly patient with cardiovascular disease. The patient comes to the clinic with a suspected respiratory infection and is diagnosed with pneumonia. As the nurse, what do you know about the altered responses of older adults? Select one: A. Treatments for older adults need to be more holistic than treatments used in the younger population. B. The altered responses of older adults define the nursing interactions with the patient. C. The altered responses of older adults reinforce the need for the nurse to monitor all body systems to identify possible systemic complications. D. Older adults become hypersensitive to antibiotic treatments for infectious disease states.

C. The altered responses of older adults reinforce the need for the nurse to monitor all body systems to identify possible systemic complications. Older people may be unable to respond effectively to an acute illness, or, if a chronic health condition is present, they may be unable to sustain appropriate responses over a long period. Furthermore, their ability to respond to definitive treatment is impaired. The altered responses of older adults reinforce the need for nurses to monitor all body system functions closely, being alert to signs of impending systemic complication. Holism should be integrated into all patients' care. Altered responses in the older adult do not define the interactions between the nurse and the patient. Older adults do not become hypersensitive to antibiotic treatments for infectious disease states.

During a morning assessment the nurse notes that the oral temperature of an older patient is 96°F (35.6°C). What guide will the nurse use to make decisions about this assessment finding? Select one: A. Older adults are at increased risk for hyperthermia B. Older adults often lose body heat in response to infection C. The normal body temperature of older adults is often lower than that of younger people D. Low temperature constitutes a risk to cardiac health

C. The normal body temperature of older adults is often lower than that of younger people

A patient who had a total laryngectomy has a nursing diagnosis of hopelessness related to loss of control of personal care. Which information obtained by the nurse is the best indicator that this identified problem is resolving? A. The patient lets the spouse provide tracheostomy care. B. The patient allows the nurse to suction the tracheostomy. C. The patient asks how to clean the tracheostomy stoma and tube. D. The patient uses a communication board to request "No Visitors."

C. The patient asks how to clean the tracheostomy stoma and tube.

The nurse interviews a patient with a new diagnosis of chronic obstructive pulmonary disease (COPD). Which information is most helpful in confirming a diagnosis of chronic bronchitis? A. The patient tells the nurse about a family history of bronchitis. B. The patient's history indicates a 30 pack-year cigarette history. C. The patient complains about a productive cough every winter for 3 months. D. The patient denies having any respiratory problems until the last 12 months.

C. The patient complains about a productive cough every winter for 3 months.

A patient is diagnosed with both human immunodeficiency virus (HIV) and active tuberculosis (TB) disease. Which information obtained by the nurse is most important to communicate to the health care provider? A. The Mantoux test had an induration of 7 mm. B. The chest-x-ray showed infiltrates in the lower lobes. C. The patient is being treated with antiretrovirals for HIV infection. D. The patient has a cough that is productive of blood-tinged mucus.

C. The patient is being treated with antiretrovirals for HIV infection.

The nurse takes an admission history on a patient with possible asthma who has new-onset wheezing and shortness of breath. Which information may indicate a need for a change in therapy? A. The patient has chronic inflammatory bowel disease. B. The patient has a history of pneumonia 6 months ago. C. The patient takes propranolol (Inderal) for hypertension. D. The patient uses acetaminophen (Tylenol) for headaches.

C. The patient takes propranolol (Inderal) for hypertension.

Crepitus

Crackling sound when palpating the skin

23. A nurse is preparing to perform an admission assessment on a patient with COPD. It is most important for the nurse to review which of the following? A) Social work assessment B) Insurance coverage C) Chloride levels D) Available diagnostic tests

D Feedback: In addition to the patients history, the nurse reviews the results of available diagnostic tests. Social work assessment is not a priority for the majority of patients. Chloride levels are relevant to CF, not COPD. Insurance coverage is not normally the domain of the nurse.

37. A nurse is explaining to a patient with asthma what her new prescription for prednisone is used for. What would be the most accurate explanation that the nurse could give? A) To ensure long-term prevention of asthma exacerbations B) To cure any systemic infection underlying asthma attacks C) To prevent recurrent pulmonary infections D) To gain prompt control of inadequately controlled, persistent asthma

D Feedback: Prednisone is used for a short-term (310 days) burst to gain prompt control of inadequately controlled, persistent asthma. It is not used to treat infection or to prevent exacerbations in the long term.

16. A patient is having pulmonary-function studies performed. The patient performs a spirometry test, revealing an FEV1/FVC ratio of 60%. How should the nurse interpret this assessment finding? A) Strong exercise tolerance B) Exhalation volume is normal C) Respiratory infection D) Obstructive lung disease

D Feedback: Spirometry is used to evaluate airflow obstruction, which is determined by the ratio of forced expiration volume in 1 second to forced vital capacity. Obstructive lung disease is apparent when an FEV1/FVC ratio is less than 70%.

1. A clinic nurse is caring for a patient who has just been diagnosed with chronic obstructive pulmonary disease (COPD). The patient asks the nurse what he could have done to minimize the risk of contracting this disease. What would be the nurses best answer? A) The most important risk factor for COPD is exposure to occupational toxins. B) The most important risk factor for COPD is inadequate exercise. C) The most important risk factor for COPD is exposure to dust and pollen. D) The most important risk factor for COPD is cigarette smoking.

D Feedback: The most important risk factor for COPD is cigarette smoking. Lack of exercise and exposure to dust and pollen are not risk factors for COPD. Occupational risks are significant but are far exceeded by smoking.

27. The case manager for a group of patients with COPD is providing health education. What is most important for the nurse to assess when providing instructions on self-management to these patients? A) Knowledge of alternative treatment modalities B) Family awareness of functional ability and activities of daily living (ADLs) C) Knowledge of the pathophysiology of the disease process D) Knowledge about self-care and their therapeutic regimen

D Feedback: When providing instructions about self-management, it is important for the nurse to assess the knowledge of patients and family members about self-care and the therapeutic regimen. This supersedes knowledge of alternative treatments or the pathophysiology of the disease, neither of which is absolutely necessary for patients to know. The patients own knowledge is more important than that of the family.

The laboratory has just called with the arterial blood gas (ABG) results on four patients. Which result is most important for the nurse to report immediately to the health care provider? a. pH 7.34, PaO2 82 mm Hg, PaCO2 40 mm Hg, and O2 sat 97% b. pH 7.35, PaO2 85 mm Hg, PaCO2 50 mm Hg, and O2 sat 95% c. pH 7.46, PaO2 90 mm Hg, PaCO2 32 mm Hg, and O2 sat 98% d. pH 7.31, PaO2 91 mm Hg, PaCO2 50 mm Hg, and O2 sat 96%

D These ABGs indicate uncompensated respiratory acidosis and should be reported to the health care provider. The other values are normal, close to normal, or compensated.

c. Hemoglobin of 9g/dL

The patient reports fatigue and shortness of breath when getting up to walk to the bathroom; however, the pulse oximetry reading is 99%. The nurse identifies a diagnosis of activity intolerance. Which laboratory value is consistent with the patient's subjective symptoms? a. BUN of 15 mg/dL b. WBC count of 8000/mm3 c. Hemoglobin of 9g/dL d. Glucose 160 mg/dL

b. 69 years

The patient reports smoking a pack of cigarettes a day for 9 years. He then quit for 2 years, then smoked 2 packs a day for the last 30 years. Whar are the pack-years for this patient? a. 39 years b. 69 years c. 19.5 years d. 41 years

The nurse is performing tuberculosis (TB) skin tests in a clinic that has many patients who have immigrated to the United States. Which question is most important for the nurse to ask before the skin test? A. "Is there any family history of TB?" B. "How long have you lived in the United States?" C. "Do you take any over-the-counter (OTC) medications?" D. "Have you received the bacille Calmette-Guérin (BCG) vaccine for TB?"

D. "Have you received the bacille Calmette-Guérin (BCG) vaccine for TB?"

A 55-year-old patient with increasing dyspnea is being evaluated for a possible diagnosis of chronic obstructive pulmonary disease (COPD). When teaching a patient about pulmonary function testing (PFT) for this condition, what is the most important question the nurse should ask? A. "Are you claustrophobic?" B. "Are you allergic to shellfish?" C. "Do you have any metal implants or prostheses?" D. "Have you taken any bronchodilators in the past 6 hours?"

D. "Have you taken any bronchodilators in the past 6 hours?"

The nurse completes discharge teaching for a patient who has had a lung transplant. The nurse evaluates that the teaching has been effective if the patient makes which statement? A. "I will make an appointment to see the doctor every year." B. "I will stop taking the prednisone if I experience a dry cough." C. "I will not worry if I feel a little short of breath with exercise." D. "I will call the health care provider right away if I develop a fever."

D. "I will call the health care provider right away if I develop a fever."

The nurse admits a patient who has a diagnosis of an acute asthma attack. Which statement indicates that the patient may need teaching regarding medication use? A. "I have not had any acute asthma attacks during the last year." B. "I became short of breath an hour before coming to the hospital." C. "I've been taking Tylenol 650 mg every 6 hours for chest-wall pain." D. "I've been using my albuterol inhaler more frequently over the last 4 days."

D. "I've been using my albuterol inhaler more frequently over the last 4 days."

A lobectomy is scheduled for a patient with stage I non-small cell lung cancer. The patient tells the nurse, "I would rather have chemotherapy than surgery." Which response by the nurse is most appropriate? A. "Are you afraid that the surgery will be very painful?" B. "Did you have bad experiences with previous surgeries?" C. "Surgery is the treatment of choice for stage I lung cancer." D. "Tell me what you know about the various treatments available."

D. "Tell me what you know about the various treatments available."

A patient scheduled for a total laryngectomy and radical neck dissection for cancer of the larynx asks the nurse, "Will I be able to talk normally after surgery?" What is the best response by the nurse? A. "You will breathe through a permanent opening in your neck, but you will not be able to communicate orally." B. "You won't be able to talk right after surgery, but you will be able to speak again after the tracheostomy tube is removed." C. "You won't be able to speak as you used to, but there are artificial voice devices that will give you the ability to speak normally." D. "You will have a permanent opening into your neck, and you will need to have rehabilitation for some type of voice restoration."

D. "You will have a permanent opening into your neck, and you will need to have rehabilitation for some type of voice restoration."

After change-of-shift report, which patient should the nurse assess first? A. 72-year-old with cor pulmonale who has 4+ bilateral edema in his legs and feet B. 28-year-old with a history of a lung transplant and a temperature of 101° F (38.3° C) C. 40-year-old with a pleural effusion who is complaining of severe stabbing chest pain D. 64-year-old with lung cancer and tracheal deviation after subclavian catheter insertion

D. 64-year-old with lung cancer and tracheal deviation after subclavian catheter insertion

The nurse is caring for a patient admitted for treatment of neck and throat cancer. Which intervention should the nurse perform? A. Encourage hydration with water. B. Feed the patient if coughing occurs. C. Encourage the patient to sit in a chair for meals. D. Encourage the patient to drink juice to address thirst.

D. ? Thick liquids

A patient arrives in the ear, nose, and throat clinic complaining of a piece of tissue being "stuck up my nose" and with foul-smelling nasal drainage from the right nare. Which action should the nurse take first? A. Notify the clinic health care provider. B. Obtain aerobic culture specimens of the drainage. C. Ask the patient about how the cotton got into the nose. D. Have the patient occlude the left nare and blow the nose.

D. Have the patient occlude the left nare and blow the nose.

A nurse recommends to an older patient's family to place a seat alongside the bathtub to enable the older adult bather to rest while drying off. What is the best explanation for the nurse's recommendation? Select one: A. Most elderly have an age-related problem discriminating hazards. B. Nonslip surfaces are essential for tubs and shower floors. C. The elderly use the bathroom often and can benefit from the rest. D. A drop in blood pressure may follow bathing.

D. A drop in blood pressure may follow bathing. A drop in blood pressure may follow bathing; sitting down after bathing lessens the risk of the elderly bather falling because of low blood pressure. Nonslip surfaces inside tubs and shower stalls are important, but this choice is not a reason for having a seat outside the tub. It is not true that the elderly use the bathroom often and can benefit from the rest. It is also not true that most elderly have an age-related problem with discriminating hazards.

The nurse is on a task force to reduce the incidence of falls among residents of a long-term care facility. Which measure should the nurse recommend to prevent the most falls? Select one: A. The use of diffuse, natural lighting on the unit B. Psychosocial interventions aimed at reducing individuals' fear of falling C. Use of physical restraints on new patients who have delirium or dementia. D. A fall history and fall risk assessment of each patient on admission

D. A fall history and fall risk assessment of each patient on admission A thorough history and fall risk assessment is central to any fall reduction initiative. This would supersede the use of diffuse lighting, though this type of lighting is appropriate. Interventions aimed at reducing the fear of falling will not necessarily reduce falls, and the liberal use of restraints is inappropriate.

A gerontologic nurse is making an effort to address some of the misconceptions about older adults that exist among health care providers. The nurse has made the point that most people aged 75 years remains functionally independent. The nurse should attribute this trend to what factor? Select one: A. Changes in the medical treatment of hypertension and hyperlipidemia B. Early detection of disease and increased advocacy by older adults C. Genetic changes that have resulted in increased resiliency to acute infection D. Application of health-promotion and disease-prevention activities

D. Application of health-promotion and disease-prevention activities Even among people 75 years of age and over, most remain functionally independent, and the proportion of older Americans with limitations in activities is declining. These declines in limitations reflect recent trends in health-promotion and disease-prevention activities, such as improved nutrition, decreased smoking, increased exercise, and early detection and treatment of risk factors such as hypertension and elevated serum cholesterol levels. This phenomenon is not attributed to genetics, medical treatment, or increased advocacy.

An alcoholic and homeless patient is diagnosed with active tuberculosis (TB). Which intervention by the nurse will be most effective in ensuring adherence with the treatment regimen? A. Arrange for a friend to administer the medication on schedule. B. Give the patient written instructions about how to take the medications. C. Teach the patient about the high risk for infecting others unless treatment is followed. D. Arrange for a daily noon meal at a community center where the drug will be administered.

D. Arrange for a daily noon meal at a community center where the drug will be administered.

A patient who has a history of chronic obstructive pulmonary disease (COPD) was hospitalized for increasing shortness of breath and chronic hypoxemia (SaO2 levels of 89% to 90%). In planning for discharge, which action by the nurse will be most effective in improving compliance with discharge teaching? A. Start giving the patient discharge teaching on the day of admission. B. Have the patient repeat the instructions immediately after teaching. C. Accomplish the patient teaching just before the scheduled discharge. D. Arrange for the patient's caregiver to be present during the teaching.

D. Arrange for the patient's caregiver to be present during the teaching.

The nurse palpates the posterior chest while the patient says "99" and notes absent fremitus. Which action should the nurse take next? A. Palpate the anterior chest and observe for barrel chest. B. Encourage the patient to turn, cough, and deep breathe. C. Review the chest x-ray report for evidence of pneumonia. D. Auscultate anterior and posterior breath sounds bilaterally.

D. Auscultate anterior and posterior breath sounds bilaterally.

When assessing the respiratory system of an older patient, which finding indicates that the nurse should take immediate action? A. Weak cough effort B. Barrel-shaped chest C. Dry mucous membranes D. Bilateral crackles at lung bases

D. Bilateral crackles at lung bases

A patient with severe chronic obstructive pulmonary disease (COPD) tells the nurse, "I wish I were dead! I'm just a burden on everybody." Based on this information, which nursing diagnosis is most appropriate? A. Complicated grieving related to expectation of death B. Ineffective coping related to unknown outcome of illness C. Deficient knowledge related to lack of education about COPD D. Chronic low self-esteem related to increased physical dependence

D. Chronic low self-esteem related to increased physical dependence

The nurse caring for older patients in an acute care facility is aware of the changes in drug metabolism that can occur in older adults. Which statement explains the most important factor that affects pharmacokinetics in older patients? Select one: A. Changes in gastrointestinal (GI) motility increase the absorption time for many drugs. B. Preexisting chronic conditions complicate the distribution and metabolism of drugs. C. Drug distribution is unpredictable due to metabolic and body-composition factors. D. Decreased renal and liver function contributes to an increased half-life for many drugs.

D. Decreased renal and liver function contributes to an increased half-life for many drugs. Changes in renal and liver function contribute significantly to the changes in pharmacokinetics that are common in older adults. While changes in GI motility, drug distribution, and preexisting conditions may be true for many patients, these factors are inconsistent.

The nurse is preparing to conduct an admission assessment on a 76-year-old man. What would be important to do before interviewing this patient? Select one: A. Make sure the door is not blocked B. Speak in a louder than normal voice C. Turn up the patient's hearing aid D. Reduce or eliminate background noise

D. Reduce or eliminate background noise It is essential to reduce or eliminate background noise as much as possible when carrying on conversations. This includes turning off the television or radio in the patient's room and closing the door to reduce sounds of telephones, beepers, alarms, or pagers. Before beginning the interview, it would not be necessary to make sure the door is not blocked or to speak in a louder than normal voice. The scenario does not say that the patient is using a hearing aid.

A gerontologic nurse has observed that patients often fail to adhere to a therapeutic regimen. What strategy should the nurse adopt to best assist an older adult in adhering to a therapeutic regimen involving wound care? Select one: A. Provide a detailed pamphlet on a dressing change. B. Delegate the dressing change to a trusted family member. C. Verbally instruct the patient how to change a dressing and check for comprehension. D. Demonstrate a dressing change and allow the patient to practice.

D. Demonstrate a dressing change and allow the patient to practice. The nurse must consider that older adults may have deficits in the ability to draw inferences, apply information, or understand major teaching points. Demonstration and practice are essential in meeting their learning needs. The other options are incorrect because the elderly may have problems reading and/or understanding a written pamphlet or verbal instructions. Having a family member change the dressing when the patient is capable of doing it impedes self-care and independence.

A gerontologic nurse is basing the therapeutic programs at a long-term care facility on Miller's Functional Consequences Theory. To actualize this theory of aging, the nurse should prioritize what task? Select one: A. Helping older adults accept the inevitability of death B. Attempting to control age-related physiological changes C. Lowering expectations for recovery from acute and chronic illnesses D. Differentiating between age-related changes and modifiable risk factors

D. Differentiating between age-related changes and modifiable risk factors The Functional Consequences Theory requires the nurse to differentiate between normal, irreversible age-related changes and modifiable risk factors. This theory does not emphasize lowering expectations, controlling age-related changes, or helping adults accept the inevitability of death.

A patient with a possible pulmonary embolism complains of chest pain and difficulty breathing. The nurse finds a heart rate of 142 beats/minute, blood pressure of 100/60 mmHg, and respirations of 42 breaths/minute. Which action should the nurse take first? A. Administer anticoagulant drug therapy. B. Notify the patient's health care provider. C. Prepare patient for a spiral computed tomography (CT). D. Elevate the head of the bed to a semi-Fowler's position.

D. Elevate the head of the bed to a semi-Fowler's position.

A gerontologic nurse is overseeing the care that is provided in a large, long-term care facility. The nurse is educating staff about the significant threat posed by influenza in older, frail adults. What action should the nurse prioritize to reduce the incidence and prevalence of influenza in the facility? Select one: A. Make arrangements for residents to limit social interaction during winter months. B. Teach staff how to administer prophylactic antiviral medications effectively. C. Ensure that residents receive a high-calorie, high-protein diet during the winter. D. Ensure that residents receive influenza vaccinations in the fall of each year.

D. Ensure that residents receive influenza vaccinations in the fall of each year. The influenza and the pneumococcal vaccinations lower the risks of hospitalization and death in elderly people. The influenza vaccine, which is prepared yearly to adjust for the specific immunologic characteristics of the influenza viruses at that time, should be administered annually in autumn. Prophylactic antiviral medications are not used. Limiting social interaction is not required in most instances. Nutrition enhances immune response, but this is not specific to influenza prevention.

Postural drainage with percussion and vibration is ordered twice daily for a patient with chronic bronchitis. Which intervention should the nurse include in the plan of care? A. Schedule the procedure 1 hour after the patient eats. B. Maintain the patient in the lateral position for 20 minutes. C. Perform percussion before assisting the patient to the drainage position. D. Give the ordered albuterol (Proventil) before the patient receives the therapy.

D. Give the ordered albuterol (Proventil) before the patient receives the therapy.

The nurse educates the staff of a residential home about an upcoming influenza season. Which of the following should the nurse include in the presentation? Select one: A. Antibiotics are an effective treatment for influenza. B. Infection control practices increase the risks of transmission of influenza. C. The influenza vaccination is not recommended for persons over the age of 64. D. Hand hygiene is one of the most important ways to prevent the spread of infections.

D. Hand hygiene is one of the most important ways to prevent the spread of infections. Health care providers should adhere to strict infection control practices and practice hand hygiene at key points in time to disrupt the transmission of microorganisms. A yearly influenza vaccination is recommended to reduce its occurrence.

A gerontologic nurse practitioner provides primary care for a large number of older adults who are living with various forms of cardiovascular disease. This nurse is well aware that heart disease is the leading cause of death in the aged. What is an age-related physiological change that contributes to this trend? Select one: A. Resting heart rate decreases with age. B. Atrial-septal defects develop with age. C. Systolic blood pressure decreases. D. Heart muscle and arteries lose their elasticity.

D. Heart muscle and arteries lose their elasticity. The leading cause of death for patients over the age of 65 years is cardiovascular disease. With age, heart muscle and arteries lose their elasticity, resulting in a reduced stroke volume. As a person ages, systolic blood pressure does not decrease, resting heart rate does not decrease, and the aged are not less likely to adopt a healthy lifestyle.

The nurse plans to teach a patient how to manage allergic rhinitis. Which information should the nurse include in the teaching plan? A. Hand washing is the primary way to prevent spreading the condition to others. B. Use of oral antihistamines for 2 weeks before the allergy season may prevent reactions. C. Corticosteroid nasal sprays will reduce inflammation, but systemic effects limit their use. D. Identification and avoidance of environmental triggers are the best way to avoid symptoms.

D. Identification and avoidance of environmental triggers are the best way to avoid symptoms.

When assessing a patient who has just arrived after an automobile accident, the emergency department nurse notes tachycardia and absent breath sounds over the right lung. For which intervention will the nurse prepare the patient? A. Emergency pericardiocentesis B. Stabilization of the chest wall with tape C. Administration of an inhaled bronchodilator D. Insertion of a chest tube with a chest drainage system

D. Insertion of a chest tube with a chest drainage system

A nurse is caring for a patient who has had a total laryngectomy and radical neck dissection. During the first 24 hours after surgery what is the priority nursing action? A. Monitor for bleeding. B. Maintain adequate IV fluid intake. C. Suction tracheostomy every eight hours. D. Keep the patient in semi-Fowler's position.

D. Keep the patient in semi-Fowler's position.

Gerontological nursing is a complex specialty. Which of the following contributes most to this fact? Select one: A. Elderly people are generally compromised in their health status B. Complications after surgery or illness result in death in most cases C. Care for the elderly costs more than care for younger patients D. Numerous health conditions can overlap in the elderly

D. Numerous health conditions can overlap in the elderly Most elderly people are well, and their care is no more expensive than that of any other person. Death is not the most frequent outcome of illness or surgery. Multiple health conditions often coexist, making it difficult for health care professionals to sort out cause-and-effect relationships in symptoms, diagnoses, and treatments.

A patient hospitalized with chronic obstructive pulmonary disease (COPD) is being discharged home on oxygen therapy. Which instruction should the nurse include in the discharge teaching? A. Storage of oxygen tanks will require adequate space in the home. B. Travel opportunities will be limited because of the use of oxygen. C. Oxygen flow should be increased if the patient has more dyspnea. D. Oxygen use can improve the patient's prognosis and quality of life.

D. Oxygen use can improve the patient's prognosis and quality of life.

A patient with end-stage lung cancer has been admitted to hospice care. The hospice team is meeting with the patient and her family to establish goals for care. What is likely to be a first priority in goal setting for the patient? Select one: A. Promotion of spirituality B. Maintenance of activities of daily living C. Social interaction D. Pain control

D. Pain control Once the phase of illness has been identified for a specific patient, along with the specific medical problems and related social and psychological problems, the nurse helps prioritize problems and establish the goals of care. Identification of goals must be a collaborative effort, with the patient, family, and nurse working together, and the goals must be consistent with the abilities, desires, motivations, and resources of those involved. Pain control is essential for patients who have a terminal illness. If pain control is not achieved, all activities of daily living are unattainable. This is thus a priority in planning care over the other listed goals.

You are the nurse caring for a female patient who developed a pressure ulcer as a result of decreased mobility. The nurse on the shift before you has provided patient teaching about pressure ulcers and healing promotion. You assess that the patient has understood the teaching by observing what? Select one: A. Patient elevates her body parts that are susceptible to edema. B. Patient performs range-of-motion exercises. C. Patient demonstrates the technique for massaging the wound site. D. Patient avoids placing her body weight on the healing site.

D. Patient avoids placing her body weight on the healing site. The major goals of pressure ulcer treatment may include relief of pressure, improved mobility, improved sensory perception, improved tissue perfusion, improved nutritional status, minimized friction and shear forces, dry surfaces in contact with skin, and healing of pressure ulcer, if present. The other options do not demonstrate the achievement of the goal of the patient teaching.

When the nurse begins taking the patient's history, the patient asks, "Did you know that I have throat cancer and may not survive?" What is the appropriate nursing response? A. "Are you having difficulty swallowing?" B. "My mother had cancer, so I know how you must be feeling right now." C. "I am sure that your cancer can be cured if you follow your doctor's advice." D. "I know you have been diagnosed with cancer. Are you concerned about what the future may hold?"

D. Psychosocial support

A nurse is caring for a client who is receiving nifedipine (Procardia). The nurse checks the client's blood pressure before administering medication and reports 98/58. Which of the following actions should the nurse take first? Select one: A. Notify the client's provider of the result. B. Instruct the client to get up from the sitting position slowly. C. Document the client's blood pressure. D. Recheck the client's blood pressure.

D. Recheck the client's blood pressure.

Using the illustrated technique, the nurse is assessing for which finding in a patient with chronic obstructive pulmonary disease (COPD)? A. Hyperresonance B. Tripod positioning C. Accessory muscle use D. Reduced chest expansion

D. Reduced chest expansion

Based on a patient's vague explanations for recurring injuries, the nurse suspects that a community-dwelling older adult may be the victim of abuse. What is the nurse's primary responsibility? Select one: A. Confront the suspected perpetrator. B. Work with the family to promote healthy conflict resolution. C. Gather evidence to corroborate the abuse. D. Report the findings to adult protective services.

D. Report the findings to adult protective services. If neglect or abuse of any kind—including physical, emotional, sexual, or financial abuse—is suspected, the local adult protective services agency must be notified. The responsibility of the nurse is to report the suspected abuse, not to prove it, confront the suspected perpetrator, or work with the family to promote resolution.

An hour after a thoracotomy, a patient complains of incisional pain at a level 7 (based on 0 to 10 scale) and has decreased left-sided breath sounds. The pleural drainage system has 100 mL of bloody drainage and a large air leak. Which action is best for the nurse to take next? A. Milk the chest tube gently to remove any clots. B. Clamp the chest tube momentarily to check for the origin of the air leak. C. Assist the patient to deep breathe, cough, and use the incentive spirometer. D. Set up the patient controlled analgesia (PCA) and administer the loading dose of morphine.

D. Set up the patient controlled analgesia (PCA) and administer the loading dose of morphine.

You are caring for a patient with a history of chronic angina. The patient tells you that after breakfast he usually takes a shower and shaves. It is at this time, the patient says, that he tends to experience chest pain. What might you counsel the patient to do to decrease the likelihood of angina in the morning? Select one: A. Shower once a week and shave prior to breakfast. B. Skip breakfast and eat an early lunch. C. Take a nitro tab prior to breakfast. D. Shower in the evening and shave before breakfast.

D. Shower in the evening and shave before breakfast. If the nurse determines that one of the situations most likely to precipitate angina is to shower and shave after breakfast, the nurse might counsel the patient to break these activities into different times during the day. Skipping breakfast and eating an early lunch would not decrease the likelihood of angina in the morning. Taking a nitro tablet before breakfast is inappropriate because the event requiring the medication has not yet occurred. Also, suggesting that the patient shower once a week and shave prior to breakfast is an incorrect suggestion because showering and shaving can both be done every day if they are spread out over the course of the day.

The nurse prepares a patient with a left-sided pleural effusion for a thoracentesis. How should the nurse position the patient? A. Supine with the head of the bed elevated 30 degrees B. In a high-Fowler's position with the left arm extended C. On the right side with the left arm extended above the head D. Sitting upright with the arms supported on an over bed table.

D. Sitting upright with the arms supported on an over bed table.

A 70-year-old client with severe dehydration is ordered an infusion of an isotonic solution at 250 mL/hr through a midline IV catheter. After 2 hours, the nurse notes that the client has crackles throughout all lung fields. Which action does the nurse take first? A. Assess the midline IV insertion site. B. Have the client cough and deep-breathe. C. Notify the health care provider about the crackles. D. Slow the rate of the IV infusion.

D. Slow the rate of the IV infusion. The presence of crackles throughout the lungs is a sign of possible fluid overload. The nurse should slow the rate of infusion and further assess for indicators of volume overload and/or respiratory distress. Assessing the site, having the client cough and deep-breathe, and notifying the provider may be appropriate, but are not the initial actions for this client.

Which intervention will the nurse include in the plan of care for a patient who is diagnosed with a lung abscess? A. Teach the patient to avoid the use of over-the-counter expectorants. B. Assist the patient with chest physiotherapy and postural drainage. C. Notify the health care provider immediately about any bloody or foul-smelling sputum. D. Teach about the need for prolonged antibiotic therapy after discharge from the hospital.

D. Teach about the need for prolonged antibiotic therapy after discharge from the hospital.

The nurse is caring for a patient who has acute pharyngitis caused by Candida albicans. Which action is appropriate for the nurse to include in the plan of care? A. Avoid giving patient warm liquids to drink. B. Assess patient for allergies to penicillin antibiotics. C. Teach the patient about the need to sleep in a warm, dry environment. D. Teach patient to "swish and swallow" prescribed oral nystatin (Mycostatin).

D. Teach patient to "swish and swallow" prescribed oral nystatin (Mycostatin).

A patient with chronic obstructive pulmonary disease (COPD) has a nursing diagnosis of impaired breathing pattern related to anxiety. Which nursing action is most appropriate to include in the plan of care? A. Titrate oxygen to keep saturation at least 90%. B. Discuss a high-protein, high-calorie diet with the patient. C. Suggest the use of over-the-counter sedative medications. D. Teach the patient how to effectively use pursed lip breathing.

D. Teach the patient how to effectively use pursed lip breathing.

The nurse analyzes the results of a patient's arterial blood gases (ABGs). Which finding would require immediate action? A. The bicarbonate level (HCO3-) is 31 mEq/L. B. The arterial oxygen saturation (SaO2) is 92%. C. The partial pressure of CO2 in arterial blood (PaCO2) is 31 mm Hg. D. The partial pressure of oxygen in arterial blood (PaO2) is 59 mm Hg.

D. The partial pressure of oxygen in arterial blood (PaO2) is 59 mm Hg.

The nurse teaches a patient how to administer formoterol (Perforomist) through a nebulizer. Which action by the patient indicates good understanding of the teaching? A. The patient attaches a spacer before using the inhaler. B. The patient coughs vigorously after using the inhaler. C. The patient activates the inhaler at the onset of expiration. D. The patient removes the facial mask when misting has ceased.

D. The patient removes the facial mask when misting has ceased.

The nurse is caring for an older adult patient who is receiving rehabilitation following an ischemic stroke. A review of the patient's electronic health record reveals that the patient usually defers her self-care to family members or members of the care team. What should the nurse include as an initial goal when planning this patient's subsequent care? Select one: A. The patient will participate in a life skills program. B. The nurse will delegate the patient's care to a nursing assistant. C. The patient's family will collaboratively manage the patient's care. D. The patient will demonstrate independent self-care.

D. The patient will demonstrate independent self-care. An appropriate patient goal will focus on the patient demonstrating independent self-care. The rehabilitation process helps patients achieve an acceptable quality of life with dignity, self-respect, and independence. The other options are incorrect because an appropriate goal would not be for the family to manage the patient's care, the patient's care would not be delegated to a nursing assistant, and participating in a social program is not an appropriate initial goal.

The nurse completes an admission assessment on a patient with asthma. Which information given by patient is most indicative of a need for a change in therapy? A. The patient uses albuterol (Proventil) before any aerobic exercise. B. The patient says that the asthma symptoms are worse every spring. C. The patient's heart rate increases after using the albuterol (Proventil) inhaler. D. The patient's only medications are albuterol (Proventil) and salmeterol (Serevent).

D. The patient's only medications are albuterol (Proventil) and salmeterol (Serevent).

A patient is admitted with active tuberculosis (TB). The nurse should question a health care provider's order to discontinue airborne precautions unless which assessment finding is documented? A. Chest x-ray shows no upper lobe infiltrates. B. TB medications have been taken for 6 months. C. Mantoux testing shows an induration of 10 mm. D. Three sputum smears for acid-fast bacilli are negative.

D. Three sputum smears for acid-fast bacilli are negative.

The nurse supervises unlicensed assistive personnel (UAP) who are providing care for a patient with right lower lobe pneumonia. The nurse should intervene if which action by UAP is observed? A. UAP splint the patient's chest during coughing. B. UAP assist the patient to ambulate to the bathroom. C. UAP help the patient to a bedside chair for meals. D. UAP lower the head of the patient's bed to 15 degrees.

D. UAP lower the head of the patient's bed to 15 degrees.

The nurse develops a teaching plan to help increase activity tolerance at home for an older adult with severe chronic obstructive pulmonary disease (COPD). Which instructions would be most appropriate for the nurse to include in the plan of care? A. Stop exercising when short of breath. B. Walk until pulse rate exceeds 130 beats/minute. C. Limit exercise to activities of daily living (ADLs). D. Walk 15 to 20 minutes daily at least 3 times/week.

D. Walk 15 to 20 minutes daily at least 3 times/week.

A nurse is assessing a patient with a stage IV pressure ulcer. What assessment of the ulcer would be expected? Select one: A. blister formation B. skin pallor C. eschar formation D. full-thickness skin loss

D. full-thickness skin loss A stage IV pressure ulcer is characterized by the extensive destruction associated with full-thickness skin loss.

After a motor vehicle accident the patient is dependent on mechanical ventilation. The nurse initiates ventilatory bundle precautions to prevent ventilator associated pneumonia, which action does the nurse perform to initiate these precautions

Elevate head of the bead between 30-45 degrees whenever possible, continuously remove subglottic secreations, perform handwashing before and after contact with each patient, no rings are worn when caring for ventilator patients, perform meticulous oral care no less frequently than every 12 hours

A client with pneumonia has difficulty clearing secretions from the airway. Which nursing intervention does the nurse include in this client's plan of care?

Encourage an intake of 2 liters of fluid per day.

A client with pneumonia is producing a smaller volume of thicker secretions than the day before. The client is receiving intravenous antibiotics. What action does the nurse take?

Encourage the client to drink more fluids

A nurse is planning care for a client after a tracheostomy. One of the client's goals is to overcome verbal communication impairment. Which intervention should the nurse include in the care plan? a) Answer questions for the client to reduce his frustration. b) Make an effort to read the client's lips to foster communication. c) Encourage the client's communication attempts by allowing him time to select or write words. d) Avoid using a tracheostomy plug because it blocks the airway.

Encourage the client's communication attempts by allowing him time to select or write words. The nurse should allow ample time for the client to respond and shouldn't speak for him. She should use as many aids as possible to assist the client with communicating and encourage the client when he attempts to communicate. When the client is ready, the nurse can use a tracheostomy plug to facilitate speech. Making an effort to read the client's lips and answering questions for the client are inappropriate.

A patient with pneumonia is producing a smaller volume of thicker secretions than the day before. The patient is receiving intravenous antibiotics. What action does the nurse take?

Encourage the patient to drink more fluid Thick secretions indicate decreased hydration and the patient is at risk for airway obstruction if these secretions cannot be cleared easily; the nurse should encourage increased fluid intake. Peak flow levels are used to monitor relative airway obstruction in patients with obstructive lung disease. A decrease in secretions does not necessarily indicate improvement in the infection, especially if the secretions are thick and not easily mobilized. The patient should continue intravenous antibiotics until there is evidence that the infection is improving.

The nurse is creating a plan of car for a patient diagnosed with acute laryngitis. What intervention should be included in the patient's plan of care?

Encourage the patient to limit speech whenever possible.

A patient with chronic obstructive pulmonary disease (COPD) has poor gas exchange. Which action by the nurse would support the patient's ventilation?

Encourage the patient to sit up at the bedside in a chair and lean forward.

Which nursing interventions are critical in caring for individuals with influenza? Select all that apply

Encouraging the patient to rest and increase fluid intake Supporting the patient and preventing the spread of the disease Monitoring pulse rate and quality and urine output during rehydration in a patient with diarrhea Encouraging rest and promoting an increase in fluids is essential to promote healing. Influenza is highly contagious and emphasis should be placed on providing symptomatic support while preventing the spread of the disease to others. Assessment of pulse rate and quality and urine output will aid the health care team in monitoring the rehydration of patients who have lost significant body fluids secondary to diarrhea with the flu. Oxygen may be indicated as part of supportive care in an individual with hypoxia secondary to respiratory infections with the flu. The patient with influenza should be placed in isolation to prevent airborne spread of the disease to others.

Which factor may contribute to sleep apnea?

Obesity Excessive adipose tissue in the neck in obese patients can contribute to an increased risk of the tongue causing a partial obstruction as it falls back in the throat. While uncontrolled hypertension and sleep deprivation can be comorbidities in an individual with sleep apnea, as a separate problem they are not a major cause of sleep apnea. Modafinil may be prescribed for individuals with significant daytime sleepiness due to sleep apnea.

The nurse is caring for a patient who has undergone a hemilaryngectomy for laryngeal cancer. Once a feeding tube has been placed and the patient's intestinal tract has recovered from the effects of anesthesia, which action by the nurse is appropriate for the next few days?

Ensuring an individualized nutrition plan is followed for calorie needs Once a feeding tube is placed, it is important for patients to receive adequate calories to promote tissue healing; a diet that provides 35-40 kcal/kg/day is recommended. Protein and fluids are also important, but a high-carbohydrate diet is not indicated. It is important to establish nutrition with adequate calories and protein, so a clear liquid diet will not be sufficient. The feeding tube should be maintained with tube feedings for 7-10 days and then removed once the patient can swallow; patients will not aspirate since the airway and the esophagus have been separated.

A student nurse is providing food orally to a patient who refuses enteral feeding. Which action made by the student nurse indicates the need for correction?

Enteral feeding is suggested to a patient who has undergone radical neck dissection. If the patient refuses food by enteral feeding, the nurse should provide the food orally. Positioning the patient in a supine position can result in aspiration; placing the patient in a high Fowler's position or elevating the bed is recommended. Adding dry milk to the food will supplement the caloric intake of the patient. Adding thickeners to the food can make the food difficult to swallow. Administering antiemetics before meals or snacks will help reduce nausea.

A nurse assesses a newly admitted client with a diagnosis of pulmonary TB, which clinical findings support this diagnosis

Fatigue, hemoptysis, night sweats

Postoperative Car of Tracheostomy (4)

Focus care on ensuring a patent airway. Confirm the presence of bilateral breath sounds. Perform a respiratory assessment at least every 2 hours. Assess the patients for complications from the procedure.

c. 12 to 20

For a healthy adult, what is the expected normal range for the respiratory rate per minute? a. 10 to 12 b. 12 to 15 c. 12 to 20 d. 20 or more

Neck Trauma assess

For other injuries (e.g., cardiovascular, respiratory, intestinal, neurologic damage) Carotid artery and esophagus For cervical spine injuries (prevent excessive neck movement)

Which risk factors can result in head and neck cancer in a patient? Select all that apply.

HPV infection, poor oral hygiene, and exposure to asbestos are risk factors that can result in head and neck cancer. Exposure to animal dander is a risk factor that results in allergic rhinitis in the patient. Exposure to coxsackie viruses can cause acute viral rhinitis in the patient.

A patient who has fallen off a roof arrives in the emergency department with possible head, neck, and chest trauma. The health care provider orders the following treatments. Which action will the nurse take first?

Immobilize the neck with a cervical collar. If the cervical spine has not already been stabilized by EMS (emergency medical services), this is the nurse's top priority. The neck should be held in place manually until a properly fitted cervical collar can be applied. Innervation of the diaphragm is between cervical spine levels 3 and 5. Oxygen administration is important; however, this is not the nurse's first priority and is considered separate from establishing an airway. Two large-bore (16- or 18-gauge IV catheter) IVs should be established and an isotonic fluid such as normal saline should be infused at a rate determined by the patient's condition and vital signs. CT scans are not the top priority and should be based on the patient's reported problems and condition.

When educating a patient about managing sinusitis without pharmacologic interventions, which instructions should the nurse include? Select all that apply.

In the case of sinusitis, steam inhalation helps to promote drainage of secretions. Sleeping with the head elevated helps to drain the sinuses and reduce congestion. Smoke is an irritant and will worsen the symptoms of sinusitis. Adequate fluid intake will decrease the symptoms of sinusitis. Applying a cold compress on the cheeks is not recommended because this worsens the symptoms. A hot compress on the cheeks will help.

What is pneumonia?

Infection that inflames air sacs in one or both lungs, which may fill with fluid

Chronic Bronchitis

Inflammation of bronchi and bronchioles caused by chronic exposure to irritants, especially cigarette smoke. Characterized by inflammation, vasodilation, congestion, mucosal edema, bronchospasm. Affects only airways not

Endotracheal Tubes: How are they inserted, where are they most commonly used, How long are they used for?

Inserted through the nose or mouth. Most commonly used in the critical care and Or setting. Used for short term airway protection and ventilation

Obstructive Sleep Apnea

Is a breathing disruptive during sleep that lasts at least seconds and occurs a minimum of 5 times in an hour

Thoracentesis

Is fluid removal by suction after the placement of a large needle of catheter into the intrapleural space

Coarse crackles and low-pitched crackles characterisitics:

Lower-pitched, coarse, rattling sounds caused by fuild or secretions in large airways; likely to change with coughing or suctioning.

How to suction

Lubricate the catheter with steril water or saline before insertion. Aplly suction only during the withdrawal of the cathe catheter. Use a twirling motion during withdrawal to prevent grabbing of the mucosa.

Head and Neck Physical Assessment/Clinical Manifestations

Lumps in mouth, throat, neck Difficulty swallowing Color changes in mouth or tongue Oral lesion or sore that does not heal in 2 weeks Persistent, unilateral ear pain Persistent/unexplained oral bleeding Numbness of mouth, lips, or face Change in fit of dentures Hoarseness or change in voice quality Persistent/recurrent sore throat Shortness of breath Anorexia and weight loss Change in fit of dentures Burning sensation when drinking citrus or hot liquids

What is the highest priority of care for a patient with head and neck cancer who underwent a total laryngectomy with neck dissection 24 hours ago?

Maintaining airway and ventilation Airway maintenance and ventilation are the highest priority. The extent of the surgery will determine the specific risk for the patient. Monitoring urinary output is a regular aspect of postoperative care, but it is not the highest priority if the airway or ventilation has the potential to be compromised. The family does require emotional support; an important aspect of this is knowing that the team will provide for the safety of their loved one. Body image changes can be challenging to deal with and must be addressed early in the future, but without an adequate airway and ventilation, the patient will develop other severe complications.

The patient is in the ICU for 3 days and then transferred back to the pulmonary step down unit. She is still slightly short of breath with exertion. Her O2 saturation is 99% on oxygen at 2 L per nasal cannula. She denies any shortness of breath when resting during the assessment. The provider plans to discharge the patient on home oxygen in the morning. What should the nurse include in this patient's discharge teaching?

New medications If distressed call 911

A client abruptly sits up in bed, reports having difficulty breathing and has an arterial oxygen saturation of 88%. Which mode of oxygen delivery is most likely to reverse these manifestations? a) Nasal cannula b) Face tent c) Simple mask d) Nonrebreather mask

Nonrebreather mask A nonrebreather mask can deliver levels of the fraction of inspired oxygen (FIO2) as high as 100%. Other modes — simple mask, face tent, and nasal cannula — deliver lower levels of FIO2.

A client who has begun standard multidrug treatment for tuberculosis (TB) reports orange-tinged sputum and urine. The nurse tells the client that this symptom represents which response to the treatment regimen?

Normal drug side effects of rifampin

A patient who has begun standard multidrug treatment for tuberculosis (TB) reports orange-tinged sputum and urine. The nurse tells the patient that this symptom represents which response to the treatment regimen?

Normal drug side effects of rifampin Orange-colored body secretions are an expected side effect of rifampin, one of the standard medications used for TB treatment. The orange color does not indicate spread of infection or hemolysis. Although alcohol and rifampin can cause hepatotoxicity, the orange color is not a sign of this complication.

Sodium (Na)

Normal level: 136-145 mmol/L functions: skeletal muscle contraction, cardiac contraction, & nerve impulse transmission

Phosphorus

Normal level: 3.0-4.5 mg/dL found in bones -- activates vitamins and enzymes -- assists in cell growth and metabolism

Potassium (K)

Normal level: 3.5-5.0 mEq/L regulates protein synthesis and glucose use and storage

Calcium

Normal level: 9.0-10.5 mg/dL absorption requires active form of vitamin D, important for maintaining bone strength and density (stored in the bones)

Chloride

Normal level: 98 to 106 mEq/L imbalance occurs as a result of other electrolyte imbalances

Epistaxis

Nosebleed is a common problem Cauterization of affected capillaries may be needed; nose is packed Posterior nasal bleeding is an emergency! Assess for respiratory distress, tolerance of packing or tubes Humidification, oxygen, bedrest, antibiotics, pain medications

A patient has recently been released from prison and has just tested positive for tuberculosis (TB). What teaching points does the community health nurse want to stress for this patient regarding medications? Select all that apply.

Not taking the medication could lead to an infection that is difficult to treat or to total drug resistance. The medications may cause nausea. The patient should take them at bedtime

A client is admitted to the hospital with a streptococcal peritonsillar abscess following incomplete treatment with an oral antibiotic. The nurse notes that the client is experiencing stridor. Which action does the nurse take next?

Notify the Rapid Response Team to assist w/ airway management

A patient is admitted to the hospital with a streptococcal peritonsillar abscess following incomplete treatment with an oral antibiotic. The nurse notes that the patient is experiencing stridor. Which action does the nurse take next?

Notify the Rapid Response Team to assist with airway management. If a patient with pharyngitis develops stridor or other indications of airway obstruction, the Rapid Response Team should be notified. Elevating the head of the bed is useful, but will not be sufficient to open the airway. Steroid medications will likely be ordered by the provider after the airway is opened. Offering liquids will increase the risk of aspiration.

The nurse is assessing a patient with chest tubes connected to a drainage system. What should the first action be when the nurse observes excessive bubbling in the water seal chamber? a) Disconnect the system and get another. b) Notify the physician. c) Place the head of the patient's bed flat. d) Milk the chest tube.

Notify the physician. Observe for air leaks in the drainage system; they are indicated by constant bubbling in the water seal chamber, or by the air leak indicator in dry systems with a one-way valve. In addition, assess the chest tube system for correctable external leaks. Notify the primary provider immediately of excessive bubbling in the water seal chamber not due to external leaks.

A postoperative patient develops stridor and becomes short of breath immediately following a thyroidectomy. What action does the nurse take?

Notify the rapid response team Vocal cord paralysis can occur following a thyroidectomy, and the combination of stridor and dyspnea is an emergency requiring endotracheal intubation or tracheostomy. A chest x-ray may be performed if aspiration is suspected after the patient is stabilized. Teaching the patient to hold the breath while swallowing is appropriate for a patient with vocal cord paralysis who is stable.

Postoperative Care after Rhinoplasty

Observe for edema and bleeding Check vital signs every 4 hours Change drip pad as needed

Nasal polyps manifestations

Obstructed nasal breathing Increased nasal discharge Change in voice quality

Which nursing action for a patient with chronic obstructive pulmonary disease (COPD) could the nurse delegate to experienced unlicensed assistive personnel (UAP)?

Obtain O2 saturation using pulse oximetry.

A public health nurse is providing education to a community about preparation for a possible influenza epidemic leading to a worldwide pandemic. What does the nurse instruct community members to do upon learning that an influenza outbreak has occurred?

Obtain a vaccine if not already vaccinated against influenza People should be taught to receive vaccinations if not already vaccinated if an outbreak occurs. People should stay home as much as possible and avoid crowds. Stockpiling food and medicines should occur in anticipation of an outbreak, not at the onset when people should be advised to stay home. Antiviral medications are given to those who contract the virus to limit symptoms.

The nurse is caring for a patient with severe COPD who is becoming increasingly confused and disoriented. What is the priority action of the nurse?

Obtain an arterial blood gas to check for carbon dioxide retention.

A client is receiving supplemental oxygen. When determining the effectiveness of oxygen therapy, which arterial blood gas value is most important? a) Partial pressure of arterial oxygen (PaO2) b) Partial pressure of arterial carbon dioxide (PaCO2) c) pH d) Bicarbonate (HCO3-)

Partial pressure of arterial oxygen (PaO2) The most significant and direct indicator of the effectiveness of oxygen therapy is the PaO2 value. Based on the PaO2 value, the nurse may adjust the type of oxygen delivery (cannula, Venturi mask, or mechanical ventilator), flow rate, and oxygen percentage. The other options reflect the client's ventilation status, not oxygenation. The pH, HCO3-, and PaCO2

Chest wall change to age-rationales

Patients may be anxious because they must work harder to breathe. Older patients have less tolerance for exercise. Calcium intake helps prevent osteoporosis by building bone in younger patients.

Which information should a nurse provide to the family of a patient who is using an endotracheal tube in an emergency department?

Patients requiring mechanical ventilation are initially managed with an endotracheal tube. The tube does not allow for much movement, and patients who have endotracheal tubes cannot eat. Endotracheal intubation is generally not performed to aspirate the trachea of a patient.

The instructor nurse is teaching a student nurse about the measures to be followed for a patient who has received radiation therapy. Which statement of the student nurse indicates effective learning?

Patients undergoing radiation therapy should use only prescribed lotions. Patients who are on radiation therapy should increase their mobility and do regular exercises to boost up their energy levels. Lotion should not be applied within two hours of treatment. The patient should refrain from rubbing the area immediately after the therapy because the irradiated skin is very sensitive and prone to injury.

A nurse practitioner has provided care for three different patients with chronic pharyngitis over the past several months. Which patients are at greatest risk for developing chronic pharyngitis?

Patients who are habitual users of alcohol and tobacco

The nurse is planning care for the non-English-speaking patient who is on complete voice rest. What alternative method of communication does the nurse implement?

Picture Board A picture board overcomes language barriers and can be used to communicate with patients who do not speak English well if a translator or a translation phone is not readily available. An alphabet board may or may not be useful if the patient does not speak English; this is not the best answer, but may be an option depending on what is available at the facility. A translator at the bedside would be beneficial for the nurse to speak with the patient, but not for the patient to ask questions or communicate concerns to the nurse. Unless the nurse is able to read the language the patient speaks, a word board would not be beneficial.

A nurse is caring for a client with a chest tube. If the chest drainage system is accidentally disconnected, what should the nurse plan to do? a) Clamp the chest tube immediately. b) Apply an occlusive dressing and notify the physician. c) Secure the chest tube with tape. d) Place the end of the chest tube in a container of sterile saline.

Place the end of the chest tube in a container of sterile saline. If a chest drainage system is disconnected, the nurse may place the end of the chest tube in a container of sterile saline or water to prevent air from entering the chest tube, thereby preventing negative respiratory pressure. The nurse should apply an occlusive dressing if the chest tube is pulled out — not if the system is disconnected. The nurse shouldn't clamp the chest tube because clamping increases the risk of tension pneumothorax. The nurse should tape the chest tube securely to prevent it from being disconnected, rather than taping it after it has been disconnected.

A client with pneumonia develops increased fever, chills, and night sweats. The nurse auscultates decreased breath sounds in the right lung and observes decreased chest wall movement in that area. The nurse reports these findings to the provider and suspects which secondary infection has likely developed?

Pulmonary Emphyema

A patient is admitted to the emergency department (ED) with a possible diagnosis of avian influenza ("bird flu"). Which of these actions included in the hospital protocol for avian influenza will the nurse take first?

Place the patient in a negative air pressure room If a patient is exhibiting symptoms of avian flu or any other pandemic influenza, he or she is assumed to be contagious until proven otherwise. Protecting the spread of disease to the community is the top priority, so placing the patient in a negative air pressure room is the nurse's first action. If avian influenza is diagnosed, it is important that those exposed receive oseltamivir or zanamivir within 48 hours of contact with the patient. Obtaining specimens will be important to determine whether the patient has avian influenza; this test takes approximately 40 minutes to complete. A patient with avian flu will become dehydrated because of diarrhea so starting an IV to administer rehydration fluid is important, but is not the first priority.

Which intervention made by the student nurse may complicate the condition of a patient with an obstructed airway and weak voice?

Place the patient in an upright and leaning forward position. An obstructed airway and weak voice are symptoms of vocal cord paralysis. Placing the patient in an upright and leaning forward position will create pressure on the head, thus aggravating the breathing difficulty. The patient should always be placed in a high-Fowler's position, which will help in easy breathing. Securing the airway is the most important step because it helps in regaining breathing. Assessing for an airway obstruction is essential to know the reason for the blockage in the airway. Proper alignment of the airway helps the air to reach the lungs.

A postoperative patient who was intubated has a breathy, weak, "wet"-sounding voice and coughing associated with swallowing. Which action by the nurse is correct?

Place the patient in high-Fowler's position Securing an airway is the main intervention. Place the patient in a high-Fowler's position to aid in breathing and proper alignment of airway structures. Assess for airway obstruction. This patient shows signs of aspiration possibly due to vocal cord paralysis. Incentive spirometry is not indicated. If the patient is aspirating, no oral liquids or solids should be given. The Rapid Response Team should be notified if the patient develops dyspnea with stridor.

An older adult patient residing in a long term care facility demonstrates new onset of coughing and sneezing and rhinorrhea, after his grandchildren came to visit him. He denies pain or fever. Which infection control procedures does the nurse instruct the LPN to initiate in order to protect the other residents

Place the patient on droplet precautions for the first 2 to 3 days

A patient calls the nurse to report a nosebleed that started with a sneeze. What does the nurse do first?

Position the patient upright and leaning forward over an emesis basin Positioning the patient in an upright, forward-leaning position prevents possible aspiration of blood. The next action would be to apply pressure to the nose for 10 minutes. If that fails, packing is necessary. After the bleeding stops, the patient should receive instructions about ways to prevent a recurrence.

A patient returns to the clinic to have the tuberculosis (TB) Mantoux test analyzed by the nurse, which was administered 2 days ago. The patient's left forearm shows a red raised area, which measures 10 mm in diameter. How does the nurse document this finding?

Positive reaction that indicates exposure to and the possible presence of TB infection An area of induration (raised soft tissue) measuring 10 mm or greater in diameter at 48-72 hours after the injection indicates exposure to and possible infection with TB. A positive reaction does not in itself mean TB is present until that has been confirmed with a chest x-ray and sputum culture. There are no false-positive readings, but the incidence of false-negative readings is greater at 48 hours and will need to be read again at 72 hours to confirm. The test will not be administered again in this situation.

Which principle about anterior versus posterior nasal bleeding must the nurse consider the priority?

Posterior bleeding is an emergency because it cannot be easily reached Posterior bleeding is not as readily seen or as easy to treat with the application of pressure or ice, thus posterior bleeds are more likely to require emergency treatment. Anterior bleeding is more easily seen and treated with pressure. Documentation is important with both types of bleeds; however, it is more difficult to assess the amount of bleeding with a posterior bleed, and blood is more likely to be swallowed. Various stimuli can trigger a nosebleed and sometimes no trigger can be identified.

Which clinical signs should the nurse monitor to ensure safety in a patient who has posterior nasal packing in the nose? Select all that apply.

Posterior nasal packing is applied to a patient to reduce bleeding from the nose. Posterior nasal packing can increase the risk of cardiovascular complications such as tachycardia and heart rhythm. The nurse should monitor heart rate and heart rhythm to check for complications. Posterior nasal packing can cause hypoxemia. Therefore the nurse should closely monitor the patient's respiratory rate. Posterior nasal packing does not alter thermoregulation. Therefore it does not result in hypothermia. Posterior nasal packing does not alter the red blood cell count.

A patient who is receiving combination chemotherapy and radiation therapy for neck cancer reports increasingly uncomfortable oral cavity effects such as stomatitis and gingivitis. What does the nurse recommend for this patient?

Providing oral care and comfort measures Stopping or taking a break from therapy is not recommended even when side effects become very uncomfortable. Patients should be taught prior to beginning therapy about ways to manage side effects.

Metabolic and respiratory acidosis (have hyperperfusion, anyone who has significant trauma, burns, sepsis, shock state= metabolic acidosis)

Pt brought to ED after being hit by car, is unresponsive, shallow breathing and has open femur fracture from which he has lost a significant amount of blood. The nurse anticipates which acid base imbalance?

A patient with pneumonia develops increased fever, chills, and night sweats. The nurse auscultates decreased breath sounds in the right lung and observes decreased chest wall movement in that area. The nurse reports these findings to the provider and suspects which secondary infection has likely developed?

Pulmonary empyema These are signs of pulmonary empyema, an infection in the pleural space. A fungal infection may occur anywhere, often as an abscess in the lungs, which is characterized by fever, cough, and foul-smelling sputum. Tuberculosis is characterized by cough and blood-tinged sputum.

c. The patient will breathe through the mouth and wear a nose clip during the test.

Pulmonary function tests are scheduled for a patient with a history of smoking who reports dyspnea and chronic cough. What will patient teaching information about this procedure include? a. Do not smoke for at least 2 weeks before the test. b. Bronchodilator drugs may be withheld 2 days before the test. c. The patient will breathe through the mouth and wear a nose clip during the test. d. The patient will be expected to walk on a treadmill during the test.

Erythroplakia

Red, velvety patches

RN provided education on INH (Isoniazid). Which statement by the patient indicates understanding of proper use of this medication?

Take it on an empty stomach

What is the most important information for the nurse to convey to a patient who is beginning pharmacological therapy for the treatment of tuberculosis to ensure suppression of the disease?

Take medication exactly as prescribed It is most important for the nurse to teach the patient to take the medication regularly, exactly as prescribed, for as long as it is prescribed to ensure adequate suppression of the disease. The patient should be instructed to eat a diet rich in Vitamins B and C. A diet rich in Vitamin K will not assist the patient in any way. Staying away from alcoholic beverages will prevent liver damage from the medications but will not ensure suppression of the disease. It is important for the patient to understand that the health care provider should be contacted in the case of illness; however, it will not ensure suppression of the disease.

a. Age, gener, race, height, weight, and smoking status

The patient is scheduled to have a pulmonary function test (PFT). Which type of information does the nurse include in the nursing history so that PFT results can be appropriately determined? a. Age, gender, race, height, weight, and smoking status b. Occupational status, activity tolerance for activities of daily living c. Medication history and history of allergies to contrast media d. History of chronic medical conditions and surgical procedures

d. "My lips and tongue are swollen." Rationale: Omalizumab is an anti-inflammatory used for long-term control of asthma. Anaphylactic reactions can occur with the administration of omalizumab. The nurse administering the medication should monitor for adverse reactions of the medication. Swelling of the lips and tongue are an indication of an adverse reaction.The client statements in options a, b, & c are not indicative of an adverse reaction.

The nurse has just administered the first dose of omalizumab (Xolair) to a client. Which statement by the client would alert the nurse that the client may be experiencing a life threatening adverse reaction? a. "I have a severe headache." b. "My feet are quite swollen." c. "I am nauseated and may vomit." d. "My lips and tongue are swollen."

What does the nurse teach a patient with sinusitis? Select all that apply.

The nurse instructs a patient with sinusitis to take adequate rest because it prevents fatigue and helps the body fight against infection-causing bacteria. The nurse also recommends that the patient drink plenty of water and remain hydrated in order to loosen nasal secretions. The nurse recommends that the patient perform large-volume nasal saline washes once or twice a day in order to facilitate the removal of nasal fluids. The nurse should suggest that the patient take hot showers twice a day to provide comfort. The patient should sleep with his or her head elevated because it helps drain the sinuses.

b. "When did you first notice these symptoms?"

The nurse is assessing an older adult patient who reports a decreased tolerance for exercise and that she must work harder to breathe. Which question assists the nurse in determining if these are normal changes related to aging? a. "How old are you?" b. "When did you first notice these symptoms?" c. "Do you or have you ever smoked cigarettes?" d. "How often do you exercise?"

C.

The nurse is caring for a patient admitted for treatment of neck and throat cancer. Which intervention should the nurse perform? A. Encourage hydration with water. B. Feed the patient if coughing occurs. C. Encourage the patient to sit in a chair for meals. D. Encourage the patient to drink juice to address thirst.

d. Percutaneous lung biopsy

The nurse is caring for several patients who had diagnostic testing for respiratory disorders. Which diagnostic test has the highest risk for the postprocedure complication of pneumothorax? a. Bronchoscopy b. Laryngoscopy c. Computed tomography of lungs d. Percutaneous lung biopsy

b. Encourage the patient to turn, cough, and deep breathe

The nurse is caring for the older adult who is temporarily confined to bed. Which intervention is important in promoting pulmonary hygiene related to age and decreased mobility? a. Obtain an order for PRN oxygen via nasal cannula b. Encourage the patient to turn, cough, and deep breathe. c. Reassure the patient that immobility is temporary d. Monitor the respiratory rate and check pulse oximetry readings.

WHEN START A PICC LINE THEY MEASURE, STOP INFUSION, CALL PHYSICIAN, WILL PROB DO CHEST XRAY TO MAKE SURE ITS STILL IN PLACE

The nurse is changing the dressing to the PICC inserted 7 days ago. The nurse notes that the length of the catheter is different than the original length of the catheter upon insertion. What should the nurse do at this point?

d. Sitting upright

The nurse is performing a physical assessment of the respiratory system. Although the patient is currently confined to bed, he has the strength and mobility to move and reposition himself. The nurse instructs him to assume which position for the assessment? a. Side-lying b. Semi-Fowlers c. Supine d. Sitting upright

a. Activities should be resumed gradually c. A sputum culture is needed every 2 to 4 weeks once medication therapy is initiated. d. Respiratory isolation is not necessary because family members already have been exposed e. Cover the mouth and nose when coughing or sneezing and put used tissues in plastic bags. Rationale: The nurse should provide the client and the family with information about tuberculosis and allay concerns about the contagious aspect of the infection. Instruct the client to follow the medication regimen exactly as prescribed and always to have a supply of the medications on hand. Advise the client of the side effects of the medication and ways of mnimizing them to ensure compliance. Reassure the client that after 2 to 3 weeks of medication therapy, it is unlikely that the client will infect anyone. Inform the client that activities should be resumed gradually and about the need for adequate nutrition and a well-balanced diet that is rich in iron, protein, and vitamin C to promote healing and prevent recurrence of infection. Inform the client and family that respiratory isolation is not necessary because family members already have been exposed. Instruct the client about thorough hand washing and to cover the mouth and nose when coughing or sneezing and to put used tissues into plastic bags. Inform the client that a sputum culture is needed every 2 to 4 weeks once medication therapy is initiated. When the results of three sputum cultures are negative, the client is no longer considered infectious and can usually return to former employment.

The nurse is preparing a list of home care instructions for the client who has been hospitalized and treated for tuberculosis. Of the following instructions, which will the nurse include on the list? a. Activities should be resumed gradually b. Avoid contact with other individuals, except family members, for at least 6 months c. A sputum culture is needed every 2 to 4 weeks once medication therapy is initiated. d. Respiratory isolation is not necessary because family members already have been exposed. e. Cover the mouth and nose when coughing or sneezing and put used tissues in plastic bags f. When one sputum culture is negative, the client is no longer considered infectious and usually can return to former employement.

A, B, C,

The nurse recognizes that a patient with sleep apnea may benefit from which intervention(s)? (Select all that apply.) A. Weight loss B. Nasal mask to deliver BiPAP C. A change in sleeping position D. Medication to increase daytime sleepiness E. Position-fixing device that prevents tongue subluxation

b. Start oxygen via nasal cannula at 2L/min

The patient demonstrates labored shallow respirations and a respiratory rate of 32/min with a pulse oximetry reading of 85%. What is the priority nursing intervention? a. Notify respiratory therapy to give the patient a breathing treatment b. Start oxygen via nasal cannula at 2L/min c. Obtain an order for a stat arterial blood gas (ABG) d. Encourage coughing and deep-breathing exercises

The nurse is reviewing the influenza criteria to see if a newly admitted patient meets vaccination requirements. Which findings would lead the nurse to recommend that the patient receive the vaccine? Select all that apply.

The patient has asthma. The patient has diabetes. The patient is being treated for canceR The patient lives in a nursing home

c. Check for a gag reflex before allowing the patient to eat

The patient has had a bronchoscopy and was NPO for several hours before the test. Now a few hours after the test, the patient is hungry and would like to eat a meal. What will the nurse do? a. Order a meal because the patient is now alert and oriented. b. Check pulse oximetry to be sure oxygen saturation has returned to normal c. Check for a gag reflex before allowing the patient to eat. d. Assess for nausea from the medications given for the test

a. ACE inhibitor

The patient has previously reported several chronic health conditions including hypertension and heart problems, and has stated a new drug was recently added to his drug regimen. Today the patient reports a new onset of cough. Which drug does the nurse suspect the patient has recently been prescribed? a. ACE inhibitor b. Vasodilator c. Diuretic d. Calcium channel blocker

a. Tuberculosis

The patient is HIV positive and reports feeling tired with shortness of breath, weight loss, and occasionally coughing up blood-tinged sputum. After considering these symptoms in conjunction with the patient's HIV status, what disorder does the nurse suspect this patient has? a. Tuberculosis b. Bronchitis c. Pneumococcal pneumonia d. Lung abscess

b. Ventilation-perfusion scanning

The patient is admitted for a deep vein thrombosis (DVT) and later becomes short of breath. A pulmonary embolus is suspected. The nurse should prepare the patient for which type of diagnostic testing? a. computed tomography b. Ventilation-perfusion scanning c. Magnetic resonance imging d. Digital chest radiography

d. Sudden onet of sharp pain after sneezing with lung sounds diminished over the left upper lobe

The patient is admitted for a pneumothorax. Which clinical assessment findings are most likely to be documented in the patient's admission record? a. Progressive fatigue and shortness of breath that has been increasing over a period of years b. Cough, high fever, rusty-colored sputum production with decreased breath sounds, particularly in lower lobes c. Frequent cough and copious sputum poduction, and wheezing and coarse crackles heard throughout the lung fields d. Sudden onset of sharp pain after sneezing with lung sounds diminished over the left upper lobe

c. The procedure is painless and the radioactive substance leaves the body in about 8 hours

The patient is scheduled or a ventilation perfusion scan. What does the nurse explain to the patient about the procedure? a. Being NPO before the examination is necessary to prevent aspiration of the dye b. After the test, isolation is necessary for 8 hours because of the radioactive dye c. The procedure is painless and the radioactive substance leaves the body in about 8 hours d. The test screens for pulmonary embolus; a CT scan will follow if needed

The nurse interviews a patient with a new diagnosis of chronic obstructive pulmonary disease (COPD). Which information is most specific in confirming a diagnosis of chronic bronchitis?

The patient reports a productive cough for 3 months every winter.

d. Visualize airway structures and obtaining tissue samples

The patient who had neck surgery for removal of a tumor reports, "not being able to breathe very well." The nurse observes that the patient has decreased chest movement and an elevated pulse. A bronchoscopy is ordered. For what reason did the physician order a bronchoscopy for this patient? a. Reverse and relieve any obstruction caused during the neck surgery b. Assess the function of vocal cords or remove foreign bodies from the larynx c. Aspirate pleural fluid or air from the pleural space d. Visualize airway structures and obtaining tissue samples

The nurse is caring for a patient who is scheduled for a lobectomy. Following the procedure, the nurse will plan care based on which of the following? a) The patient will return from surgery with no drainage tubes. b) The patient will return to the nursing unit with two chest tubes. c) The patient will require mechanical ventilation following surgery. d) The patient will require sedation until the chest tube (s) are removed.

The patient will return to the nursing unit with two chest tubes. The nurse should plan for the patient to return to the nursing unit with two chest tubes intact. During a lobectomy, the lobe is removed, and the remaining lobes of the lung are re-expanded. Usually, two chest catheters are inserted for drainage. The upper tube is for air removal; the lower one is for fluid drainage. Sometimes, only one catheter is needed. The chest tube is connected to a chest drainage apparatus for several days.

The nurse completes an admission assessment on a patient with asthma. Which information given by patient is indicates a need for a change in therapy?

The patient's only medications are albuterol (Ventolin HFAl) and salmeterol (Serevent).

c. Assess the patient for respiratory distress and recheck the oximeter reading

The patient's pulse oximetry reading is 89%. What is the nurse's priority action? a. Recheck the reading with a different oximeter b. Apply supplemental oxygen and recheck the oximeter reading in 15 minutes c. Assess the patient for respiratory distress and recheck the oximeter reading d. Place the patient in the recovery position and monitor frequently

A, C, D, E.

The provider discusses radiation therapy with the patient because her lesion is small and the cure rate is 80% or higher. The patient asks if her voice will return to normal. What is the appropriate nursing response? (Select all that apply.) A. "At first the hoarseness may become worse." B. "The more you use your voice, the quicker it will improve." C. "Gargling with saline may help decrease the discomfort in your throat." D. "Your voice will improve within 4 to 6 weeks after completion of the therapy." E. "You should rest your voice and use alternative communication during the therapy."

While caring for a patient with an endotracheal tube, the nurses recognizes that suctioning is required how often? a) When the nurse needs to stimulate the cough reflex b) Every 2 hours when the patient is awake c) When there is a need to prevent the patient from coughing d) When adventitious breath sounds are auscultated

When adventitious breath sounds are auscultated It is usually necessary to suction the patient's secretions because of the decreased effectiveness of the cough mechanism. Tracheal suctioning is performed when adventitious breath sounds are detected or whenever secretions are present. Unnecessary suctioning, such as scheduling every 2 hours, can initiate bronchospasm and cause trauma to the tracheal mucosa.

A client is being discharged home with active tuberculosis. Which information does the nurse include in the discharge teaching plan?

You will need to have your household undergo TB testing

c. Liver function test Rationale: Zafirlukast (Accolate) is a leukotriene receptor atagonist used in the prophylaxis and long-term treatment of bronchial asthma.Azfirlukast is used with caution in clients with impaired hepatic function.Liver function laboratory tests should be performed to obtain a baseline, and the levels should be monitored during administration of the medication.

Zafirlukast (Accolate) is prescribed for a client with bronchial asthma. Which laboratory test does the nurse expect to be prescribed before the administration of this medication? a. Platelet count b. Neutrophil count c. Liver function test d. Complete blood count

noncoring

a _____________ needle (e.g., Huber) is used to access an implanted port

What information is most important to teach the client going home with a peripherally inserted central catheter (PICC) line? a. "Avoid carrying your grandchild with the arm that has the IV." b. "Be sure to place the arm with the IV in a sling during the day." c. "Flush the IV line with normal saline daily." d. "You can use the arm with the IV for most of the activities of daily living."

a. "Avoid carrying your grandchild with the arm that has the IV." Rational: A properly placed PICC (in the antecubital fossa or the basilic vein) allows the client considerable freedom of movement. Clients can participate in most activities of daily living; however, heavy lifting can dislodge the catheter or occlude the lumen. Although it is important to keep the insertion site and tubing dry, the client can shower. The device is flushed with heparin.

A nursing student asks why midline catheters need strict sterile dressing changes when short peripheral IVs do not. Which answer by the experienced nurse is most accurate? a. "Because of the length of time they stay inserted." b. "They really don't need strict sterile technique." c. "Because the tip is in the right atrium of the heart." d. "The tonicity of the fluids used promotes infection."

a. "Because of the length of time they stay inserted." Rational: Midline catheters can stay in place for as long as 4 weeks, so dressing changes must be done with strict sterile technique to reduce the incidence of infection. The tip does not lie in the right atrium; it resides no farther than the axillary vein. These catheters are used for a wide range of fluids and medications, so tonicity would not be a factor in infection risk.

A student nurse is preparing to take a blood pressure (BP) on a client who has a peripheral IV line in the left arm. What instruction by the faculty member is most important? a. "Use the arm that doesn't have the IV site in it." b. "Don't inflate the cuff too high if you use the left arm." c. "Make sure the IV line is secure before taking the BP." d. "While the BP is taken, a little backflow of the IV is okay."

a. "Use the arm that doesn't have the IV site in it." Rational: Nurses should not take blood pressure on arms that have IVs because increased pressure can cause infiltration and can cause fluid to leak from the insertion site. Because the affected arm should not be used for BP, none of the other options can be correct.

The nurse is preparing to administer a medication IV push. What information does the nurse need to know before beginning the infusion? (Select all that apply.) a. Any dilution required b. Rate of administration c. Compatibility with infusions d. Other routes of administration e. Specific monitoring needed

a. Any dilution required b. Rate of administration c. Compatibility with infusions e. Specific monitoring needed Rational: Giving IV push medications requires specific knowledge about each drug, including dilution, rate of administration, compatibility, and monitoring. pH and osmolarity and specific infusion sites appropriate for giving the specific drug are also important to know. When giving an IV push medication, it is not necessary to know whether other routes of administration are possible.

What action does the nurse take to prevent infection in the older adult receiving IV therapy? a. Applying skin protectant before applying the dressing b. Avoiding the use of alcohol pads when removing tape c. Shaving the skin before attempting the venipuncture d. Using maximum friction to cleanse the skin

a. Applying skin protectant before applying the dressing Rational: The skin of an older adult may be more delicate and compromised. Avoidance of a disruption in skin integrity lessens the chance of an infection occurring with an IV catheter. A barrier applied to the skin before the IV dressing is placed helps maintain skin integrity.

The nurse is caring for a client who is receiving an epidural infusion for pain management. Which action has the highest priority? a. Assessing the respiratory rate b. Changing the dressing over the site c. Using various pain management therapies d. Weaning the pain medication

a. Assessing the respiratory rate Rational: Complications from an epidural infusion can be caused by the type of medication being infused, or they can be related to the catheter. When used for pain management, the client needs to be assessed for level of alertness, respiratory status, and itching. Using other pain management therapies and weaning the pain medication are important, but monitoring respiratory status has the highest priority in the nursing care of this client.

A nurse is preparing to administer two drugs at the same time to a client via a double-lumen midline catheter. Which action by the nurse is most important? a. Check the two drugs for compatibility. b. Compare the recommended infusion times. c. Schedule any post-infusion lab draws. d. Flush both lumens with saline before starting the infusion.

a. Check the two drugs for compatibility. Rational: Because midline catheters dwell in the peripheral, not central, circulation, incompatible drugs should not be given together via a double-lumen midline catheter because the flow rate of the blood is not high enough to dilute the drugs before they mix. The other options are valid interventions before starting the infusion, but they do not take precedence over determining whether the drugs may be infused at the same time.

The nurse is preparing to give a client an IV push medication through an intermittent IV set (saline lock) using a needleless system. Which actions by the nurse are most appropriate? (Select all that apply.) a. Cleanse the access port vigorously for at least 30 seconds. b. Use an antimicrobial agent when cleansing the port. c. Clean the ridges in the Luer-Lok connection well. d. Rinse the antimicrobial agent off with saline. e. Allow the antimicrobial agent to dry before using IV.

a. Cleanse the access port vigorously for at least 30 seconds. b. Use an antimicrobial agent when cleansing the port. c. Clean the ridges in the Luer-Lok connection well. Rational: Needleless systems need careful cleansing before use. Guidelines include scrubbing the connection vigorously with an antimicrobial agent for 30 seconds, and paying special attention to the ridges in the Luer-Lok device. Rinsing and drying are not necessary.

The nurse is assessing several clients receiving intravenous therapy. Which client situation requires immediate intervention? a. Completion of an intermittent medication into a Groshong catheter b. Physician's order to discontinue a peripheral intravenous catheter c. Nonaccessed implanted port placed 1 month ago without problem d. Peripheral IV catheter dated 5 days ago used for once-daily antibiotics

a. Completion of an intermittent medication into a Groshong catheter Rational: A Groshong catheter is a peripherally inserted catheter that needs to be flushed with saline after intermittent use. Peripheral IV catheters should be discontinued after 4 days, so this one should be changed; however, this is not the priority. An order to discontinue the peripheral catheter requires intervention, but flushing of the Groshong catheter is more of an immediate intervention to prevent clotting of the catheter. A nonaccessed implanted port site needs to be assessed, but this is not an immediate intervention.

The RN assigned a new nurse to a client who was receiving chemotherapy through an intravenous extension set attached to a Huber needle. Which information about disconnecting the Huber needle is most important for the RN to provide to the new nurse? a. "Apply topical anesthetic cream to the area after discontinuing the system." b. "Be aware of a rebound effect when discontinuing the system." c. "Be sure to flush the system with saline after removing the Huber needle." d. "Place pressure over the site to prevent bleeding."

b. "Be aware of a rebound effect when discontinuing the system." Rational: Huber needles are used to access implanted ports placed under the skin. Because the dense septum holds tightly to the needle, a rebound can occur when it is pulled from the septum, often resulting in needle stick injury to the nurse. Topical anesthetic cream can be used when accessing the system. Flushing is carried out when the system is accessed and once monthly. Because the implanted port is not being removed, there is no need for a pressure dressing.

The home care nurse is about to administer IV medication to the client and reads in the chart that the PICC line in the client's left arm has been in place for 4 weeks. The IV is patent, with a good blood return. The site is clean and free from manifestations of infiltration, irritation, and infection. Which action by the nurse is most appropriate? a. Notify the physician. b. Administer the prescribed medication. c. Discontinue the PICC line. d. Switch the medication to the oral route.

b. Administer the prescribed medication. Rational: A PICC line that is functioning well without inflammation or infection may remain in place for months or even years. Because the line shows no signs of complications, it is permissible to administer the IV antibiotic. The physician does not have to be called to have the IV route changed to an oral route.

After discontinuing a nontunneled, percutaneous central catheter, it is most important for the nurse to record which information? a. Application of a sterile dressing b. Length of the catheter c. Occurrence of venospasms d. Type of ointment used to seal the tract

b. Length of the catheter Rational: After removal of a catheter, measure the catheter length and compare it with the length documented on insertion. If the entire length has not been removed, the nurse should contact the physician immediately because some of the catheter may still be in the client's vein.

A client is to receive 10 days of antibiotic therapy for urosepsis. The nurse plans to insert which type of intravenous catheter? a. Hickman b. Midline c. Nontunneled central d. Short peripheral

b. Midline Rational: Midline catheters are used for therapies lasting from 1 to 4 weeks. Short peripheral catheters can be inserted by the nurse for use with antibiotic therapy, but they can stay in only for up to 96 hours. If the length of IV therapy is longer than 6 days, a midline catheter should be chosen. Nontunneled central catheters and Hickman catheters are inserted by a physician.

The nurse is caring for a client admitted yesterday with an intraosseous (IO) infusion after a car crash. Which action by the nurse takes priority? a. Ensure that the IV flow rate has been recalculated for an IO infusion. b. Plan to insert another kind of IV line during the shift. c. Determine which IV medications can be given safely via the IO. d. Monitor the site and dressings routinely for hemorrhage.

b. Plan to insert another kind of IV line during the shift. Rational: IO infusions, although valuable in an emergency, should be left in place for only 24 hours. The nurse should plan to insert a peripheral IV sometime during the shift. IV solutions, flow rates, and medications are given the same way that they are given IV. Hemorrhage is not a complication of IO infusion.

The nurse is caring for a client with an intraosseous catheter placed in the leg 20 hours ago. Which assessment is of greatest concern? a. Length of time catheter is in place b. Poor vascular access in upper extremities c. Affected leg cool to touch d. Site of intraosseous catheter placement

c. Affected leg cool to touch Rational: Compartment syndrome is a condition in which increased tissue perfusion in a confined anatomic space causes decreased blood flow to the area. A cool extremity can signal the possibility of this syndrome. All other distractors are important. However, the possible development of a compartment syndrome requires immediate intervention because the client could require amputation of the limb if the nurse does not pick up this perfusion problem.

Five days after the start of intraperitoneal therapy, the client reports abdominal pain and "feeling warm." The nurse prepares to assess the client further for evidence of which condition? a. Allergic reaction b. Bowel obstruction c. Catheter lumen occlusion d. Infection

d. Infection Rational: Fever, abdominal pain, abdominal rigidity, and rebound tenderness may be present in the client who has peritonitis related to intraperitoneal therapy. Peritonitis is preventable by using strict aseptic technique in handling all equipment and infusion supplies. An allergic reaction would occur earlier in the course of treatment. Bowel obstruction and catheter lumen occlusion can occur but would present clinically in different ways.

The nurse is caring for four clients receiving IV therapy. Which client does the nurse assess first? a. Client with a newly inserted peripherally inserted central catheter (PICC) line waiting for x-ray b. Client with a peripheral catheter for intermittent infusions c. Older adult client with a nonaccessed implanted port d. Older adult client with normal saline infusion

d. Older adult client with normal saline infusion Rational: Older adults are more prone to fluid overload and resulting congestive heart failure. Because this client is receiving continuous IV fluid, he or she is at risk for fluid overload and needs to be assessed. All other clients would need to be assessed for complications of IV catheters. However, they do not need immediate assessment.

The nurse is caring for a client with a radial arterial catheter. Which assessment takes priority? a. Amount of pressure in fluid container b. Date of catheter tubing change c. Checking for heparin in infusion container d. Presence of an ulnar pulse

d. Presence of an ulnar pulse Rational: An intra-arterial catheter may cause arterial occlusion, which can lead to absent or decreased perfusion to the extremity. Assessment of ulnar pulse is one way to assess circulation to the arm in which the catheter is located.

Before the administration of intravenous fluid, it is most important for the nurse to obtain which information from the health care provider's orders? a. Intravenous catheter size b. Osmolarity of the solution c. Vein to be used for therapy d. Specific type of IV fluid

d. Specific type of IV fluid Rational: An order for infusion therapy must contain the following to be complete: specific type of fluid, rate of administration, and drugs added to the solution. Osmolarity of the solution is not necessary because it is incorporated into the specific type of fluid. It is the nurse's independent decision about the most appropriate vein to cannulate and the catheter size to use.

hyponatremia (due to the loss of wound fluids from the large wounds)

would you expect hyponatremia or hypernatremia in a patient suffering from severe burns?


Ensembles d'études connexes

Introduction to Nutrition Chapter 8

View Set

6 What are the characteristics of lymphatic capillaries?

View Set